Download as pdf or txt
Download as pdf or txt
You are on page 1of 154

PREVENTIVE MEDICINE 3 - [APPLIED EPIDEMIOLOGY] AY 20-21

diagnostic tests and treatment, study designs, statistical


analysis, disease treatment, prevention and prognosis 2.01
27 JAN 21 | 03 FEB 21
Ma. Victoria Rondaris, MD
TABLE OF CONTENTS II. CLINICAL EPIDEMIOLOGY
I. EPIDEMIOLOGY ........................................................................ 1 ● The application of epidemiological principles and methods to the
A. AIMS OF EPIDEMIOLOGY ...................................................... 1 practice of clinical medicine
B. EARLY DIAGNOSIS ................................................................ 1
● The science of making predictions about individual patients by
C. APPLIED EPIDEMIOLOGY ...................................................... 1
counting clinical events in similar patients, using scientific
II. CLINICAL EPIDEMIOLOGY ....................................................... 1
A. SOURCES OF DATA USEFUL FOR EPIDEMIOLOGIC methods for studies of groups of patients to ensure that the
STUDIES ......................................................................................... 1 predictions are accurate.
B. MEASURING HEALTH AND DISEASE .................................... 1 ● Purpose: to develop and apply methods of clinical observations
C. MEASURE OF DISEASE FREQUENCY .................................. 2 that will lead to valid conclusions by avoiding being misled by
D. DIAGNOSIS ............................................................................. 3 systematic error and chance
E. DISEASE ................................................................................. 4 ● To make good decisions in the care of the patients
F. EARLY DIAGNOSIS ................................................................ 5 ● The relationship between Epidemiology + Clinical Medicine |
III. EPIDEMIOLOGICAL STUDY (OBSERVATIONAL STUDIES) ..... 6
A. CROSS-SECTIONAL STUDY .................................................. 6 Table 1. Relationship between Epidemiology and Clinical Medicine
B. CASE-CONTROL STUDY........................................................ 7 Epidemiology Clinical Medicine
C. COHORT STUDY .................................................................... 7 Population Individuals
D. EXPERIMENTAL STUDY ........................................................ 8 ● Studies/assessments ● Diagnosis
IV. CAUSE ........................................................................................... 9 ● Prevention ● Treatment
A. CONCEPT OF CAUSE .......................................................... 10 ● Evaluation ● Curing
B. SYNERGISM ......................................................................... 10 ● Planning ● Caring
C. GUIDELINES FOR CAUSATION ........................................... 10
V. NATURAL HISTORY OF A DISEASE AND PROGNOSIS.............. 10 Table 2. Clinical Questions
VI. TREATMENT ................................................................................ 11 Issue Question
A. TYPES OF CLINICAL TRIALS ............................................... 13 Abnormality Is the patient sick?
VII. PROGNOSIS ............................................................................... 13 Diagnosis How accurate are tests used to diagnose
A. FIRST RULE OF THUMB....................................................... 14 disease?
B. SECOND RULE OF THUMB .................................................. 14 Frequency How often does a disease occur?
VIII. PREVENTION ............................................................................. 14
Risk What factors are associated with an increased
A. PRIMARY PREVENTION....................................................... 14
risk of disease?
B. SECONDARY PREVENTION ................................................ 14
C. TERTIARY PREVENTION ..................................................... 14 Prognosis What are the consequences of having a
disease?
This trans is purely based on the lecture and the powerpoint of Dra. Rondaris.
Treatment How does treatment change the course of
disease?
I. EPIDEMIOLOGY Prevention Does an intervention on well people keep
● The study of the distribution and determinants of health-related disease from arising?
states or events in specified populations, and the application of Does early detection and treatment improve
this study to control of health problems the course of disease?
Cause What conditions lead to disease?
A. AIMS OF EPIDEMIOLOGY What are the pathogenetic mechanisms of
● To understand the course of the disease (natural history of the disease?
disease) Cost How much will care for an illness cost?
● To identify the causes or risk factors
● To provide effective measures of treatment and prevention A. SOURCES OF DATA USEFUL FOR EPIDEMIOLOGIC
STUDIES
B. EARLY DIAGNOSIS ● Data on vital events – birth and death
● Causation ● Morbidity or disease statistics
→ Factors affecting good health (genetic factors or environmental ● Data on physiologic and or pathologic condition
factors including lifestyle) → ill health ● Statistics on health resources and services
● Natural History ● Statistics pertaining to the environment
→ Good health → Subclinical changes → Clinical changes → ● Demographic data
Death or Recovery ● Socio-cultural data
● Description of Health status of the population
→ Proportion with ill health, change over time, change with age, B. MEASURING HEALTH AND DISEASE
etc. ● Clinical question: Is the patient sick or well?
● Evaluation of intervention ● Diagnostic tests
→ Health promotion → Qualitative diagnostic tests
→ Preventive measures ■ Presence or absence of a disease according to presence
→ Public health services or absence of a clinical s/sx.
■ Example: x-ray confirms or disproves the presence of a
C. APPLIED EPIDEMIOLOGY
fracture.
● Clinical epidemiology → Quantitative diagnostic tests
● Communicable disease epidemiology ■ Above or below a preselected cutoff value known as the
● Environmental and occupational epidemiology positivity criterion
● Molecular epidemiology

TUBAN, UY H. | TRIA, TRINIDAD, TROPICALES, TUASON, TUAZON, UY A., UY K., UY N., UY Q., UY W., UYCOCO, VALDECANAS Page 1 of 14
PREV MED 3 - [Applied Epi] 2.01 – Diagnostic Tests and Treatment, Study Designs, Statistical Analysis, Disease Treatment, Prevention and
Page 2 of 14
Prognosis (27 JAN 2021, 3 FEB 2021)

■ Normal (Gaussian) distribution method


− Frequency distribution of values of the test follows a
normal distribution.
■ Percentile method
■ Therapeutic method
■ Predictive value method
Table 3. Six Definitions of Normal in Common Clinical Use |
Property Term Consequences of its
Clinical Application
The distribution of Gaussian Ought to occasionally
diagnostic test results obtain minus values for
has a certain shape hemoglobin, etc.
Lies within a preset Percentile All diseases have the same
percentile of previous prevalence. Patients are
diagnostic test results normal only until they are
worked up
Carries no additional Risk factor Assumes that altering a
risk of risk factor alters risk
morbidity or mortality Figure 1. Gaussian Distribution Curves. Upper: actual clinical distributions of the
Socially or politically Culturally Confusion over the role of different laboratory parameters. Bottom: negatively skewed distribution to the
left; positively skewed distribution to the right; normal or no skew that represents
aspired to desirable medicine in society
a symmetrical distribution.
Range of test results Diagnostic Need to know predictive
beyond values that apply in your C. MEASURE OF DISEASE FREQUENCY
which a specific practice ● Clinical question: How often does a disease occur?
disease is, with
known probability, Prevalence Rate (P)
present or absent ● The number of cases in a defined population at a specified point
Range of test results Therapeutic Need to keep up with new in time
beyond knowledge about therapy ● Old + new cases |
which therapy does → Point prevalence (e.g. summer, winter)
more good than harm → Period prevalence (e.g. entire 2nd half of the year in the PH -
Diagnostic criteria are usually based on symptoms, signs and rainy season)
test results # of people with disease
P= x(factor)
● Hepatitis - presence of antibodies in the blood # of people in the pop. at risk at the specified time
● Asbestosis - symptoms and signs of specific changes in lung
● Example: no. of women with mastitis/ no. of women in QC (at
function
risk population) OR no. of people with colds/ no. of people in
→ Radiographic demonstration of fibrosis of the lung tissue or
QC (whether young or old, still at risk to have colds!)
pleural thickening
● Prevalence studies do not usually provide strong evidence of
→ History of exposure to asbestos fibers
● The Jones Criteria (revised) for Guidance in the Diagnosis of causality
● It is helpful in assessing the need for health care and the
Acute Rheumatic Fever
→ A high probability of rheumatic fever is indicated by the planning of health services
● Prevalence rates are often used to measure the occurrence of
presence of two major or one major and two minor,
manifestations, if supported by evidence of a preceding Group conditions for which the onset of disease may be gradual
● Factors influencing observed prevalence rate |
A streptococcal infection
→ Increased by:
Table 4. Major and minor manifestations of ARF in the Jones Criteria ■ Longer duration of the disease
Major Minor ■ Prolongation of life of patient without cure
● Carditis Clinical ■ Increase in new case(increase in incidence)
● Polyarthritis ● Fever ■ In-migration of cases
● Chorea ● Arthralgia ■ Out-migration of healthy people
● Erythema marginatum ● Previous rheumatic fever or ■ In-migration of susceptible people
● Subcutaneous nodules rheumatic heart disease ■ Improved diagnostic facilities (better reporting)
Laboratory
→ Decreased by:
● Acute phase reactants: abnormal
■ Shorter duration of disease
ESR, CRP, leukocytosis
■ High case-fatality rate from disease
● Prolonged P-R interval
■ Decrease in new cases(decrease in incidence)
● WHO Case-Definition For AIDS ■ In-migration of healthy people
→ Presence of: ■ Out-migration of cases
■ Disseminated Kaposi’s sarcoma OR ■ Improved cure rate of cases
■ Cryptococcal meningitis OR Incidence Rate (I)
■ 2 major signs + ≥1 minor sign
● Number of new cases arising in a given period in a specified
Table 5. Major and minor signs of AIDS
time
Major Minor ● New cases only |
• Weight loss > 10% • Persistent cough > 1 month ● Numerator – number of new events that occur in defined time
• Fever > 1 month • General pruritic dermatitis period
• Chronic diarrhea >1 • Recurrent herpes zoster ● Denominator – population at risk of experiencing the event
month • General lymphadenopathy during this period
• Chronic herpes simplex/ oral candidiasis ● Most accurate way of calculating incidence rate is to calculate
the person-time incidence rate (incidence density)
PREV MED 3 - [Applied Epi] 2.01 – Diagnostic Tests and Treatment, Study Designs, Statistical Analysis, Disease Treatment, Prevention and
Page 3 of 14
Prognosis (27 JAN 2021, 3 FEB 2021)

● An incidence rate describes how quickly disease occurs in a Gold standard


population. It is based on person-time, so it has some ● A sounder indication of truth or a standard of accuracy
advantages over an incidence proportion. Because person-time is ● A new diagnostic test is compared
calculated for each subject, it can accommodate persons coming ● Elusive (not available)
into and leaving the study ● Expensive and risky – biopsy, surgical exploration, autopsy
# of people who get a dx in a specified period
→ Hard to be a gold standard
𝐈= x(factor) ● Sometimes simple – throat swab culture
sum of the length of tume during wc each person in the pop.
→ Preferred
is at risk
Table 6. Target disorders with their corresponding gold standard for diagnosis
Cumulative Incidence or Risk (CI) Target Disorder Gold Standard
● Unlike incidence rate, it measures the denominator only at the Breast CA Excisional Biopsy
beginning of a study Prostate CA Transrectal Biopsy
● This rate has a simplicity that makes it suitable for the Coronary Stenosis Coronary angiography
communication of health information to decision makers Myocardial Infarction Catheterization
● Easy to interpret and provide a useful summary measure
Strep Throat Throat Culture
● It is useful approximation of incidence rate when the rate is low or
when the study period is short
● No need to count for the time period |
● Dra. Rondaris: aalamin kung saan lang nag-simula
# of people who get a dx in a specified period
𝐂𝐈 = x(factor)
No. of people free of the dx in pop. at risk
at the beginning of the period

Figure 5. Bell-shaped curve of normal and abnormal test group. At the point
where they transect each other are the cut-off points.
Figure 2. Review of the spectrum of illness from communicable disease.
Analysis of a Diagnostic Test |
Disease No Disease
Test Positive a b
true positives false positives
Test Negative c d
false negatives true negatives
a (true-positives): individuals with the disease, and for whom the test is positive
b (false-positives): individuals without the disease, but for whom the test is
positive
c (false-negatives): individuals with the disease, but for whom the test is
Figure 3. Example of an occurrence of disease in 100 people. negative
d (true-negatives): individuals without the disease, and for whom the test is
negative
a + c = total number of individuals with the disease
b + d = total number of individuals without the disease

Figure 4. Example of calculation of disease occurrence.

D. DIAGNOSIS
● Clinical question: How accurate are tests used to diagnose
disease?
Diagnostic test
● Objective is to diagnose any treatable disease present Figure 6. Formulas used for answering the clinical question on diagnostics.
● Characteristics of a diagnostic test Se: sensitivity; Sp: specificity; P: prevalence; PV: predictive value
→ Reliable/repeatability – gives the same measurement when
repeated more than once ● Validity of a Diagnostic Test
→ Valid - measures what it intends to measure → a = no. of true positives, b = no. of false positives c = no. of
→ Accurate – correctly determines those with disease and those false negatives, d = no. of true negatives
without → Sensitivity
→ Easy to use – can be performed by other people without ■ probability of a positive test in people with the disease
difficulty ■ a / (a + c)
→ Not expensive – affordable → Specificity
→ Safe and acceptable ■ probability of a negative test in people without the disease
■ d / (b + d)
PREV MED 3 - [Applied Epi] 2.01 – Diagnostic Tests and Treatment, Study Designs, Statistical Analysis, Disease Treatment, Prevention and
Page 4 of 14
Prognosis (27 JAN 2021, 3 FEB 2021)

→ Positive predictive value → A specific test is most helpful when the test result is positive
■ probability of the person having the disease when the test is (to confirm or “rule in” the disease). Very high Sp → face of a
positive child with Down’s syndrome rules in diagnosis of Down’s
■ a / (a + b) ● Extremely high Sn or Sp are RARE
→ Negative predictive value ● Use of multiple diagnostic tests
■ probability of the person not having the disease when the → Use of imperfect diagnostic tests, with less than 100%
test is negative sensitivity and specificity, a single test frequently results in a
■ d / (c + d) probability of disease that is neither very high nor very low.
Serial Testing
● Consecutively, based on previous test result
● Used when rapid assessment is not required
● Used when some of the tests are expensive and risky
● Maximizes specificity and positive predictive value but lowers
sensitivity and the negative predictive value.
● The process is more efficient if the test with the highest
specificity is used first.
● Example: serial platelet count for Dengue monitoring

Figure 7. Sample computation. *not discussed because the same with clin ep

E. DISEASE
● Clinical question: How often are tests used to diagnose
disease?
● Statements about validity test: |
→ Sensitivity and specificity are inversely related!
→ A sensitive test can pick up most cases of the disease but it will
erroneously label as positive many persons who do not have the
disease.
→ A highly specific test will correctly label as negative those who
do not have the disease but it will miss many cases.

Figure 9. Example of serial test - 2hr OGTT.

Likelihood Ratios
● Alternative way of describing the performance of a diagnostic
test
● Summarize the same kind of information as sensitivity and
specificity
● Used to calculate the probability of disease after a positive and
negative test (positive or negative predictive value)
Figure 8. Evidence that sensitivity and specificity are inversely related.
● Advantage - can be used at multiple level of test results
● Use of likelihood ratios depend on odds |
→ A very sensitive test gives a low positive predictive value since ● Probability
it produces many false positives. Conversely, a very specific → Used to express sensitivity, specificity, and predictive value
test gives a high positive predictive value. → Is the proportion of people in whom a particular
■ Sensitivity and specificity are unaffected by the prevalence characteristic, such as positive test, is present
of the disease or condition. Since sensitivity depends only ● Test’s positive likelihood ratio (LR+)
on those with the disease or condition and specificity only on → The ratio of the proportion of diseased people with a positive
those without the disease or condition. test result (sensitivity) to the proportion of non-diseased with
■ The positive predictive value of a test increases with the a positive test result (1 - specificity)
prevalence of the disease. ● Test’s negative likelihood ratio (LR-)
■ Relationship of Disease Prevalence to Positive → The proportion of diseased people with a negative test result
Predictive Value (1 - sensitivity) divided by the proportion of non-diseased
− Disease Prevalence - 1% people with a negative test result (specificity)
− Positive Predictive value – 17%
− Disease Prevalence – 5% Sensitivity
− Positive Predictive value – 51% 𝐋𝐑+=
1 − Specificity
● Uses of sensitive tests
→ A sensitive test should be chosen when there is an important 1 − Sensitivity
penalty for missing a disease (dangerous but treatable 𝐋𝐑−=
Specificity
condition)
→ A sensitive test is most helpful to the clinician when the test PROBLEM 1
result is negative (to rule out disease) = SnNout ● The sensitivity of ESR for spinal malignancy is 78%; and the
■ Very high Sn approaching 100% specificity is 67%, giving a false-positive rate of 100%-
→ Loss of retinal pulsation in increased ICP → presence of 67%=33%. The Likelihood ratio (LR) for a positive test is
pulsation (r/o inc. ICP) therefore:
● Uses of specific tests → LR=0.78/0.33=2.36
→ Highly specific tests are needed when false-positive results can
harm the patient physically, emotionally, or financially.
PREV MED 3 - [Applied Epi] 2.01 – Diagnostic Tests and Treatment, Study Designs, Statistical Analysis, Disease Treatment, Prevention and
Page 5 of 14
Prognosis (27 JAN 2021, 3 FEB 2021)

● It helps to keep in mind that the probability is a proportion; It is the Reliability and Validity
number of times a given outcome occurs divided by all the ● Measurement error
occurrences → Instrument: the means of making the measurement
● If we take a sample of blood from a patient 5 times, and the → Observer: the person making the measurement
sample is positive 1 time, we can think of the probability as being ● Biologic variation
1 in 5 or 0.20 → Within individuals: changes in people with time and
● The odds on the other hand is a ratio; it is the number of times a situation (environment)
given outcome occurs divided by the number of time that specific → Among individuals: biologic differences from person to
outcome does not occur person
● With the blood sample example, the odds of a positive sample is 1
to 4 or 1/(5-1)
● Example: Diagnostic Characteristics of a D-dimer Assay in
Diagnosing Deep Venous Thrombosis (DVT) (see figure 10)

Figure 10. Likelihood Ratio Sample Computation. Figure 11. Reliability vs. validity (accuracy). Bottom image: A, high validity and
reliability. B, low validity and high reliability. C, high validity and low reliability. D,
Simple Rule of Thumb low validity and low reliability. The dotted lines represent the true values.
● Mnemonic F. EARLY DIAGNOSIS
→ Likelihood ratio of 2, 5, 10 increases the probability of disease
approximately 15%, 30% and 45% respectively, and ● Strategies
→ The inverse of these likelihood ratios of 0.5, 0.2, and 0.1 → Screening test (uni- or multi-phasic)
decrease the probability → Periodic health examination (e.g. yearly blood chem testing
● Interpretation of Likelihood ratios among ≥40 yo px)
→ The larger the value of LR+, the stronger the association → Case finding
between having a positive test result and having the disease of ● Objectives
interest → Early detection of asymptomatic disease
→ The larger the size of the LR+ the better the diagnostic value of → Identification of predictors or risk factors of disease
the test. Although somewhat arbitrary, and LR+ value of 10 or
greater is often perceived as an indication of a test of high
diagnostic value
→ An LR- with a value of 1 indicates a test with no value in sorting
out persons with and without the disease of interest as the
probability of a negative test result is equally likely among
persons affected and and unaffected
→ The smaller the value of LR-, the stronger the association
between having a negative test result and not having the
disease of interest
→ The smaller the size of the LR-, the better diagnostic value of
the test.
→ On somewhat arbitrary grounds, an LR- value of 0.1 or less is
Figure 12. Natural History of Disease Timeline.
often perceived as an indication of a test with high diagnostic
value Natural History of Disease (4 Stages)
Clinical question: How accurate are tests used to diagnose 1. Biologic onset
disease? → Initial interaction between man, causal factors, and the rest of
the environment
Problems → Cannot detect the presence of disease
● Lack of information on negative tests 2. Early diagnosis possible
● Lack of information on test results in the non-diseased → Mechanisms of disease produce structural or functional
● Lack of objective standards for disease changes
● Consequences of imperfect standards → Individual remains free of any symptoms
→ If a new test is compared with an old (but inaccurate) standard 3. Usual clinical diagnosis
test, the new test may seem worse even when it is actually → Disease progresses to the point where symptoms appear and
better the affected individual becomes ill
4. Outcome
→ Recovery, permanent disability or death
PREV MED 3 - [Applied Epi] 2.01 – Diagnostic Tests and Treatment, Study Designs, Statistical Analysis, Disease Treatment, Prevention and
Page 6 of 14
Prognosis (27 JAN 2021, 3 FEB 2021)

Types of Epidemiological Study (Descriptive Studies)


● Case reports
→ Detailed presentations of a single case or a handful of cases
→ Means of describing rare clinical events
→ Describe unusual manifestations of disease
→ Elucidate the mechanisms of disease and treatment
→ Place issues before medical community and often trigger
more decisive studies
→ Susceptible to bias
→ Lower part in the hierarchy of evidence pyramid
→ Examples of case reports:
■ 58 y/o woman with Parkinsonism → Amantadine HCl 100
Figure 13. Stages of Disease History. mg bid → remission in her rigidity, tremor and akinesia
■ Minoxidil for HPN → improve male pattern baldness
Critical Points in the Natural History Of Disease ■ Tamoxifen for contraception → prevent breast CA
● Position 1 ● Case-series
→ The screening test and case finding would be too late to be of → A simple descriptive account of interesting characteristics
help in early detection of disease observed in a group of patients
● Position 2 → Study larger group of patients (lecture: ≥10; clin epi: ≥8) with
→ The test will have a promise of improving the outcomes of particular disease
those who have the target disorder → Describe the clinical manifestations of disease and
→ Do Diagnostic Tests treatments in a group of patients assembled at one point in
● Position 3 time
→ Early detection of the disease is a waste of time → Absence of a comparison group, not conclusive
→ Dra. Rondaris: no need for diagnostic test → Hypothesis-generating
→ Selection bias (researcher chooses which case to place)
→ Examples of case series
■ Silicone breast implants suspected of causing connective
tissue disease like scleroderma and rheumatoid arthritis
■ 156 women with silicone breast implants and rheumatic
disease complaints
Table 8. Analytic Study Design
Type of Study Alternative Name Unit of Study
Observational Studies
Descriptive
Analytical
• Ecological Correlational Population
Figure 14. Critical Points in the Natural History of Disease. • Cross-sectional Prevalence Individuals
• Case-control Case-reference Individuals
III. EPIDEMIOLOGICAL STUDY (OBSERVATIONAL STUDIES) • Cohort Follow-up Individuals
Classification of Epidemiological Study Experimental studies Interventional studies
Randomized control trials Clinical trials Patients
● Descriptive: characterize a phenomenon Field trials Healthy people
→ Case report/case series Community trials Community Communities
→ Prevalence survey intervention studies
→ Ecological study
● Analytic: determine the presence of a relationship between an A. CROSS-SECTIONAL STUDY
exposure(s) and an outcome(s)
● Prevalence Study
→ Observational: exposure variable is not available
● No direction of inquiry
■ Cross sectional study:
● Measurements of exposure and effect are made at the same
■ Case control study
time |
■ Cohort study
● Useful for investigating exposures that are fixed characteristics
→ Experimental: exposure variable is manipulated
of individuals, such as ethnicity, socio-economic status and
■ Randomized controlled trials
blood group, or chronic disease or stable conditions
■ Field trials
● Short-term and therefore less costly
■ Community trials
● Provide no direct estimate of risk
Table 7. Analytic Study Design ● Prone to bias from selective survival
Cross-sectional study Longitudinal studies ● Estimates prevalence may be biased by the exclusion of cases
Exposure and outcome are Exposure and outcome are measure in which death or recovery are rapid
measure at the same time in at different times in the life of the ● In sudden outbreaks of disease it is the most convenient first
the life of the participant participant step in an investigation into the cause
● Rare disease, conditions of short duration or diseases with high
case fatality are often not detected |
Cross sectional study Case control study
Cohort study
Randomized controlled trials
Field trials
Community trials
PREV MED 3 - [Applied Epi] 2.01 – Diagnostic Tests and Treatment, Study Designs, Statistical Analysis, Disease Treatment, Prevention and
Page 7 of 14
Prognosis (27 JAN 2021, 3 FEB 2021)

Example 2:
● Habitual, vigorous physical activity (exposure) vs. primary
cardiac arrest
→ Case: out-of-hospital primary cardiac arrest
→ Control: telephone calls
● Criteria for entry:
→ Age 25-75 y/o
→ No clinical recognizable heart disease
→ No prior disease with limited activity
→ Have a spouse
Figure 15. Cross-sectional study. Advantages of case-control study
● Relatively efficient, requiring smaller sample than cohort study
● Completed faster and more economical
● Earliest practical observational strategy for determining an
association
● Antecedent-consequence uncertainty
Odds Ratio (OR)
● Ratio of the odds that the cases were exposed to the odds that
the controls were exposed
● Interpretation of Odds Ratio
→ Value of OR less than 1 indicates a negative association
(i.e., protective effect) between the risk factor and the
Figure 16. Basic formulation of a cross-sectional study. disease
→ For rare disease (e.g., most chronic diseases with disease
B. CASE-CONTROL STUDY prevalence of less than 10%), OR approximates RR
● Case-control study is observational, analytical and
longitudinal. |
● Study population is classified according to disease status
→ Cases: those with the disease interest
→ Control: those without the disease
● Longitudinal studies: looking backward from the disease to a
possible cause; looking for exposure, observe what happened in
the past
● Use new (incident) cases
● Used to investigate cause (etiology) of disease, esp. rare
diseases |
● Uses odds ratio |
Figure 19. Table arrangement and formula for Odds Ratio |
Table 9. Interpretation of Odds Ratio |
Odds Ratio Interpretation
1.0 No association
>1.0 Risk Factor
<1.0 Protective Factor

C. COHORT STUDY
● Longitudinal studies (forward)
● Provide the best information about the causation of disease
● Most direct measurement of the risk of developing disease
Figure 17. Case-control study. ● Provide the possibility of estimating the attributable risks/ RISK
Examples of case-control study DIFFERENCE (RISK IN THE EXPOSED GROUP MINUS THE
RISK IN THE UNEXPOSED GROUP
● Example 1: Association between recent meat consumption and
● Use relative risk |
enteritis necroticans in Papua New Guinea
● Most closely resemble experimental studies

Figure 20. Cohort Study. Cohort and case control studies are the ONLY
longitudinal studies. Direction is forward for cohort, backward for case control.
Figure 18. Example of a Case-control Study computation using Odds Ratio. In Prospective over time → people without the disease → divided into exposed and
dealing with OR, use a 2x2 table. Remember “ABCD”! Odds ratio between who not exposed → want to know who got sick later on
had enteritis and ate meat. Remember only the last one: AD/BC *Doc said typo
● The cases were 11.6 times more likely than the controls to have
recently ingested meat.
PREV MED 3 - [Applied Epi] 2.01 – Diagnostic Tests and Treatment, Study Designs, Statistical Analysis, Disease Treatment, Prevention and
Page 8 of 14
Prognosis (27 JAN 2021, 3 FEB 2021)

Disadvantages of cohort study Confounding


● Long-term, not always feasible ● Confusion of two supposedly causal variables, so that part or all
● Sample size required for the study extremely large of the purported effect of one variable is actually due to the
● Attrition (dropout) is most serious problem other
Types of cohort studies ● Can result in distortion of the true measure of the effect
between the exposure and outcome
● Retrospective Cohort (RC)
● Identification of confounders in research is based primarily on
→ Exposure and outcome already occurred when the study is
literature search or expert knowledge
initiated
● Definition
● Prospective Cohort (PC)
→ Common cause of both exposure and outcome
→ Exposure may or may not have occurred, but outcome has not
yet occurred when the study is initiated.
● Ambidirectional Cohort (ADC)
→ Data collected both retrospectively and prospectively on the
same cohort.
Figure 23. Confounding. Confounding variable affects the outcome of the study.
→ Especially useful when studying exposures with short- and
long-term effects. Table 11. Confounding as mixing effects - Water Pipe Analogy |
Usually an
exposure will lead
to an outcome but
at times a
“confounder” will
interrupt →
outcome will be
different

If confounder is
Figure 21. Prospective vs. retrospective cohort studies. Time frames for a
removed, real
hypothetical prospective cohort study and a hypothetical retrospective cohort study association will be
begun in 2012. seen/clearer →
will not affect the
outcome.

See Appendix for Applications of different observational study designs

D. EXPERIMENTAL STUDY

Figure 22. Table arrangement and formula for Relative Risk |


IE+
𝐑𝐢𝐬𝐤 𝐫𝐚𝐭𝐢𝐨 =
IE−
𝐑𝐢𝐬𝐤 𝐝𝐢𝐟𝐟𝐞𝐫𝐞𝐧𝐜𝐞 = IE+ − IE−
Table 10. Interpretation of Risk Ratio |
Risk Ratio Risk Difference
RR = 1.0 RD = 0
No association No association
RR > 1.0 RD > 0
Harmful association Harmful association
RR < 1.0 RD < 0
Beneficial association Beneficial association
OR & RR point of reference is always 1. RD point of reference is always 0.
Figure 23. Experimental Study,
See Appendix for summary of advantages and disadvantages of different Basic population → make criteria → randomize → perform intervention
observational study designs
Bias Randomized Controlled Trials (RCTs)
● Selection bias ● Gold standard or reference in medicine |
→ The way the cases and controls exposed and non-exposed ● Provide the greatest justification for concluding causality
individuals were selected. ● Subject to the least number of problems or biases
→ Example: Oral contraceptive use and thrombophlebitis (doctor ● Best study design to establish the efficacy of a treatment or a
monitors OC users more closely procedure
● Recall bias
→ Miscalculation of exposure more than controls.
→ Example: women with a baby with a malformation tend to
remember more mild infections during their pregnancies than
mothers of normal infants
PREV MED 3 - [Applied Epi] 2.01 – Diagnostic Tests and Treatment, Study Designs, Statistical Analysis, Disease Treatment, Prevention and
Page 9 of 14
Prognosis (27 JAN 2021, 3 FEB 2021)

● Selection bias
→ Occurs when comparisons are made between groups of
patients that differ in determinants of outcome other than the
one under study
● Measurement bias
→ Occurs when the methods of measurement are dissimilar
among groups of patients
● Confounding bias
→ Occurs when two factors are associated (“travel together”)
and the effect of one is confused with or distorted by the
effect of the other

IV. CAUSE
● Clinical question:
Figure 24. Randomized Controlled Trial. → What conditions lead to disease?
Target population → screen (included/excluded) → baseline measurement → → What are the pathogenic mechanisms of disease?
randomization → 1. New treatment; 2. Standard or placebo (not recommended but ● Concept of Cause
there are exceptions, better if standard is used as comparator) → measure outcome → A cause must precede a disease
Disadvantages of RCT → A cause is termed sufficient when it inevitably produces or
● Expensive and time-consuming initiates a disease
● Difficult to obtain approval to perform properly designed clinical → A cause is termed necessary if a disease cannot develop in
trials its absence

● A sufficient cause is not usually a single factor, but often
Table 12. Relative ability of different types of study to “prove” causation
comprises several components
Type of Study Ability to “prove” causation
● It is not necessary to identify all the components of a sufficient
Randomized controlled trials Strong
cause before effective prevention can take place
Cohort studies Moderate
● Each sufficient cause has a necessary cause as a component
Case-control studies Moderate ● A causal factor on its own is often neither necessary nor
Cross-sectional studies Weak sufficient
Ecological studies Weak
See Appendix for Type of Research and Study Design
Bias in Clinical Observation
Table 13. Methods of Controlling Selection Bias
Method Description PHASE OF STUDY
Design Analysis
Randomization Assign patients to groups +
in a way that gives each
patient equal chance of
falling into one or the other
group
Restriction Limit the range of +
characteristics of patients
Figure 25. Cause. Causes of Cholera
in the study
Matching For each patient in one + Necessary cause and Sufficient cause
group select one or more
patients with the same ● Presence of an organism is a necessary cause for disease to
characteristics (except for occur but not necessarily a sufficient cause
the one under study) for a ● Example:
comparison group → AIDS cannot occur without exposure to HIV, but exposure to
Stratification Compare rates within + the virus does not necessarily result in disease
subgroups (strata) with → Exposure to HIV rarely results in AIDS after needle stick (3/
otherwise similar 1,000) because the virus is not as infectious as Hep B virus
probability of the outcome
Adjustment
Simple Mathematically adjust +
crude rates for one or few
characteristics so that
equal weight is given to
strata of similar risk
Multiple Adjust for difference in +
large number of factors
related to outcome, using
mathematical modeling
techniques
Best Describe how different the +
case/worse results could be under the
case most extreme or simply
very unlikely conditions of Figure 26. Cause. Causes of Tuberculosis
selection bias
PREV MED 3 - [Applied Epi] 2.01 – Diagnostic Tests and Treatment, Study Designs, Statistical Analysis, Disease Treatment, Prevention and
Page 10 of 14
Prognosis (27 JAN 2021, 3 FEB 2021)

A. CONCEPT OF CAUSE ● Dose-response relationship


● Interaction → “Is increased exposure to the possible cause associated with
→ Interplay of multiple causes increased effect?”
● Synergism → Example: Number of cigarettes smoked vs lung cancer
→ The joint effect is greater than the sum of the effects of the ● Reversibility
individual causes → “Does the removal of a possible cause lead to reduction of
● Antagonism disease risk?”
→ The joint effect is lesser → Example: declining mortality from lung CA in ex-cigarette
● Effect Modification smokers
→ A special type of interaction; the strength of the relationship ● Study design
between 2 variables is different according to the level of some → “Is the evidence based on a strong study design?”
third variable, called an effect modifier. ● Judging the evidence
● A substantial impact on a patient’s healthy by changing only one → “How many lines of evidence lead to conclusions?”
or a small number of the causes
V. NATURAL HISTORY OF A DISEASE AND PROGNOSIS
B. SYNERGISM
● Clinical question: What are the consequences of having a
● Probability of developing coronary artery disease over a 5-year disease?
period among men aged 55. ● Prognosis
→ Men with normal systolic BP, normal cholesterol, no history of → Is a prediction of the future course of disease following its
diabetes, or LVH on ECG, and no history of smoking have a onset
low chance (2.1%) of developing CHD over the next 5 years. ● Outcomes of Disease (5Ds) |
→ Risk increases, up to 7.3%, when the risk factors are present → Death
individually ■ A bad outcome if untimely
→ But when all risk factors are present together, the absolute risk → Disease
of developing CAD in the next 5 years (43.4%) is almost twice ■ A set of symptoms, physical signs, and laboratory
the sum of the individual risks abnormalities
→ Discomfort
■ Symptoms such as pain, nausea, dyspnea, itching, and
tinnitus
→ Disability
■ Impaired ability to go about usual activities at home, work,
or recreation
→ Dissatisfaction
■ Emotional reaction to disease and its care, such as
sadness or anger
● Natural history of disease
→ Refers to the stages of disease
● Prognostic factors |
→ Are conditions that are associated with a given outcome of
the disease
Figure 27. Study of breast cancer in relation to alcohol consumption.
→ A variety of consequences of disease are counted (usually
Folate intake to see if the risk for breast cancer diminishes even if you drink alcohol.
So the higher your folate intake, your risk for breast cancer is lowest (RR <1). after the disease process)
→ Describe relatively frequent events (frequency)
Comparing disease occurrence among exposed and unexposed ■ E.g. How many live through breast cancer? How many are
● Risk ratio or relative risk in remission? How many died?
→ The ratio of the risk of occurrence of a disease among exposed → Sick people
people to that among the unexposed → From disease to outcome
→ Better indicator of the strength of an association than the risk → Frequent events
difference ● Risk factors |
→ Used in assessing the likelihood that an association represents → Events being counted is the onset of disease
a causal relationship → Predict low probability events (probability of having the
→ Example: RR = 49.6 / 17.7 = 2.8 disease)
C. GUIDELINES FOR CAUSATION ■ E.g. How many of these women do cigarette smoking?
→ Healthy people
● Temporal relationship ■ Seemingly health (have family history for diabetes → risk
→ “Does the cause precede the effect? (essential)” factor)
● Plausibility → From onset to disease
→ “Is the association consistent with other knowledge? Makes → Yearly rates (low probability events)
sense, according to biologic knowledge of the time (mechanism
of action; evidence from experimental animals”
● Consistency
→ “Have similar results been shown in other studies?”
→ Repeatedly observed by different persons, in different places,
circumstances, and times.
→ Example: increase tobacco vs. increase lung CA
● Strength
→ “What is the strength of the association between the cause and
the effect? (large relative risk)” Figure 28. Natural history of disease (stages of disease).
→ Example: 10-fold higher incidence of lung cancer among male
smokers compared to non-smokers)”
PREV MED 3 - [Applied Epi] 2.01 – Diagnostic Tests and Treatment, Study Designs, Statistical Analysis, Disease Treatment, Prevention and
Page 11 of 14
Prognosis (27 JAN 2021, 3 FEB 2021)

VI. TREATMENT
● Clinical question: How does treatment change the course of
disease?
● Three ways of picking up therapy |
→ Induction method
■ Your own uncontrolled clinical experience
− “kutob mo or feel mo lang”
■ Outcomes of patients with new treatments and outcomes
of patients treatment in other ways before new treatment
was available
■ Use of historical comparisons
■ Retrospective analysis
■ “seems to work or ought to work”
→ Deduction method
■ Formal randomized clinical trials
Figure 29. Schematic representation of the natural history of disease. − based on scientifically proven readings
■ Prospective analysis
Table 14. Risk and prognostic factors for myocardial infarction
■ Select treatment if worthless
Risk Factors Prognostic Factors for Poor Outcome
→ Abdication/Seduction method
↑Age ↑Age
■ Recommendations of others
Male Female
− Asking colleagues
Cigarette smoking Cigarette smoking
HPN Hypotension ■ Abdication - accept treatment on faith
↑LDL/↓HDL Anterior infarction − “Bahala na si Lord”
Inactivity CHF ■ Seduction - “to fit the rhyme”
Inflammation Ventricular arrhythmia − “Sabi ng iba kaya gagayahin ko”
Coagulation disorders ● Hypothetico-deductive method
There are prognostic factors that are also risk factors (ex: increasing age and → Preferred method for selecting specific treatments (From the
cigarette smoking). hypothesis and what you read about it)
Table 15. Independent prognostic factors for AIDS → The best information on whether a given treatment does
Factor Poor Prognostic Level more good than harm to patients with a given disorder is the
Age ≥37yo result of a randomized clinical trial (Not just because it is
Initial presentation Multiple diagnoses RCT means it is the best but still read the journal if they
Single diagnosis other - followed the inclusion/exclusion, randomization, no selection
than Kaposi’s sarcoma or bias, if treatment is new or just modified and was it compared
P. carinii pneumonia to the standard)
CD4+ T lymphocytes Low Six Guides to Distinguish Useful from Useless or Even
Hemoglobin Low Harmful Therapy
Rates Commonly Used to Describe Prognosis 1. Was the assignment of patients to treatments really
● 5-year survival randomized?
→ Percent of patients surviving 5 years from some point in the 2. Were all clinically relevant outcomes reported?
course of their disease 3. Were the study patients recognizably similar to your own?
● Case fatality 4. Were both clinical and statistical significance considered?
→ Percent of patients with a diseases who die of it 5. Is the therapeutic maneuver feasible in your practice?
● Disease-specific mortality 6. Were all the patients who entered the study accounted for at its
→ Number of people per 10,000 population dying of a specific conclusion?
disease ● Guides 1 & 6
● Response → Deals mostly with validity (can be computed and measured)
→ Percent of patients showing some evidence of improvement → Are the article’s conclusions true?
following an intervention ● Guides 2, 3, & 5
● Remission → Deals mostly with applicability (can be applied to your own
→ Percent of patients entering a phase which disease is no longer patient
detectable (commonly used in cancer) → Are the article’s conclusions relevant to your own patients?
● Recurrence ● Guide 4
→ Percent of patients who have return of disease after a disease- → Deals with both validity and applicability
free interval → Statistical and clinical significance
● Survival analysis (Kaplan-Meir analysis) ● Clinical significance
→ A way of estimating the survival of a cohort over time → Refers to the importance of a difference in clinical outcomes
● Life table analysis between treated and control patients
→ Also used in estimating survival → Usually described in terms of the magnitude of a result
Guides for Reading Articles to Learn the Clinical Course and → Goes beyond arithmetic and is determined by clinical
Prognosis of Disease judgment
1. Was an “inception cohort” assembled? ● Statistical significance
2. Was the referral pattern described? → Tells us whether the conclusions (from computations) the
3. Was complete follow-up achieved? author have drawn are likely to be true
4. Were objective outcome criteria developed and used? → Regardless of whether or not they are clinically important
5. Was the outcome assessment “blind”?
6. Was the adjustment for extraneous prognostic factors carried
out?
PREV MED 3 - [Applied Epi] 2.01 – Diagnostic Tests and Treatment, Study Designs, Statistical Analysis, Disease Treatment, Prevention and
Page 12 of 14
Prognosis (27 JAN 2021, 3 FEB 2021)

Were Both Clinical and Statistical Significance Considered?


● An article that reports on a randomized double-blind clinical trial
comparing a new drug (Drug A) with an identical appearing
placebo (Drug B) for the control of an important clinical disorder.
● Based on the results, the authors of the article will have drawn
one of two conclusions: either Drug A is better than Drug B or
Drug A is no better than Drug B.

Figure 33. Occurrence of death, stroke, or other major complications - Absolute


Risk Reduction. The decimal form of the absolute risk reduction (ARR) is
foreign to most clinicians.

Figure 30. Comparing the conclusions of clinical trials.

Figure 34. Occurrence of death, stroke, or other major complications -


Number Needed To Treat. The measure of clinical significance is called the
number needed to treat (NNT).

Figure 31. Naming the erroneous conclusions from a clinical trial. ● For easy interpretation of absolute risk reduction, we take the
FP (False Positive) = Type 1 error; FN (False Negative) = Type 2 error reciprocal of it.
● The reciprocal of the absolute risk reduction is the number
● The relationship between Type 1 and Type 2 errors are used in
of patients we need to treat in order to prevent one
both planning and interpreting randomized trials.
complication of their disease.
● In planning such a trial, investigators can decide beforehand just
how great a risk they are willing to run of drawing erroneous Table 16. Measures of Effect
conclusions of both sorts. Expression Question Definition
● Most authors decide to set the false-positive (alpha) risk at 0.05 (Important column (Formulas not usually
and the false-negative (beta) risk at 0.20-conventional levels of to remember! asked in the exams)
Usually asked when
statistical significance (standard or commonly used)
to use and how to
interpret)
Absolute Risk What is the incidence # of new cases over
(Incidence) of disease in a group
of initially free of the 𝐈=
a given period of time
condition? # of people in the group
Attributable Risk What is the incidence
(Risk difference) of disease attributable 𝐀𝐑 = IE− IĒ
to exposure?
Relative Risk How many times
(Risk Ratio) more likely are IE
𝐑𝐑 =
exposed persons to IĒ
become diseased,
relative to non-
exposed persons?
Population- What is the incidence
Figure 32. Occurrence of death, stroke, or other major complications. “How might Attributable Risk of disease population, 𝐀𝐑 𝐏 = AR × P
these benefits be expressed in terms of clinical significance?” associated with the
*FP = 0.08, FN = 0.10; these relative risk reductions (RRR) mean that the risk of prevalence of a risk
death, stroke, or other complications of hypertension was reduced by almost two- factor?
third through active treatment.
Population- What fraction of
Attributable disease in a AR p
Fraction population is 𝐀𝐅𝐏 =
IT
attributable to
exposure to a risk
factor?
IE = incidence in exposed persons; IĒ = incidence in non-exposed persons
P = prevalence of exposure to a risk factor; IT = total incidence of a dx in a pop.
PREV MED 3 - [Applied Epi] 2.01 – Diagnostic Tests and Treatment, Study Designs, Statistical Analysis, Disease Treatment, Prevention and
Page 13 of 14
Prognosis (27 JAN 2021, 3 FEB 2021)

Is the Therapeutic Maneuver Feasible in your Practice? VII. PROGNOSIS


● Contamination ● What to do for difficult situations:
→ In which control patients accidentally receive the experimental → Seek an expert opinion
treatment (Supposed to be the standard) → Read up on clinical literature about clinical course and
→ Reduction in the difference in clinical outcome between prognosis
experimental and control groups → Prognosticate based on you own clinical experience
● Co-Intervention
→ The performance of additional diagnostic or therapeutic acts on Guides for Reading Articles to Learn the Clinical Course and
experimental but no the control patients (Do more on the Prognosis of Disease
experimental group when it should be done in both groups) 1. Was an “inception cohort” assembled?
→ Increase in the difference in clinical outcomes between 2. Was the referral pattern described?
experimental and control groups 3. Was complete follow-up achieved?
4. Were objective outcome criteria developed and used?
A. TYPES OF CLINICAL TRIALS 5. Was the outcome assessment “blind”?
● Intervention Studies 6. Was adjustment for extraneous prognostic factors carried out?
→ Clinical Trials: experimental studies in medicine that involve
1. Was an “inception cohort” assembled?
humans are called
→ Controlled Trials: are studies in which the experimental drug ● Patients should have been identified at an early and uniform
or procedure is compared with another drug or procedure, point (inception) in the course of their disease (e.g. onset of
sometimes a placebo and sometimes the previously accepted symptoms, time of diagnosis or beginning of treatment), so that
treatment those who succumbed or completely recovered are included
→ Uncontrolled Trials: are studies in which the investigator’s with those whose disease persisted
experience with the experimental drug or procedure is → The starting point is called zero time
described, but the treatment is not compared with another drug ● Descriptions of prognosis should include the full range of
→ Concurrent Control: is the control that is given intervention for manifestations that would be considered important to patients
the same period of time as the study group ● Failure to start a study of clinical course and prognosis with an
● Types according to purpose: inception cohort has an unpredictable effect on its result.
→ Prophylactic trials (e.g. immunization, contraception) ● Failure to assemble a proper inception cohort of patients
→ Therapeutic trials (drug treatment, surgical procedures) constitutes a fatal flaw in studies of prognosis.
→ Safety trials (side-effects of drug) ● Many studies of prognosis are done backwards
→ Effectiveness trials (theoretical, use, and extended use ● “Tama ba yung pagkakuha ng study population?”
effectiveness of contraceptive methods) 2. Was the referral pattern described?
→ Risk factor trials (proving etiology of disease) ● The pathway by which patients entered the study sample
→ Efficiency trials should be described.
→ Did they come from a primary care center or were they
assembled in a tertiary care center?
Table 17. Phases of Clinical Trials | ● It is in the assembly of patients that studies of the course and
Phase Description prognosis of disease often flounder
1 • Small number of healthy volunteers ● “How were they referred for the study?”
• Non-blind or “open” Table 18. Different forms of Bias based on Referral Pattern |
• Establish limits of the safe clinical dosage range Bias Description
2 • Patients with the target disease to determine efficacy Centripetal Center’s reputation results in part from its particular
• Single-blind trial Bias expertise in a specialized area of clinical medicine, it
will be referred problem cases likely to benefit from
3 • Classical phase
its expertise
• Larger number of patients with the target disease to
Popularity Experts may preferentially admit and keep track of
establish safety and efficacy
Bias these cases over other less challenging or less
• Performed on patients with consent interesting cases
• Carried out mostly on hospital in-patients (controlled Referral Filter Selection that occurs at each stage of referral
environment) Bias process can generate patient samples at tertiary
4 • Post marketing surveillance care centers that are much different from those
• A trial in normal field or program setting found in the general population
• Reassess effectiveness, safety, acceptability and Diagnostic Px differ in their financial and geographic access to
continued use of the drugs. Access Bias clinical technology that identifies them as eligible for
studies of the course and prognosis of disease
3. Was complete follow-up achieved?
● All members of the inception cohort should be accounted
for at the end of the follow-up period, and their clinical status
should be known.
● This is because patients do not disappear from a study for trivial
reasons (refuse therapy or recover or die or retire or simply
grow tired of being followed.)
→ Exhaust all means to know why they left the study and
publish it as part of documentation
● Difficult for the authors to achieve perfection, they are bound to
lose a few members of their inception cohort
Figure 35. Phases of clinical trials in a new vaccine. FDA still continues data ● “Sinabi ba na may follow-up and nacomplete ba yung follow-
collection even if the vaccine is already in the community set-up.
up?”
PREV MED 3 - [Applied Epi] 2.01 – Diagnostic Tests and Treatment, Study Designs, Statistical Analysis, Disease Treatment, Prevention and
Page 14 of 14
Prognosis (27 JAN 2021, 3 FEB 2021)

4. Were objective outcome criteria developed and used? APPENDIX


● The prognostic outcomes should be stated in explicit,
Table 19. Advantages and Disadvantages of Different Observational Study
objective terms so that you, as the reader of the subsequent designs
report, will be able to relate them to your own practice Ecological Cross- Case-control Cohort
● The criteria are applied in a consistent manner. sectional
● “Sinama ba yung inclusion/exclusion and nasa results ba?” Selection NA medium high low
5. Was the outcome assessment “blind”? bias
Recall bias NA high high low
● The examination for important prognostic outcomes should have
Loss to NA NA low high
been carried out by clinicians who were “blind” to the other
follow-up
features of these patients.
Confounding high medium medium low
● Also reduces bias
Time required low medium medium high
● Diagnostic-suspicion Bias
→ Clinician who knows that a patient possesses a prognostic Cost low medium medium high
factor of presumed importance may carry out more frequent Table 20. Applications of Different Observational Study Designs
and more detailed searches for relevant prognostic outcome Ecological Cross- Case- Cohort
● Expectation Bias sectional control
→ Pathologists and others who interpret diagnostic specimens Investigation ++++ - +++++ -
can have their judgments dramatically influenced by prior of rare dx
knowledge of clinical features of the case Investigation ++ - _ +++++
of rare cause
A. FIRST RULE OF THUMB Testing + ++ - +++++
● If the article concludes that some constellation of symptoms, multiple effect
signs, and laboratory results accurately predicts a certain of cause
prognosis, demand evidence that the authors have confirmed the Study of ++ ++ ++++ +++
constellation’s predictive power in a second independent sample multiple
of patients (the test sample) exposure &
determinants
B. SECOND RULE OF THUMB Measurements ++ - + +++++
● It has to do with the numbers of patients that should have been of time
included in the training and test samples. relationship
● There should at least be 10 patients for every prognostic factor the Direct - - + +++++
authors studied. measurement
of incidence
VIII. PREVENTION Investigation - - +++ -
of long latent
A. PRIMARY PREVENTION periods
● Immunization
Table 21. Type of Research and Study Design |
● Target: Total populations, selected groups, healthy
Objective Study Design
individuals
● Phase of Disease: Specific causal factor/s Disease description / spectrum Case study
Case series
B. SECONDARY PREVENTION Cross-sectional
● Pap Smear Disease prevalence Cross-sectional
● Screening test/s Operating characteristics of a Cross-sectional
→ Identification of unrecognized disease or risk factor diagnostic test Cohort
■ History taking Disease incidence Cohort
■ Physical examination Prognosis Cohort
■ Laboratory tests Causation / etiology / harm Cohort
■ Ancillary procedures Case-control
● Target: Patients Clinical Trials (RCT)
● Phase of Disease: Early stage of disease
C. TERTIARY PREVENTION
● Limitation of Disability
● Rehabilitation
● Goal here is not to prevent death but to maximize the amount of
high-quality time patient has left
● Target: Patients
● Phase of Disease: Late stage of disease (Treatment,
Rehabilitation)
REFERENCE
Good luck, mga Dokies!
Rondaris, M., (2020), Applied Epidemiology [PowerPoint Presentation]. Manila,
Philippines: Faculty of Medicine and Surgery, University of Santo Tomas,
Preventive Medicine
PREVENTIVE MEDICINE [APPLIED EPIDEMIOLOGY] AY 20-21

CRITICAL APPRAISAL
Ma. Victoria Rondaris, MD
2.0210 FEB 21

TABLE OF CONTENTS B. Critical Appraisal of the METHODOLOGY


● Is the study design valid for your question?
I. CRITICAL APPRAISAL ............................................................... 1 ● Are both inclusion and exclusion criteria described?
II. CRITICAL APPRAISAL: A CHECKLIST BY ROBERT WILL ● Is there an attempt to limit bias in the selection of participating
(2016) .................................................................................................... 1
groups?
A. Critical Appraisal of the INTRODUCTION ............................. 1
● Are there methodological protocols (i.e. blinding) used to limit
B. Critical Appraisal of the METHODOLOGY ............................ 1 other possible biases?
C. Critical Appraisal of the RESULTS ........................................ 1
● Do the research methods limit the influence of confounding
variables?
D. Critical Appraisal of the DISCUSSION/CONCLUSION ......... 1 → If in the methodology the researchers applied strategies to
control the confounders.
● Are the outcome measures valid for the health condition you are
MUST KNOW BOOK PREVIOUS TRANS
searching?
U & 4
C. Critical Appraisal of the RESULTS
This trans is purely based on the lecture and powerpoint of Dra. Rondaris.
● Is there a table that describes the subjects’ demographics?
● Are the baseline demographics between groups similar?
I. CRITICAL APPRAISAL ● Are the subjects generalizable to your patient?
● A process of evaluating a research journal or article ● Are the statistical tests appropriate for the study design and
systematically to determine its reliability and validity and its clinical question?
application in clinical practice. ● Are the results presented within the paper?
● Patient application of journals ● Are the results statistically significant and how large is their
● An important element of Evidence-Based Medicine (EBM) difference between groups?
● Is there evidence of significance fishing (changing statistical
tests to ensure significance)?

D. Critical Appraisal of the DISCUSSION/CONCLUSION


● Do the authors attempt to contextualize non-significant data in
an attempt to promote significance (talking about findings
which had a trend toward significance as if they were
significant)?
● Do the authors acknowledge limitations of the study?
● Are there conflicts of interest which were noted?

REFERENCES
Figure 1. Process of Critical Appraisal. Rondaris, M.V. (2021). Critical Appraisal. [Powerpoint Presentation]. Manila,
Philippines: Faculty of Medicine and Surgery, University of Santo Tomas, PREV.
MED. 3
5 Steps in Evidence-Based Medicine | U
1. Making an answerable question from an uncertain clinical
problem.
2. Search for the evidence
3. Critical Appraisal
4. Application of results in practice
5. Evaluation of new practices
→ Done in new research of methods that are not the gold
standard.

II. CRITICAL APPRAISAL: A CHECKLIST BY ROBERT WILL


(2016)

A. Critical Appraisal of the INTRODUCTION


● Does the article attempt to answer the same questions as your
clinical question?
● Is the article recently published (within 5 years) or it is seminal
(an earlier article but which has strongly influenced later
developments)?
● Is the journal peer-reviewed?
● Do the authors present a hypothesis/ses?

TUBAN, UY H. | UY K., UY N., UY Q., UY W., UYCOCO, VALDECANAS Page 1 of 1


PREVENTIVE MEDICINE 3 [FAMILY HEALTH] AY 20-21

Eva Irene Yu Maglonzo, MD, FPAFP, MHPEd, FPCGM 19 AUG 20

TABLE OF CONTENTS
I. INTRODUCTION .................................................................................. 1
• 3 Biggest Worries about having Chronic Illness (Age 60+) ................ 1
• Fundamental Practice Changes to Change Outcomes ...................... 1
• Red Flags for Old People .................................................................. 1
• Comprehensive Geriatric Assessment .............................................. 1
• PFC Matrix........................................................................................ 1
II. GERIATRIC SYNDROMES .................................................................. 1
A. FALL ............................................................................................... 1
B. DEMENTIA ..................................................................................... 2
C. DEPRESSION ................................................................................ 3
D. INCONTINENCE ............................................................................. 3
E. OSTEOPOROSIS ........................................................................... 4 Figure 1. Components of a Comprehensive Geriatric Assessment
F. POLYPHARMACY .......................................................................... 4
G. INSOMNIA ...................................................................................... 4 PFC Matrix
H. CONSTIPATION (IMPACTION) ...................................................... 5
I. SARCOPENIA & MALNUTRITION................................................... 5 ● Patient-centered, family-focused, community-oriented diagnosis
J. FRAILTY ..................................................................................... 6 and intervention
III. OTHER ASSESSMENTS & ADVOCACIES......................................... 6 Table 1. PFC Matrix
Family Community
Components Patient Centered
MUST KNOW BOOK PREVIOUS TRANS Focused Oriented
Diagnosis Anxiety
Dementia Access to
(Medical and Financial
This trans is based on the powerpoint of Dra. Maglonzo supplemented with her Depression Care
Psychosocial) challenge
notes and A2021 trans.
Intervention
Support
(Medical and
I. INTRODUCTION Anticholinesterase Family group
Psychosocial)
Antidepressant meeting Refer to
● Dealing with chronic illness is like piloting a small plane. How? to Diagnosis
NGO
→ Flight instruction → Self-Management Support (above)
→ Preventive Medicine → Effective Clinical Management
→ Safe Flight Plan → BPS Plan – PFC II. GERIATRIC SYNDROMES
→ Air Traffic Control Surveillance → Close Follow-up ● Fall
→ Usual care works when plane is about to crash. Designed ● Dementia
FOR CRISIS! ● Depression
3 Biggest Worries about having Chronic Illness (Age 60+) ● Incontinence
1. Losing independence ● Osteoporosis
2. Being a burden to family and friends ● Polypharmacy
3. Not being able to afford needed medical care ● Insomnia
● Constipation (Impaction)
Fundamental Practice Changes to Change Outcomes ● Sarcopenia and Malnutrition
● Interventions A. FALL
→ Focused on guidelines, feedback and role changes can
improve processes Diagnosis
→ Address more than one area have more impact ● Timed Up and Go Test
→ Focus on patient-centered change outcomes → From patient in sitting position → Stand without using arms,
→ e.g. Diabetes: long term care should be evident, and walk 3 meters, turn around → walk back, and sit down.
guideline must be implemented in practice to change → Doctor will time the process and observe of gait abnormalities
outcomes in certain diseases. → Older adult who takes >12 seconds to complete test is at high
● We need to look at causes of diseases. risk for falling
→ Ex: Fall → Caused by Hypoxia, CHF or Medications
Red Flags for Old People
● >75 years
● Needs help with ADLs/IADLs by CCAC or caregiver
● Lives alone
● Falls
● Delirium/confusion
● Incontinence
● >2 admissions to acute care hospital/year
● “Failure to thrive”
● WHEN SEEN: Investigate and give necessary intervention
Comprehensive Geriatric Assessment
Figure 2. Timed Up and Go Test
● Expands scope of interest to include caregiver and
environment |
● Emphasis: optimization of function & increase in life
expectancy |
DE GUZMAN AO, ESTRADA PD, ESTRELLA AV | DURAN, DY ECHO Page 1 of 7
[PREV MED3] 1.01 – Geriatrics (19 AUG 2020) Page 2 of 7
● Functional Reach Test ● MMSE Interpretations
→ Yard stick on wall parallel to floor, at height of acromion of a. MMSE Interpretation via SINGLE CUT-OFF
subject’s dominant arm. Subject with feet distanced apart,
Single Cut Off Abnormal (≤24)
make a fist, forward flex to 90 degrees.
→ Subject will reach forward without taking a step or touching b. MMSE Interpretation via RANGE
the wall
Increased odds of dementia (<21)
→ Measure distance between start and end using head of the
Decreased odds of dementia (>25)
metacarpal of the 3rd finger as reference point.
→ NOTE: Be cautious because patient might fall!
c. MMSE Interpretation via EDUCATION
Abnormal for 8th grade education (21)
Abnormal for high school education (<23)
Abnormal for college education (<24)

d. MMSE Interpretation via SEVERITY


Normal (24-30)
Mild Cognitive Impairment (18-23)
Severe Cognitive Impairment (0-17)

Figure 3. Functional Reach Test e. Clock Drawing Test


Table 2. Results of Functional Reach Test ● Direction: Draw the face of a clock, put all the numbers, set the
Age Men Women clock hands to read ten after eleven
20-40 16.73 in 14.64 in ● Scoring: 0-4 point method is brief, sensitive, and easy to apply
41-69 14.98 in 13.81 in → Draw closed circle 1 point
70-87 13.16 10.47 → Places numbers correctly 1 point
→ Cut off: 13.16 inches for MEN, 10.47 inches for WOMEN → Includes 12 numbers 1 point
→ Places hand correctly 1 point
→ Below means that patient is at RISK OF FALLING
● Mild AD: cholinesterase inhibitor treatment dose, increase over
ASAD CONFERENCE 2013
time (Donepezil 5 mg initially for 1 month then 10 mg thereafter)
● Moderate – Severe: cholinesterase inhibitor best dose + ● Normal at Risk: assess all risk factors and intervene
memantine 5 mg 1st week then 10 mg thereafter appropriately
● Profound AD: continue as long as there are abilities to retain ● MCI: individualize approach
● Mild AD: cholinesterase inhibitor treatment dose, increase over
● In patients >60 years old, what are the benefits of Vit D on time (Donepezil 5 mg initially for 1 month then 10 mg thereafter)
treatment of falls, fractures and mortality? ● Moderate – Severe: cholinesterase inhibitor best dose +
→ Vit D use of around 800-1000 IU to reduce fractures and memantine 5 mg 1st week then 10 mg thereafter
overall mortality in older patients ● Profound AD: continue as long as there are abilities to retain
→ No high-level evidence to support regularly testing older
patients for Vitamin D insufficiency ● What are the benefits and harms of antipsychotics for agitation
in dementia?
B. DEMENTIA → Strong placebo effect explains most of the perceived efficacy,
Diagnosis with antipsychotics providing little additional improvement
● MINI COG – Three item questionnaire for initial screening over placebo on agitation scales (~3 additional points out of
● MOCA – Monreal Cognitive Assessment Tool 144).
● MMSE – Mini-mental State Examination → 50% improvement in behavior occurs in ~46% on
● CDT – Clock drawing Test, supplement to MMSE antipsychotic versus ~33% on placebo
→ Harms are serious (increased death or cerebrovascular
events for somnolence or gait troubles 1 in ~10 or 20, for
each). Antipsychotics should be reserved for cases of severe
aggression and withdrawal attempted as soon as possible.

● Do statins have any effect on cognitive function or dementia?


→ Evidence indicates that statins do not prevent, treat, or cause
cognitive impairment or dementia
→ FDA warning seems to be based primarily on case reports
that may reflect idiosyncratic short-term “fuzzy” thinking
→ Decisions to prescribe statins should not be altered.

Family Meeting
● Allay fears of the Unknown
→ Involve the family in cases that they have a family member
Figure 4. Dementia Screening Results who has a dementia
● MMSE cannot measure Semantic memory and Executive → Address the questions and the needs of the family members
function but CDT can so they are used together ■ “Will I inherit it?”
● MMSE Parts: Orientation, Registration, Attention and ■ “Will they recognize me in the future?”
Calculation, Recall, Language ■ “Will they become crazy? Is it fatal?”
■ “How long will they be around us?”

DE GUZMAN AO, ESTRADA PD, ESTRELLA AV | DURAN, DY ECHO Page 2 of 7


[PREV MED3] 1.01 – Geriatrics (19 AUG 2020) Page 3 of 7
● Handle Stigma Table 4. Management in Depression
→ Not all dementia is Alzheimer (there are other forms of Non-pharmacologic Pharmacologic
dementia like vascular dementia, Parkinson’s dementia, ● Psychotherapy ● Sertraline 50 mg OD
dementia with Lewy body, or your frontal temporal dementia) ● Family support ● Escitalopram 10 mg OD
→ Orient pedigree ● Lifestyle changes ● Mirtazipine 30 mg OD
→ Deterministic vs risk genes ● How effective are depressants for treating depression in the
→ Not communicable (e.g. like TB) elderly?
● Inform about Prognosis → Efficacy of antidepressants in the elderly is inconsistent and
→ There is no cure for AD. Current treatment is focused on may decrease as patients age
cognitive, functional and behavioral intentions. (prevent → 80% to 40% of elderly patients will recover with
progression of the disease but you cannot revert back the antidepressants, with some studies showing no difference
memories of the patient to their usual memory) from placebo response rates
→ The average lifespan of patient with Alzheimer’s disease is 4- → Harms antidepressants are common, with ~20% stopping
8 years after diagnosis. due to adverse effects
→ Some can live for as long as 20 years depending on their age
and health D. INCONTINENCE
● Explain Treatment Diagnosis
→ Treatment is BOTH drug and non-drug therapy ● Ask for symptoms
→ No cure for AD and other dementias Table 5. Signs and Symptoms of Incontinence
→ Manage expectations from treatment Signs and Symptoms
→ Emphasize on the role of families/ caregivers Post-micturition
Storage symptoms Voiding symptoms
→ Cost of drugs symptoms
→ Adherence and persistence ● Altered bladder ● Hesitancy ● Feeling of
→ Side effects, share personal experience sensation ● Intermittency incomplete
→ When to start, on whom and until when? ● Increased ● Slow stream bladder emptying
daytime ● Splitting/ spraying ● Post-micturition
Community Interventions frequency ● Straining dribble
● Nocturia ● Terminal dribble
● Organize support groups
● Urgency
● Let them join a club
● Urinary
● Conduct lay for a on geriatric care and wellness
incontinence

C. DEPRESSION ● Physical Examination


→ Males
Diagnosis
■ Digital Rectal examination to identify prostate size,
● Geriatric Depression Scale contour and consistency for possible BPH as a cause
Table 3. Geriatric Depression Scale
of incontinence
Parameters Yes No
1. Are you basically satisfied with 1
your life?
2. Have you dropped your 1
activities and interests?
3. Do you feel that your life is 1
empty?
4. Do you often get bored? 1
5. Are you in good spirits most of 1
the time?
6. Are you afraid that something 1
bad is going to happen to you?
7. Do you feel happy most of the 1
time?
8. Do you often feel helpless? 1
9. Do you prefer to stay at home 1
rather than going out?
10. Do you feel you have 1
problems with memory than most?
11. Do you think it’s wonderful to 1
be alive now?
12. Do you feel worthless? 1
13. Do you feel full of energy? 1
14. Do you feel your situation is 1
hopeless?
15. Do you feel other people are 1
better than you?
→ A score of > 5 suggests depression
→ A score of > 10 confirms depression

z Figure 5. International Prostate Symptom Score (I-PSS)

DE GUZMAN AO, ESTRADA PD, ESTRELLA AV | DURAN, DY ECHO Page 3 of 7


[PREV MED3] 1.01 – Geriatrics (19 AUG 2020) Page 4 of 7
Table 6. Management in Incontinence Table 8. Management in Polypharmacy
Non-pharmacologic Pharmacologic Pharmacologic Non-pharmacologic
● Lifestyle: no smoking, alcohol ● Men with BPH: 5- ARI ● Medication review (at every office ● Write out schedules
& caffeine (Finasteride or Dutasteride 5 visit and/or after every ● Write out indications
● Pelvic muscle exercises mg OD) to decrease size hospitalization) for each medication
(Kegel’s) ● Alpha receptor blocker ● Eliminate medications with duplicate ● Use pill boxes to track
● Timed voiding (Tamsulosin, Terazosin 400 effects adherence
ug OD) to relieve symptoms) ● Stop ineffective medications or those ● Detailed explanations
● Women: Estrogen cream with sub-optimal therapeutic effect of each medication
● Add new medications one at a time and the indication to
● How effective are alpha-blockers in reducing lower urinary
● Advise: “start low and go slow” when increase adherence
tract symptoms (LUTS) in men with benigh prostatic
starting new medications
hypertrophy?
● Know all non-prescription drugs
→ Effective as first line therapy for LUTS- BPH (herbal medicines, vitamins)
→ Around 1 in 10 will have improved symptoms and/or avoid
symptom progression G. INSOMNIA
→ Around 1 in 50 will experience hypotension or dizziness Diagnosis
→ Mainly indirect comparisons suggest doxazosin and
● Complaint of difficulty initiating sleep, difficulty maintaining
terazosin may be slightly more effective but have increased
sleep, or waking up too early, or sleep that is chronically non-
risk of adverse events
restorative and poor in quality
E. OSTEOPOROSIS → Occurs despite adequate opportunity and circumstances for
Diagnosis sleep
● Bone Densitometry Table 9. Daytime impairment related to nighttime sleep difficulty
■ T-score Daytime Impairment
Patient experiences at least one of the ffg:
>-1: normal ● Fatigue or malaise
● Attention, concentration and memory impairment
-1 to -2.5: osteopenia (low bone mass) ● Social or vocation dysfunction or poor school performance
● Mood disturbance or irritability
-2.5 to -4: osteoporosis ● Daytime sleepiness
● Motivation, energy, or initiative reduction
● Proneness to errors/accidents at work or while driving
● Tension, headaches or GI symptoms in response to sleep loss
● Follow-up/Monitoring: Patients who HAVE NOT sustained a ● Concerns or worries about sleep
fracture (Group Health, 2013) Table 10. Management in Insomnia
→ Frequency of screening by densitometry (costly, no Non-pharmacologic Pharmacologic
evidence for the need to screen often) ● Melatonin 1 tab, 2-3h before
● Avoid caffeine, alcohol and
■ ≥ 65 y/o – screen not less than every 2 yrs bedtime
nicotine
■ 60-64 y/o – if with fracture risk ● Avoid benzodiazepines and
● Avoid heavy meals 2-3h
hypnotics
before sleeping
● Zolpidem 10mg may be given
● Avoid bright light at nighttime
if no response to
● Allot a comfortable sleeping
nonpharmacologic or
environment
melatonin
● Are Z-drugs (zopiclone, zolpidem and eszopiclone) safe
and effective in insomnia?
→ Z-drugs help people fall asleep faster (~13-22 mins) and
perhaps ~5% more sleeping time while in bed.
→ May increase the risk of mild infections (1 in 43 patients)
→ Inconsistent cognitive effects (reduced verbal memory or
attention)
Figure 6. Recommended Screening Interval.
● Is melatonin effective for sleeping disorders?
F. POLYPHARMACY → Quality of melatonin research is poor and at high risk of bias
Diagnosis → If research is believable, melatonin may help people fall
● 4 or more prescription medications or 3 or more new asleep faster (~10 mins) and spend more time asleep (~15
prescription medications in 24 hours mins)
→ 4 meds increase the risk of fall ■ Both of these amounts may be of limited clinical value
→ 5 meds increase the risk of adverse events
Table 7. Signs and Symptoms of Polypharmacy ● For people with primary insomnia (not related to other
Signs and Symptoms of Polypharmacy conditions), can sleep restriction therapy (SRT) improve
● Dry mouth ● Orthostatic hypotension outcomes?
● Tachycardia ● Hypoglycemia → SRT improves time to fall asleep by 12 mins and time asleep
● Confusion ● Congestive heart while in bed by 5-10%
● Diarrhea failure/pulmonary edema → SRT improves sleep for 1 in 2-6 patients compared to sleep
● Constipation ● Flatulence hygiene alone
● Peripheral edema ● Bloating
● Extrapyramidal side effects ● Somnolence
● Syncope ● Lethargy
DE GUZMAN AO, ESTRADA PD, ESTRELLA AV | DURAN, DY ECHO Page 4 of 7
[PREV MED3] 1.01 – Geriatrics (19 AUG 2020) Page 5 of 7

H. CONSTIPATION (IMPACTION) ■ 89 cm (35 in) in nonpregnant women


Diagnosis ■ >102 cm (40 in) in men
● Mid-Arm Circumference
● Rome IV Criteria → NV: 26cm
→ Fulfilled for the last 3 months with symptom onset at least 6
months prior to diagnosis
Table 11. Rome IV Criteria
Rome IV Criteria for Constipation
1. Must have 2 or more of the following in 25% of defecation
● Straining
● Lumpy or hard stools
● Sensation of incomplete evacuation
● Sensation of anorectal obstruction
● Need for manual maneuvers
● Fewer than 3 defecations per week
2. Loose stools rarely present without the use of laxatives
3. Insufficient criteria for IBS
Table 12. Pharmacologic Interventions for Constipation (Impaction)
Drug Dose & Time of Onset Adverse Effect
Polyethylene 17 g/day (1 sachet) Minimal bloating/gas
glycol | 24-48 h
1 tsp 1-3x/day Bloating, abdominal
Psyllium
12-24 h distension
Figure 7. Steps in obtaining Mid-Arm Circumference. Normal value is 26 cm.
Senna 15-30 mg/day Abdominal pain
Diarrhea, abdominal ● Calf Circumference
Bisacodyl 5-15 mg/day → <30.5 cm
pain
Lactulose 15-30 mg/day Bloating, cramping ■ Suggests protein energy malnutrition in elderly
● Pinggang Pinoy (Filipino Plate) for Senior Citizens
Table 13. Non-pharmacologic Interventions for Constipation (Impaction)
Intervention Intake Notes
Water Improves stool frequency if on a
1.5 to 2 L
supplementation high-fiber diet
Intake should be slowly increased
Daily Fiber 20 to 35 over several weeks to decrease
Intake g/day adverse effects (flatulence,
abdominal cramping, bloating)
● Is docusate effective in prevention and treatment of
constipation?
→ Appears similar to placebo in increasing stool frequency
Figure 8. Pinggang Pinoy for Older Persons ≥60y/o. From FRNI (2016)
→ Inferior to other products for treating functional, medication-
induced, or post-op constipation Management of Obesity in elderly
● Achieve weight loss of 3-5%
I. SARCOPENIA & MALNUTRITION
→ if BMI is 30 and above or
Screening tools to assess geriatric nutritional status → if BMI is 25-29, and has 1 risk factor
● Mini Nutritional Assessment (MNA) ■ HPN
→ Used if presenting problem is loss of appetite or BMI of 18 ■ DM
| ■ Dyslipidemia
→ Screening score for the SHORT FORM ● Decrease caloric intake of 500 calories and moderate intensity
■ 12-14 – normal nutritional status exercises tailored to condition of patient
■ 8-11 – at risk of malnutrition Sarcopenia Algorithm in the elderly
■ 0-7 – malnourished
→ Screening score for the LONG FORM
■ 24-30 – normal nutritional status
■ 17-23.5 – at risk of malnutrition
■ <17 – malnourished
● Body Mass Index
o Kg/m2
Table 14. BMI Classification
Classification WHO ASIA-PACIFIC
Underweight <18.5 <18.5
Normal 18.5 – 24.9 18.5 – 22.9
Overweight 25 – 25.9 23 – 24.9
Obese ≥30 ≥25
● Waist Circumference
→ Midpoint between the lowest rib and the iliac crest
→ Optimal cut-off for ≥70 y/o
■ Males: 109 cm (43 in) Figure 9. EWGSOP-suggested algorithm for sarcopenia case finding in older
■ Females: 98 cm (38 in) individuals.
→ Values that increases risk in diabetes, hypertension, and
cardiovascular diseases
DE GUZMAN AO, ESTRADA PD, ESTRELLA AV | DURAN, DY ECHO Page 5 of 7
[PREV MED3] 1.01 – Geriatrics (19 AUG 2020) Page 6 of 7
→ (Left side) If gait speed is >0.8 m/s, measure grip strength Warning Signs
using a Jamar Hand Dynamometer | ● Low level of activity
■ Grip strength NV ● Exhaustion
− Males: >30 kg ● Unintentional weight loss
− Females: >20 kg ● Atypical mood swings
→ (Right side) If gait speed is ≤0.8m/s, measure muscle mass Risk Factors
using Dual X-ray Absorptiometry or DEXA scan ● Chronic diseases
(appendicular muscle mass) | ● Physiologic impairment
■ Muscle Mass NV
− Males: >7.23 kg/m2 A. OTHER ASSESSMENTS & ADVOCACIES
− Females: >5.67 kg/m2 Environmental Assessment
Interventions in elderly with Sarcopenia ● Carpets, slippery floor, reach
● Let caregivers look into their home environment for safety and
risks of falls and other accidents
Social Assessment
● Caregiver support
● Educate them on older person’s rights
Modified Caregiver Strain Index (MCSI)
Apart from the elderly, our other patient are the caregivers who will experience
fatigue at some point.

Figure 10. Interventions in Elderly with Sarcopenia


● Common Intervention
→ Diet, Exercise
■ No evidence for using hormonal replacement
therapy, drugs, supplements
● Protein Requirement
→ 1 g/kg/day
■ Higher than the level currently suggested by the
Institute of Medicine (0.8 g/kg/day) Figure 11. Modified Caregiver Strain Index (MCSI)
→ Ingested amino acids stimulate muscle protein synthesis and ● Ask the caregiver all the questions listed in the index and
maintain muscle mass in elderly categorize whether s/he experiences it whether madalas,
● Is testosterone supplementation effective and safe in paminsan-minsan or halos hindi. Total the score.
androgen decline in aging males? ● MCSI scoring
→ In older men, testosterone increases some muscle strength → ≤ 23 – normal
by 7% → 24-28 – predisposition to strain
→ Moderate improvements in erectile function and libido → ≥ 29 – severe caregiver strain
→ Adverse events: cardiovascular in those with higher risk RA 9994 Expanded Senior Citizens Act of 2010
→ Many results are inconsistent, at high risk of bias, and difficult It is also important to let the elderly know of their rights.
to quantify in real world application 1. 20% discount and VAT exemption for:
● Purchase of medicine invluding the flu and pneumococcal
J. FRAILTY vaccines and other essential medical supplies, accessories
End result of unmanaged sarcopenia and equipment
Clinical Syndrome ● Professional fees of MD in all private hospitals, medical
● Including 3 or more of the following: facilities, OPD and home health care
→ Unintentional weight loss of >10lbs in the previous year ● Medical and dental services, diagnostic and laboratory fees
→ Self-reported exhaustion ● Actual transportation fare for land travel and domestic air
→ Weakness (as measured by grip strength in the lowest 20% transport and sea shipping vessels
by gender and BMI) ● Hotel and lodging establishments, restaurants
→ Slow walking speed (lowest 20% by gender & height) ● Funeral and burial services
→ Low physical activity (as measured by kcal; lowest 20%) ● Admissions to cinema
2. Payment of individual income tax below minimum wage
Red Flags 3. 5% on water and electricity registered on their name not
● Slowness in walking exceeding 100 kwh and 30 mm3
● Sarcopenia 4. Educational assistance to pursue secondary, tertiary,
● New onset depression vocational and technical courses and short-term courses in
● > 5 chronic medication schools
5. Exemption from training fees for socioeconomic programs
6. Death benefit of Php 2000 to nearest surviving relative
7. Express lanes in establishments
8. Mandatory Philhealth coverage for indigenous senior citizens
DE GUZMAN AO, ESTRADA PD, ESTRELLA AV | DURAN, DY ECHO Page 6 of 7
[PREV MED3] 1.01 – Geriatrics (19 AUG 2020) Page 7 of 7
Wellness
Advocate wellness for older persons.
Laboratory Tests
● FOBT
→ 60-75 y/o – annually
→ 76-85 y/o – if with risk
→ >85 y/o – do not screen
● Bone Densitometry
→ ≥ 65 y/o – screen not less than every 2 yrs
→ 60-64 y/o – if with fracture risk
● Pap Smear
→ 60-65 y/o – every 5 years
→ > 65 y/o with 3 adequate screening – do not screen
● Mammography
→ 60-74 y/o – every 2 yrs
→ ≥ 75 y/o – no recommendation
● If at risk, do FBS, lipid profile, ECG

Vaccines
● Pneumococcal vaccine (PCV13 followed by PPSV23 after 6-
12 mos)
→ If PPSV23 was administered first, give PCV13 after 1 yr
● Influenza vaccine annually
● Herpes Zoster vaccine single dose
● Tdap
→ Tdap, TD, Tt – 3 doses (1 Tdap first then 2 Td) at 0, 1, 6-12
mos
→ Booster every 10 yrs
Optimizing the Role of Physicians
● Increase collaboration among physicians and other healthcare
providers around the current health initiative within their
community
● Advocate for rights of older persons
● Participate in Comprehensive Geriatric Assessment
● Research on Geriatric Medicine & Gerontology

REFERENCES
Maglonzo, E.I.Y., (2020), Jeopardy in the Care of Older Persons [PowerPoint
Presentation]. Manila, Philippines: Faculty of Medicine and Surgery,
University of Santo Tomas, MED 2
A2020 trans

DE GUZMAN AO, ESTRADA PD, ESTRELLA AV | DURAN, DY ECHO Page 7 of 7


PREVENTIVE MEDICINE 3 AY 20-21

Ma. Victoria Pilares-Cruz, MD, DPAFP, FPAFP & Ma. Teresa Tricia G. Bautista, MD, FPAFP, FPCGM, MHA 26 AUG 20

TABLE OF CONTENTS Table 1. Advantages and Disadvantages of Home Care


I. HOME CARE ....................................................................................... 1 Advantages Disadvantages
A. ROLE OF THE PHYSICIAN ............................................................ 1 ● Less costly ● Time consuming
B. CONCEPT or MODELS OF CARE .................................................. 1 ● Enhances physician- ● Minimal Resources
C. UST FAMILY HEALTHCARE PROGRAM ....................................... 1
patient relationship
D. ISSUES IN HOME CARE ................................................................ 2
E. HOME CARE GOALS ..................................................................... 2 ● Continuity of care
F. BASIC HOME CARE SKILLS.......................................................... 2
II. DIFFICULT CLINICAL ENCOUNTERS................................................. 3 A. ROLE OF THE PHYSICIAN
A. COMPONENTS OF A DIFFULT CLINICAL ENCOUNTER .............. 3 ● Physician is the overall manager
B. CASE SIMULATION ....................................................................... 4 ● Goals:
C. Intervention to Patient Adherence ................................................... 5
D. BATHE Technique .......................................................................... 5
→ Appropriate home care assessment skills
E. EMPATHY vs. SYMPATHY............................................................. 5 → Assessment of family caregivers and resources
F. 10 Useful Coping Skills for Physicians ............................................ 5 → Knowledge of community resources and home care
technology
MUST KNOW BOOK PREVIOUS TRANS → Ability to integrate home and hospital care
→ Lead the home care team
This trans is based on the PPT of Dr. Cruz’s and Dr. Bautista’s lectures, American Medical Association: Roles of Physician
supplemented with A2020 Trans. In their “Guidelines for the Medical Management of Home Care
I. HOME CARE Patients”
1. Main management of medical problems
● National Association for Home Care
2. Identification of homecare needs of the patient
→ Approximately 7.6 million people in the US require some
3. Establishment or approval of a plan of treatment with both short
form of home health care
and long-term goals
→ More than 20,000 home health care providers exist today
4. Evaluation of new, acute or emergent medical problems
→ Almost two-thirds (62.3%) of home health care recipients
supplied by other team members
are women
5. Provision for the continuity of care to and from all settings
→ More than two-thirds (68.6%) of home health care
(Institution, home and community)
recipients are over age 65
6. Communication with the patient and other team members
● Home Care
7. Participation, as needed, in homecare/family conferences
→ Provision of comprehensive health care, services are
8. Reassessments of care plan and outcomes of care
provided in places of residence
9. Evaluation of quality of care
→ Goals
10. Documentation in appropriate medical records
■ To promote, maintain or restore health of the patient
11. Provision of 24-hour on-call coverage by a physician
■ To minimize the effect of disability
→ Allows a person with special needs to stay in their home B. CONCEPT or MODELS OF CARE
→ Might be good for people who are getting older, are ● System I (Same Doctor)
chronically ill, recovering from surgery, or disabled → Doctor orders home care service
● Different Aspects of Care | → Then, work cooperatively with the Home Care Program
→ Preventive (FHCP) staff to plan and provide necessary home care
→ Therapeutic ● System II (Different Doctors)
→ Diagnostic → A physician of the Home Care Program (FHCP) provides
→ Rehabilitative home care
→ Long term maintenance → He is the overall manager but plans together with the
● Home care is defined as diagnosis, treatment, and ongoing patient and other FHCP staff.
monitoring of the patient in the home.
→ Usually less expensive, more convenient, and just as C. UST FAMILY HEALTHCARE PROGRAM |
effective as care you get in a hospital or skilled nursing Assessment
facility (SNF) ● Discharge Planning
→ House calls: episodic care intervention or aid to practice → Prepared by the discharge coordinator who will identify
● Skilled home health services include: the patient’s need for continued care
→ Wound care for pressure sores or a surgical wound → Importance: Allay patient’s and family’s anxiety about
→ Patient and caregiver education post-hospital care
→ Intravenous or nutrition therapy → SCREEN who needs to be admitted to the program
→ Injections based on the ff:
→ Monitoring serious illness and unstable health status ■ Medical Merit
● Home Care Services Four Point Program include: ■ Psychosocial merit
→ Comfortable transition to Home ■ Cooperative Patient / Family
→ Maintenance of optimal clinical condition ■ Residence location (5km radius from the base
→ Ongoing Health Education and Reinforcement of hospital)
Learning ■ Positive economic balance
→ Continuity of Care

DE GUZMAN AO, ESTRADA PD, ESTRELLA AV | GABITO, GALLANO, GARCES, GARCIA EJ, GARCIA IC Page 1 of 5
[PRV-FH] 1.02 – HOME CARE & DIFFICULT CLINICAL ENCOUNTERS (26 AUG 2020) Page 2 of 5
Admission F. BASIC HOME CARE SKILLS |
● Checking the referral form Range of Motion Exercises
→ Purpose: for assessment, preparation, management ● To maintain muscle tone and joint mobility |
planning, and proper education of the patient and family ● Joint movements performed by a patient / caregiver to maintain
members muscle tone & joint mobility that has been lost through disease,
● Communication with the referring doctor injury, or lack of use
→ Assess extent of involvement ● Joint that has not been moved sufficiently
● Meeting the patient and the family → Can begin to stiffen within 24 hours
→ Establish rapport ● Active, Active-Assistive, Passive ROM
→ Know patient and family’s expectation
→ Do goal setting Hygiene
Planning ● Complete Bed Bath
→ Proper draping
● Family Health care Plan → Patient handling
→ Backbone of the patient medical record → Systematic and proper techniques from face down
→ Assess the following: → Proper grooming
■ Medical history ● Back rub
■ Family assessment ● Oral care
■ Environmental condition
■ Socioeconomic factor Elimination and Perineal Care
■ ADLs ● Materials needed for this care are:
− + instrumental ADLs esp. for geriatrics
● Data collection
→ Identify patient’s problems, goals and expected
outcomes of health care service
Implementation
● Home Care Visit
→ Each home visit shall become a means of working
towards the end goal
Evaluation
A continuing process
● 2 Methods of Continuing Care: Figure 1. Materials for elimination and perineal care
→ Formal conferences
■ Involve all the members of the team Transfers
■ Patient’s progress is evaluated, and modifications ● Importance
are made → Restores muscle tone
→ Informal conferences → Stimulated respiratory system
■ Impromptu chat on the phone between health care → Stimulates circulatory system
team members and personnel regarding a particular → Improves elimination
aspect of patient care ● Bed to Chair techniques
■ Must be documented → One-person assist
D. ISSUES IN HOME CARE → Two-person assist
● Legal Issues
→ Home Care Policies must be properly drafted
→ Documentation of all examination, conversations and
care rendered
→ Constant surveillance and attention to quality care
● Ethical Issues
→ Informed consent
● Financial Issues
→ Physician should discuss the financial agreement
E. HOME CARE GOALS
● Feeding Figure 2. Bed to Chair transfer techniques.
● Bed Sore Care (L→R one-person assist, two-person assist)
● Rehabilitation ● Moving Patient in Bed techniques
→ Physical, pulmonary → To one side of bed
● Reintegration into Society → Up in bed
→ maximize patient capacity → Using a draw sheet
● Counseling → Turning the patient (Logrolling)
→ Genetic, Primary care
● Spiritual Care
● Patient and Family Education
→ Teach patient how to participate in the management
→ Self-monitoring, sexual activity, energy conservation, CPR
→ Minimal cost strategy
→ Prevention - progression of disease

DE GUZMAN AO, ESTRADA PD, ESTRELLA AV | GABITO, GALLANO, GARCES, GARCIA EJ, GARCIA IC Page 2 of 5
[PRV-FH] 1.02 – HOME CARE & DIFFICULT CLINICAL ENCOUNTERS (26 AUG 2020) Page 3 of 5

II. DIFFICULT CLINICAL ENCOUNTERS


Physicians often find themselves in difficult clinical encounters. These encounters
often leave the physician feeling frustrated. The patient may also be dissatisfied
with these encounters because of unmet needs, unfulfilled expectations, and
unresolved medical issues. It is therefore not enough to be clinically competent if
we want better health outcomes. If we cannot engage with our patients and their
significant others, there must be certain skills to acquire. What is the use of our
expertise if we cannot gain the patient’s trust or if we cannot convince them to
undergo our treatment plan? As physicians, it is imperative to equally nurture our
people skills and the art of medicine.

● Difficult clinical encounters


→ Attributable to factors associated with physician, patient,
situation or a combination
→ Personalities of individual patients and doctors and the
range of personal, social, and professional expectations
that each brings to the interaction always have the
potential to turn the medical encounter into a difficult one
→ In the emergency department where patients are
frequently hostile, angry, combative, or abusive if special
care is not taken to avoid potential problems
→ Also be replicated when there are sensitive concerns to be
Figure 3. Moving Patient in Bed. (Upper L→R: to one side of bed, up in bed;
dealt with, matters of decision making, and delivering bad
Lower L→R: Using a draw sheet, turning the patient) news.
● Other variations (please refer to the images below) A. COMPONENTS OF A DIFFULT CLINICAL ENCOUNTER
→ Supine ● Physician characteristics
→ Side-lying → Pre-existing bias
→ Prone ■ Negative bias towards specific conditions
→ Poor communication skills
→ Situational stressors
→ Angry or defensive
→ Fatigue or harried
→ Dogmatic or arrogant
● Situational issues
→ Time pressure during visits
→ Patient and staff conflicts
→ Complex social issues
→ Language and literacy issues
→ Environmental issues
→ Multiple people
→ Breaking the bad news
● Patient characteristics
→ Personality disorders
→ Multiple and poorly defined symptoms
→ Non-adherence to medical advice
→ Self-destructive behaviors
→ Angry
→ Manipulative
→ Somatizing
→ Demented
Profile of Difficult Patients
These patients were less likely to fully trust or be satisfied with their
physician, and they were more likely to report the worsening of
symptoms 2 weeks after their visit.
● Had >5 symptoms
● Severe symptoms
● Take more medications
● Underlying mental health issues
● They are older
● Different sexual orientation from the provider
Figure 4. Other variations in positioning (supine, side-lying & prone)
● More acute, chronic psychosocial problems
● More frequently divorced or widowed
● From a social class
Physician and Practice Characteristics Associated with HIGH
Frustration
Doctors play a crucial role as well. Researchers found that doctors
involved in difficult encounters were generally less experienced and
had fewer psychosocial skills. When you have someone who has

DE GUZMAN AO, ESTRADA PD, ESTRELLA AV | GABITO, GALLANO, GARCES, GARCIA EJ, GARCIA IC Page 3 of 5
[PRV-FH] 1.02 – HOME CARE & DIFFICULT CLINICAL ENCOUNTERS (26 AUG 2020) Page 4 of 5
15-20 years of experience, they have learned how to deal with these ● Use an open-ended question.
patients. Indeed, experience comes with time and practice. → “You seem quiet today. Is there a reason for this?”
● Age & experience → “You seem sad today. Are you depressed?”
→ <40 y/o ● Encourage a productive clinic visit
→ Less experience → “Is there someone close to you who can help to make
● Stress level: Above average decisions with regards to health care?”
● Specialty: Medicine subspecialty ● Identify cause of silence
● Hours worked per week: >55 hours ● Possible fear of authority figure
● Patients with psychosocial problems: more than average ● Recognize barriers
● Patients with substance usage: more than average → language, personality, cultural
● Medical reasons
B. CASE SIMULATION → hearing loss, medications which cause drowsiness,
DEMANDING PATIENT concurrent health problem
● Previous negative experience with a health care provider or
| CASE:
A top executive sees you for the first time due to head and neck pain service
that started yesterday. There were no red flags such as vomiting, HARASSING/SEDUCTIVE PATIENT
dizziness, blurring of vision and the like. History and PE were
otherwise normal. He insists on having a Cranial CT scan done to | CASE:
make sure that it is not worrisome because he cannot afford to be How to handle a patient whose actions, words, looks or gestures
sick. How should you approach the situation? insinuate malice?
● Explain the situation to patient Patient to provider sexual harassment is an underreported
● Address the patient’s doubts regarding your ability phenomenon. A rough estimate of 67% is experienced in the
● Explore the patient’s concerns and fears workplace, with female providers experiencing more common than
→ “You seem doubtful with the plan that I am suggesting. Is males (40%).
there anything that is worrying you?” ● Be pleasant but firm. Focus on the problem at hand, be
→ Consider negative past experience with a doctor or fear of straightforward.
dying after going through a relative’s death ● This may be a psychological issue in the patient’s history which
● Empathize needs exploration.
→ “I realize this symptom is alarming because your uncle ● Emphasize your professional role.
who recently passed suffered from aneurysm.” ● You may opt to have a secretary, or another person at hand to
● Last resort is to suggest a second opinion. dissipate the awkward mood.
● If truly uncomfortable, you may refer the patient to another
ANGRY PATIENT provider.
● A romantic relationship will lead to
| CASE: → Loss of objectivity
We have a patient who waited for you for four hours in your clinic. For → Imbalance of authority
some reason you are extremely late. The patient was not feeling well
and drove two hours to see you. Finally, you arrive, but he is really EMOTIONAL PATIENT
mad. How would you attend to the patient?
| CASE:
● Allow complete angry outburst. A patient breaks down in front of you.
● Make a conscious effort to slow down response. Do not reply
immediately. Pay special attention to the speed and volume of ● Use active listening skills
your speech. ● Recognize the patient’s emotions
● Validate the frustration with empathic comments: ● Know the value of a good pause.
→ “I understand that you are upset at having to wait, and I → Let the patient drink a glass of water, hand a tissue, ensure
apologize for the delay.” privacy
→ “I can understand why you are upset. I appreciate your ● Explore and address psychological issues raised
waiting for me.” ● Give time to ventilate, validate and process the emotional
● Involve the patient in the outcome. Make a compromise, find a process.
way to console, such as a peace offering like how one will make ● Only when the patient is calm, is the time to process the thoughts
it up to him. and feelings, together with the patient.
→ If one feels angry with what the patient is telling you, learn ● Encourage open communication without being judgmental
to disengage. (“time out”) SOMATIZING PATIENT
● Provide the opportunity to discuss his concerns.
→ “How can I help you today?” | CASE:
You have a 34-year old who has been coming to your clinic in the
SILENT/DETACHED PATIENT past months due to unexplained paresthesia. Two weeks ago, she
was complaining of an upset stomach and gas. She has been
| CASE: complaining of chronic and intermittent headaches. All tests have
This time you are confronted with two brothers. The younger 18-year turned out normal. You are the third doctor she consulted in a span
old sibling was complaining of flank pains. You sensed him to be of 4 months.
unusually quiet and seemingly detached. You can’t elicit a reliable ● Collecting a detailed history of physical symptoms can help
history.
the patient feel that you are listening to him/her
● In this context, the adolescent may appreciate that he tells his → A detailed review of psychiatric symptoms should be
story in private. Ask for his brother to step out for a while, and deferred for later in the examination
that he will be called in at the conclusion of the consultation. → Asking questions about psychiatric symptoms early on
● Establish rapport in order to gain confidence. could lead to further resistance by reinforcing negative
● Explain the importance of sharing information in order to help the preconceptions that the patient may have towards mental
patient. illness
DE GUZMAN AO, ESTRADA PD, ESTRELLA AV | GABITO, GALLANO, GARCES, GARCIA EJ, GARCIA IC Page 4 of 5
[PRV-FH] 1.02 – HOME CARE & DIFFICULT CLINICAL ENCOUNTERS (26 AUG 2020) Page 5 of 5
● Explicitly express empathy towards physical symptoms Note: The following notes are purely from A2020 Trans.
throughout the history
→ To acknowledge any real suffering and contradict the C. Intervention to Patient Adherence
notion that the symptom is imaginary ● Simplifying regimen characteristics
● Ask, “how has this illness affected your life?” ● Imparting knowledge
→ This question helps make the connection between the ● Modifying patient beliefs
patient’s physical state and milieu ● Patient communication
● Explore the response to the previous questions ● Leaving the bias
→ If there is somatization, you should assist in reversing the ● Evaluating adherence
arrow of causation. D. BATHE Technique
→ Expand upon it to elicit a detailed history picking up social
● 5-step technique is designed to help physicians uncover patient’s
stressors
emotional issues quickly during an encounter
● Conducting further physical and cognitive examination
● 1st 4 letters of BATHE Acronym prompt physician to ask
→ Shows patient you are not ignoring complaints
questions that elicit the context of visit
● Educate the patient mind and body are connected.
● Final Step = EMPATHY
→ Emotions affect how body reacts physically
● BATHE Technique:
→ “My headache hurts more at the work than at the beach”
→ B – assess the Background situation
or “when I am nervous my heart beats faster”
→ A – assess the patient’s Affect
● Elicit feedback and questions from the patient
→ T – determine the problem that is most Troubling
● Discuss your treatment plan with the patients
→ H – evaluate how the patient is Handling the problem
→ With confirmed somatization, the patient may require
→ E – convey Empathy
psychiatric care
● Address the issue directly
→ “I noticed that you have seen several physicians and have E. EMPATHY vs. SYMPATHY
had extensive medical tests to try to uncover the cause of
Table 2. Empathy vs. Sympathy
your symptoms. I recognize that the symptoms are a real
EMPATHY SYMPATHY
difficulty for you, but I believe that these tests have ruled
Definition Understanding what Acknowledging
out any serious medical problems.” others are feeling another person’s
UNHYGIENIC/MALODOROUS PATIENT because you have emotional hardships
● Although commonsense tells us that we should be prudent and experienced it and providing
yourself or can put comfort and
patient with these patients it is still a very real struggle
yourself into their assurance
● Do not show dislike
shoes
● Carry on with the interview and treat the problem Relationship Personal Understanding the
● Do your best in conducting interview! understanding experience of others
● Educate the patient of caregiver on the importance of hygiene Nursing context A doctor relating Doctors comforting
● Explore possible reasons for it with a patient the patient or their
● May be a sign of caregiver neglect because he or she families
FREQUENT “FLIER” PATIENT | has been in the
similar situation
● Begin by acknowledging the pattern of frequent visits Scope Personal: can be From either one to
● May be lonely, dependent or too afraid/embarrassed to ask one to many in another person or
questions they really want answered some circumstances one to many
● Identify the underlying reasons for the frequent visits:
→ Need to talk F. 10 Useful Coping Skills for Physicians
→ Need for reassurance ● Allow patients to vent their feelings
→ Need for relief from chronic pain ● Strengthen your communication skills
→ Concern about undiagnosed symptoms ● Try not to judge
● Become a more effective history taker
NON-COMPLIANT PATIENT | ● Remain calm and confident
● Factors: ● Be patient
→ Patient – knowledge, motivation, health belief model ● Be proactive
→ Disease – chronicity, disability, # of symptoms ● Understand your own strengths and vulnerabilities
→ Regimens - # of drugs, side effects, dosing ● Become an enabler
→ Physician – personal relationship & meeting expectation ● Respect your patients
IN SUMMARY,
● To overcome difficult patient encounters, one should use END OF TRANSCRIPT
empathic listening skills and non-judgmental caring attitudes
● Attempt to evaluate for underlying psychiatric or medical REFERENCES
disorders for previous or current physical and social
constraints Bautista, TTG, (2020), Difficult Clinical Encounters [PowerPoint Presentation].
Manila, Philippines: Faculty of Medicine and Surgery, University of Santo
● Establish clear boundaries
Tomas, MED 2
● Adhere to a patient-centered communication to agree upon Cruz, MVP, (2020), Home Care: The Heart of Practice [PowerPoint Presentation].
a management plan Manila, Philippines: Faculty of Medicine and Surgery, University of Santo
● Timing of visits and duration and expected conducts must Tomas, MED 2
be discussed and negotiated
● Understanding and managing factors contributing to a
difficult encounter leads to a more satisfactory experience
for the physician, patient and his loved ones

DE GUZMAN AO, ESTRADA PD, ESTRELLA AV | GABITO, GALLANO, GARCES, GARCIA EJ, GARCIA IC Page 5 of 5
PREVENTIVE MEDICINE 3 [FAMILY HEALTH] AY 20-21

Care of the Dying


Assoc. Prof. Ma. Victoria Pilares-Cruz, MD, DFM, FPFAP
1.03
26 AUG 20

TABLE OF CONTENTS II. COMMUNICATION


I. CARE OF THE DYING ............................................................................1
A. Issues in Terminal Illness ...................................................................1 B. Communication Problems
II. COMMUNICATION ..................................................................................1
B. Communication Problems ..................................................................1 ● The Physician
C. Bad News ...........................................................................................1 → Feeling helpless withdrawal from the patient
D. The Six Steps of SPIKESS ................................................................2 ■ Occurs when the physician knows that his/her patient
E. Principle of Informed Consent and Patient Autonomy.......................3 is dying and starts to feel helpless about it |
A. Benefits of Disclosure (Nice to Know) ...............................................3
B. Stages of Death and Dying (Stages of Grief) ....................................3 → May lose enthusiasm for care or use
C. Psychosocial Care .............................................................................3 → May use closing lines of communication
III. MANAGEMENT OF SYMPTOMS (SYMPTOM CONTROL) ..................4 ■ e.g. “don’t worry” (translates to “don’t bother me”) or
A. Pain ....................................................................................................4 “everything will be alright”
B. Anorexia .............................................................................................5
C. Nausea and Vomiting .........................................................................5
→ It is not appropriate to “break a bombshell” and walk away
D. Constipation .......................................................................................5 from the scene.
E. Dyspnea .............................................................................................6 ● The Patient
F. Cough .................................................................................................6 → Usually have an idea of what they could be suffering from.
G. Hiccups ...............................................................................................6 ■ Although are unlikely to initiate discussions
H. Bed Sores...........................................................................................6
I. Mucositis ............................................................................................7
→ There could be periods of excessive denial.
J. Urinary Incontinence ..........................................................................7 → For those patients who have accepted the diagnosis, they
K. Insomnia .............................................................................................7 just want their physicians to be with them to “WATCH
L. Confusion ...........................................................................................7 WITH ME” |
IV. HOSPICE CARE AND END-OF-LIFE CARE ..........................................7
A. Hospice Care .....................................................................................7 | Disclosure: Don’t Tell!
B. Palliative Care ....................................................................................7 ● In North America, principles of informed consent, patient
C. The Two Roads to Death ...................................................................7 autonomy and case law have created clear ethical and legal
D. Medications ........................................................................................7 obligations to provide patients with as much info as they
E. Advance Care Planning .....................................................................7
F. Advance Directive ..............................................................................7 desire about their illness and treatment.
V. MANAGING DEATH AND BEREAVEMENT CARE ................................8 ● Physicians may not hold medical information even if they
A. End of Life ..........................................................................................8 suspect it will have a negative effect on the patient. Yet, a
B. Bereavement ......................................................................................8 mandate to disclose the truth without regard or concern for
END OF TRANSCRIPT .......................................................................................8
REFERENCES ....................................................................................................8
the sensitivity with which it was done and the obligation to
APPENDIX ..........................................................................................................8 support the patients and assist them in decision making can
result in the patient being upset as if they were lied to.
● i.e., “do not reveal the cancer diagnosis to the patient”, “do
MUST KNOW MENTIONED BY DOC PREVIOUS TRANS not tell my wife or husband because s/he won’t handle it”
Dealing with Family Collusion
● Occurs when the family would want to conceal information
I. CARE OF THE DYING
from the patient or the other way around. |
● At one point, every physician or clinician will be faced with the ● How do you deal with family collusion?
following questions: | → Acknowledge that they (the family) know the patient better.
→ How can I be most helpful when hope appears lost? → Find out the reason for keeping the news from the patient
→ What is the best way to deliver news of a terminal → Determine the cost of keeping the news from the patient.
diagnosis? → Ask for permission to speak to the patient alone.
→ Should you asses someone with depression when he or → Acknowledge any identified emotional or psychological
she is terminally ill? distress in the family (e.g. guilt, anger, anxiety,
→ How can you develop a reasonable and thoughtful plan for helplessness)
end of life care? → Reassure the family that you will update them with regard
A. Issues in Terminal Illness to the details of your conversation with the patient.
Conspiracy of Silence
● In the management of terminally ill patients, health professional
needs many skills: | ● When you conspire with the family, you are withholding
→ The ability to deliver bad news information that really belongs to the patient.
→ The knowledge to provide appropriate optimal end-of-life ● Under such condition, patient becomes more and more isolated
care → Leads to a great deal of tension because of trying to keep
→ The compassion to allow the person to retain his or her information from one another
dignity → Decrease in the quality of relationship (which important in
● Stages of care of the dying: | the time remaining)
→ Communication C. Bad News
■ Disclosure ● Any information which adversely or seriously affects an
■ Stages individual’s view of his or her future
→ Management of Symptoms ● How bad news is discussed can affect: |
■ Pain control → The patient’s comprehension of the information
→ Hospice care → The satisfaction with medical care, the level of
→ End-of-life hopefulness and subsequent psychological adjustments
→ Bereavement Care
DE GUZMAN AO, ESTRADA PD, ESTRELLA AV | Elazegui, Encinares Page 1 of 8
[PREV MED3 – Family Health] X.02 – Care of the Dying (26 August 2020) Page 2 of 8
Breaking Bad News → Before discussing medical information, the clinician uses
● How do you properly break the bad news? open-ended questions to create a reasonably accurate
→ Respond to patients’ emotional reactions picture of how the patient perceives the medical situation.
→ Involve the patient in decision making (particularly the ■ e.g. “What have you been told about your medical
forms of treatment) situation so far?” or “What is your understanding of
■ Disclose the information first in order to do this. the reasons we did the MRI?”
→ Deal with the stress created by patients’ expectations for → Based on this, you can correct any misinformation and
cure tailor the bad news to what the patient understands
→ Involve multiple family members during the disclosure. ● It can also accomplish the important task in determining if the
● Most of the time, there is a dilemma of how to give hope when patient is engaged in any variation of illness denial like: |
the situation appears bleak. → Wishful thinking
● The biggest concern in breaking the bad news: → Omission of the essential but unfavorable medical details
→ Physicians, usually young doctors, may feel of the illness
uncomfortable in delivering the bad news. → Unrealistic expectations of treatment
→ May either avoid discussing distressing information such | CEA for Counselling Skills – C: Catharsis
as poor prognosis or start conveying unwarranted ● Can be used for the following questions:
optimism to the patient → What came to your mind when you started feeling your
→ Majority of the problems with breaking the bad news symptoms? (Ano ang naisip mo noong nakaramdam ka
are comprised of the following (based on a survey): ng sakit?)
■ “How to be honest with the patient and not destroy → What feelings came out when these thoughts came to
hope” (50%) your mind? (Ano ang naging damdamin mo noong naisip
■ “Dealing with the patient’s emotions” (25%) mo ang mga ito?)
■ “Finding the right amount of time” (10%) → What consequence of your illness makes you feel this way
D. The Six Steps of SPIKESS the most? (Ano ang pinakanakakatakot na maaring
mangyari dahil sa sakit mo?)
● Goal: Enable the clinician to fulfill the foremost important
objectives of the interview disclosing bad news: | Obtaining the Patient’s INVITATION
→ Gather information from the patient ● Some patients desire full information about their condition, but
→ Transmit medical information some do not.
→ Provide support ● Discussing information at a time of ordering tests can cue the
→ Elicit patient’s collaboration in developing a strategy or physician to plan the next discussion with the patient.
treatment plan for the future. → “How would you like me to give the information about the
● Oncologists and medical students that have been taught the test results? Would you like me to give you all the
protocol have reported increase confidence in their ability to information or sketch out the results and spend more time
disclose unfavorable medical information to patients. | discussing the treatment plan?”
● Basic steps of SPIKESS: | Giving KNOWLEDGE and Information to the Patient
→ S – SETTING UP the Interview
● Warning the patient that bad news is coming may lessen the
→ P – assessing the patient’s PERCEPTION
shock following the disclosure. This may also facilitate
→ I – obtaining the patient’s INVITATION
information processing. |
→ K – giving KNOWLEDGE and information to the patient
→ “Unfortunately, I’ve got bad news to tell you” or “I’m sorry
→ E – addressing the patient’s E•MO•TIONS with emphatic
to tell you that…”
responses
● Giving medical facts, the one-way part of the physician-patient
→ S – STRATEGY and SUMMARY
dialogue, may be proved with a few simple guidelines:
SETTING UP the Interview → Start at the level of comprehension and vocabulary of
● Unless there is a semblance of privacy and the setting is the patient |
conducive to an undistracted and focused discussion, the goals → Try to use nontechnical words such as metastasize or
of the interview may not be met. | biopsy. |
● Guidelines: ■ Instead of “metastasize”: use “spread”
→ Arrange for some privacy. ■ Instead of “biopsy”: use “sample of the tissue”
■ Make sure interview room is ideal. → Avoid excessive bluntness like “You have a very bad
■ Have tissues ready in case patients become upset cancer and unless you undergo treatment, you are going
→ Involve significant others. to die.”
■ Most patients want to have others with them, but this ■ This is likely to leave the patient feeling isolated and,
should be the patient’s choice. later, angry with a tendency to blame the messenger
■ When there are several family members, ask the of the bad news (which is the doctor).
patient to choose 1 or 2 family representatives. → Give information in small chunks and check periodically
→ Sit down. as to the patient’s understanding
■ Relaxes the patient → When the prognosis is poor, avoid using phrases such
A sign that you will not rush as “there is nothing more we can do for you”
■ Try not to have barriers between you and the patient. ■ This attitude is inconsistent with the fact that patients
→ Make a connection with the patient. often have other therapeutic goals such as good pain
■ Uncomfortable, but it is an important way of control and symptom relief
establishing rapport with your patient Addressing the Patient’s EMOTIONS with Emphatic
■ Touching the patient on the arm or holding a hand are Responses
other ways to accomplish this
● When patients get bad news, their emotional reaction is often
→ Manage time constraints and interruptions
an expression of shock, isolation, and grief. |
■ Inform the patient of any time constraints and
→ Offer support and solidarity to the patient by making an
interruptions that you may expect.
empathic response.
Assessing the Patient’s PERCEPTION ● Four Steps in Making an Empathic Response:
● The dictum here is: “before you tell, ask.” → Observe for any emotion on the part of the patient
■ A look of sadness, silence or shock
[PREV MED3 – Family Health] X.02 – Care of the Dying (26 August 2020) Page 3 of 8
→ Identify the emotion experienced by the patient by G. Stages of Death and Dying (Stages of Grief)
naming it to oneself.
■ If patient appears sad and silent, use open-ended ● Coping ability depends on:
questions to ask the patient as to what they are → Previous experiences with death
thinking or feeling → Cultural attitudes
→ Identify the reason for the emotion. → Religious and traditional beliefs
■ If you are not sure, ask the patient again. ● Note for the succeeding part: bullets with “| ” are the only
→ Lt the patient know that you have connected the descriptors that Doc mentioned for each stage
emotion with the reason for the emotion by making a Denial
connecting statement ● "This is not happening to me.“
■ Done after you have given the patient a brief period
→ Initial reaction of shock
to express his or her feelings, ● Refusal to believe diagnosis |
■ e.g. Doctor: I am sorry to say that the X-ray shows
● Moving the thought away
that chemotherapy doesn’t seem to be working.
● Isolation
Unfortunately, your tumor has grown somewhat.
Patient: “I’ve been afraid of this” Anger
■ e.g. Doctor: *He then moves his chair closer and ● “Why me?”
says”: “I know this is isn’t what you wanted to hear. I ● "How dare God do this to me!“
wish the news were better” ● Expression for frustration |
Table 1. Other examples of empathic vs. Validating Responses → and irritability towards God, their fate, family member,
Empathic statements Validating responses friend, or to self
“I can see how upsetting this is “I can understand how you felt ● Desire for control in a situation in which they feel completely
to you.” that way.” out of control |
“I can tell you weren’t expecting “I guess anyone might have that ● Can also be an assertion of life
to hear this.” same reaction.” Bargaining
“I know this is not good news for “You were perfectly correct to ● "Just let me live to see my son graduate.“
you.” think that way.” ● "I promise I'll be a better person if...."
“I’m sorry to have to tell you “Yes, your understanding of the ● Attempt to negotiate in return for cure |
this.” reason for the tests is very → Pledge to fulfill promises or make a deal in order to get
good/” what is wanted like postponing death
“I was also hoping for a better “It appears that you’ve thought Depression
result.” things through very well.”
“This is very difficult for me
● "I can't bear to face going through this, putting my family
also.” through this."
● Reaction to the effects of illness |
| STRATEGY AND SUMMARY ● Anticipation of the approaching death |
● Ask patients if they are ready for discussion on treatment ● Inability to sustain hope |
options ● Clinical signs |
● Share responsibility for decision-making with the patient → Withdrawal
● Check for the patient’s understanding of the discussion → Psychomotor retardation
→ Sleep disturbance
E. Principle of Informed Consent and Patient Autonomy
→ Hopelessness
● Clear ethical and legal obligations to provide patients with as → Suicidal ideation
much information as they desire about their illness and its Acceptance
treatment.
● "I'm ready, I don't want to struggle anymore."
● Physicians may not withhold medical information even if they
● The final rest before the long journey
suspect it will have a negative effect on the patient. Yet a ● Realization that death is inevitable |
mandate to: ● Importance of company, family, and enjoying the quality of life
→ Disclose the truth, with regard or concern for the sensitivity remaining.
with which it is done ● Note: not all patients will accept the diagnosis, some will have
→ Obligation to support the patients and assist them in long periods of denial, anger, and so on
decision-making
H. Psychosocial Care
F. Benefits of Disclosure (Nice to Know)
● Provision of holistic care that try to address more than the
● In a study by Centeno-Cortes et al., there was no increase in physical needs of patients
perceived symptoms of anxiety, despair, sadness, depression, Most Common Psychological Problems
insomnia or fear in the group of informed patients compared to
● Depression
the uninformed patients
→ Provide psychological support
● 75% of informed patients were able to share their concerns
→ Antidepressants
about the illness and its consequences with their relatives.
■ Sedating tricyclic antidepressants: for patients with
● Families of informed patients felt that they were in a better
insomnia and anxiety; given at bedtime
position to care for the patient.
■ Methylphenidate (psychostimulant): for patients who
● Allows the patient to make practical (e.g. making a will, settling
are withdrawn or who display vegetative signs; given
bank accounts and mortgages) and emotional adjustments to
2 times a day (breakfast and lunch)
the illness; thereby providing closure to his or her life.
■ SSRI (paroxetine, and sertraline): first-line agents for
● Diagnosis disclosure in cancer is also correlated with improved
treatment of depression in terminally ill patients when
symptom control and patient satisfaction with care.
immediate onset of action is not essential
● Disadvantage:
● Stress
→ Gerle et al., found that patients who were not informed of
→ Patient may have an altered body image and loss of self-
the diagnosis tended to be more anxious and at times,
esteem, fears of abandonment and separation, anxieties,
felt desperate.
and feelings of hopelessness.
[PREV MED3 – Family Health] X.02 – Care of the Dying (26 August 2020) Page 4 of 8
→ Family members may also experience physical and → Neurosurgical Approaches
emotional stress ■ Pain pathways can be cut or interrupted (ablated) by
→ Treated with counselling, sedatives, spiritual advice and neurosurgery
support groups → Psychological Approaches
● Grief ■ Relaxation and imagery, cognitive distraction
→ Usually begins before the anticipated death | (focusing attention on stimuli other than pain),
■ Explain the course of their illness and how death will support groups, and pastoral counselling
come → Complementary Therapies
■ Assure them that their physical symptoms will be ■ Relaxation techniques, mediation, movement
controlled therapies, and massage
■ Develop regular family meetings WHO 3-Step Process |
III. MANAGEMENT OF SYMPTOMS (SYMPTOM CONTROL) ● The Pain Relief Ladder

A. Pain

Figure 1 (L-R). Causes of cancer pain and The Concept of Total Pain.
● Concept of Total Pain |
→ Includes not only the physical symptoms from the disease
or the treatment, but also the financial, physical and Figure 2. WHO Pain Relief Ladder
spiritual components.
→ Step 1: NSAIDS
● Pain Rating Scales – verbal, numerical, visual analogue,
→ Step 2: WEAK OPIOIDS
Wong-Baker Faces
→ Step 3: STRONG OPIOIDS
● Prevalence of Cancer Pain
● Factors to consider:
→ Wide variability in published rates
→ Absorption and Convenience: Oral>Rectal, IV>IM
→ Early disease
→ Efficiency and Half life
■ 48% (range: 38%-100%)
Table 2. Choice of analgesics based on the severity of pain
→ Advanced disease
■ 74% (range: 53%-100%) Mild pain Moderate pain Severe pain
<3 out of 10 on 3-6 out of 10 on NRS >6 out of 10 on NRS
→ Pancreas: 72-85%
NRS
→ Esophagus: 71%-77%
Step 3
→ Head and neck: 67%-91%
Step 1 Step 2 Strong opioids
→ Genitourinary: 58%-90% Non-opioids Weak opioids Morphine
→ Prostate: 56-94%
Diamorphine
→ Breast: 50-89%
Codeine Fentanyl
● Causes of Pain in People with Cancer: Etiology
Paracetamol* Dihydrocodeine Hydromorphone
→ Pain due to cancer (85-93%) – most common!
NSAIDs* Tramadol Oxycodone
■ Pain due to cancer dx and treatment (17%-21%)
■ Pain unrelated to cancer or its treatment (2%-9%) Table 3. WHO Analgesic Ladder
● Common Pain Syndromes in Cancer by Pathophysiology WHO Analgesic Ladder
→ Somatic Analgesics Durg of Alternative
■ Bone metastases choice
■ Skin lesions Step 1 Mild non-opioid ± NSAID paracetamol
→ Visceral Pain (Score 1 adjuvant aspirin
■ Malignant bowel obstruction -2)
→ Neuropathic Step 2 Mild weak opioid ± codeine oxycodone
■ Chemotherapy-induced peripheral neuropathy to Moderate non-opioid ± tramadol
■ Postherpetic neuropathy (Score 3 – 6) adjuvant
● 5 Principles in Pain Control (WHO) | Step 3 strong opioid morphine oxycodone
→ Define the type and site of pain Moderate to ± non-opioid tramadol
■ Pain is considered to be the "fifth vital sign“ | Severe ± adjuvant
■ Then get the PQRST (Palliative/Provoking, Quality, (Score 7 -
Radiation Severity, Timing) 10)
→ Anticipate pain breakthrough ** Oral medication should only be abandoned if the patient is
→ Reviewing regimen frequently and regularly unable to take or retain them
→ Treat “total” pain ** Drugs given on an "as required" basis or pro re nata (PRN)
→ Treat each pain specifically usually results in poor pain control
● Approaches in Pain Management
→ Removal or Reduction of the Underlying Cancer Commonly Used Analgesics |
■ Surgery, radiation therapy, chemotherapy, and ● Morphine
immune therapy → Most commonly used opioid
→ Pharmacologic & Anaesthetic Approaches → SE: Constipation – most common side effect of opioids
■ Administered by different routes
[PREV MED3 – Family Health] X.02 – Care of the Dying (26 August 2020) Page 5 of 8
■Can be lessened by an increase in fluids, dietary Visceral Mild Non-opioid (opioid
fiber, and moderate exercise if required)
■ Or by laxatives or stool softeners Moderate, Opioid + non-opioid
→ 2-dosing rules: severe
■ If respiratory depression develops= >2x stables dose Neuropathic Pain
■ Reestablishing pain control requires not less 1.5x Nerve compression Corticosteroid +/- opioid
previous dose Nerve infiltration, Antidepressant or anticonvulsant
→ Hospital setting: IV or Subcutaneous (PCA-Patient damage Or local anesthetic
controlled anesthesia) Or NMDA receptor antagonist
● Fentanyl Sympathetic type of Sympathetic nerve block
→ Topically (patch) pain
→ At least 24 hours to reach maximum analgesia Other Types of Pain
→ Can cause confusion and delirium Raised intracranial Corticosteroid
● Meperidine pressure
→ Not recommended for persistent pain since it is short Muscle spasm Muscle relaxant
acting
→ Must be avoided due to accumulation of its metabolite ● Strong evidence from trials of patients with CA supports the use
normeperidine, which is not reversed by naloxone - of NSAIDs, opioids, bisphosphonates, and radiotherapy or
→ Produces neurotoxicity (e.g., seizures, hallucinations, and radiopharmaceuticals for pain
delirium) → Bisphosphonates: specifically effective for bone pain
→ Insufficient evidence for exercise or acupuncture in pain
Breakthrough Pain mx
B. Anorexia
● Causes: mouth infection, nausea, constipation, depression,
drugs, radiotherapy, or the disease itself
● Tips to get the patient to eat well:
→ Know the patient’s preference
Figure 3. Visual representation of breakthrough pain. American Pain Foundation. → Serve food that s/he eats when s/he wants to
● Sudden, temporary flares of severe pain → Offer small portions on a small plate
● May be unpredictable or associated with movement or activity → Offer visually appearing food
● Occurs in up to 2/3 of those with cancer → Prepare food out of the patient’s smelling distance
● Affects all aspects of the patient’s life → Serve hot soups, ice cream or fruit
● Treatment: immediate-release opioids or rescue → Have the patient eat sitting up in a room free of odors
medications | → Allow someone to eat with the patient
→ Quick-acting, potent analgesics (e.g. oral morphine) → Give supplemental vitamins and minerals
→ For times when pain breaks through normal pain control C. Nausea and Vomiting
→ Act quickly and clear the body relatively soon
→ Additive to the dose taken regularly for persistent pain. ● Causes: GI problems (constipation and gastritis),
hypercalcemia, uremia, drug side effects, increased ICP,
| On Addiction: Will I become addicted if I’m prescribed
psychosocial stress
strong opioids like morphine?
● To alleviate:
● Pain specialists view addiction as a series of specific
→ Offer small meals, liquid and soft foods, very cold drinks in
behavior including:
between meals, and ice chips
→ Compulsive use of medication
→ Let the patient sit up after eating
→ Loss of control over medications
→ Let patient keep distance from smelly food and colors
→ Insistence on taking meds despite deterioration in the
→ Keep the patient away from fatty, sweet, and spicy foods
quality of life (QOL)
→ Schedule eating before radiotherapy or chemotherapy
● Physical dependence on opioids occurs when they are taken
→ Offer relaxation and breathing exercises
for more than twenty-four hours.
● Treatment:
→ Note: it is a normal, expected part of proper pain rx.
→ H2 blockers: gastritis
→ Meds are slowly tapered over time to ensure that the
→ Metoclopramide (p.o. or s.c.): nausea due to gastric
patients do not experience severe withdrawal symptoms
distention and reflux
due to dependence.
→ Promethazine 25mg, qid or Prochlorperazine (10mg)
● Adjunct Analgesics p.o. before meals: nonspecific nausea and vomiting
→ Drugs with a primary indication other than pain that have → Scopolamine, Meclizine: recurrent nausea prevention
analgesic properties in some painful conditions
■ Corticosteroids: pain d/t inflammation and swelling D. Constipation
■ Tricyclic antidepressants (amitriptyline, nortriptyline, ● Causes: inactivity, opioids, anticholinergic drugs, decreased
doxepin) fluid, decreased dietary fiber intake
■ Anticonvulsants (valproate, carbamazepine, ● Common in dying patients
gabapentine): radiculopathies ● Treatment:
■ Benzodiazepines (diazepam): anxiolytic → Laxatives – to prevent fecal impaction
■ Muscle relaxants (tizanidine, lioresal) ■ Casantaranol
■ Anesthetics: for indwelling epidural catheter ■ Senna
Table 4. Types of Pain and Their Corresponding Drugs | ■ Osmotic laxative (lactulose or sorbitol) – cheaper but
Pain Regimen equally effective
Nociceptive Pain → Stool softeners
● Supportive measures:
Bone, Soft Tissue Mild, moderate Non-opioid (opioid
→ Increase dietary fiber
if required)
→ Increase fluids
Severe Opioid + non-opioid
■ Drink hot beverages (calamansi juice)
→ Avoid constipating foods (e.g. guava)
[PREV MED3 – Family Health] X.02 – Care of the Dying (26 August 2020) Page 6 of 8

E. Dyspnea → Inadequate blood supply Ischemia Tissue damage


Cell death
● Several causes
● Etiology
● Pharmacologic treatment:
→ Pressure: caused by the force of bone against a surface
→ Antibiotics: Pneumonia
→ Shear force: force created when the skin of a patient stays
→ Thoracentesis: pleural effusion
in one place as the deep fascia and skeletal muscle slide
→ Oxygen – for hypoxemia (via nasal cannula)
down with gravity
■ Psychologically comforting
→ Friction: force resisting shearing o the skin
→ Morphine
● Differentiate the stages of pressure ulcers. |
■ 2-10mg sublingual or 2-4mg SQ q 2-4H
− Reduces tachypnea and breathlessness Stage 1 Pressure Ulcers
decreased anxiety ● Skin is intact but shows a persistent pink or red area that
■ Low dose morphine; blunts medullary response to does not turn white when pressed (does not blanch)
co2 retention or oxygen decline ● Resembles a mild sunburn
→ Diuretics: volume depletion to treat congestion ● Skin maybe tender, painful, or itchy; may feel warm, spongy, or
→ Benzodiazepines: anxiety associated with dyspnea firm to touch
→ Drying agents ● Superficial; heals spontaneously when pressure is relieved |
■ Topical scopolamine gel (0.25-.5mg q 8-12H) Stage 2 Pressure Ulcers
■ Hyoscanine (0.125mg sublingual q 8H) ● Wound looks like an abrasion, blister (broken or unbroken), or
■ Diphenhydramine (25-100mg IM q 4-6H) a shallow crater
→ Nebulized saline: viscous secretions ● Surrounding tissues may show areas of pale, red, or purple
→ Nebulized albuterol and corticosteroids: discoloration
bronchospasm, bronchial inflammation ● Swelling and/or oozing may be present
● Non-pharmacologic treatment: ● Ulcer is an open sore that does not extend through the full
→ Position: upright in bed or chair leaning against a wall thickness of the skin
→ Adequate ventilation
→ Keep the patient’s room cool Stage 3 Pressure Ulcers
→ Wear loose and comfortable clothing ● Wound extends through all layers of the skin
→ Humidify inspired air (vaporizer) → Crater involves damage or necrosis of subcutaneous
→ Help patient expectorate tissues
→ Soft music (alleviate anxiety) → Deeper and very difficult to treat
→ Wound becomes a primary site for a serious infection to
F. Cough occur |
● Management will depend on the type: | Stage 4 Pressure Ulcers
→ Productive
● Full thickness skin loss with extension beyond the deep
■ Encouraged
fascia |
■ Expel secretions
● Ulcer is extremely deep and now involves:
■ Rx: carbocisteine (375mg, 2 capsules tid)
→ Muscle layers
→ Dry cough
→ Underlying organs
■ Useless
→ Bone, and tendon or joint space
■ Rx: cough suppressant or antitussives
● Deep, open wound may show a blackened tissue called eschar
→ Persistent
● Management:
■ Muscle pain
→ Surgery – normal course of treatment
■ Muscle Exhaustion
■ Surgical removal of necrotic or decayed tissue
■ Should be treated
■ Rx: dextromethorphan (10mg/tab, 1 tab tid)
− Cough suppressant
− May also give a bronchodilator
● Supportive measures
→ Steam inhalation and soothing syrup
→ Warm fluid mixture of ginger, calamansi, honey
→ Mentholated candies Figure 4. Stages of Pressure Ulcers.
→ Chest pulmo-physiotherapy tid Prevention
G. Hiccups ● Relieve pressure by rotating the patient every:
● Supportive measures → 2 hours when lying down
→ 2 tsp sugar in mouth and let patient swallow → 15 minutes when seated
→ Sip small amounts of water from a glass at a sequence ● Use specialized mattresses or continuously inflated, air-
counting 1 up to 25 or more | suspension beds
→ Rebreathe into a bag | ● Urinary catheter: last resort to keep the patient dry |
→ Massage roof of mouth with wet cotton swab | → Only when there is pain occurring with bedding changes
→ Sit upright and try to slowly breathe deeply and slowly or when the patient and the family have a strong
release breath preference
● Skin care
H. Bed Sores
→ Clean daily
● aka Decubitus or Dermal Ulcers, or Pressure → Apply baby lotion or powder 3x/day in a circular, gently
Ulcers/Sores/Wounds massaging motion
● Common especially among the debilitated and bedridden → Prevent excess sweating and drying of the skin
patients → Make sure that linens are always dry
● Due to unrelieved pressure to any part of the body, especially → Let patients wear adult diapers
portions over bony or cartilaginous areas ● Proper nutrition and hydration
● Pathophysiology:
[PREV MED3 – Family Health] X.02 – Care of the Dying (26 August 2020) Page 7 of 8

I. Mucositis → Affirms life and regards death as a normal process


→ Neither hastens nor postpones death
● Cause: injury to rapidly dividing epithelial cells lining the oral
→ Provides relief from pain and other distressing symptoms
cavity physiological changes ranging from mild atrophy to
→ Integrates the psychological and spiritual aspects of
severe ulceration
patient care
● Common among cancer patients |
→ Offers a support system to help patients live as actively as
● Associated with pain and increased risk for infection |
possible until death
→ Can lead to inadequate nutrition and hydration
→ Offers a support system to help the family cope during the
● Management
patient’s illness and in their bereavement
→ Encourage proper oral hygiene
→ Vitamin C supplementation C. The Two Roads to Death
→ If (+) Candidiasis: Miconazole gel or Nystatin tablets ● There are two possible roads to death: the difficult road or the
J. Urinary Incontinence usual road
→ Usual road: patient goes into lethargy semi-coma
● Consequence of the terminal phase of life coma death
● Management: → Difficult road: hallucinations/delirium/convulsions first
→ Decrease fluid intake comatose stage death
→ Pads
→ Frequent changing of sheets
→ Indwelling catheterization
K. Insomnia
● Assess the cause:
→ Pain
→ Bladder or bowel problems
→ Cough
→ Difficulty breathing
→ Psychological
● Management is the directed at the cause. Figure 5. Two roads to death.
→ Midazolam 7.5mg before bedtime
→ Amitryptiline 25-75mg at night D. Medications
→ Lombitol 1 cap o.d. or bid ● Reassess the need for each medication and minimize the
number of medications that the patient is taking.
L. Confusion ● Continue only those medications needed to manage symptoms
● aka terminal restlessness (e.g. pain, breathlessness, excess secretions, and terminal
● Impaired consciousness delirium)
● Symptoms: ● Medications to manage chronic conditions (i.e. hypertension,
→ Agitation, muscle twitching, occasional convulsions DM, hyperlipidemia, osteoporosis) should be stopped.
● Management: Benzodiazepines ● Choose the least invasive route of administration for all meds.
| On Feeding E. Advance Care Planning
● Studies demonstrate that parenteral or enteral feeding of
● Chance to make decisions about the care you want to receive
patients near death neither improves symptom control nor
at a time when you might be unable to speak for yourself
lengthens life.
● Process that involves you, your family, loved ones, and health
● Anorexia may be helpful as the resulting ketosis can lead to
care providers
a sense of well-being and diminish discomfort.
● Goal: maintain and maximize your health and independence
● Families and caregivers realize that food pushed upon the
as you face the end of your life
unwilling patient may cause problems such as aspiration.
● Should be a compassionate process and with support and
● Help them to find alternative ways to nurture the patient.
education as your health changes
IV. HOSPICE CARE AND END-OF-LIFE CARE ● Encourage patients to choose a surrogate decision-maker
→ Important component of advance care planning
A. Hospice Care
→ Discuss values and preferences
● Consists of palliative and supportive services for dying
persons and their families F. Advance Directive
→ Usually those who have less than 6 months to live. ● Written statement on what type of medical treatment you want
● It highlights quality rather than length of life at the end of life
● Provided by interdisciplinary team of professionals and ● Confirms that your wishes are carried out if you are not able to
volunteers (care coordinator, nurse, physician, counselor, communicate them to a healthcare professional
volunteer coordinator, and spiritual support) ● Ongoing process that can change over time
● Provides continuity of care Types of Advance Directives
● It provides family-centered care involving the patient and family ● The Living Will
in making decisions → Written or video statement about the kinds of medical care
● Available 24/7 you do/do not want to receive if you are no longer able to
● Provides follow-up bereavement care for up to 1 year after the express your consent.
patient’s death → Includes: resuscitation, desired QOL, end-of-life
B. Palliative Care treatments, etc.
→ When creating this, you should know how independent t
● Any form of medical care or treatment that concentrates on want to be and what you want out of your life |
reducing the severity of disease symptoms rather than ■ Do you want your life extended for any reason or only
striving to halt, delay, or reverse the progression of the disease if a cure is possible?
itself, or provide a cure → Having a treatment or not is your decision. Nothing is right
● It should not be confused with hospice care or wrong. This is your choice. |
→ Delivers palliative care to those at the end of life |
● Palliative Care Treatment
[PREV MED3 – Family Health] X.02 – Care of the Dying (26 August 2020) Page 8 of 8
● Durable power of attorney for health care (Medical Power of The Brief Bereavement Interview
Attorney)
● Responses to Grief
● POLST (Physician Orders for Life-Sustaining Treatment)
→ “You’ve faced a lot over the past several weeks. How has
● Do not resuscitate (DNR) orders
that been for you?”
● Organ and tissue donation
→ “How have things been different for you?”
V. MANAGING DEATH AND BEREAVEMENT CARE → “Is there anything that has been especially troubling to
● Grief: the normal process of reacting, both internally and you?”
externally, to the perception of loss | ● Social Support
→ Psychological/emotional reactions: anger, guilt, anxiety, → “Has anyone been particularly helpful to you in the past
sadness, despair month?”
→ Physical reactions: sleep difficulties, appetite changes, ● Coping resources
somatic complaints, illness → “Are there any activities that have made this less difficult
→ Social reactions: feelings about taking care of others in the for you?”
family, the desire to see/not see family and friends, or the ● Practical difficulties
desire to return to work → “How are things around the house? With your finances?”
● Mourning: the process by which people adapt to a loss | Factors that contribute to how individuals will grieve (and how
→ Different cultural customs, rituals, or rules for dealing with they are affected by their loss)
loss ● Nature of the relationship with the person who died
A. End of Life ● Circumstances surrounding the death
● Where to die ● Circumstances surrounding the grieving person’s support
● What to expect system
● Advance directives ● The griever’s unique personality
→ By creating one, you are making preferences about ● Cultural background
medical care known before you’re faced with a serious ● Religious or spiritual background
injury or illness. ● Biological sex
→ Spares your loved the stress of making decisions about ● Ritual or funeral experience
your care while you are sick. ● Other crises or stresses in one’s life
→ Any person 18 years and above can prepare an advance END OF TRANSCRIPT
directive. |
B. Bereavement
REFERENCES
● The period after a loss during which grief is experienced and
Cruz, M.V.P. (2020). Care of the dying [Video Presentation]. Manila, Philippines:
mourning occurs Faculty of Medicine and Surgery, University of Santo Tomas, Preventive
● Length of time spent in a period of bereavement depends on: Medicine 3.
→ Intensity of attachment to the deceased
→ How much was involved in anticipation of the loss

APPENDIX

Table. Changes in the Dying


Change Manifested by / Signs of: Change Manifested by / Signs of:
Fatigue Decreasing function, hygiene Decreasing level of Increasing drowsiness
Inability to move around the bed consciousness Difficulty awakening
Inability to lift the head off the pillow Unresponsive to verbal or tactile stimuli
Cutaneous ischemia Erythema over bony prominences Decreasing ability to Difficulty finding words
Skin breakdown, wounds communicate Monosyllabic words, short sentences
Decreasing Anorexia Delayed or inappropriate responses
appetite/food intake, Poor intake Verbally unresponsive
wasting Weight loss, muscle and fat, notable in Terminal delirium Day-night reversal
temples Confusion
Decreasing fluid intake, Poor intake Agitation, restlessness
dehydration Peripheral edema owing to Purposeless, repetitive movements
hypoalbuminemia Moaning, groaning
Dehydration, dry mucus membranes Respiratory Change in RR (increasing first then slowing)
and conjunctiva dysfunction Decreasing tidal volume
Cardiac dysfunction, Tachycardia Abnormal breathing patterns – apnea,
renal failure Hypertension followed by hypotension Cheyne-Stones respirations, agonal breaths
Peripheral cooling Loss of ability to Dysphagia
Peripheral and central cyanosis (bluish swallow Coughing, choking, aspiration
color of extremities) Loss of gag reflex
Mottling of the skin (livedo reticularis) Buildup of oral and tracheal secretions
Venous pooling along dependent skin Loss of sphincter Gurgling
surfaces control Incontinence of urine or bowels
Dark urine Skin maceration
Oliguria, anuria
PREVENTIVE MEDICINE 3 [FAMILY HEALTH]

OPTIMAL PRIMARY CARE FOR LGBTQ+ AY 20-21

PATIENTS 1.04
13 AUG 20
Ma. Teresa Tricia G. Bautista, MD, FPAFP, FPCGM, MHA
TABLE OF CONTENTS → Can result to chronic anxiety from expectations of
I. UNDERSTSANDING LGBTQ+ ISSUES | ............................................................ 1
A. Case 1................................................................................................................ 1
rejection based on their sexuality
B. Case 2................................................................................................................ 1 ● Minor stress and rejection sensitivity are associated with
II. PERFORMING A SENSITIVE HISTORY AND PHYSICAL EXAMINATION ............. 2 higher rates of:
A. Case 1................................................................................................................ 2
B. Case 2................................................................................................................ 4 → Depression
C. Case 3................................................................................................................ 4 → Anxiety
III. OTHER NOTES ........................................................................................................ 5
A. Health and Social Issues and Screening for the LGBT Community ................... 5
→ Substance abuse
→ Suicide
MUST KNOW ADDED NOTES BY PREVIOUS TRANS → Cardiovascular outcomes
DOC What else in Tessa’s social history would you like to discuss
further?
This transcript is based on the two video lectures by Dra. Bautista. The first part a) Cigarette Use
of this trans corresponds to the first video Lec 04: Understanding LGBT Issues, b) Alcohol Use
while the second corresponds to Lec 04-A on Performing a Hx and PE. c) Both
I. UNDERSTSANDING LGBTQ+ ISSUES | ● As primary care physicians, it is our responsibility to motivate
our patients to stay away from risky and unhealthy behavior.
● Creating awareness and understanding on matters pertaining ● Young members of the LGBTQ community are at a higher
to the LGBT Community is essential to: risk for cigarette use
→ promoting cultural competence among healthcare → Tend to self-report poor health status in general
providers ● In communities with higher levels of approval for same sex
→ ensuring sensitivity towards them relationships, there was a lower prevalence of smoking
● When addressing LGBT individuals, healthcare providers among LGBTQ individuals.
should always ask clients how they identify and/or wish to be → Supportive of the hypothesis that structural stigma
addressed. | contributes to smoking and alcohol use
A. Case 1 ● LGBT population has higher rates of:
→ Depression
| TESSA → Suicide
● Tessa is a 25-year old masteral student with generalized → Intimate partner violence
anxiety disorder who presents for routine follow-up. She is → Obesity
due for Pap Smear and would like refills of her SSRI. ● LGBT population have higher rates of breast cancer:
● She reveals to you that she is doing well in school and → Due to decreased parity
excited about her classes this semester. → Lower use of hormonal contraception
● She drinks 2-3 bottles of light beer with friends 3-5 times a → Lower rates of screening mammogram and clinical breast
week. She smokes 5-10 cigarette sticks a day. exam
● She states that she had previously been sexually active with
men, but 6 months ago she began a monogamous B. Case 2
relationship with a female partner. | ARIS
● This relationship caused some conflict with her family and ● Aris is a 32-year old man who consulted you before due to
friends. She is worried about rejection and prejudice from tension headache and is back at your clinic.
them. ● He left home 2 years ago and has been living with his friends,
What might Tessa’s stress and anxiety represent or emanate often moving from one house to another.
from? ● He identifies as a male and expressed himself as a male in
● Members of a group that is stigmatized by and face prejudice terms of clothes, haircut and tattoos. He told you that he has
from society may be more vulnerable to psychological distress. been intimate with both men and women for the last 2 years
● Minority Stress | with intermittent use of barrier protection. Now he prefers to
→ It is a theory that can be applied to various types of be in relationship with men.
stigmatized population. ● His preference in men is the reason why he moved out. His
→ In the LGBT community, it is characterized by the father, a high-ranking military, is not in favor of his lifestyle
experience of: and is often the reason of their disagreement. |
■ External stressors: violence, harassment, and What is meant by the terms “gender expression” and “gender
discrimination identity”? |
■ Internal stressors: expectation of rejection, hiding ● The human experience has been described as consisting of
and concealing one’s identity, internalized sexual orientation, gender identity, gender expression, and
homophobia and coping processes associated with sexual behaviour.
being an LGBT ● Gender Identity
● Rejection Sensitivity | → The person’s internal sense of their own gender
→ A psychological condition that causes a person to feel ■ Cisgender: natal sex concordant with internal sense
oversensitive to rejection or perceived rejection in of gender
relationships and social interactions.
DE GUZMAN AO, ESTRADA PD, ESTRELLA AV | ESTRELLA AP, ESTRELLA AM, ESTRERA (BARANGAY ESTRELLA) Page 1 of 5
[PRV-FH] 1.04 – Optimal Primary Care for LGBTQ+ Patients Page 2 of 5
■ Transgender: natal sex discordant with internal How would you categorize Aris’ sexual orientation and sexual
sense of gender behaviour?
■ Genderqueer: a term used by persons who may not
● Sexual orientation: gay
entirely identify as either male or female
→ Aris finds himself attracted primarily to men.
■ Bigender: a person whose gender identity
● Sexual behavior: men who have sex with men (MSM)
encompasses both male and female genders.
→ Even though he was previously intimate with both men and
− Some may feel that one identity is stronger, but
women.
both are present |
● Gender Expression | ARIS (continued)
→ The spectrum of masculine and feminine characteristics in ● Aris also reports that he smokes about ½ pack day, drinks
how one dresses, acts or presents his/herself heavily on weekends, snorts shabu, and tried marijuana a
→ Can be masculine, feminine or somewhere in between few times
→ Gender nonconforming: ● He gives a history of depression with anhedonia, insomnia,
■ People whose gender expression does not conform and weight loss. He does not have contact his family
to society’s norm anymore, as his father does not accept his lifestyle.
■ e.g. women who wear men style of clothes, ● You discussed with him STI screening, safer sexual
hairstyles, or men who wear more feminine types of practices, as well as substance abuse and mental health.
make-up, nails, clothing You would like to follow him closely to consider starting
treatment of his depression with his medications.
How does Aris seem to identify in terms of his Gender Identity
● Aris states that he might be open to that and will stop by the
vs. his Gender Expression?
clinic sometime in the future. He is unable to commit today,
● In Aris’ case: because he is not sure where he will be living next month.
→ Gender identity: man
→ Gender expression: gender conforming/masculine What are some common barriers to the health care of LGBT
→ Biological sex: male population?
● Is his gender identity cis or transgender? ● Physical Barriers
→ As his gender identity and biological sex are the same, this → Lack of LGBT-friendly facilities (treatment rooms, wards,
is cisgender (as opposed to transgender). toilets, registration forms, etc)
● Terms and Definitions Specific to Gender Expression ● Behavioral Barriers/Behavior of the health care team
→ Passing → Verbal harassment
■ used by transgender people to mean that they are → Physical abuse
seen as the gender with which they self-identify or → Denial of service in the health care setting
they want to be seen as → Discrimination in health care
■ e.g. a transwoman using the female’s bathroom and ● Psychological Barriers
being seen as female by those around her; a → Fear of being blamed for their sexual/gender identity
transman being perceived as a cisman | → Fer of being discriminated on the basis of sexual
→ Transition orientation and gender identity
■ describes the period during which a transgender → Doubt of confidentiality about sexual/gender identity
person begins to express their gender identity → Fear that the health care staff are unable to understand
■ A person may change their name, take hormones, health issues
have surgery or change legal documents to reflect Why is knowledge of these barriers so essential?
their gender identity |
● It is simple. Health is a right. | ,
What are the differences between sexual behaviour and ● As physicians, we can work together to improve access to
sexual orientation? | health care for the LGBT population by:
● Sexual Orientation → Creating a welcoming environment
→ Based on how a person identifies attraction on a physical → Decreasing all these barriers
and emotional basis
II. PERFORMING A SENSITIVE HISTORY AND PHYSICAL
■ In other words, it refers to a person’s emotional,
EXAMINATION
sexual, or relational attraction to others. |
→ May be described by many terms: ● Gender Sensitivity
■ Heterosexual, bisexual, homosexual (lesbian or gay), → Informed by knowledge and understanding of the
queer, pansexual and asexual differences, inequalities and varying needs of people of all
→ Not a choice, but a person’s self-awareness and gender identities.
acceptance can be influenced by many factors including → People of all genders have a right to access effective
cultural environment, stage of development, and beliefs. care that is responsive to the differing needs of people of
● Sexual Behavior and Orientation different genders, who may experience mental health
→ Do not always align and can evolve over time differently.
→ e.g. a man who does not identify himself as gay, may at → For gender-sensitive healthcare, we not only need to
times be intimate with men change systems and structures, but also to enhance
→ e.g. a woman who identifies as lesbian may have both understanding, raise awareness, and develop skills
female and male partners among health professionals. |
● Other terms and definitions specific to sexual identity and A. Case 1
orientation
→ Coming out | CECIL
■ The process through which a person identifies, ● Cecil is a 46-year old woman with a history of type 2 diabetes
acknowledges, and decides to share information mellitus. She has been generally healthy in the past without
about their own sexual orientation and/or gender major illness.
identity with others ● Her family history is remarkable for diabetes and
→ Outing hypertension in both of her parents, but no history of cancer.
■ The act of exposing information about a person’s ● She works as a manager at the local fast food chain/
sexual orientation and/or gender identity without their ● She drinks alcohol on social occasions, and smokes 1 pack
consent per day for the past 20 years. She denies illicit drug use.
[PRV-FH] 1.04 – Optimal Primary Care for LGBTQ+ Patients Page 3 of 5

What open-ended and gender-neutral language could you use Figure 1. The HITS Screening Tool for Domestic Violence
when taking a sexual history?
● Gender neutral language or gender-inclusive language
→ Avoids bias towards a particular sex or social gender
→ Use of nouns that are not-gender specific when referring
to roles or professions
→ Avoidance pronouns to refer to people of unknown or
indeterminate gender.
■ Use gender-neutral alternatives (i.e., they, them
theirs, and themselves) to avoid misgendering |
Table 1. Gender-Neutral Terms for Specific Jobs
Note: A total score of >10 is suggestive of intimate partner violence
Gender-specific Job Titles
Gender-neutral Job Titles (IPV). |
Male Female
Policeman Policewoman Police Officer | CECIL (cont’d)
Waiter Waitress Server ● ROS revealed insomnia because of worrying about financial
problems. Despite these worries, her mood is generally good
Fireman Firewoman Fire Fighter
and she continues to enjoy her hobbies and family time. She
Salesman Saleswoman Salesperson
denies any suicidal or homicidal ideations. She scored 2 in
Air Steward Air Stewardess Flight Attendant the GAD-2 assessment, so you did not prescribe anything for
• Some terms, such as chairman, that contain the component, her anxiety
man, but have been traditionally used to refer to persons ● Generalized Anxiety Disorder
regardless of sex, are now seen by some as gender-specific. → Over the past 2 weeks, how often have you been bothered
You may use chairperson or chair instead. by the following problems?
The Sexual History Nearly
● Should include sexual behavior, orientation, and gender Several >1/2 of
Not at all every
days the days
identity. day
● Instead of asking “Are you married?” Feeling nervous,
→ Ask “do you have a partner?” or “are you in a relationship?” anxious, or on 0 1 2 3
● Next you could ask “have you had female partners, male edge
partners, or both?” Not being able to
→ Alternatively, you could start by asking “are you sexually stop or control 0 1 2 3
active?” worrying
→ Then ask, “when you have had sex, what are the genders Note: Total GAD-2 score: __+__+__ |
of your partners?” | CECIL (cont’d)
→ If patients are confused by this question, you could ask ● You referred her to your OB friend for Pap smear.
“Do you have sex with men, women, or both?” ● A month later, Cecil returns but without the Pap smear result.
● At the end of the sexual history: She said that when your OB friend asked her if she had any
→ “Do you have any concerns or questions about your children, and she said she has one, she went on to ask about
sexuality, sexual orientation, or sexual health?” her child’s father. Cecil avoided the question, asked to be
● You should also ask whether patients identify as transgender excused and did not return.
or have a gender-related concerns:
→ “Since many people are affected by gender issues, I ask How could this initial encounter with the OB have been
patients if they have any concerns about this. If this topic different?
is not relevant to you, tell me and I will move on.” ● The OB should not have asked a question that assumed
heterosexuality. |
| CECIL (cont’d)
● Did the OB display any bias? What assumptions should we
● Cecil revealed that she is currently exclusively sexually
avoid as healthcare providers to be inclusive?
active with her female partner of 15 years, Grace. They have
→ Open-ended questions that are gender-affirming and
adopted a 10-year old child whom they raised together since
free from heterosexist bias should be used.
birth.
→ Questions should be modified to achieve neutrality and
Why is it important to take a sexual history in a sensitive and remove bias.
non-judgmental manner from this patient? ● Common Assumptions to Avoid: |
● Responding in a non-judgmental way to the information → Do not assume that all patients use traditional labels.
received facilities the delivery of optimal health care for several Several people prefer more inclusive/indefinite labels or
reasons: do not like to use labels at all.
→ Helps to establish trusting relationships → Do not assume sexual orientation based on appearance,
■ Patients feel safe and are more likely to respond sexual behavior, or partner’s gender.
honestly to sensitive questions. → Do not assume sexual behavior or orientation has not
→ Allows us to create a more positive clinical encounter, changed since the last visit.
which LGBT patients will more likely want to repeat. → Do not assume transgender patients are gay, bisexual, or
● Lesbians who were “out” to their primary care provider lesbian.
were significantly more likely to: → In other words, never assume. | .
→ Seek health and preventive care ● When referring to a patient’s sexual identity:
→ Have Pap Smear → We should learn to listen to how patients describe
→ Be comfortable discussing sensitive issues themselves and their partners, and then follow their lead.
● If further screening for intimate partner violence (IPV) is → If in doubt on how to refer to your patient’s sexual identity
negative: or partner, ask the patient what terms they prefer.
→ One of the tools used is HITS ■ You can be curious without offending them.
■ Stands for and explores Hurts, Insults, Threats,
and Screams
[PRV-FH] 1.04 – Optimal Primary Care for LGBTQ+ Patients Page 4 of 5
→ Sometimes, the LGBT population use terms for ● She recently moved in the city and currently lives with a
themselves that may sound derogatory if used by an supportive partner. She denies tobacco and alcohol use.
outside source.
What are some words you might use to elicit information
■ If you are unsure if the patient will be comfortable with
about her partner?
you using this language, ask the patient first. | .
● What do you do if you offended a patient? ● Instead of the stereotype questioning, consider asking:
→ Apologize and ask the patient what they prefer. → “What is your current relationship status?”
→ Most patients will appreciate your sincerity and good ■ Are you married?
intention ■ Do you have a boyfriend?
→ “Do you have a partner” or “Are you in a relationship?” → “Who are the important people in your life?”
instead of asking “Are you married?” ● “Tell me more about your partner.”
● Ask open-ended questions |
B. Case 2 → It will elicit the most information.
→ It provides an open environment and prevents
| YANNA
assumptions that may hinder therapeutic alliance.
● Yanna is a 30-year old woman with an unremarkable past
medical history who came to you for a routine Pap smear. How would you approach the physical exam?
She is accompanied by her boyfriend, Josh, who remains in ● Don’t forget to ask permission. |
the room during the procedure. → Ask if the patient wants anyone in the room with her.
● You leave the room to give Yanna some privacy to get ● The patient might also not want a physical exam.
dressed. When you re-enter the room to discuss follow-up, ● Remember to use gender affirming language
she says, “Doctor, my boyfriend and I were talking, and we → Use the term “chest” instead of “breasts”
were wondering if he should get a Pap smear too?” → “Genitals” and “private parts” instead of gender-specific,
How would you respond to Yanna’s question? anatomic parts such as “uterus, cervix, or penis”
● Draping and examining one organ/part at a time should be
● You could respond by looking non-judgmentally at Josh and
a standard practice. |
affirming the appropriateness of the question.
→ “You’re wondering what medical care you need to stay | DEREK (cont’d)
healthy, right? What makes you think Pap smear is ● As you become more engaged with the patient, you learned
important to you?” that Derek is in a relationship with a male who is aware of her
transition. Screening for intimate partner violence is negative.
What is the concept of gender affirmation?
● History and physical exam showed that she did not undergo
● Gender affirmation any gender affirming surgeries. She is happy with her breast
→ Interpersonal, interactive process where a person size which has increased since taking estrogen.
receives social recognition and support for their chosen ● As the consultation ended, she asks, “Can you refill my
gender identity and expression estrogen and spironolactone? I have never been off them. I
● Physician’s Role in Gender Affirmation am still looking for a new endocrinologist.”
→ Simply supporting these LGBT patients
→ In some settings or other cultures, prescribing hormonal As a primary care provider not familiar with transgender care,
therapy to patients or performing surgical gender how would you respond to her request?
transformation could be a demonstration of this ● A primary care provider may sometimes be asked to refill
affirmation. | medications, particularly in transitions of care.
● Writing a prescription really depends on the provider’s
C. Case 3 comfort and knowledge of drug monitoring and side effects.
| DEREK What are ways to promote gender affirmation in the clinical
● Derek is a 36-year old patient with medium-length hair and setting?
baggy clothing who consulted you for the first time for a ● Understand the barriers to inclusion of the LGBT community.
medication refill. You noticed that the patient has not ● Exercise prudence to ensure that they feel accepted by being
selected “M” or “F” on the patient data sheet. sensitive and open to their concerns about gender and
● Past medical history is noted for asthma and current sexuality in a non-judgmental manner and neutral language.
medications include salbutamol MDI, spironolactone, and ● Identify issues with open-ended questions about gender and
monthly estradiol injection. sexuality in neutral language.
● Vital signs and anthropometrics showed: ● Clinic infrastructures should be gender neutral or “all-gender” if
→ Height: 5’7” possible.
→ Weight: 174 lbs
→ BP: 130/80
→ HR: 74 bpm
How would you ask Derek abut gender identity?
• As in the previous case, asking open-ended questions allows
the patient to identify preference.
• Ask the patient to describe how he or she would like to be
addressed. It is acceptable to clarify for the sake of
understanding.
| DEREK (cont’d)
● Derek reveals that she identifies as a woman. She reports
feeling ‘trapped in the wrong body’ since she was in
elementary school and she began transitioning to a female
when she was in college.
● She also struggled with depression during this period, but for ● As society is evolving, we need to open doors and make them
the past 3 years, she has been meeting a psychiatrist and feel safe and validated.
describes her mood as good and stable.
[PRV-FH] 1.04 – Optimal Primary Care for LGBTQ+ Patients Page 5 of 5

III. OTHER NOTES |


● Keypoints in History Taking: |
→ Use open-ended questions
→ Reflect the terminology used by your patients
→ Bisexuals are majority of LGBT and yet get less attention
→ Don’t make assumptions about sexual orientation, identity,
expression, or sexual behaviors
→ Be sensitive to the process of “coming out”; some of
individuals discloses their sexual orientation first to
clinician
→ Reassuring responses from health care providers to build
trust and make health services more accessible to LGBT
→ Coming out is a personal process. |
■ Don’t try to speed or delay
■ Let your patient set the pace.
A. Health and Social Issues and Screening
for the LGBT Community
Table 2. Health and Social Issue Prevalent in LGBT
Age Group Issues
• HIV (young MSM)
Childhood and
• Suicide attempts and depressions
Adolescence
• Homelessness, violence, harassment
• Mood and anxiety disorders
• Lesbians and bisexual women
Early-Middle
o Use less preventive health services
Adulthood
o More prone to obesity, breast CA
• HIV, family planning
• Implications of long-term hormone use
in transgender people
• Limited social support
Late Adulthood
• Greater fear of stigma and
discrimination compared to younger
members of the community
• Stigma, violence, and discrimination
All Age Groups • Smoking, alcohol consumption,
substance abuse (lesbian and bisexual
women are best to be known at risk)
● Screening and Preventive Measures
→ Mammogram for female patients over 40/50 (lesbians,
bisexual transman even after breast reduction/removal)
→ Transwoman using hormones
→ Prostate exam for male patients (including transwoman
even after vaginoplasty)
→ Pap smear – recommended for every patient at risk of
cervical cancer
→ Anal pap smear (no universal guidelines)

END OF TRANSCRIPT

REFERENCES
Bautista, M.T.G. (2020). Optimal Primary Care for LGBTQ+ Patients:
Understanding LGBTQ+ Issues and Performing a Sensitive History and
Physical Examination [PowerPoint Presentation]. Manila, Philippines:
Faculty of Medicine and Surgery, University of Santo Tomas, Preventive
Medicine 3
PREVENTIVE MEDICINE 3 [COMMUNITY HEALTH]
AY 20-21

27 JAN 21
Dr. Alejandro V. Pineda, Jr.

TABLE OF CONTENTS → Employees accomplish tasks within the stipulated time frame
as a result of effective organization management
I. INTRODUCTION.................................................................... 1
● Employees stay loyal towards their job and do not treat work
II. MANAGEMENT ..................................................................... 1
III. PLANNING............................................................................. 2 as a burden
A. RATIONAL PLANNING ................................................ 2 Staffing (Human Resource)
B. STRATEGIC PLANNING .............................................. 2 ● Manning and keeping positions provided for by the
C. MISSION AND VISION ................................................. 3 organization structure
D. PLANNING MODEL ..................................................... 3 ● Defining manpower requirements for jobs
IV. PHILIPPINE HEALTH AGENDA............................................. 4
● Selecting candidates for positions
V. HEALTH ECONOMICS, MEDICAL ECONOMICS,
PHARMACOECONOMICS .............................................................. 6
● Training or developing candidates and incumbents to
A. HEALTH ECONOMICS ................................................ 6 accomplish tasks effectively
B. MEDICAL ECONOMICS............................................... 6 Directing
C. PHARMACOECONOMICS ........................................... 8 ● Guiding and leading subordinates
END OF TRANSCRIPT.................................................................. 11 ● Subordinates must learn the organizational structure,
REFERENCES .............................................................................. 11
interdepartmental relationship of activities and personalities and
their duties
MUST KNOW BOOK PREVIOUS TRANS
Controlling or Evaluating
● Measures performance against goals and plans, reveals
deviations, helps assure accomplishment of plans
I. INTRODUCTION
● “5-Star Family Physician/Doctor” II. MANAGEMENT
→ Health care provider Basic Steps of Control or Evaluation
→ Teacher/Educator ● Determining models or basis for control which may be based on
→ Scientist/Researcher appraisal of past experiences
→ Administrator/Manager ● Measurement of performance by observations, reports,
→ Social mobilizer statistical data, determine deviations of differences
Organization ● Institution of corrective or remedial measures
● Gone are the days where we rely on attendance/attendance.
● An act of putting into systematic relationships the elements and
Now, the attendance is tied up with productivity.
activities essential to the achievement of an objective or goal.
● Structural framework of management involved in determining Functions of Management
and grouping the different basic tasks, assigning authority ● Planning
and responsibilities, as well as the establishment of → Selecting objectives and strategies, policies, programs and
relationships procedures for achieving them, either for the entire enterprise
● “Backbone of management” or for any organized part
Administration → Undertaking of a predetermined course of action
intended to facilitate the accomplishment of a task,
● Complex process aimed at achieving objectives with the least
work, or mission
possible expenditure of time, energy, and money consistent with
→ “The process of making decisions as to what topic to address
the agreed quality of work
or what problems to attack, and where to direct time and
● Under administration, we look at cost/budget as a relative thing,
resources”
meaning we can still pursue progress/development even with
→ “Making current decisions in the light of their future effects”
meager income if we’re agreed on areas for work
→ Key Elements in Planning
● According to Newman, “administration means guidance,
■ Plan
leadership, and control of the efforts of the groups towards
− How to get from your starting point to your end point
some common goals”.
and what you want to achieve
● According to Theo Haimann, ”administration means overall
determination of policies, setting of major objectives, the
■ Strategy
identification of general purposes and laying down of broad
programmes and projects”. − Broad framework for action which indicates goals,
● It refers to the activities of higher level. It lays down basic methods, and underlying principles
principles of the enterprise. ■ Policy
− Guidelines for practice which set broad goals and the
Management framework for action
● Technique by which purposes and objectives of a particular ■ Programme
human group are determined, clarified, and effectuated − Overall outline of action; collection of activities in a
● Effective management planned sequence leading to a defined goal or goals
→ Required for better coordination among various ■ Priority
departments − The first claim for consideration
● Leads to a peaceful and positive ambience at the workplace ■ Aim or goal
● Gives a sense of security and oneness to the employees − Broad statement of what is to be achieved
RAPACON, VILLANUEVA, A., | VILLAFUERTE, S., VILLEGAS, S., VIZCARRA, WAHAB, XU, YONZON Page 1 of 11
PRV - Community Health 1.01 – Health Administration, Managed Care, and Health Economics (27 JANUARY 2021) Page 2 of 11

■ Objective or target
− Specific goal to be achieved
− SMART objectives (specific, measurable, attainable,
realistic, time-bound)
Principles of Management
● “The principles of management are flexible, not absolute.”
● Division of work
→ Specialization essential to efficiency
■ Efficiency: gaining maximum benefit after a particular
program is implemented with meager resources
● Authority and responsibility
→ Accountability
■ You are accountable for your own contribution
■ Your leader is accountable for everything
■ i.e. Chief Operating Officer (COO), Department Directors Figure 1. Planning paradigm. Obtained from Dr. Pineda’s PowerPoint
● Discipline Presentation
→ Respect for agreements
A. RATIONAL PLANNING
● Unity of command
→ Receive orders from one superior ● “Rational planning models provide a means to guide choices so
● Unity of Direction that decisions are made which represent the best way to achieve
→ One objective, one plan, one head desired results” (Naidoo & Wills, 2000)
● Subordination ● “‘Rational’ approach suggests that the whole range of options
→ For the general interest or welfare should be identified and considered before a comprehensive
● Scalar chain program is drawn up” (McArthy, 1982)
→ Chain of superiors ● Naidoo & Wills, 2000
→ E.g. CEO → COO → Department directors → Unit Heads → Steps in Rational Planning
Committee Heads → Supervisors → Subalterns 1. Assessing needs
● Order 2. Setting aims - what it is you intend to achieve
→ Organization, management 3. Setting objectives - precise outcomes; SMART (Specific,
● Equity Manageable/Measurable, Achievable/Appropriate, Realistic,
→ Justice + kindness Time-limited)
→ Equal opportunities 4. Deciding which methods or strategies will achieve your
● Stability of tenure objectives Definition/Supporting Point
→ Unnecessary turnover → GANTT CHART: plots charts and responsible people for
→ Predictability tasks against timescale
■ If you do well, you will last long 5. Implementing plan and evaluating outcomes in order to make
● Initiative improvements in the future
→ Thinking out and execution
→ Industriousness, creativity, resourcefulness B. STRATEGIC PLANNING
● Esprit de Corps ● “Refers to planning a large-scale activity involving different
→ Team & communication from top to lower level partners and staged interventions” (Naidoo & Wills, 2000)
■ The top level knows what the needs are at the lower level ● Ideal strategic planning is set to 5 years
● Remuneration ● National Objectives for Health—set from 2017 to 2022
→ Maximum possible satisfaction Strategic Management
→ Clear, equitable, real-estate renumeration commensurate ● The set of managerial decisions and actions that determines the
with the position appointed to, and associated task and long-run performance of an organization
required output
Importance of Strategic Management
● Centralization
→ Concentrated vs. dispersed authority 1. It forms the basis of the decision-making process
→ Decentralization of Healthcare in Ph (RA 7160) 2. It results in higher organizational performance
■ Functions of the government are decentralized to the 3. It requires that managers examine and adapt to business
LGUs environment changes
4. It coordinates diverse organizational units, helping them focus
III. PLANNING on organizational goals
Advantages of Management SWOT Analysis
● Achievement of the objectives in the most efficient and ● Tool that identifies the strengths, weaknesses, opportunities
economical manner and threats of an organization
● Use of efficient methods and the development of standards ● Takes the information from an environmental analysis and
necessary for accurate control separate it into internal and external issues
● Integration of the activities of the different units → Internal: issues that we can control
● Reduction of emergency and unexpected problems → External: issues that we cannot control, but can be identified
● Basic, straightforward model that can assess what an
organization can and cannot do
● Determines what may assist the firm in accomplishing its
objectives, and what obstacles must be overcome or minimized
to achieve desired results
PRV - Community Health 1.01 – Health Administration, Managed Care, and Health Economics (27 JANUARY 2021) Page 3 of 11

Mission Statement
● A declaration as to why an organization exists
● Defines the business the organization is currently in
● Concentrates on the present
● Reflects an organization’s core competencies, the basic skills or
products provided
Strategic Planning
● A mission statement identifies a starting point or current state
of business, but a vision statement is necessary for an
organization to determine the direction that should be pursued
D. PLANNING MODEL
Project Planning
● “Refers to planning a specific project which is time-limited and
Figure 2. SWOT Analysis. Obtained from Dr. Pineda’s PowerPoint Presentation aims to bring about a defined change” (Naidoo & Wills, 2000)
Ewles & Simnett, 1999
1. Identify needs and priorities
● Green & Kreuter, 1999
→ Which problem has the greatest impact?
→ Are certain populations at special risk?
→ Which problems are most susceptible to intervention?
→ Which problem is not being addressed by other agencies?
→ Which problem, when addressed appropriately, has the
greatest potential for an attractive yield in benefits?
2. Set aims and objectives
● J. Thomas Butler, 2001
→ Program goals: broad statements of the program’s
intended achievements
→ Objectives: precise statements that map out the tasks
necessary to reach each goal
Figure 3. Application of SWOT Analysis in Strategy Formulation. Obtained from → Educational objectives: broad statements of the programs
Dr. Pineda’s PowerPoint Presentation effect on the agency or on the client
3. Identify appropriate methods for achieving the objectives
Strategic Management Process
● Line of Attack
→ External:
■ Amount of funding
■ Particular expertise
→ Internal:
■ Set SMART objectives
● Approaches
→ Participative small group work
■ Effective at changing attitudes
→ Formal teaching method
■ Effective at imparting specific knowledge
→ Community development
■ Effective at increasing community involvement and
Figure 4. Strategic Management Process. Obtained from Dr. Pineda’s participation
PowerPoint Presentation → Mass media
● Identify the organization’s current mission, goals and strategies ■ Effective in raising people’s awareness on health
● External Analysis issues
→ Explore the environment 4. Identify resources
→ Opportunities, Threats ● Consideration for resources
● Internal Analysis → Funding/Budget (Direct/Fixed costs)
→ Strengths, Weaknesses → People’s skills and expertise
● Formulate Strategies (Stage Interventions) → Materials
● Implement Strategies → Existing policies, plans, facilities and resources
● Evaluate Results 5. Plan evaluation methods
● Evaluation
C. MISSION AND VISION
→ Needed to assess results, determine whether objectives
● “How can any organization, regardless of the type or size, are being met or have been met and find out if the
become truly successful if they cannot answer the fundamental methods used were appropriate and efficient.
questions “why do we exist?” And “where are we going?” → Findings can then be fed back into the planning process in
→ Mission and vision statements answer these order to progress practice
Vision → About identifying values or criteria which will be used to
● Ideals, standards and desired future states determine success
● Encourages everyone in the organization to think about ● Reasons for Evaluations (3Es)
possibilities → To assess what has been achieved – did an intervention
● Communicates what makes the organization unequalled and set have its intended effects? (Effectiveness)
apart from others → To measure its impact and whether it was worthwhile
● Statements of destinations (Efficiency)
PRV - Community Health 1.01 – Health Administration, Managed Care, and Health Economics (27 JANUARY 2021) Page 4 of 11

→ To judge its cost-effectiveness and whether the time, → Core Principles of Quality
money and labor were well-spent (Economy) ■ Equity – users have equal access and benefit from
● Criteria for Evaluation services
→ Effectiveness ■ Effectiveness – services achieve their intended
■ The extent to which aims and objectives are met objectives
→ Appropriateness ■ Efficiency – services achieve maximum benefit for
■ The relevance of the intervention to needs stated costs
→ Acceptability ■ Accessibility – a service is easily available to users in
■ Whether it is carried out in a sensitive way terms of time, distance and ethos.
→ Efficiency ■ Appropriateness – service that which users require
■ Whether time, money and resources are well spent, ■ Acceptability – services satisfy the reasonable
given the benefits expectations of users.
→ Equity ■ Responsiveness – services adapt to the expressed
■ Equal provisions for equal need needs of users.
6. Set an action plan
● Detailed Written Plan
→ Identifies the task, the person responsible for each task,
resources to be used, timescale and means of evaluation
→ Include interim indicators of progress to show if
implementor is proceeding as planned
7. Action, or implement the plan including evaluation
● Keep a log book or diary
→ Note for unexpected problems and how they were dealt
with
→ Note unintended benefits
● Feed notes in the evaluation procedure
● Process evaluation
→ Formative or illuminative evaluation
→ Concerned with assessing the process of programme
implementation
→ Addresses participants perceptions and reactions to Figure 5. Quality Assurance or Audit cycle. Obtained from Dr. Pineda’s
PowerPointPresentation
health promotion intervention, and identifies the factors
which support or impede these activities IV. PHILIPPINE HEALTH AGENDA
→ Useful means to assess acceptability, appropriateness
and equity of an intervention ● PHA 2016-2022 Theme: All For Health Towards Health For All
→ Employs wide range of qualitative or ‘soft’ methods such (Lahat Para sa Kalusugan! Tungo sa Kalusugan Para sa Lahat!)
as interviews, diaries, observations and content analysis ● Persistent Inequities in Health Outcomes
of documents → Every year, around 2000 mothers die due to pregnancy-
● FOUR main questions in process evaluation (WHO): related complications. (maternal mortality rate)
→ Is the program reaching the target group? (Program → A Filipino child born to the poorest family is 3 times more
acceptability) likely to not reach his 5th birthday, compared to one born to
→ Are the participants satisfied with the program? (Program the richest family. (under 5 mortality)
reach) → Three out of 10 children are stunted. (malnutrition,
→ Are all the activities of the program being implemented? deficiency)
(Program integrity) ● Restrictive and Impoverishing Healthcare Costs
→ Are all the materials and components of the program of → Every year, 1.5 million families are pushed to poverty due to
good quality? (Program quality) health care expenditures
● Impact Evaluation → Filipinos forego or delay care due to prohibitive and
→ IMPACT – the immediate effects such as increased unpredictable user fees or co-payments
knowledge or shifts in attitude ■ Leads to Filipinos consulting physicians in the setting of the
■ Baseline data use to forecast a better future state, clinical course of disease (full presentation of signs and
determine whether the set goals are present or not symptoms)
→ Tends to be the most popular choice, as it is easier to do ■ Remember the three stages in the natural history of
→ Can be built into programme as the end stage disease: healthy, preclinical, clinical
● Outcome Evaluation → Php 4,000/month healthcare expenses considered
→ OUTCOME – the longer-term effects including changes in catastrophic for single income families
lifestyle or behavior modification ■ Total fertility rate 3, 1 mother, 1 father = 5 average family
→ More difficult as it involves an assessment of longer-term size with single breadwinner
effects ■ If the breadwinner earns minimum wage or even below
→ More complex and costly minimum (Php 600.00 per 8-hour workday x 5 days in a
→ Often the preferred evaluation method as it measures week x 4 weeks in a month = Php 12,000.00), Php 4,000.00
sustained changes that have stood the test of time monthly healthcare expenses will be catastrophic
● Quality and Audit ● In this situation, the Duterte administration wants to handle our
→ Quality Assurance concerns through the following key areas:
■ Ongoing process of continual assessment and → Universal Health Coverage
improvement of practice → Strengthen Implementation Of RPRH Law
■ Involves setting the standards specifying quality and → War Against Drugs
ensuring consistency → Additional Funds From PAGCOR
→ Audit ● Endpoint of the agenda: Attain Health-Related Sustainable
■ Systematic process of scrutinizing a service or program Development Goal Targets
in order to improve performance
■ May focus on a particular aspect
PRV - Community Health 1.01 – Health Administration, Managed Care, and Health Economics (27 JANUARY 2021) Page 5 of 11

→ Financial Risk Protection – the impoverished should have → Located close to the people (mobile clinic or subsidize
opportunities for medical consultation, pick-up, and transportation cost)
management → Enhanced by telemedicine
→ Better Health Outcomes 3. Universal Health Insurance (Financial Freedom When
→ Responsiveness of the system Accessing Services)
● Values: Equity, Quality, Efficiency, Transparency, Accountability, ● Services are financed predominantly by PhilHealth
Sustainability, Resilience → PhilHealth As The Gateway To Free Affordable Care
Three Guarantees ■ 100% of Filipinos are members
■ Formal sector premium paid through payroll
1. All Life Stages & Triple Burden of Disease (Services for Both
■ Non-formal sector premium paid through tax subsidy
the Well & the Sick)
→ Simplify PhilHealth Rules
● First 1000 days; Reproductive and Sexual Health, Maternal,
■ No balance billing for the poor/basic accommodation &
Newborn, and Child Health, Exclusive Breastfeeding, Food &
fixed co-payment for non-basic accommodation
Micronutrient Supplementation, Immunization, Adolesent
→ PhilHealth As Main Revenue Source For Public Health
Health, Geriatric Health, Health Screening, promotion, and
Care Providers
Information
■ Expand benefits to cover comprehensive range of
● Communicable Diseases
services
→ HIV/AIDS, TB, Malaria, Diseases for Elimination, Dengue,
■ Contracting networks of providers within SDNS
Leptospirosis, Ebola, Zika
● Non-communicable diseases & Malnutrition Strategies / Stage Interventions: ACHIEVE
→ Cancer, Diabetes, Heart disease and their risk factors ● Advance quality, health promotion and primary care
(obesity, smoking, diet, sedentary lifestyle), Malnutrition → Conduct annual health visits for all poor families and special
● Diseases of Rapid Urbanization & Industrialization populations (NHTS, IP, PWD, Senior Citizens)
→ Injuries, Substance abuse, Mental Illness, Pandemics → Develop an explicit list of primary care entitlements that will
(e.g. COVID-19), Travel Medicine, Health consequences become the basis for licensing and contracting arrangements
of climate change / disaster (e.g. injuries from → Transform select DOH hospitals into mega-hospitals with
earthquakes, landslides, typhoons) capabilities for multi-specialty training and teaching and
reference laboratory
→ Support LGUs in advancing pro-health resolutions or
ordinances (e.g. city-wide smoke-free or speed limit
ordinances)
→ Establish expert bodies for health promotion and surveillance
and response
● Cover all Filipinos against health-related financial risk
→ Raise more revenues for health, e.g. impose health promoting
taxes, increase NHIP premium rates, improve premium
collection efficiency.
→ Align GSIS, MAP, PCSO, PAGCOR and minimize overlaps
with PhilHealth
→ Expand PhilHealth benefits to cover outpatient diagnostics,
Figure 6. Leading causes of morbidity. Hypertension is number 3, while the rest
medicines, blood, and blood products aided by health
are communicable diseases. ARI, ALTRI, and Pneumonia still top the list. technology assessment
→ Update costing of current PhilHealth case rates to ensure that
it covers full cost of care and link payment to service quality
→ Enhance and enforce PhilHealth contracting policies for better
viability and sustainability
● Harness the power of strategic HRH development
→ Revise health professions curriculum to be more primary care-
oriented and responsive to local and global needs
→ Streamline HRH compensation package to incentivize service
in high-risk or GIDA areas
→ Update frontline staffing complement standards from
profession-based to competency-based
→ Make available fully funded scholarships for HRH hailing from
GIDA areas or IP groups
→ Formulate mechanisms for mandatory return of service
Figure 7. Leading causes of mortality. Cardiovascular diseases top the list, schemes for all heath graduates
represented by diseases of the heart (#1) and diseases of the vascular system ● Invest in eHealth and data for decision-making
(#2). Most of the diseases on this list are non-communicable diseases. → Mandate the use of electronic medical records in all health
2. Service Delivery Network (Functional Network Of Health facilities
Facilities) → Make online submission of clinical, drug dispensing,
● Services are delivered by networks that are administrative and financial records a prerequisite for
→ Fully functional (complete equipment, medicines, health registration, licensing and contracting
professional) → Commission nationwide surveys, streamline information
→ Compliant with clinical practice guidelines systems, and support efforts to improve local civil registration
→ Available 24/7 & even during disasters and vital statistics
→ Practicing gatekeeping – contextualized in the setting of → Automate major business processes and invest in
the Universal Healthcare Law (signed February 20, 2019) warehousing and business intelligence tools
■ Each Filipino will have a doctor who will keep a → Facilitate ease of access of researchers to available data
registry, and will take care of the needs of their patients ● Enforce standards, accountability, and transparency
at primary care → Publish health information that can trigger better performance
and accountability
PRV - Community Health 1.01 – Health Administration, Managed Care, and Health Economics (27 JANUARY 2021) Page 6 of 11

→ Set up dedicated performance monitoring unit to track Well-being


● Value all clients and patients, especially the poor, marginalized, ● “Well-being is the experience of health, happiness and
and vulnerable prosperity. It includes having good mental health, high life
→ Prioritize the poorest 20 million Filipinos in all health programs satisfaction, a sense of meaning or purpose and ability to
and support them in non-direct health expenditures manage stress” – Psychology Today
→ Make all health entitlements simple, explicit, and widely
Health – Economist’s View
published to facilitate understanding, & generate demand
→ Set up participation and redress mechanisms ● A type of capital that provides services
→ Reduce turnaround time and improve transparency of ● The flow of services from which the stock of services from which
processes at all DOH health facilities the stock of health or “capital” is consumed continuously over a
→ Eliminate queuing, guarantee decent accommodation, and period of time
clean restrooms in all government hospitals ● Stock of health depreciates over time and may be augmented
● Elicit multi-sectoral and multi-stakeholder support for health by medical services
→ Harness and align the private sector in planning supply side Health
investments ● A nebulous concept that defies measurement
→ Work with other national government agencies to address ● In terms of measurement, depends as much on the quantity of
social determinants of health life (number of life-years remaining) as it does on quality of life
→ Make health impact assessment and public health ● Like any durable good, generates flow of services that yield
management plan a prerequisite for initiating large-scale, satisfaction or “utility”
high-risk infrastructure projects
→ Collaborate with CSOs and other stakeholders on budget Economics
development, monitoring and evaluation ● Economics is the study of how society decides what gets
produced, how and for whom” – Fischer and Dornbush
→ Finite resources
→ Buy a European assembled car or a Japanese brand?
→ Buy a local ice cream or an imported brand?
● It examines the costs and benefits of resource allocation
Applied Economics
● Practical application of the principles and concepts of
economics
A. HEALTH ECONOMICS
● Is a discipline that analyzes and supply and demand for health
Figure 8. Summary of the Philippine Health Agenda 2016-2022.
care and provides a structure to understand the choices made
● The National Objectives for Health (NOH) 2017-2022 therein
→ Serves as the medium-term roadmap of the Philippines ● Studies the supply and demands of health care resources and
towards achieving universal healthcare (UHC) the impact of health care resources on the population
→ It specifies the objectives, strategies, and targets of the
Department of Health FOURmula One Plus for Health (F1 B. MEDICAL ECONOMICS
Plus for Health) built along the health system pillars of ● Deals with the health care delivery system and practice of
financing, service delivery, regulation, governance, and medicine
performance accountability ● Why Medical Economics?
→ Academic subject prescribed by law and medical curriculum
→ To enable the young medical graduates to cope with the real
life practice environment
→ To define the role of the physician in the existing health care
delivery system in the country
→ To appreciate the importance of economic and financial
perspectives in effecting better patient care (preventive,
curative, rehabilitative and pallative aspects)
→ To recognize the interplay of various factors that affect the
practice of medicine
Table1. Three Types of Economic Systems
Economic
Definition Examples
Figure 9. Cover of National Objectives for Health 2017-2022. The Philippine System
Heath Agenda is incorporated in this important document. This is the bible for Capitalism Business Orientation Philippines
health delivery in terms of the system, economics, reduction in the burden of Socialism Taxation and Capitation United Kingdom
people, & disease state for Filipinos.
Communism Government Controlled China, Cuba
V. HEALTH ECONOMICS, MEDICAL ECONOMICS, Economic Terms
PHARMACOECONOMICS
● Demand
Health → How many of the product will be brough at a particular time at
● “State of complete physical, mental and social well-being and a certain price?
not merely the absence of disease or infirmity” - WHO ● Supply
● State of being associated with freedom from disease and illness → How many of the products will be offered for sale at particular
that includes a component (WELLNESS) that is linked with time at a certain price?
positive well-being ● Prices
Wellness → Determined by the forces of demand and supply
● Individual and personal state of positive health exemplified by ● Gross Domestic Product (GDP)
biological and psychological well-being and quality of life → The value of all goods produced and services rendered in a
year
PRV - Community Health 1.01 – Health Administration, Managed Care, and Health Economics (27 JANUARY 2021) Page 7 of 11

● Gross National Product (GNP)


→ The gross domestic product adjusted with the net factor
income from the rest of the world
● Recession
→ A slow down or decline in the GNP
● Inflation
→ Rapid rise in price caused by a total demand exceeding supply
● Business
→ Goods
→ Service
● Consumption
→ The process of using goods and services
● Consumers
→ The people who used goods and services
● Consumer Goods and Services Figure 10. Generic Structure of An HMO. Obtained from Dr. Pineda’s
→ Goods and Services which satisfy people’s economic wants PowerPoint Presentation
and difficulties
● Business Classification of HMO’s as per Ownership
→ A term used to identify the efforts of a group of individual, or a ● Investor – Based HMO’s
single individual to produce and distribute useful goods or ● Community – Based HMO’s
services ● Cooperative – Based HMO’s
● Capitalist Basic Misconceptions of HMO’s
→ Anyone who owns a business or share a business, has taken ● It is a health insurance
the risk of investing money in the hope of producing a profit, ● It is a pre-need insurance
and earning an income from such
Table 3. Important Difference Between an HMO and Health Insurance
Objectives of Business Features HMO Insurance
● Profit Method of Payment Direct Settlement Reimbursement
→ Difference between costs and receipts Scope of Services More Limited
→ Primary objectives of business comprehensive
● Social Responsibility Choice of Hospital Limited Wider
→ Obligation of business to contribute to the good of society in a and Physician
meaningful way Membership Fee Higher Freedom of Choice
Types of Business Cost Effectiveness Present Not present
and Quality
● Industrial Business
Assurance
→ Production (Mining, Manufacturing) Mechanisms
● Agriculture Business Supervising DOH/IC IC
→ Production (Farming, Livestock) Government Agency
● Commercial Business
→ Marketing (Wholesaler, Retailers)
→ Finance (Banks/Investments)
→ Services (Hotels, Medical Practice)
Social/Insurance
● PhilHealth (philhealth.gov.ph)
→ Philippine Health Insurance Commission
● ECC (evv.gov.ph)
→ Employees Compensation Comission
● Private Sector
→ Out of pocket Practive) – Most common in PH
→ Health Insurance
→ Managed Care (HMO’s)
→ Employer’s Based Health Care Program
→ Schools
→ NGO’s
Managed Care Figure 11. HMO Basic Health Care Package
● “An organized system of health care delivery, offering a Benefits Package
comprehensive set of benefits, in which members are voluntarily ● Preventive Health Care
enrolled, and paying for a fixed, prepaid period.” ● Outpatient Services
Table 2. HMO’s in the Philippines (2011+) ● Inpatient Services
Health Maintenance Organization ● Emergency Care
PHILCARE (PhilamCare) ● Optional Benefits
MEDICARD → Dental
MAXICARE → Maternity
HMI → Prescription Medicines
INTELLICARE (Pryce Care) → Term Insurance
I.CARE → Annual PE/Pre-Employment
COCOLIFE Important Features in HMO
MEDOCARE
CARITAS HEALTH SHIELD ● Pre-exisiting diseases disclosure
MED ASIA ● Exclusions
● Limitations
PRV - Community Health 1.01 – Health Administration, Managed Care, and Health Economics (27 JANUARY 2021) Page 8 of 11

Preferred Provider Organizations ■ Intangible costs


− Pain, worry and distress experienced by the patient
● Combines the philosophy of fee for service with an HMO type of
and family (impossible to measure).
health care plan
● Price
● System that comprise a panel of health care providers and
→ The amount of money required to purchase an item
includes physicians, hospitals, diagnostic center and other
entities to form a contractual health team concept Economic Evaluation
● Fee schedule for each services is negotiated and agreed upon ● Provides us with a framework for drawing up a balance sheet of
before the services is provide costs and benefits
● Patients are given a choice of services within the system ● All economic evaluations have a common structure in that they
● Cost-effective health care is encouraged involve explicit measurements of inputs (‘costs’) and outcomes
Economic Evaluation In Healthcare (‘benefits’)
Other key definitions
● Medicine effectiveness
→ The effects of a medicine when used in real-life situations
● Medicine efficacy
→ The effects of medicine under clinical trial conditions
How to measure benefits from medical intervention
● Natural units
→ i.e., strokes prevented, years of life saved, ulcers healed, etc.
● Utility units
→ QALY (quality-adjusted life-year) is a widely used measure
that combines quality and quantity of life in a single score
● Associated Economic Benefit
Figure 12. Personal Health Care. Obtained from Dr. Pineda’s PowerPoint
→ Usually measured in money, as in the economic benefit for an
Presentation employee returning to work after an illness.
Pharmacoeconomic Evaluation
● Dominated by one simple theoretical concept: COST
EFFECTIVENESS
Pharmacoeconomics
● Evaluates the cost (expressed in monetary terms) and effects
(expressed in terms of monetary value, efficacy or enhanced
quality of life) of a pharmaceutical product.
Table 4. Pharmacoeconomic Methodologies
Methodology Cost measurement Outcome unit
unit
Cost-minimization Pesos (assumed
to be equivalent in
comparative
groups)
Figure 13. Out of Pocket Expenditure. Obtained from Dr. Pineda’s PowerPoint Cost-benefit pesos
Presentation
Cost-effectiveness Natural units (life-
C. PHARMACOECONOMICS pesos years gained,
● The description and analysis of the costs of drug therapy to mg/dL blood
health care systems and society (Townsend RJ, Drug Intell clin glucose, mmHg
blood pressure)
pharmacology 1987).
● Is a sub-discipline under health economics. Cost utility Utility units
(QALY)
● It compares the value of one pharmaceutical drug to drug therapy
to another (Pharmacoeconomics, Walley, Haycox, Bolland, Cost Minimization Analysis
Elsevier Science Ltd, 2004).
● Simplest tool
Key definitions → i.e., this is applied when comparing the cost of two drugs of
● Utility equal efficacy and tolerability
→ “usefulness” that individuals placed on goods and services ● When conducting this study, the author needs to measure all
(i.e., Ferrari = “speed”, Toyota = “safety”). costs (resource expenditures) inherent to the delivery of the
● Efficiency therapeutic intervention and relevant to the pharmacoeconomic
→ Measures how well resources are used in order to achieve a perspective.
desired output ● It assumes that all clinical outcomes are equivalent
● Cost ● Bioequivalence
→ The total resources consumed in producing a good or service → Clinical Importance
→ Cost classifications: ■ Bioequivalence studies are usually performed to compare
■ Direct costs the rate and extent of absorption of a new drug product with
that of a recognized standard.
− Staff costs, drug acquisition costs, capital costs from
■ The standard is usually the innovator product, which has
the perspective of health care provider
undergone extensive clinical testing.
■ Indirect costs
■ Two products are considered bioequivalent when there is
− Costs experienced by the patient, patient’s family or
absence of a significant difference in the rate and extent to
society (loss of productivity, loss of earnings, etc), from
which the active ingredient becomes available at the site of
the perspective of society as a whole.
action, when administered at the same dose under similar
conditions.
PRV - Community Health 1.01 – Health Administration, Managed Care, and Health Economics (27 JANUARY 2021) Page 9 of 11

Table 5. Pharmacoeconomic Methodologies Table 8. QALY “League table”


Cost categories Drug A Drug B Drug C
Acquisition cost
Medrep salary
per coverage
GMP cost
Marketing cost
Total cost
Cost Benefit Analysis (CBA)
● Health care is a human right
● However, because of limited resources, it must be allocated
● How much will it cost to save a human life?
● Allocation of health resources- costs, benefits
● In health care, is the analysis of health care resource
expenditures relative to possible medical benefit.
● This analysis may be helpful and necessary in setting priorities How do we apply the principles of Pharmacoeconomics to the
when choices must be made in the face of limited resources. use of antihypertensive agents?
● Is used in determining the degree of access to, or benefits of
health care to be provided
Cost Effectiveness Analysis
● Primary tool for comparing the cost of a health intervention with
the expected health gains
● An intervention can be understood to be any activity using
human, financial, and other inputs, that aims to improve health.
● The health gain might be reducing the risk of a health problem,
reducing the severity or duration of an illness or disability, or
preventing death. Figure 14. Hypertension as a risk factor for CVD. Obtained from Dr. Pineda’s
● Example: PowerPoint Presentation
→ If the health outcome is the same, say preventing death
from hypertension either by modifying risk factors or by
treating the disease, then analysts need only compare the
costs of different interventions that can achieve that
outcome.
→ Desired outcome:
■ Preventing death from hypertension
■ Reducing mortality
→ Cost:
■ Primary prevention program
Figure 15. Hypertension as a leading cause of morbidity and mortality. Obtained
■ Management program from Dr. Pineda’s PowerPoint Presentation
Cost Utility Analysis (CUA)
● In health economics, the purpose of CUA is to estimate the ratio
between the cost of a health-related intervention and the benefit
it produces in terms of the number of years lived in full health by
the beneficiaries (QALY).
● Hence it can be considered a special case of cost-effectiveness
analysis.
● We measure cost in terms of monetary values and outcome
in terms of life-years saved.
● Intervention consequence is measured in terms of quality and
Figure 16. Elevated BP among Adults (LEFT); Elevated BP among Elderly
quantity of life. (RIGHT). Grouped by age and sex in the Philippines, 2015 vs 2018 data.
● How much will it cost to save the life of one person for one year Obtained from Dr. Pineda’s PowerPoint Presentation
or extend the life of a person for one year?
Quality Adjusted Life Years (QALY)
● Measure of health stock
● Commonly employed in evaluating cost-effectiveness of
expenditure on health.
● The values of the state (s) of health that a person experiences
are weighted by the length of time spent in each state.
● WHO-DALY (extent of loss due to morbidity and mortality)
Figure 17.Hypertension profile in the Philippines. Obtained from Dr. Pineda’s
PowerPoint Presentation
PRV - Community Health 1.01 – Health Administration, Managed Care, and Health Economics (27 JANUARY 2021) Page 10 of 11

● Cost Benefit Analysis


Table 6. Cost of initial hypertension laboratory work-up in 3 representative
institutions

Table 7. Annual cost of hypertension treatment in 3 representative institutions


Figure 18. Lowering BP is imperative in reducing CV risks.

● Cost Minimization

● Cost Effectiveness Analysis


→ Clinical considerations:
■ Drug A is better than drug B as far as clinical outcome is
concerned
■ Drug A can lower BP better than drug B, but drug A costs
Figure 19. JNC-8 Hypertension Treatment Choices
more than drug B.
■ Question: Am I willing to pay the extra cost of Drug A
→ Bioequivalence Study for the additional benefit that I get from it?
Table 9. Using cost-effectiveness analysis

Figure 20. Semilogarithmic plots of the mean losartan plasma concentration vs.
sampling time of New losartan and originator drug 50mg tablets. The lower the
upper limits of the 90% CI for the logarithmically-transformed data lie within the
Figure 21. Cost-effectiveness of certain Interventions for CVD.
bioequivalence criteria of 80-125%. (The clinical phase of the study was done at
DR. Victor R. Potenciano Medical Center and approved by the Medical Ethics
Committee in July 2004)
PRV - Community Health 1.01 – Health Administration, Managed Care, and Health Economics (27 JANUARY 2021) Page 11 of 11

Table 10. Benefits of various treatments in terms of NNT

● Physicians are best advised to understand the costs and quality-


of-life issues necessary to make cost-effective treatment
decisions while finding the best outcome for patients.
● Evidence based practice + Pharmacoeconomincs + Clinical
Experience = QUALITY CARE
→ Using evidences and factoring pharmacoeconomic issues into
treatment options should result in even better medical care for
our patients.
Sound recommendations to reduce the cost of hypertension
treatment
● Understand principles and strategies under pharmacoeconomics
● Emphasize and maximize the role of lifestyle changes and non-
pharmocological options including DASH diet, physical activity
and lifestyle changes
● Request labs and tests only when they will affect the
management of the hypertensive patient
● Work with a accredited reliable laboratory that charges
reasonably
● Use anti-HTN and other drugs preribed for co-morbidities with
evidence-based indications
● Use drugs with evidence-based indications
● Use cheaper but equivalent forms of the drug (Bioequivalent)
● Use the least amount of drug necessary to achieve the desired
effect and if necessary and advantageous combination regimen
(separate or fixed dose)

END OF TRANSCRIPT

REFERENCES
Pineda, A. V., Jr. (2021), Health Administration, Managed Care, and Health
Economics [PowerPoint Presentation]. Manila, Philippines: Faculty of
Medicine and Surgery, University of Santo Tomas, PrevMed 3
PREVENTIVE MEDICINE 3 [COMMUNITY HEALTH] AY 20-21

Iluminada Meneses Lerma, MD 3 FEB 21

TABLE OF CONTENTS II. HEALTH PROMOTION TRIAD


I. HEALTH PROMOTION .......................................................... 1
A. DEFINITION .................................................................... 1
B. BASIC CHARACTERISTICS ............................................ 1
C. WHO APPROACHES IN HEALTH PROMOTION............. 1
II. HEALTH PROMOTION TRIAD............................................... 1
A. DISEASE PREVENTION ................................................. 1
B. HEALTH EDUCATION ..................................................... 1
C. HEALTH PROTECTION................................................... 2
III. STRATEGIES FOR HEALTH PROMOTION .......................... 2
A. EDUCATIONAL ............................................................... 2
B. ORGANIZATIONAL ......................................................... 2
C. POLITICAL/LEGISLATIVE ............................................... 2
D. COMMUNITY/SOCIAL ..................................................... 2
E. ECONOMIC ..................................................................... 2
REFERENCES ................................................................................ 2

MUST KNOW BOOK PREVIOUS TRANS


Figure 1. Health Promotion Triad.
● Composed of health education, prevention and health protection.

I. HEALTH PROMOTION A. DISEASE PREVENTION


● There are three levels:
A. DEFINITION → Primary
● “A planned combination of educational, political, regulatory, and → Secondary
organizational supports for actions and conditions of living → Tertiary
conducive to the health of the individuals, groups or communities”
(Green & Kreuter, 1999)
● Definition by WHO, 1986
→ The process of enabling people to increase control over and
improve their health…
→ A commitment to dealing with challenges of reducing
inequities, extending the scope of prevention, and helping
people to cope with their circumstances…
→ Creating environments conducive to health, in which people
are better able to take care of themselves.
B. BASIC CHARACTERISTICS
● According to the Working Group on Concepts and Principles in
Health Promotion, 1987
→ Enabling the people to take control over, and responsibility for,
their health as an important component of everyday life.
→ Requiring the close cooperation of sectors beyond their health
services.
Figure 2. The Phases of Prevention in relation to natural history of disease.
→ Combining diverse, but complimentary methods or
approaches. Primary Prevention
→ Encouraging effective and concrete public participation. ● Action taken to avert the occurrence of disease
C. WHO APPROACHES IN HEALTH PROMOTION → Immunization and health education
● There are three approaches to health promotion: Secondary Prevention
→ Setting Approach ● Action taken to identify diseases at their earliest stages and to
■ Healthy cities, communities, work places, schools and apply appropriate treatments to limit their consequences and
hospitals severity
→ Population-based Approach → Screening and surveillance
■ Children, adolescents, women, elderly, workers, etc.
Tertiary Prevention
→ Issues-based Approach
■ Healthy practices addressing major and common risk ● Specific interventions to assist diseased or disabled persons in
factors in areas of diet and nutrition, physical activities, limiting the effects of their diseases or disabilities; also may
tobacco, injury prevention, safe sex, patient safety, food include activities to prevent recurrences of a disease.
safety and others. → Rehabilitation and physical therapy
B. HEALTH EDUCATION
● Planned process
● Combines a variety of educational experiences
● Facilitates voluntary adaptations or establishment of behavior
conducive to health
RAPACON, RAMOS, P. Page 1 of 3
PRV - Mod 1.02 – Health Promotion and Interventions (03 FEBRUARY 2021) Page 2 of 3

● Three learning domains: B. ORGANIZATIONAL


→ Cognitive ● May be from the following:
→ Psychomotor → Government agencies
→ Affective → Private sector
Cognitive Domain → Non-Governmental Organizations (NGO)
● Comprises information and knowledge ● Examples:
● Information gained from health education can be → Annual hearing and vision screening in schools
→ A new information → Automobile, bicycle, and firearm safety programs conducted
→ A reinforcement by law enforcement agencies
→ Development of support groups by nonprofit organizations
Psychomotor Domain
and facilities that provide services to people with special
● Deals with skill acquisition and reinforcement needs
Affective Domain
● Mainly concerned in
→ Habit formation
→ Behavior change
→ New practice
C. HEALTH PROTECTION
● Comprises legal or fiscal controls, other regulation policies, and
voluntary codes of practice, aimed at the enhancement of
positive health and the prevention of ill-health.
→ Examples:
■ Philippine Medical Act of 1959
■ PHILHEALTH
■ Generics Law; National Drug Policy and Formulary
■ Dangerous Drug Act
■ Cheaper Medicine Bill
Figure 3. 2011 DOH Health Promotions.
■ Code of Sanitation of the Philippines
■ Disability Act C. POLITICAL/LEGISLATIVE
■ Senior Citizens Law
■ Child Protection Laws and Policies ● May be from the following:
■ Clean Air Act → Individuals
■ MMDA Waste Management Policies ■ President
■ Executive Order No. 663 ■ Governor
■ Mayor
− Implementing the National Commitment for “Bakuna and
→ Senate/Congress
Una sa Sanggol at Ina”, attaining WHO’s goals to
● Examples:
eliminate measles and neonatal tetanus, eradicate polio,
→ Passage of laws requiring use of helmets while driving
control hepatitis B, and other vaccine-preventable
motorcycles and bicycles
diseases.
→ Regulations requiring agencies and companies to monitor air
■ Presidential Proclamation No. 1168
pollution and governmental actions to reduce it
− November 10, 2006
→ Regulations aimed at reducing youth access to tobacco
− Declaring the month of November of every year as products and alcohol.
Malaria Awareness Month
■ Presidential Decree No. 996 D. COMMUNITY/SOCIAL
− Providing for compulsory basic immunization for infants
● May be from the following:
and children below eight years of age
→ Individuals
III. STRATEGIES FOR HEALTH PROMOTION → Communities
● Classified as: ● Initiatives of non-specific groups
→ Educational → Community and Social ● Examples:
→ Organizational → Economic → Organization and training of out of school youth to reduce
→ Political/Legislative vulnerability to sex or drug crimes
● Classification of these strategies may overlap → Formation of neighborhood walking clubs
→ E.g. Senior Citizens Act and Disability Act → Health fairs at shopping malls
■ Economic and Legislative strategies E. ECONOMIC
A. EDUCATIONAL ● Involves giving incentives in cash or in kind
● Aims primarily at the voluntary actions people can take on their ● Examples:
own part, individually or collectively, for their own health or the → Tax incentives to landlords of low-income housing to
health of others and the common good of the community. encourage maintenance of property and reduction of pest
● Examples: infestation
→ Stress management classes for middle-management → Incentives from insurance companies to those who practice
employees in the work place healthy lifestyles
→ Educational programs designed to reduce personal → Incentives from employers to employees who stay healthy
vulnerability to crime and do not miss work
→ Primary school programs to develop the skills to cope with REFERENCES
peer pressure Lerma, I.M., (2021), Health Promotion [PowerPoint Presentation]. Manila,
Philippines: Faculty of Medicine and Surgery, University of Santo Tomas,
PREVMED 3
PREVENTIVE MEDICINE 3 [COMMUNITY HEALTH] AY 20-21

26 AUG 20
Sandra S. Hernandez, MD, MPH 3 FEB 21

TABLE OF CONTENTS ● 1998


→ Medicare functions for the private sector workers previously
I. HISTORY ............................................................................... 1
administered by Social Security System (SSS)
II. ENABLING LAWS .................................................................. 1
III. GUIDING PRINCIPLES AND OBJECTIVES .......................... 1 → Indigent programs in partnership with local government units
A. GUIDING PRINCIPLES.................................................... 1 ■ Covers all families in the DSWD’s national household
B. GENERAL OBJECTIVES ................................................. 2 targeting system for poverty reduction
IV. HEALTH CARE PROVIDERS ................................................ 2 ● 1999
A. HEALTH CARE INSTITUTIONS ...................................... 2 → Covers self-employed and the informal sector
B. HEALTH CARE PROFESSIONALS ................................. 2 ● 2005
V. MEMBERSHIP AND CONTRIBUTIONS................................. 2 → Medicare functions from the Overseas Welfare
A. FORMAL ECONOMY....................................................... 2 Administration (OWA) for Overseas Filipino Workers
B. INFORMAL ECONOMY ................................................... 2
C. INDIGENT MEMBERS ..................................................... 3 II. ENABLING LAWS
D. SPONSORED MEMBERS ............................................... 3 ● 1995
E. CESSATION FROM FORMAL EMPLOYMENT OR → RA 7875 “National Health Insurance Program (NHIP) of
COVERAGE ............................................................................... 3 1995”
F. LIFETIME MEMBERS ...................................................... 3 ■ Established the Philippine Health Insurance Corporation
G. PHILHEALTH IDENTIFICATION NUMBER AND HEALTH (Phil Health)
INSURANCE ID CARD ............................................................... 3 ● 2003 amendment
VI. BENEFIT ENTITLEMENTS .................................................... 3 → RA 9241 and its IRR
VII. PAYMENT CLAIMS................................................................ 3 ● 2012 amendment
A. REPLACEMENT OF FEE FOR SERVICE ........................ 3
→ RA 10606 “National Health Insurance Act”
B. CASE BASED PAYMENTS .............................................. 3
→ Quasi-Judicial Provisions of the revised IRR of R.A. 10606
C. GUIDELINES FOR CLAIMS PAYMENT ........................... 6
D. Z BENEFIT PACKAGES .................................................. 6 ● 2013
E. MANDATORY PHILHEALTH COVERAGE ...................... 6 → Revised IRR of the National Health Insurance Act of 2013
VIII. COVID-19 .............................................................................. 7 (RA 7875 As Amended by RA 9241 and 10606)
A. INPATIENT CARE: GUIDELINES AND BENEFIT ● 2019
AVAILMENT ............................................................................... 7 → RA 11223 “Universal Health Care Act”
B. BENEFIT PACKAGE FOR TESTING FOR SARS-COV-2 7 ■ Every Filipino citizen shall be included in the NHIP
IX. RELATED LEGISLATIONS .................................................... 8
A. REPUBLIC ACT NO. 6675 “GENERICS ACT OF 1988”... 8 III. GUIDING PRINCIPLES AND OBJECTIVES
B. REPUBLIC ACT NO. 9502 “UNIVERSALLY ACCESSIBLE
A. GUIDING PRINCIPLES
CHEAPER AND QUALITY MEDICINES ACT OF 2008” .............. 8
● Allocation of National Resources for Health
→ Give priority to health as a strategy for bringing about faster
MUST KNOW BOOK PREVIOUS TRANS economic development and improving quality of life
● Universality
UPDATE: Information from the uploaded powerpoint presentation on Related → Provide all citizens with the mechanism to gain financial
Legislations were added to this updated trans. access to health services in combination with other health
programs
I. HISTORY ● Equity
→ Provide for uniform basic benefits
→ Access to care must be a function of a person’s health needs
than ability to pay
● Responsiveness
→ Adequately meet the needs for personal health services at
various stages of a member’s life
● Social Solidarity
→ Guided by community spirit
→ Enhance risk sharing among income groups, age groups and
persons of differing health status and residing in different
geographic areas
Figure 1. History
● Effectiveness
● 1995 → Balance economical use of resources with quality of care
→ National Health Insurance Program ● Innovation
■ Administered by the Philippine Health Insurance → Adapt to changes in medical technology, health service
Corporation (Phil Health) organizations, health care provider payment systems, scopes
■ Mandates to provide health insurance coverage to all of professional practice and other trends in the health sector
Filipinos ● Devolution
● 1997 → Implemented in consultation with local government units
→ Medicare functions for government workers from the → Subject to overall policy directions that are set forth by the
Government Service Insurance System (GSIS) national government
● Fiduciary Responsibilities

DE GUZMAN AO, ESTRADA PD, ESTRELLA AV | DELA PENA, A, DIMALIBOT, A, DELA CRUZ, F Page 1 of 8
Updated by D2022 (3 Feb 21): RAPACON, RAMOS, P.
[PREV MED3 – Community Health] 1.02 – PHILHEALTH AND RELATED LEGISLATIONS (26 August 2020; 3 February, 2021) Page 2 of 8
→ Provide effective stewardship, funds management and V. MEMBERSHIP AND CONTRIBUTIONS
maintenance of reserves ● Member
● Informed choice → Any person whose premium contributions have been
→ Choose from among accredited health care providers regularly paid to the program
● Maximum Community Participation → May be a paying member, an indigent member, a sponsored
→ Build on existing community initiatives for its organization member, or a lifetime member or otherwise known as
and human resource requirements covered member
● Compulsory Coverage ● Dependent
→ Enroll to all citizens of the Philippines in order to avoid → Legal dependents of the member
adverse selection in social inequity ■ Legitimate spouse who is not a member
● Cost sharing ■ Unmarried and unemployed legitimate, legitimated,
→ Continuously evaluate its cost sharing schedule to ensure acknowledged, illegitimate children and legally adopted or
that costs borne by the members are fair and equitable and stepchildren below twenty-one (21) years oof age
that the charges by health care providers are reasonable ■ Children who are twenty-one (21) years old or above but
● Professional responsibility of health care providers suffering from congenital disability, either physical or
→ Assure that all participating health care providers are mental, or any disability acquired that renders them totally
responsible and accountable in all their dealings with the dependent on the member for support, as determined by
corporations and its members the Corporation
● Public health services ■ Foster child as defined in RA 10165, otherwise known as
→ Focus on the provision of benefit packages for personal the Foster Care Act of 2012
health services while the government shall provide public ■ Parents who are sixty (60) years old and above, not
health services for all groups otherwise an enrolled member, whose monthly income is
● Quality of services below an amount to be determined by the Corporation in
→ Promote the improvement and the quality of health services accordance with the guiding principles set forth in the Act
● Care for the indigent ■ Parents with permanent disability regardless of age as
→ Provide basic package of needed personal health services to determined by the Corporation, that renders them totally
indigents through premium subsidy or direct supervision from dependent on the member for subsistence
the government
A. FORMAL ECONOMY
B. GENERAL OBJECTIVES
● Those with formal contract and fixed terms of employment
● Provide all citizens of the Philippines with the mechanism to including workers in the government and private sector, whose
gain financial access to health services premium contribution payments are equally shared by the
● Establish the program to serve as the means to help the people employee and the employer
pay for health care services
● Prioritize and accelerate the provision of health services to all Members in the Formal Economy
Filipinos, especially that segment of the population who cannot ● Government employee
afford these services ● Private employee
● All other workers rendering services, whether in government or
IV. HEALTH CARE PROVIDERS private offices, such as job order contractors, project-based
contractors, and the like
A. HEALTH CARE INSTITUTIONS ● Owners of Micro Enterprises
● Hospitals ● Owners of Small, Medium, and Large Enterprises
● Out-patient clinics ● Household Help – as defined in RA 10361 or “Kasambahay
→ Rural health unit or health center Law”
→ Dispensary or infirmary ● Family Drivers
→ Birthing home Formal Economy Monthly Contributions
→ Medical outpatient clinic ● Shared equally by the employer and employee at a prescribed
● Free standing dialysis clinics rate set by the Corporation not exceeding five percent (5%) of
● Ambulatory surgical clinics their respective basic monthly salaries
● Health maintenance organizations (HMO) ● Deducted and withheld automatically by the employer from the
● Community-based Health Care Organizations (CBHCOs) former’s salary, wage or earnings
● Pharmacies ● Employer’s counterpart shall not, in any manner be charged to
● Other health care institutions licensed by DOH the employee
B. HEALTH CARE PROFESSIONALS ● Remitted by the employer on on or before the date prescribed
● Physicians by the Corporation
● Dentists ● Failure of the employer to remit the required contribution and
● Nurses submit the remittance list shall make the employer liable for
● Midwives reimbursement of payment of a properly filed claim
● Pharmacists B. INFORMAL ECONOMY
● Other duly licensed health care professionals ● Contributions shall be based primarily on household earnings
Accreditation Requirements for Health Care Professionals and assets
● They must be duly licensed to practice in the Philippine by PRC → A member who has missed or has an unpaid premium
● They must be members of the Program with qualifying premium contribution is allowed to pay retroactively as prescribed the
contributions corporation
● They must comply with the provisions set forth in the Members in the Informal Economy
performance commitment for professionals ● Migrant workers
● They must comply with any other requirements that may be
determined by the corporation
[PREV MED3 – Community Health] 1.02 – PHILHEALTH AND RELATED LEGISLATIONS (26 August 2020; 3 February, 2021) Page 3 of 8
→ Maybe documented or undocumented Filipinos who are → Out-Patient care
engaged in a renumerated activity in another country of ■ Services of health care professionals
which they are not a citizen ■ Diagnostic, laboratory, and other medical examination
● Informal sector services
→ Includes street hawkers, market vendors, pedicab and ■ Personal preventive services
tricycle drivers, small construction workers and home-based ■ Prescription drugs and biologicals, subject to the
industries and services limitations of the Act
● Self-earning individuals ■ Health Education
→ Those who render services or sell goods as a means of → Emergency and transfer services
livelihood outside of an employee-employer relationship → Health education packages
→ Includes professional practitioners such as doctors, lawyers, → Other health care services that the corporation and the DOH
engineers, artists, architects, businessmen, actors, shall determine to be appropriate and cost-effective
professional athletes, coaches, etc. ● Entitlement to Benefits
● Filipino with dual citizenship → Paid premium contribution for at least three (3) months within
● Naturalized Filipino citizens the six (6) months prior to the first day of availment
● Citizens of other countries working and/or residing in the → Paid in full the required premium for the calendar year
Philippines → The following need not pay the monthly contributions to be
entitled to the Program’s benefits
C. INDIGENT MEMBERS ■ Retirees and pensioners of the SSS and GSIS prior to
● A person who has no visible means of income, or whose March 4, 1995
income is insufficient for family subsistence, as identified by the ■ Members of PhilHealth who have reached the age of
DSWD, based on specific criteria set for this purpose in retirement as provided for by law, not gainfully employed
accordance with the guiding principles set forth in Article 1 of or continuing their practice as professional and have met
the Act the required premium contributions of at least 120 months
D. SPONSORED MEMBERS → In case of death of the member, the dependents of the
deceased member shall continue to avail of the benefits for
● A member whose contribution is being paid by another
the unexpired portion of the coverage or until the end of the
individual, government agency, or private entity according to the
calendar year, whichever comes first.
rules as may be prescribed by the Corporation
VII. PAYMENT CLAIMS
CESSATION FROM FORMAL EMPLOYMENT OR
E.
COVERAGE ● Provider Payment Mechanisms
→ Fee-for-Service (FFS) payments
● Indigent, Sponsored Member or Migrant Workers
→ Case Rate/Based payments
● A member separated from formal employment or whose
→ Capitation of health care professionals and institutions, or
coverage as a Sponsored member or as an Indigent or as a
networks of the same including HMOs, medical cooperatives,
Migrant worker has ceased should pay the required premium as
and other legally formed health service groups
self-earning individuals to ensure continuous entitlement to
→ Global budget
benefits
→ Such other provider payment mechanisms that may be
F. LIFETIME MEMBERS determined
● Retirees/ pensioners (Government/Private) A. REPLACEMENT OF FEE FOR SERVICE
● Uniformed members of the AFP, PNP, BJMP, and BFP
● Shift from Fee-for-Service to Case Rate payments for medical
● Members off PhilHealth who have reached the age of retirement
and surgical cases because of developments that were taking
as provided by law and have met the required premium
place in the healthcare industry.
contributions of at least 120 months, regardless of their
● The most important of which was to provide optimal financial
employer/s’ or sponsor’s arrears in contributions and is not
protection, especially to the most vulnerable groups
included in the sponsored program nor declared as dependent
● Free for service was replaced by case rate or case based
by their spouse or children payments
G.PHILHEALTH IDENTIFICATION NUMBER AND HEALTH → Inefficiency
INSURANCE ID CARD → Overutilization of diagnostic procedures
● A permanent and unique PhilHealth Identification Number (PIN) → Unnecessary health care services
for every member → Wasteful payments
● Absence of the ID card shall not prejudice the right of any → Inequity when comparing payments to private and
member to avail of benefits or medical services under the government health care institutions
program → Experience form other countries all over the world shows that
Case Based Payment is preferred over FFS
VI. BENEFIT ENTITLEMENTS
B. CASE BASED PAYMENTS
● Benefits
→ Services that the Program offers to members, subject to the ● General Policies
classification and ualifications provided for in this Rules → All claims for medical conditions and procedures
● Benefit Package submitted to PhilHealth shall be paid using case rates
→ In-Patient care → All ICD (medical conditions) and RVS codes
■ Room and board (procedures) will be given rates
■ Services of health care professionals → The objective is to reduce the out-of-pocket expenditures
■ Diagnostic, laboratory, and other medical examination of patient-members
services → In no instance shall case rates be added to the expenses
■ Use of surgical or medical equipment and facilities → CR payment is not an add-on to hospitalisation and PF
■ Prescription drugs and biologicals, subject to the fees
limitations of the Act → The case rates are the only reimbursement rates for all
■ Health Education specified cases
[PREV MED3 – Community Health] 1.02 – PHILHEALTH AND RELATED LEGISLATIONS (26 August 2020; 3 February, 2021) Page 4 of 8
● Scope and Coverage ● Generates the list of conditions associated with that ICD code
→ All medical conditions and procedures, regardless of ● When answering a case-based analysis for the PhilHealth
member category, that are admitted in accredited health cases and to know the corresponding benefits package use:
care institutions ■ Case rate search system
→ All identified day surgeries and select procedures done in ■ Annex (list of medical case rates)
accredited health care institutions Computation of Reimbursement
General Policies ● When a patient has multiple conditions that are actively being
● The No Balance Billing (NBB) shall apply to all indigents and managed during one confinement, the health care provider may
sponsored sectors claim two case rates relevant to the conditions of the patient
● Case rates are paid to the health care institutions and shall ● The first case = medical condition or procedure that used the
include the professional fees (PF) most resources (drug and medicines, laboratories and
● Medical conditions and procedures that are not in the list shall diagnostics, professional fees, etc) in managing the patient)
not be reimbursed ● The second case rate = medical condition, or procedure with
● Admission due to patient’s choice shall NOT be reimbursed by the second most resources used
the Corporation ● For a claim with a combination of case rates, the provider shall
→ Ex. A patient wants to be admitted per request, that be paid the full (100%) case rate amount for the first case rate
should not be covered by PhilHealth plus 50% of the second case rate
● Combination:
Coverage → Medical condition and medical condition.
● Professional fees → Medical condition and procedure.
● HCI charges, including but not limited to: → Procedure and procedure
→ Room and board → Supplies
Matrix of Payment for Combination of Case Rates
→ Diagnostics and laboratories → Operating room fees
→ Drugs/medicines → Other fees and charges ● If a medical condition is claimed as the 1st Case Rate payment
● Pre-operative diagnostics done prior to confinement are not will be:
covered → Case Rate (100%)
Computation of Reimbursement → HCI (70%)
→ PF (30%)
● For MEDICAL case rates, the HCI fee and the PF shall be 70%
● If it is claimed as the 2nd Case Rate
and 30% of the case rate amount respectively
→ Case Rate (50%)
● For procedure case rates, the following shall be the basis for
→ HCI (20%)
computation except for specified cases:
→ PF (30%)
→ PF= RVU x 56 x 1.5 (except for specified procedure
case rates)
→ HCI fee= case rate amount – PF
List of Medical Case Rates
● The table shows a list of ICD codes and corresponding
description of the medical case.
● Also includes the case rate amount if claimed as the first case
rate
● A column shows
→ Total amount
→ Professional fee at 30%
→ Health care institution fee at 70%

Figure 3. Matrix of payment for combination of case rates

● Sample Claim
→ Total benefit
■ 100% of the first case rate + 50% of the second
case rate

Figure 2. List of medical case rates for primary care


facilities/infirmaries/dispensaries
PhilHealth Case Rates Search
● Case rate search system where you may encode the:
→ Disease/ Medical case
→ ICD code
■ Ex. Pneumonia, ICD code J18 (you may encode just
the disease of the ICD code) Figure 4. Sample Claim (taken from ppt)
− On the right side you will see high risk
pneumonia its different categories with
corresponding case rates amounts, the HCI fee
(70%) and PF (30%)
[PREV MED3 – Community Health] 1.02 – PHILHEALTH AND RELATED LEGISLATIONS (26 August 2020; 3 February, 2021) Page 5 of 8
Professional Fees (because appendectomy is different illness and
● The entire case rate amount, including the pf, shall be paid hemodiaysis is exempted from SPC)
directly to HCI
● The HCI shall act as the withholding tax agent for the PF CASE B
● The governemnt HCI shall facilitate the payment of the pooled
PF share to the health personnel
● The claims shall still be reimbursed even if managed by several
doctors (accredited and non-accredited) provided the said case
is attended by at least one (1) PhilHealth accredited doctor
Procedure List A Exemptions
● Aside from being exempted from 50% rule, claims of multiple
sessions of the following procedures (Procedure List A) shall be
reimbursed evenif claimed as second case:
→ Blood transfusion, outpatient
→ Brachytherapy
→ Chemotherapy
→ Dialysis other than hemodialysis ● Man is admitted with stroke as 1st case along with IHD with MI
→ Hemodialysis as the 2nd case
→ Radiotherapy → The decision was to pay for the Stroke in full at 100%,
Single Period of Confinement while IHD with MI was to be paid at 50% of the case rate
→ Readmission w/in 79 from the previous: due to
● Admissions and readmissions due to the same illness or
pneumonia (1st case) which is a different illness. The
procedure within a 90-calendar day period shall only be
SPC rule does not apply here and the pneumonia may
compensated with one (1) case rate benefit
be paid in full. As a 2nd case rate a claim for IHD with MI
● First and second case rates both evaluated for compliance with
has been made. This time this is covered by the SPC
the SPC rule
rule so this claim is denied
● Cases exempted from the SPC Rule:
45 Day Benefit Limit
● 45 days confinement for members, and 45 days for dependents
per calendar year
● The total number of confinement days shall be deducted from
the 45-day benefit limit of the beneficiary except for the
following medical/procedure case rates with pre-determined
number of days deduction:
→ Dialysis other than hemodialysis e.g., peritoneal dialysis
(rvs 90945)
→ Six days of dialysis, regardless of the number of
exchanges per day, shall be equivalent to one day
deduction from the 45 days allowable benefit per year
● 45 DAYS CHEMOTHERAPY
→ One cycle of chemotherapy is equivalent to two (2) days
Figure 5. Cases exempted from SPC Rule. (taken from ppt) deduction from the 45 days allowable benefit per year
Case examples (SPC rule and 50% 2nd case rule) regardless of the number of days of confinement per
cycle
CASE A ● 45 DAYS RADIOTHERAPY
→ One session for each procedure above is equivalent to
one day deduction from the 45 days allowable benefit per
year
● 45 DAYS BLOOD TRANSFUSION
→ One session for each procedure is equivalent to one day
deduction from the 45 days allowable benefit per year
Special Reimbursement Rules: Referral Package (P00001)
● Reimbursement of the full case rate package shall be paid to
the referral (receiving) hospital
→ So this applies in patients who were admitted initially in
one health care institution and transferred to another
● First admission: px is admitted on January for pneumonia, institution (possibly for further management)
during the said admission he underwent hemodialysis ● Claims filed by the referring hospital shall be reimbursed a fixed
→ With pneumonia as the 1st case rate and hemodialysis as amount of 4,000 pesos
the 2nd, both case rates are paid in full (hemodialysis is ● The HCI fee and PF shall be 70% and 30% respectively
exempted from the 50% 2nd case rate rule) ● Claims for referrals shall only be allowed if the transfer is to a
● However, the px is readmitted 59 days from the prev. higher level hospital except in Level 3 hospitals where transfer
confinement therefore the case rates have to be evaluated to the same level is allowed
under the SPC rule → Level 1 to Level 2
→ Admitted for an appendectomy (1st case rate) which is → Level 1 to Level 3
different from the first illness (pneumonia). He also → Level 2 to Level 3 or
undergoes hemodialysis during this admission (same as → Level 3 to Level 3
the prior 2nd case rate). Both case rates are paid in full
[PREV MED3 – Community Health] 1.02 – PHILHEALTH AND RELATED LEGISLATIONS (26 August 2020; 3 February, 2021) Page 6 of 8
● Claims for referrals shall be limited to the conditions. Listed in
ANNEX 8 (Medical conditions allowable for referral package)
● Only the first and last hospitals to handle the patient shall be ● The corporation shall penalize health care providers for
reimbursed. Claims of the facilities in between shall be denied claims involving:
→ Over-utilization of services
→ Unnecessary diagnostic and therapeutic procedures and
intervention
→ Irrational medication and prescriptions
→ Fraudulent, false or incorrect information as determined
by the appropriate office.
→ Gross, unjustified deviations from currently accepted
standards of practice and/or treatment protocols.
→ Inappropriate referral practices
→ Use of fake, adulterated or misbranded pharmaceuticals,
or unregistered drugs.
→ Use of drugs other than those recognized in the latest
PNF and those for which exemptions were granted by
Figure 6. Medical conditions allowable for referral package.(taken from ppt) the Board.
Confinement Abroad ● All claim applications for drugs and medicines shall be in
● For confinements abroad, the claim shall be reimbursed the full generic terminology in conformity with DOH regulations
case rate amount based on the final diagnosis/es and the law.
● Confinement of less than twenty-four (24) hours shall only
Death of patient
be compensated under the following instances:
● Immediate cause of death (disease, injury, or complication that → If the patient is transferred to another healthcare
caused death) shall be the basis for the case rate/s that will be institution
claimed → In emergency cases as defined by the Corporation
● This does not mean the mode of dying, e.g., Cardio-Respiratory → If the patient expired
Arrest → Other cases as may be determined by the Corporation
→ If a procedure has been done, the procedure and/or the
immediate cause of death may be claimed as first and/or D. Z BENEFIT PACKAGES
second case rate (The procedure or the case that used ● Include mandatory services for the totality of care,
the most resources shall be claimed as the first case essential for the treatment of the condition, hospital
rate). The rule on the first and second case rate applies services such as accommodation, medicines, laboratories
No Balance Billing for Indigent and Sponsored Members in and professional fees and other services or alternative
the Government Health Care Institutions guideline recommendations that may be needed by the
● No other fee of expense shall be charged to indigent in patient
government health care institutions, subject to the guidelines → Acute Lymphocytic Leukemia
issued by the Corporation → Breast Cancer
● Health care institutions must give indigent members → Teratology of Fallot
preferential access to their social welfare funds → Ventricular Septal Defect
● Health care professionals must not charge over and above → Kidney Transplant
the professional dees provided by the program for members → Coronary Artery Bypass Graft
admitted to a service bed. → Prostate Cancer
→ Cervical Cancer
C. GUIDELINES FOR CLAIMS PAYMENT → Z MORPH and Expanded Z MORPH
● Payment for Health Care Professionals in the Health Care → Selected Orthopedic Implant
Institutions → Peritoneal Dialysis
→ All payments for professional services rendered by → Colon and Rectum Cancer
salaried public providers shall be retained by the health → Premature and Small Newborns
facility in which services are rendered and be pooled and → Children with:
distributed among health personnel. ■ Developmental Disabilities
● Income Retention by Government Health Care Institutions ■ Disabilities Mobility Impairment
→ Reimbursements paid to public facilities shall be retained ■ Disabilities Hearing Impairment
by the individual facility in which services were rendered ■ Visual Disabilities
and for which payment was made ● REPUBLIC ACT NO. 9994: “Expanded Senior Citizens Act
■ The funds are used for the upkeep or for the of 2010”
operating expenses of the facility → An act granting additional benefits and privileges to
● Reimbursement and Period to File Claims senior citizens, further amending Republic Act No. 7432,
→ All claims for reimbursement or payment for services as amended, otherwise known as “an act to maximize
rendered shall be filed within a period of sixty (60) the contribution of senior citizens to nation building, grant
calendar days from the date of discharge of the patient benefits and special privileges and for other purposes”
from the health care provider
→ The health care provider shall deduct from the total E. MANDATORY PHILHEALTH COVERAGE
charges all expenses reimbursable by the Corporation ● All indigent senior citizens shall be covered by the national
upon discharge of the patient health insurance program of PhilHealth.
→ The payment of benefits shall be made directly to the ● The LGUs where the indigent senior citizens resides shall
health care provider.4 allocate the necessary funds to ensure the enrollment of their
indigent senior citizens in accordance with the pertinent laws
and regulations.
[PREV MED3 – Community Health] 1.02 – PHILHEALTH AND RELATED LEGISLATIONS (26 August 2020; 3 February, 2021) Page 7 of 8
Medical Detoxification Package (2017) VIII. COVID-19
● Created in keeping with the government’s trust in ending drug
abuse in the country A. INPATIENT CARE: GUIDELINES AND BENEFIT
● Mandatory services include: AVAILMENT
→ Screening ● PhilHealth Circular no. 2020-0009
→ Diagnostics ● Filipinos not registered in PhilHealth shall be automatically
→ Medicines covered
● Other services include: ● Single period of confinement and 45 days annual benefit shall
→ Activated charcoal not comply
→ Sodium sulfate ● All COVID-19 benefits for inpatient care shall have no co-
→ Vitamin B complex payment from the patient for direct health care services, both
→ Benzodiazepine public and private.
→ Antipsychotic medicines ● Mandatory services include accommodation, management and
→ D50-50 monitoring of illness, laboratory/diagnostics/imaging, medicines
→ Acidification medicines included in the guidelines and protocols of DOH, supplies and
→ Ascorbic acid equipment (including PPE)
● Benefit packages for in-patient case management:
→ Php 43, 997 – mild pneumonia in the elderly or with co-
morbidities
→ Php 143, 267- moderate pneumonia
→ Php 333, 519- severe pneumonia
→ Php 786, 384- critical pneumonia

Figure 7. Benefit Packages for In-patient Case Management for Probably and
Confirmed COVID-19 Developing Severe Illness/ Outcomes.

Figure 7. Mandatory and Other Services for Medical Detoxification B. BENEFIT PACKAGE FOR TESTING FOR SARS-COV-2
● Medical detoxification (ICD Code: MD01) has a case rate of Php ● All Filipinos who are classified as eligible for testing based on
10,000 and any co-morbidity (ICD Code: ICD-10) associated DOH guidelines as well as COVID-19 cases that require repeat
with withdrawal symptoms during drug treatment may be filed testing
as an additional claim to the medical detoxification package ● Filipinos not registered in PhilHealth shall be automatically
covered
● Single period of confinement and 45 days annual benefit shall
not apply
● Benefit package services includes: screening/clinical
assessment, diagnostic workup, specimen collection, specimen
Figure 8. Claim Application, Code and Filling Schedule of Medical Detoxification
transport, conduct of RT-PCR, analysis and reporting of results
● All services for the testing are procured and provided by the
Persons with Disability testing laboratory has a package amount of Php 8,150
● Republic Act No. 11228
→ An act providing mandatory PhilHealth coverage for all
persons with disability (PWDs), amending for the
purpose Republic Act No. 7277, as amended, otherwise
known as the “Magna Carta for Persons with
Disability”.
→ Mandatory PhilHealth coverage
→ Premium contributions shall be paid by the National
Government
→ Premium contributions of Persons with Disability in the
formal economy shall be shared equally by their Figure 8. Packages for SARS-COV-2 Testing
employes and the National Government
[PREV MED3 – Community Health] 1.02 – PHILHEALTH AND RELATED LEGISLATIONS (26 August 2020; 3 February, 2021) Page 8 of 8
● The COVID-19 Community Isolation Benefit Package shall B. REPUBLIC ACT NO. 9502 “UNIVERSALLY ACCESSIBLE
include all identified services to effectively manage cases CHEAPER AND QUALITY MEDICINES ACT OF 2008”
needing isolation services based on applicable guidelines ● An act providing for cheaper and quality medicines, amending
adopted by DOH, whether suspect, probable, confirmed or for the purpose Republic Act No. 8293 or the Intellectual
otherwise. Property Code, Republic Act No. 6675 or the Generics Act of
1988, and Republic Act No. 5921 or the Pharmacy Law, and for
other purposes.
Drugs and Medicines Price Regulation
● The President of the Philippines, upon recommendation of the
Secretary of the Department of Health, shall have the power to
impose maximum retail prices over any or all drugs and
medicines as enumerated in Section 23.
● When the public interest so requires, the Secretary of the
Department of Health shall have the power to determine the
maximum retail prices of drugs and medicines which shall be
recommended to the President of the Philippines for approval.
Cheaper Medicines Act
Figure 9. COVID-19 Community Isolation Benefit Package
● No retailer shall sell drugs and medicines at a retail price
IX. RELATED LEGISLATIONS exceeding the maximum retail price approved by the President
of the Philippines as provided in Section 17 of this Act.
A. REPUBLIC ACT NO. 6675 “GENERICS ACT OF 1988”
● An act to promote, require and ensure the production of an Section 23: List of Drugs and Medicines that are Subject to
adequate supply, distribution, use and acceptance of drugs and Price Regulation
medicines identified by their generic names. ● All drugs and medicines indicated for treatment of chronic
illnesses and life-threatening conditions
Generic Name or Generic Terminology ● Drugs and medicines indicated for the prevention of diseases
● Identification of drugs and medicines e.g. vaccines, immunoglobulin, anti-sera
→ by their scientifically and internationally recognized ● Drugs and medicines indicated for prevention of pregnancy e.g.
active ingredients oral contraceptives
→ by their official generic name as determined by the ● Anesthetic agents
Bureau of Food and Drugs of the Department of Health ● Intravenous fluids
● Drugs and medicines that are included in the Philippine National
Essential Drugs List or National Drug Formulary Drug Formulary (PNDF) Essential Drug List; and
● All other drugs and medicines which, from time to time, the
● List of drugs Secretary of the DOH determines to be in need of price
● Prepared and periodically updated by DOH regulation.
→ On the basis of health conditions obtaining in the
Philippines as well as on an internationally accepted Amendments to Republic Act No. 6675
criteria ● Otherwise known as The Generics Act of 1988
● Consists of a core list and a complementary list ● There shall appear prominently on the label of a generic drug
Generics Act of 1988 the following statement:
→ “THIS PRODUCT HAS THE SAME THERAPEUTIC
● The exclusive use of generic terminology in the manufacture, EFFICACY AS ANY OTHER GENERIC PRODUCT OF
marketing and sales of drugs and medicines, particularly those THE SAME NAME. SIGNED: BFAD”
in the Essential Drugs List. ● Every drug manufacturing company operating in the
● Who shall use generic terminology? Philippines shall be required to produce, distribute and make
→ All government health agencies and their personnel widely available to the general public an unbranded generic
as well as other government agencies counterpart of their branded product.
→ All medical, dental and veterinary practitioners,
including private practitioners REFERENCES
→ Any organization or company involved in the Hernandez (2020), PhilHealth [Powerpoint Presentation]. Manila, Philippines:
manufacture, importation, repacking, marketing and/or Faculty of Medicine and Surgery, University of Santo Tomas,
distribution of drugs and medicines PREVMED 3.
→ Drug outlets, including drugstores, hospital and non- Hernandez (2020), PhilHealth and Related Legislations Powerpoint
Presentation]. Manila, Philippines: Faculty of Medicine and Surgery,
hospital pharmacies and non-traditional outlets such as
University of Santo Tomas, PREVMED 3.
supermarkets and stores
PREVENTIVE MEDICINE 3

QUIZ 1: DIAGNOSTIC TESTS AND TREATMENT, STUDY


DESIGNS, STATISTICAL ANALYSIS, DISEASE
TREATMENT,
10 FEBRUARY 2021
PREVENTION, AND PROGNOSIS
QUESTION ANSWER RATIONALE
1. The Receiver Operational Curve (ROC) uses the: D
a. Prevalence
b. LR+
c. LR-
d. Sensitivity

The figure above shows a generic ROC curve, where sensitivity and
specificity are both used to measure the accuracy of a diagnostic test.

2. The study design which provides the greatest D Table 12. Relative ability of different types of study to “prove” causation
justification for concluding causality Type of Study Ability to “prove” causation
a. Cohort Randomized controlled trials Strong
b. Case-control Cohort studies Moderate
c. Cross-sectional Case-control studies Moderate
d. RCT Cross-sectional studies Weak
Ecological studies Weak
3. Stage of disease in which structural changes may B Natural History of Disease (4 Stages)
occur 1. Biologic onset
a. Biologic onset → Initial interaction between man, causal factors, and the rest of
b. Early diagnosis possible the environment
c. Usual clinical diagnosis → Cannot detect the presence of disease
d. Outcome 2. Early diagnosis possible
→ Mechanisms of disease produce structural or functional
changes
→ Individual remains free of any symptoms
3. Usual clinical diagnosis
→ Disease progresses to the point where symptoms appear and
the affected individual becomes ill
4. Outcome
→ Recovery, permanent disability or death

4. Probability of a negative test in people without the B ● Sensitivity


disease → probability of a positive test in people with the disease
a. Sensitivity ● Specificity
b. Specificity → probability of a negative test in people without the disease
c. PPV ● Positive predictive value
d. NPV → probability of the person having the disease when the test is
positive
5. Probability of a positive test in people with the disease A ● Negative predictive value
a. Sensitivity → probability of the person not having the disease when the test
b. Specificity is negative
c. PPV
d. NPV

Page 1 of 4
PREVMED3 APPLIED EPIDEMIOLOGY: Quiz 1 – DIAGNOSTIC TESTS AND TREATMENT, STUDY DESIGNS, STATISTICAL ANALYSIS, DISEASE
Page 2 of 4
TREATMENT, PREVENTION, AND PROGNOSIS (10 FEB 2021)

6. The question: does an intervention on well people C Table 2. Clinical Questions


keep disease from coming? Issue Question
a. Diagnosis Abnormality Is the patient sick?
b. Treatment Diagnosis How accurate are tests used to diagnose disease?
c. Prevention Frequency How often does a disease occur?
d. Prognosis Risk What factors are associated with an increased risk of
disease?
Prognosis What are the consequences of having a disease?
Treatment How does treatment change the course of disease?
Prevention Does an intervention on well people keep disease
from arising?
Does early detection and treatment improve the
course of disease?
Cause What conditions lead to disease?
What are the pathogenetic mechanisms of disease?
Cost How much will care for an illness cost?
7. If the objective of the research is operating D Table 21. Type of Research and Study Design | U
characteristics of a good diagnostic test, the study Objective Study Design
design is Disease description / spectrum Case study
a. Case study Case series
b. Case-control Cross-sectional
c. Clinical trial Disease prevalence Cross-sectional
d. Cross-sectional Operating characteristics of a Cross-sectional
8. Study design of disease prognosis D diagnostic test Cohort
a. RCT Disease incidence Cohort
b. Case-control Prognosis Cohort
c. Case series Causation / etiology / harm Cohort
d. Cohort Case-control
Clinical Trials (RCT)
9. The study design of a rare cause of disease B A. CASE-CONTROL STUDY
a. Cohort ● Case-control study is observational, analytical and longitudinal.
b. Case-control |U
c. Cross-sectional ● Study population is classified according to disease status
d. RCT → Cases: those with the disease interest
→ Control: those without the disease
● Longitudinal studies: looking backward from the disease to a
possible cause; looking for exposure, observe what happened in
the past
● Use new (incident) cases
● Used to investigate cause (etiology) of disease, esp. rare diseases
|U
● Uses odds ratio | U
10. The study design which provides the best D B. COHORT STUDY
information about disease causation ● Longitudinal studies (forward)
a. RCT ● Provide the best information about the causation of disease
b. Case-control ● Most direct measurement of the risk of developing disease
c. Cross-sectional ● Provide the possibility of estimating the attributable risks/ RISK
d. Cohort DIFFERENCE (RISK IN THE EXPOSED GROUP MINUS THE
RISK IN THE UNEXPOSED GROUP
● Use relative risk | U
● Most closely resemble experimental studies
11. Cross-sectional studies use A C. CROSS-SECTIONAL STUDY
a. Prevalent cases ● Prevalence Study
b. Incident cases ● No direction of inquiry
c. New cases ● Measurements of exposure and effect are made at the same
d. Case reports time | U
12. A prevalence study is also known as B ● Useful for investigating exposures that are fixed characteristics of
a. Cohort individuals, such as ethnicity, socio-economic status and blood
b. Cross-sectional group, or chronic disease or stable conditions
c. Case series ● Short-term and therefore less costly
d. Case-control ● Provide no direct estimate of risk
13. A study design useful in the initial phase of outbreak A ● Prone to bias from selective survival
investigation. ● Estimates prevalence may be biased by the exclusion of cases in
a. Cross-sectional which death or recovery are rapid
b. Case series ● In sudden outbreaks of disease it is the most convenient first step
c. Case-control in an investigation into the cause
d. Cohort ● Rare disease, conditions of short duration or diseases with high
case fatality are often not detected | U
PREVMED3 APPLIED EPIDEMIOLOGY: Quiz 1 – DIAGNOSTIC TESTS AND TREATMENT, STUDY DESIGNS, STATISTICAL ANALYSIS, DISEASE
Page 3 of 4
TREATMENT, PREVENTION, AND PROGNOSIS (10 FEB 2021)

14. Method of controlling selection bias which limit the B Table 13. Methods of Controlling Selection Bias
range of characteristics of patients in the study Method Description
a. Randomization
b. Restriction Randomization Assign patients to groups in a way that gives
c. Matching each patient equal chance of falling into one or
d. Stratification the other group
Restriction Limit the range of characteristics of patients in the
study
Matching For each patient in one group select one or more
patients with the same characteristics (except for
the one under study) for a comparison group
Stratification Compare rates within subgroups (strata) with
otherwise similar probability of the outcome
15. Selection and recall bias are high in D Table 19. Advantages and Disadvantages of Different Observational Study designs
a. Cohort Ecological Cross- Case- Cohort
b. Cross-sectional sectional control
c. Case series Selection bias NA medium high low
d. Case-control Recall bias NA high high low
Loss to follow-up NA NA low high
Confounding high medium medium low
Time required low medium medium high
Cost low medium medium high
16. One of the following is NOT a characteristic of a good D Characteristics of a diagnostic test
diagnostic test ● Reliable/repeatability – gives the same measurement when
a. Reliable repeated more than once
b. Valid ● Valid - measures what it intends to measure
c. Available ● Accurate – correctly determines those with disease and those
d. Expensive without
● Easy to use – can be performed by other people without difficulty
● Not expensive – affordable
● Safe and acceptable

17. WHO growth chart in children is an example of B


a. Gaussian method
b. Percentile method
c. Therapeutic method
d. Predictive value method

Recall: weight and age are plotted into the graph and their percentile
is derived.
Example of Gaussian method: IQ tests

18. Prevalence of disease is increased by C Factors influencing observed prevalence rate


a. High case fatality rate ● Increased by:
b. Out-migration of cases → Longer duration of the disease
c. In-migration of cases → Prolongation of life of patient without cure
d. Improved cure rate → Increase in new case(increase in incidence)
→ In-migration of cases
→ Out-migration of healthy people
→ In-migration of susceptible people
→ Improved diagnostic facilities (better reporting)
● Decreased by:
→ Shorter duration of disease
→ High case-fatality rate from disease
→ Decrease in new cases(decrease in incidence)
→ In-migration of healthy people
→ Out-migration of cases
→ Improved cure rate of cases
PREVMED3 APPLIED EPIDEMIOLOGY: Quiz 1 – DIAGNOSTIC TESTS AND TREATMENT, STUDY DESIGNS, STATISTICAL ANALYSIS, DISEASE
Page 4 of 4
TREATMENT, PREVENTION, AND PROGNOSIS (10 FEB 2021)

19. A screening test for a disease is promising between B ● Position 1


a. Biologic onset and early diagnosis → The screening test and case finding would be too late to be of
b. Early diagnosis and clinical diagnosis help in early detection of disease
c. Clinical diagnosis and outcome ● Position 2
d. Outcome and death → The test will have a promise of improving the outcomes of
those who have the target disorder
→ Do Diagnostic Tests
● Position 3
→ Early detection of the disease is a waste of time
→ Dra. Rondaris: no need for diagnostic test

20. The most serious problem in a cohort study is Attrition Disadvantages of cohort study
*Choices were not noted ● Long-term, not always feasible
● Sample size required for the study extremely large
● Attrition (dropout) is most serious problem
PREVENTIVE MEDICINE 3

QUIZ 1: DIAGNOSTIC TESTS AND TREATMENT,


STUDY DESIGNS, STATISTICAL ANALYSIS, DISEASE
TREATMENT, PREVENTION, AND PROGNOSIS
28 APRIL 2021
QUESTION ANSWER RATIONALE
1. The Receiver Operational Curve (ROC) uses the: D
A. Prevalence
B. LR+
C. LR-
D. Sensitivity

The figure above shows a generic ROC curve, where sensitivity and
specificity are both used to measure the accuracy of a diagnostic test.

2. The study design which provides the greatest D Table 12. Relative ability of different types of study to “prove” causation
justification for concluding causality Type of Study Ability to “prove” causation
A. Cohort Randomized controlled trials Strong
B. Case-control Cohort studies Moderate
C. Cross-sectional Case-control studies Moderate
D. RCT Cross-sectional studies Weak
Ecological studies Weak
3. Stage of disease in which structural changes may B Natural History of Disease (4 Stages)
occur 1. Biologic onset
A. Biologic onset → Initial interaction between man, causal factors, and the rest of
B. Early diagnosis possible the environment
C. Usual clinical diagnosis → Cannot detect the presence of disease
D. Outcome 2. Early diagnosis possible
→ Mechanisms of disease produce structural or functional
changes
→ Individual remains free of any symptoms
3. Usual clinical diagnosis
→ Disease progresses to the point where symptoms appear, and
the affected individual becomes ill
4. Outcome
→ Recovery, permanent disability or death

4. Probability of a negative test in people without the B ● Sensitivity


disease → probability of a positive test in people with the disease
A. Sensitivity ● Specificity
B. Specificity → probability of a negative test in people without the disease
C. PPV ● Positive predictive value
D. NPV → probability of the person having the disease when the test is
positive
5. Probability of a positive test in people with the disease A ● Negative predictive value
A. Sensitivity → probability of the person not having the disease when the test
B. Specificity is negative
C. PPV
D. NPV

Page 1 of 4
PREVMED3 APPLIED EPIDEMIOLOGY: Quiz 1 – DIAGNOSTIC TESTS AND TREATMENT, STUDY DESIGNS, STATISTICAL ANALYSIS, DISEASE
Page 2 of 4
TREATMENT, PREVENTION, AND PROGNOSIS (28 APR 2021)

6. The question: does an intervention on well people C Table 2. Clinical Questions


keep disease from coming? Issue Question
A. Diagnosis Abnormality Is the patient sick?
B. Treatment Diagnosis How accurate are tests used to diagnose disease?
C. Prevention Frequency How often does a disease occur?
D. Prognosis Risk What factors are associated with an increased risk of
disease?
Prognosis What are the consequences of having a disease?
Treatment How does treatment change the course of disease?
Prevention Does an intervention on well people keep disease
from arising?
Does early detection and treatment improve the
course of disease?
Cause What conditions lead to disease?
What are the pathogenetic mechanisms of disease?
Cost How much will care for an illness cost?
7. If the objective of the research is operating D Table 21. Type of Research and Study Design |
characteristics of a good diagnostic test, the study Objective Study Design
design is Disease description / spectrum Case study
A. Case study Case series
B. Case-control Cross-sectional
C. Clinical trial Disease prevalence Cross-sectional
D. Cross-sectional Operating characteristics of a Cross-sectional
8. Study design of disease prognosis D diagnostic test Cohort
A. RCT Disease incidence Cohort
B. Case-control Prognosis Cohort
C. Case series Causation / etiology / harm Cohort
D. Cohort Case-control
Clinical Trials (RCT)
9. The study design of a rare cause of disease A
A. Cohort
B. Case-control
C. Cross-sectional
D. RCT

10. The study design which provides the best D A. COHORT STUDY
information about disease causation ● Longitudinal studies (forward)
A. RCT ● Provide the best information about the causation of disease
B. Case-control ● Most direct measurement of the risk of developing disease
C. Cross-sectional ● Provide the possibility of estimating the attributable risks/ RISK
D. Cohort DIFFERENCE (RISK IN THE EXPOSED GROUP MINUS THE
RISK IN THE UNEXPOSED GROUP
● Use relative risk |
● Most closely resemble experimental studies
11. Cross-sectional studies use A B. CROSS-SECTIONAL STUDY
A. Prevalent cases ● Prevalence Study
B. Incident cases ● No direction of inquiry
C. New cases ● Measurements of exposure and effect are made at the same
D. Case reports time |
12. A prevalence study is also known as B ● Useful for investigating exposures that are fixed characteristics of
A. Cohort individuals, such as ethnicity, socio-economic status and blood
B. Cross-sectional group, or chronic disease or stable conditions
C. Case series ● Short-term and therefore less costly
D. Case-control
PREVMED3 APPLIED EPIDEMIOLOGY: Quiz 1 – DIAGNOSTIC TESTS AND TREATMENT, STUDY DESIGNS, STATISTICAL ANALYSIS, DISEASE
Page 3 of 4
TREATMENT, PREVENTION, AND PROGNOSIS (28 APR 2021)

13. A study design useful in the initial phase of outbreak A ● Provide no direct estimate of risk
investigation. ● Prone to bias from selective survival
A. Cross-sectional ● Estimates prevalence may be biased by the exclusion of cases in
B. Case series which death or recovery are rapid
C. Case-control ● In sudden outbreaks of disease, it is the most convenient first step
D. Cohort in an investigation into the cause
● Rare disease, conditions of short duration or diseases with high
case fatality are often not detected |
14. Method of controlling selection bias which limit the B Table 13. Methods of Controlling Selection Bias
range of characteristics of patients in the study Method Description
A. Randomization
B. Restriction Randomization Assign patients to groups in a way that gives
C. Matching each patient equal chance of falling into one or
D. Stratification the other group
Restriction Limit the range of characteristics of patients in the
study
Matching For each patient in one group select one or more
patients with the same characteristics (except for
the one under study) for a comparison group
Stratification Compare rates within subgroups (strata) with
otherwise similar probability of the outcome
15. Selection and recall bias are high in D Table 19. Advantages and Disadvantages of Different Observational Study designs
A. Cohort Ecological Cross- Case- Cohort
B. Cross-sectional sectional control
C. Case series Selection bias NA medium high low
D. Case-control Recall bias NA high high low
Loss to follow-up NA NA low high
Confounding high medium medium low
Time required low medium medium high
Cost low medium medium high
16. One of the following is NOT a characteristic of a good D Characteristics of a diagnostic test
diagnostic test ● Reliable/repeatability – gives the same measurement when
A. Reliable repeated more than once
B. Valid ● Valid - measures what it intends to measure
C. Available ● Accurate – correctly determines those with disease and those
D. Expensive without
● Easy to use – can be performed by other people without difficulty
● Not expensive – affordable
● Safe and acceptable

17. WHO growth chart in children is an example of B


A. Gaussian method
B. Percentile method
C. Therapeutic method
D. Predictive value method

Recall: weight and age are plotted into the graph and their percentile
is derived.
Example of Gaussian method: IQ tests

18. Prevalence of disease is increased by C Factors influencing observed prevalence rate


A. High case fatality rate ● Increased by:
B. Out-migration of cases → Longer duration of the disease
C. In-migration of cases → Prolongation of life of patient without cure
D. Improved cure rate → Increase in new case (increase in incidence)
→ In-migration of cases
→ Out-migration of healthy people
→ In-migration of susceptible people
PREVMED3 APPLIED EPIDEMIOLOGY: Quiz 1 – DIAGNOSTIC TESTS AND TREATMENT, STUDY DESIGNS, STATISTICAL ANALYSIS, DISEASE
Page 4 of 4
TREATMENT, PREVENTION, AND PROGNOSIS (28 APR 2021)

→ Improved diagnostic facilities (better reporting)


● Decreased by:
→ Shorter duration of disease
→ High case-fatality rate from disease
→ Decrease in new cases (decrease in incidence)
→ In-migration of healthy people
→ Out-migration of cases
→ Improved cure rate of cases
19. A screening test for a disease is promising between B ● Position 1
A. Biologic onset and early diagnosis → The screening test and case finding would be too late to be of
B. Early diagnosis and clinical diagnosis help in early detection of disease
C. Clinical diagnosis and outcome ● Position 2
D. Outcome and death → The test will have a promise of improving the outcomes of
those who have the target disorder
→ Do diagnostic tests
● Position 3
→ Early detection of the disease is a waste of time
→ Dra. Rondaris: no need for diagnostic test

20. The most serious problem in a cohort study is C Disadvantages of cohort study
A. Selection bias ● Long-term, not always feasible
B. Confounding bias ● Sample size required for the study extremely large
C. Attrition ● Attrition (dropout) is most serious problem
D. Non-hypothesis generating
PREVENTIVE MEDICINE 3 [APPLIED EPIDEMIOLOGY]

20 FEBRUARY 2021, 24 MARCH 2021, 12 MAY 2021

DISCLAIMER: NO ANSWER KEY WAS GIVEN. STUDY AT YOUR OWN RISK.


Some items do not have a key as the ratio team/editor is unsure due to lack of an answer key. We hope you understand. Thank you.

QUESTION ANSWER RATIONALE


1. Which of the following is NOT an advantage of a A Cohort Study
prospective cohort study? • Provide the best information about the causation of disease
a. It usually costs less than a case-control study • Most direct measurement of the risk of developing disease
b. Precise measurement of exposure is possible • Provide the possibility of estimating the attributable risks/ RISK
c. Incidence rates can be calculated DIFFERENCE (RISK IN THE EXPOSED GROUP MINUS THE
d. Many disease outcomes can be studied
RISK IN THE UNEXPOSED GROUP
simultaneously
• Use relative risk |
• Most closely resemble experimental studies
Disadvantages of Cohort Study
• Long-term, not always feasible
• Sample size required for the study extremely large
• Attrition (dropout) is most serious problem
*Case-control studies are quick and cheap as compared to cohort.
Additionally, only fewer subjects are needed.
Batch 2022 Trans on Diagnostic Tests, Treatment, Prevention and Prognosis
DKA Clinical Epidemiology p.3
2. The statistical measurement to determine B Case-Control Studies
association in a case-control study is: • Uses new (incident) cases
a. Relative risk • Used to investigate cause (etiology) of disease, esp. rare
b. Odds ratio diseases
c. Number needed to treat • Uses odds ratio
d. Monitoring of pulmonary function Batch 2022 Trans on Diagnostic Tests, Treatment, Prevention and Prognosis

3. This is considered a primary level of prevention in B Primary Prevention


the management of Covid 19 infection ● Immunization
a. Treatment with Avigan ● Target: Total populations, selected groups, healthy
b. Contact tracing individuals
c. Adjunctive treatment with convalescent ● Phase of Disease: Specific causal factor/s (before the disease
plasma occurs)
d. Mechanical ventilation → Social distancing
→ Travel restriction
→ Contact tracing and quarantine
Secondary Prevention
● Pap Smear
● Screening test/s
→ Identification of unrecognized disease or risk factor
■ History taking
■ Physical examination
■ Laboratory tests
■ Ancillary procedures
● Target: Patients
● Phase of Disease: Early stage of disease
● Favipiravir (Avigan) is an oral antiviral approved for treatment of
influenza in Japan. It selectively inhibits RNA polymerase, which is
necessary for viral replication. It may serve an option for
“avoidance or control of outbreaks in elderly residents living in
long-term care facilities.”
● CHOICE C and A
Tertiary Prevention
● Limitation of Disability
● Rehabilitation
● Goal here is not to prevent death but to maximize the amount of
high-quality time patient has left
● Target: Patients
● Phase of Disease: Late stage of disease (Treatment,
Rehabilitation)
● CHOICE D

Page 1 of 8
PRV-AE-Shifting Exam (20 FEBRUARY 2021) Page 2 of 8
Batch 2022 Trans on Diagnostic Tests, Treatment, Prevention and Prognosis.
Joshi S, Parkar J, Ansari A, Vora A, Talwar D, Tiwaskar M, Patil S, Barkate H. Role of favipiravir in the
treatment of COVID-19.
Int J Infect Dis. 2021 Jan;102:501-508. doi: 10.1016/j.ijid.2020.10.069. Epub 2020 Oct 30. PMID: 33130203;
PMCID: PMC7831863.
https://www.contagionlive.com/view/fda-clears-favipiravir-covid19-facility-outbreak-prevention-study
4. In vaccine production, this phase of clinical trial is C Phase 3
the administration of the vaccine to in-hospital • Classical phase
patients • Large number of patients with the target disease to establish safety
a. Phase 1 and efficacy
b. Phase 2 • Performed on patients with consent
c. Phase 3 • Carried out mostly on hospital in-patients (controlled
d. Phase 4 environment)
Phases of Clinical Trials
Phase Description
1 • Small number of healthy volunteers
• Non-blind or “open”
• Establish limits of the safe clinical dosage range
2 • Patients with the target disease to determine efficacy
• Single-blind trial
3 • Classical phase
• Larger number of patients with the target disease to
establish safety and efficacy
• Performed on patients with consent
• Carried out mostly on hospital in-patients (controlled
environment)
4 • Post marketing surveillance
• A trial in normal field or program setting
• Reassess effectiveness, safety, acceptability and
continued use of the drugs.

Batch 2022 Trans on Diagnostic Tests, Treatment, Prevention and Prognosis p.13
5. A study design which strongly proves causality A Randomized Controlled Trial
between a variable and an outcome is: • Gold standard or reference in medicine
a. RCT • Provide the greatest justification for concluding causality
b. Case-control • Subject to the least number of problems or biases
c. Cross-sectional • Best study design to establish the efficacy of a treatment or a
d. Cohort procedure
Batch 2022 Trans on Diagnostic Tests, Treatment, Prevention and Prognosis p.8
6. Which of the following is a good index of the C Case fatality rate = (Deaths due to the particular disease / Total
severity of a short term, acute disease? number of cases of the particular disease) X 100
a. Cause-specific death rate • It represents the killing power of the disease.
b. 5-year survival rate
• Percent of patients with a disease who die of it
c. Case-fatality rate
Doc got this question from flashcards available online: https://quizlet.com/96690525/chapter-6-review-
d. Standardized mortality rate questions-flash-cards/
Batch 2022 Trans on Diagnostic Tests, Treatment, Prevention and Prognosis p.11
7. Screening test for diseases is most helpful in this B Natural History of Disease
stage of the natural history of disease 1. Biologic onset
a. Biologic onset → Initial interaction between man, causal factors, and the rest of
b. Pre-clinical the environment
c. Clinical → Cannot detect the presence of disease
d. Outcome 2. Early diagnosis possible / Pre-clinical
→ Mechanisms of disease produce structural or functional
changes
→ Individual remains free of any symptoms
* A screening test is done to detect potential health disorders or
diseases in people who do not have any symptoms of disease.
3. Usual clinical diagnosis
→ Disease progresses to the point where symptoms appear and
the affected individual becomes ill
4. Outcome
→ Recovery, permanent disability or death
Batch 2022 Trans on Diagnostic Tests, Treatment, Prevention and Prognosis p.5
https://www.hopkinsmedicine.org/health/treatment-tests-and-therapies/screening-tests-for-common-
diseases
8. An example of Secondary Level of disease A Choices A to D are outright under tertiary prevention by the word
prevention *A was considered as “treatment”
the correct answer
a. Treatment of pneumonia with amoxicillin according to the
Type Objective Disease Phase
available post-test
b. Treatment of Covid-19 with Remdesivir key on March 24 and Primary Immunization Causal factors
c. Treatment of breast cancer with Tamoxifen upon discussing with
Secondary Screening test/s Early stage
PRV-AE-Shifting Exam (20 FEBRUARY 2021) Page 3 of 8
Doc Santos after the
d. Treatment of sepsis with IVIG exam
Tertiary Treatment, Rehab Late stage
According to Doc Santos, A is the best option since choices B and D
are for treating terminal stage of illnesses to limit the effect of the
disease. We tried to argue that all choices were already to initiate
treatment, which falls under tertiary. Doc couldn’t give us a more
elaborate explanation as to why A is considered the correct answer
and asked us to just raise the concern to the dept head sorry

To try and rationalize why A might be the best option, we use this
figure lifted from the trans on Health Promotion and Interventions
(PRV-CH). According to this figure, measures on diagnosis,
treatment, compliance and adherence to target the clinical course of
the disease in its early stage are not considered part of tertiary
prevention. However, once a disability sets in, all interventions done
would be for the tertiary level of prevention. This may be the reason
why choice A was considered as the best option as amoxicillin is
given empirically in those suspected to have low-risk CAP.
Batch 2022 Trans on Diagnostic Tests, Treatment, Prevention and Prognosis p.14
Batch 2022 PRV-CH Trans on Health Promotion and Interventions p.1
9. The bias being controlled in Best case/worst case A* Best case/worst case
Most probable
approach is: answer, but was
- Describe how different the results could be under the most
a. Selection noted as incorrect extreme or simply very unlikely conditions of selection bias
during the exam
b. Confounding Batch 2022 Trans on Diagnostic Tests, Treatment, Prevention and Prognosis p.9
c. Recall C was the
d. Centripetal answer in
the May 12,
2021 exam
10. The Best case/Worst case approach is best A Controlling Bias in Cohort
applied in this type of study Method Description
Random selection Randomly select patients or randomly assign them
a. Cohort to groups
b. Cross-sectional Restriction Limit the range of patient characteristics
c. Case-control Matching For each patient in one group select one or more
d. RCT patients with the same characteristics to the other
group
Stratification Compare rates within subgroups
Simple adjustments Adjust crude rate by assigning weight for one or few
characteristics
Multivariate Adjust for differences in multiple factors using
statistical modelling techniques
Sensitivity analysis Best case / Worst case
DKA Clinical Epidemiology II p.12
11. A screening test for breast cancer was C With disease W/o disease
administered to 400 women with biopsy proven
(+) test 100 (a) 50 (b)
breast cancer and to 400 women without breast
(-) test 300 (c) 350 (d)
cancer. The test results were positive for 100 of the
proven cases and 50 of the normal women. What Sensitivity: probability of a positive test in people with the disease
is the sensitivity of this screening test? = a/(a+c)
a. 0.88 = 100/(100+300)
b. 0.67 = 0.25
c. 0.25
d. 0.33
PRV-AE-Shifting Exam (20 FEBRUARY 2021) Page 4 of 8

12. The WHO Growth chart for children with the z B


scores is an example of:
a. Gaussian distribution method
b. Percentile method
c. Therapeutic method
d. Predictive value method

Recall: weight and age are plotted into the graph and their percentile
is derived.
Example of Gaussian method: IQ tests
Batch 2022 Quiz 1 Ratio
13. The hallmark feature of an analytic epidemiologic A “The key feature of analytic epidemiology is a comparison group.”
study is: CDC Principles of Epidemiology in Public Health Practice, Third Edition
a. Use of an appropriate comparison-group https://www.cdc.gov/csels/dsepd/ss1978/lesson1/section7.html
b. Laboratory confirmation of a diagnosis
c. Publication in a peer reviewed journal
d. Statistical analysis using logistic regression

14. A type of study design which has the highest C Disadvantages of Cohort Study
attrition rate • Long-term, not always feasible
a. RCT • Sample size required for the study extremely large
b. Case-control • Attrition (dropout) is most serious problem
c. Cohort Batch 2022 Trans on Diagnostic Tests, Treatment, Prevention and Prognosis p.8
d. Ecological

15. The major purpose of random assignment in a D Table 13. Methods of Controlling Selection Bias
clinical trial is to: Method Description PHASE OF STUDY
a. Help ensure that the study subjects are Design Analysis
representative of the general population Randomization Assign patients to groups in a +
b. Facilitate double blinding way that gives each patient
equal chance of falling into
c. Facilitate the measurement of outcome
one or the other group
variables
Restriction Limit the range of +
d. Reduce selection bias in the allocation of characteristics of patients in
treatment the study
Matching For each patient in one group +
select one or more patients
with the same characteristics
(except for the one under
study) for a comparison
group
Stratification Compare rates within +
subgroups (strata) with
otherwise similar probability
of the outcome
Batch 2022 Trans on Diagnostic Tests, Treatment, Prevention and Prognosis p.9
16. Two of these measurements are applied in the use A
of the Receiver's Operator Curve (ROC)
a. Sensitivity and specificity
b. Incidence and prevalence
c. Likelihood ratio (+) and likelihood ratio (-)
d. Positive and negative predictive values

The figure above shows a generic ROC curve where sensitivity and
specificity are both used to measure the accuracy of a diagnostic test.
Batch 2022 Quiz 1 Ratio
PRV-AE-Shifting Exam (20 FEBRUARY 2021) Page 5 of 8

17. All of the following are TRUE of odds ratio, D Choice A-C: correct
EXCEPT: Choice D: Ratio of the odds that the cases were exposed to the odds
a. It is an estimate of relative risk that the controls were exposed
b. It is the only measure of risk that can be
obtained directly from a case-control study
c. It tends to be biased towards 1 (neither risk or
protection at high rates of disease)
d. It is the ratio of incidence in exposed divided
Batch 2022 Trans on Diagnostic Tests, Treatment, Prevention and Prognosis p.7
by incidence in non-exposed

18. In the construction of a contingency table, A Validity of a Diagnostic Test


sensitivity pertains to: • a = no. of true positives, b = no. of false positives c = no. of false
a. True positive cases negatives, d = no. of true negatives
b. False negative cases • Sensitivity
c. Positive predictive value → probability of a positive test in people with the disease
d. Negative predictive value → a / (a + c)
• Specificity
→ probability of a negative test in people without the disease
→ d / (b + d)
• Positive predictive value
→ probability of the person having the disease when the test is
positive
→ a / (a + b)
• Negative predictive value
→ probability of the person not having the disease when the test is
negative
→ d / (c + d)

Batch 2022 Trans on Diagnostic Tests, Treatment, Prevention and Prognosis p.3-4

19. The BEST study design for this research on the A* Longitudinal studies
effect of breastfeeding on the intelligence of Most probable
answer, but was
• Exposure and outcome are measured at different times in the life of
children is: noted as incorrect the participant
during the exam
a. Case-control • Case-control, cohort, RCT, field trial, community trials
b. Case series B/D
c. Cross-sectional
d. Predictive value

Batch 2022 Trans on Diagnostic Tests, Treatment, Prevention and Prognosis p.7
PRV-AE-Shifting Exam (20 FEBRUARY 2021) Page 6 of 8

20. All of the following are important criteria when D* Guidelines for Causation
making causal inferences EXCEPT: Most probable ● Temporal relationship
answer, but was
a. Consistency with existing knowledge noted as incorrect → “Does the cause precede the effect? (essential)”
during the exam
b. Dose-response relationship ● Plausibility
c. Strength of association → “Is the association consistent with other knowledge? Makes
d. Prevalence risk B was the sense, according to biologic knowledge of the time (mechanism
answer in the of action; evidence from experimental animals”
May 12, 2021 ● Consistency
exam → “Have similar results been shown in other studies?”
→ Repeatedly observed by different persons, in different places,
circumstances, and times.
→ Example: increase tobacco vs. increase lung CA
● Strength
→ “What is the strength of the association between the cause and
the effect? (large relative risk)”
→ Example: 10-fold higher incidence of lung cancer among male
smokers compared to non-smokers)”
● Dose-response relationship
→ “Is increased exposure to the possible cause associated with
increased effect?”
→ Example: Number of cigarettes smoked vs lung cancer
● Reversibility
→ “Does the removal of a possible cause lead to reduction of
disease risk?”
→ Example: declining mortality from lung CA in ex-cigarette
smokers
● Study design
→ “Is the evidence based on a strong study design?”
● Judging the evidence
→ “How many lines of evidence lead to conclusions?”
Batch 2022 Trans on Diagnostic Tests, Treatment, Prevention and Prognosis p.10

21. A relative risk of equal to 1 means: C Risk Ratio Risk Difference


a. Harmful to the subjects RR = 1.0 RD = 0
b. Protective to the subjects No association No association
c. No effect to the subjects RR > 1.0 RD > 0
d. Prevalence Harmful association Harmful association
RR < 1.0 RD < 0
Beneficial association Beneficial association
OR & RR point of reference is always 1. RD point of reference is always 0.
Batch 2022 Trans on Diagnostic Tests, Treatment, Prevention and Prognosis p.8

22. British investigators conducted a study to compare C Advantages of Case-control Study


MMR vaccine history among 1,294 children with • Relatively efficient, requiring smaller sample than cohort study
pervasive development disorder, i.e autism, • Completed faster and more economical
Asperger's syndrome and 4, 469 children without • Earliest practical observational strategy for determining an
such disorders. They have found no association. association
This is an example of which type of study? • Antecedent-consequence uncertainty
a. RTC
b. Cohort
c. Case-control
d. Cross-sectional Batch 2022 Trans on Diagnostic Tests, Treatment, Prevention and Prognosis p.7

23. Wearing of face mask and face shield with proper A Primary Prevention
physical distancing is a type of this level of • Immunization
prevention • Target: Total populations, selected groups, healthy
a. Primary individuals
b. Secondary • Phase of Disease: Specific causal factor/s
c. Tertiary
*See no. 3
d. Primary and secondary
Batch 2022 Trans on Diagnostic Tests, Treatment, Prevention and Prognosis p.14
24. A characteristic of a good diagnostic test when it C Diagnostic Tests
correctly determines those with and without the • Objective is to diagnose any treatable disease present
disease • Characteristics of a diagnostic test
a. Reliability → Reliable/repeatability – gives the same measurement when
b. Validity repeated more than once
c. Accuracy → Valid - measures what it intends to measure
PRV-AE-Shifting Exam (20 FEBRUARY 2021) Page 7 of 8

d. Positive predictive value → Accurate – correctly determines those with disease and those
without
→ Easy to use – can be performed by other people without
difficulty
→ Not expensive – affordable
→ Safe and acceptable
Batch 2022 Trans on Diagnostic Tests, Treatment, Prevention and Prognosis p.3
25. Risk factor on a disease increases this measure of *A was incorrect
• Prevalence - the number or proportion of cases or events or
disease B was the attributes among a given population
a. Prevalence answer in • Attack rate - a form of incidence that measures the proportion of
b. Attack rate the May 12, persons in a population who experience an acute health event
c. Case-fatality 2021 exam during a limited period (e.g., during an outbreak)
d. Outcome o Calculated as the number of new cases of a health problem
during an outbreak divided by the size of the population at the
beginning of the period, usually expressed as a percentage or
per 1,000 or 100,000 population (see also incidence proportion)
• Case-fatality - the proportion of persons with a particular condition
(e.g., patients) who die from that condition. The denominator is the
number of persons with the condition; the numerator is the number
of cause-specific deaths among those persons
• Outcome - any or all of the possible results that can stem from
exposure to a causal factor or from preventive or therapeutic
interventions; all identified changes in health status that result from
the handling of a health problem
CDC Glossary. https://www.cdc.gov/csels/dsepd/ss1978/glossary.html
26. What phase of clinical trial is the field study for the D See no . 4
safety of a drug?
a. Phase 1
b. Phase 2
c. Phase 3
d. Phase 4

27. What type of bias is involved in a prognosis study A Different forms of Bias based on Referral Pattern
if the patients were taken from a tertiary center Bias Description
which is known for having a high success rate Centripetal Center’s reputation results in part from its particular
of a procedure? Bias expertise in a specialized area of clinical medicine, it will
be referred problem cases likely to benefit from its
a. Centripetal bias
expertise
b. Diagnostic suspicion bias
Popularity Experts may preferentially admit and keep track of these
c. Popularity bias Bias cases over other less challenging or less interesting
d. Selection bias cases
Referral Filter Selection that occurs at each stage of referral process
Bias can generate patient samples at tertiary care centers
that are much different from those found in the general
population
Diagnostic Px differ in their financial and geographic access to
Access Bias clinical technology that identifies them as eligible for
studies of the course and prognosis of disease
Batch 2022 Trans on Diagnostic Tests, Treatment, Prevention and Prognosis p.14
28. A longitudinal or prospective study is also referred C Analytic Study Design
as a/an: Cross-sectional study Longitudinal studies
a. Ecological study Exposure and outcome are Exposure and outcome are
b. Cross-sectional study measure at the same time in measure at different times in the
c. Cohort study the life of the participant life of the participant
d. Observational study Cross sectional study Case control study
Cohort study
Randomized controlled trials
Field trials
Community trials
Batch 2022 Trans on Diagnostic Tests, Treatment, Prevention and Prognosis p.6
29. Several studies have found that approximately B Attributable risk (risk difference)
85% of cases of lung cancer are due to cigarette • “What is the incidence of disease attributable to exposure?”
smoking. This measure is an example of: • In this case, the exposure is cigarette smoking and the given
a. Incidence rate percent is the incidence of the disease (lung cancer) that is
b. Attributable risk due/related to the exposure (smoking).
c. Relative risk Relative rate
d. Prevalence rate • “How many times more likely are exposed persons to become
diseased, relative to non- exposed persons?”
Prevalence rate
PRV-AE-Shifting Exam (20 FEBRUARY 2021) Page 8 of 8

• Number of cases in a defined population at a specified point in time.


Incidence rate
• Number of new cases arising in a given period of time in a specified
time. (Describes how quickly disease occurs in a population)
Batch 2022 Trans on Diagnostic Tests, Treatment, Prevention and Prognosis p.13

30. The purpose of a double blind study is to: C A double-blind study blinds both the subjects (sample) as well as
a. Achieve comparability of treated and the researchers (observer) to the treatment allocation.
untreated subjects
b. Reduce the effects of sampling variation
c. Avoid observer and subject bias
d. Avoid observer bias and sampling variation https://www.ncbi.nlm.nih.gov/books/NBK546641/
PREVENTIVE MEDICINE 3 (FAMILY HEALTH)

17 FEBRUARY 2021

COVERAGE
● Geriatrics
● Home Care
● Care of the Dying
● Difficult Clinical Encounters
● Optimal Primary Care for LGBTQ+ Patients

DISCLAIMER: Some of the answers cannot be found in the transes.


QUESTION ANSWER RATIONALE
1. Georgia believes in her mind that she is a woman. This is B Gender identity: the person’s internal sense of
a reflection of her their own gender
A. Biologic sex
B. Gender identity Gender expression: the spectrum of masculine
C. Gender expression and feminine characteristics in how one dresses,
D. Gender orientation acts, or presents his/herself
Sexual orientation: a person’s emotional, sexual,
or relational attraction to others

[PRV-FH] 04 Optimal Primary Care for LGBTQ+


Patients (Batch 2022), p. 2
2. A 75 year old female has the following CGA results: C/D Mild Alzheimer’s Disease is treated with a
MMSE- 15, GDS- 6. Which of the following cholinesterase inhibitor (donepezil) treatment
pharmacologic management is correct? dose, increased over time.
A. Donepezil 5 mg OD and Zolpidem 10 mg OD
B. Donepezil 10 mg OD and Memantine 10 mg OD
C. Donepezil 5 mg OD then increase to 10 mg OD
after 3 months
D. Donepezil 5 mg OD then increase to 10 mg OD
after 3 months

[PRV-FH] 01 Geriatrics (Batch 2022), p. 2


3. Adjuvant analgesics are drugs that are not primarily used A In the trans, they are referred to as “Adjunct
as analgesics but can produce analgesia in certain types Analgesics”: drugs with a primary indication other
of pain hence can be added to the pain regimen. than pain that have analgesic properties in some
A. True painful conditions. Ex. corticosteroids for pain due
B. False to inflammation and swelling.

[PRV-FH] 03 Care of the Dying (Batch 2022), p. 5


4. Predictor of difficult clinical encounters: A Profile of Difficult Patients:
A. > 5 physical symptoms ● Had >5 symptoms
B. Physicians working less than 40 hours/week ● Severe symptoms
C. severity of symptoms <6/10 point-scale ● Take more medications
D. Physician’s level of stress is low to average ● Underlying mental health issues
● They are older
● Different sexual orientation from the provider
● More acute, chronic psychosocial problems
● More frequently divorced or widowed
● From a social class

[PRV-FH] 02 Home Care & Difficult Clinical


Encounters (Batch 2022), p. 3
5. In this patient type, use open ended questions and A Silent/Detached Patient
explain the importance of sharing information in order for ● They may appreciate it if they get to tell their
you to help him or her: story in private.
A. Silent ● Establish rapport to gain confidence.
B. Seductive ● Explain the importance of sharing
C. Somatizing information in order to help the patient.
D. Demanding ● Use an open-ended question.
● Encourage a productive clinic visit

TRANS TEAMS 3A AND 4B Page 1 of 18


PREV MED 3-FH SHIFTING EXAM (17 FEBRUARY 2021) Page 2 of 18

● Identify cause of silence


○ Possible fear of authority figure
○ Barriers
■ Language, personality, cultural
○ Medical reasons
○ Previous negative experience with a
healthcare provider/service

[PRV-FH] 02 Home Care & Difficult Clinical


Encounters (Batch 2022), p. 4
6. The living will is only used at the end of life if a person is A The Living Will
terminally ill or permanently unconscious to describe ● Written or video statement about the kinds
under what conditions an attempt to prolong life should of medical care you do/do not want to
be started or stopped according to the patient's wishes. receive if you are no longer able to express
A. True your consent.
B. False ● Includes: resuscitation, desired QOL, end-of-
life treatments, etc.
● When creating this, you should know how
independent want to be and what you want out
of your life
● Do you want your life extended for any reason
or only if a cure is possible?
● Having a treatment or not is your decision.
Nothing is right or wrong. This is your choice.
● Durable power of attorney for health care
(Medical Power of Attorney)
● POLST (Physician Orders for Life-Sustaining
Treatment)
● Do not resuscitate (DNR) orders
● Organ and tissue donation

[PRV-FH] 03 Care of Dying (Batch 2022), pp. 7-8


7. Which of the following is NOT a parameter in the Mini C Mini Nutritional Assessment
Nutritional assessment? The MNA is able to classify older people as well
A. Calf circumference nourished, at risk for malnutrition or malnourished.
B. Self view of nutritional status The MNA consists of 18 self reported questions
C. Demispan derived from four parameters of assessment:
D. Immobility anthropometric assessment, general assessment,
dietary assessment and self-assessment
administered in two steps. The MNA-SF is a
screening tool composed of the first six items (of
the 18 questions) that permits detection of a
decline in ingestion over the past three months
(loss of appetite, decline of food intake, digestive
problems, chewing or swallowing difficulties),
weight loss in the past three months, current
mobility impairment, an acute illness or major
stress in the past three months, a
neuropsychological problem (dementia or
depression) and a decrease in body mass index
(BMI). If indicated after Step 1 (MNA-SF), the risk
of malnutrition should be assessed using the full
MNA. The MNA evaluates living arrangements, the
presence of polypharmacy or pressure ulcers, the
number of full meals eaten daily, the amount and
frequency of specific foods and fluids, and the
mode of feeding. The patient reports nutritional
and health status, and the practitioner
determines weight and height (to calculate
BMI), and mid-arm and mid-calf
circumferences.
PREV MED 3-FH SHIFTING EXAM (17 FEBRUARY 2021) Page 3 of 18

● Demispan was not mentioned in this


assessment tool.

Source: Nestlé nutrition Institute - Mna® elderly -


Overview. (n.d.). Retrieved February 20, 2021,
from https://www.mna-elderly.com/

Source: M. S., M. S., H. V., G. V., & B. V. (2007).


The Mini Nutritional Assessment (MNA) after 20
years of research and clinical practice. Reviews in
Clinical Gerontology, 293-310.
8. For moderate to severe pain control, weak opioids with B
adjuvants are recommended in the WHO Step Ladder
A. True
B. False

The Pain Relief Ladder


Step 1: NSAIDS
Step 2: WEAK OPIOIDS
Step 3: STRONG OPIOIDS

[PRV-FH] 03 Care of Dying (Batch 2022), p. 4


9. Which Republic act covered the mandatory Philhealth B RA 9994 Expanded Senior Citizens Act of 2010
coverage for indigent senior citizens - It is also important to let the elderly know
A. RA 6111 of their rights.
B. RA 6994 (Typo, should be 9994) 1. 20% discount and VAT exemption for:
C. RA 7277 ■ Purchase of medicine including the
D. RA 9649 flu and pneumococcal vaccines
and other essential medical
supplies, accessories and
equipment
■ Professional fees of MD in all
private hospitals, medical facilities,
OPD and home health care
PREV MED 3-FH SHIFTING EXAM (17 FEBRUARY 2021) Page 4 of 18

■ Medical and dental services,


diagnostic and laboratory fees
■ Actual transportation fare for land
travel and domestic air transport
and sea shipping vessels
■ Hotel and lodging establishments,
restaurants
■ Funeral and burial services
■ Admissions to cinema
2. Payment of individual income tax below
minimum wage
3. 5% on water and electricity registered on
their name not exceeding 100 kwh and
30 mm3
4. Educational assistance to pursue
secondary, tertiary, vocational and
technical courses and short-term courses
in schools
5. Exemption from training fees for
socioeconomic programs
6. Death benefit of Php 2000 to nearest
surviving relative
7. Express lanes in establishments
8. Mandatory Philhealth coverage for
indigenous senior citizens

[PRV-FH] 01 Geriatrics (Batch 2022), p. 6


10. Accepting hospice care means stopping all medical B Hospice Care
treatment ● consists of palliative and supportive services
A. True for dying persons and their family
B. False ● highlights quality rather than length of life
● provides continuity of care
● provides family centered care involving the
patient and family in making decisions
● available 24/7
● Provides follow-up bereavement care up to
1 year after the patient’s death

[PRV-FH] 03 Care of the Dying (Batch 2022), p. 2


11. Doctor said: “I have the results of the tests, do you want C Obtaining the Patient’s Invitation
all the details explained to you?" Identify step in the ● Some patients desire full information about
SPIKES model their condition, but some do not.
A. Patient’s perception ● Discussing information at a time of ordering
B. Knowledge tests can cue the physician to plan the next
C. Invitation discussion with the patient.
D. Patient’s emotions ● “How would you like me to give the
information about the test results?
● “Would you like me to give you all the
information or sketch out the results and
spend more time discussing the
treatment plan?”

Patient’s perception
● The dictum here is: “before you tell, ask.”
● Before discussing medical information, the
clinician uses open-ended questions to create
a reasonably accurate picture of how the
patient perceives the medical situation.
● Based on this, you can correct any
misinformation and tailor the bad news to what
the patient understands
● It can also accomplish the important task in
determining if the patient is engaged in any
variation of illness denial (e.g. wishful thinking
omission, unrealistic expectation of treatment)
PREV MED 3-FH SHIFTING EXAM (17 FEBRUARY 2021) Page 5 of 18

Knowledge
● Warning the patient that bad news is coming
may lessen the shock following the disclosure.
This may also facilitate information
processing.
○ “Unfortunately, I’ve got bad news to tell
you” or “I’m sorry to tell you that…:”
● Giving medical facts, the one-way part of the
physician-patient dialogue, may be proved
with a few simple guidelines:
○ Start at the level of comprehension and
vocabulary of the patient
○ Try to use nontechnical words
○ Avoid excessive bluntness
○ Give information in small chunks and
check periodically as to the patient’s
understanding
○ When the prognosis is poor, avoid using
phrases such as “there is nothing more
we can do for you”

Patient’s emotion
● When patients get bad news, their emotional
reaction is often an expression of shock,
isolation, and grief
○ Offer support and solidarity to the
patient by making an empathic
response.
● Four Steps in Making an Empathic Response:
○ Observe for any emotion on the part of
the patient
○ Identify the emotion experienced by the
patient by naming it to oneself
○ Identify the reason for the emotion.
○ Let the patient know that you have
connected the emotion with the reason
for the emotion by making a connecting
statement

[PRV-FH] 1.03 Care of the Dying (Batch 2022), p.


2
12. Which is NOT a common health issue among the LGBT B
population?
A. Family planning
B. GUT malignancy
C. Homelessness, violence
D. Suicide attempts and depression

Choice A is an issue in early to middle adulthood


Choice C and D are issues in childhood and
adolescence

[PRV-FH] 1.04 Optimal Primary Care for LGBTQ+


Patients (Batch 2022), p. 5
13. A 75-year-old teacher has memory problem. She was C Until the last 20 years, deficits in executive
noted to have progressive forgetfulness and functions were rarely considered in the early
disorientation for approximately 4 years. Assistance was stages of Alzheimer's disease (AD). Some studies
necessary in handling money and managing suggested that these were relatively preserved
PREV MED 3-FH SHIFTING EXAM (17 FEBRUARY 2021) Page 6 of 18

appointments. Medical work-up was normal. Brain MRI during the pre-clinical phase of the disorder.
showed presence of atrophy in both hippocampi. What is However, over the last years, this view has
the cognitive domain that is most frequently affected first changed, and more recent studies have confirmed
in patients with this condition? the presence in the AD of early impairment in a
A. Language variety of tasks aimed at investigating
B. episodic memory executive functions. These findings confirm that
C. Executive function in the AD, executive functions are impaired
D. visuospatial function from the early stages, primarily due to
degeneration of the prefrontal cortex.

Guarino, A., Favieri, F., Boncompagni, I., Agostini,


F., Cantone, M., & Casagrande, M. (2019).
Executive Functions in Alzheimer Disease: A
Systematic Review. Frontiers in aging
neuroscience, 10, 437.
https://doi.org/10.3389/fnagi.2018.00437
14. Which of the following statements expresses empathy A
rather than sympathy?
A. "I feel your loss"
B. “I know how you feel.”
C. "I feel sorry for you."
D. “He is in a better place now”

Choice A: “I feel your loss.”


Shows a more personal understanding wherein the
physician puts himself/herself into the patient’s
shoes.

Choice B: “I know how you feel.”


Acknowledges what the patient feels. The
physician understands the patient’s experience.
This shows sympathy.

Choice C: “I feel sorry for you.”


The physician has a feeling of pity or sympathy for
the patient rather than empathy.

Choice D: “He is in a better place now.”


The physician tries to provide comfort and
assurance to the patient or family.

[PRV-FH] 02 Home Care and Difficult Clinical


Encounters (Batch 2022), p. 5
15. Early recognition and assessment of dementia is C Advance Care Planning
beneficial because the person with dementia has the ● Chance to make decisions about the care
opportunity to: you want to receive at a time when you
A. Appoint people to be proxy decision-makers might be unable to speak for yourself
and to give up driving ● Process that involves you, your family,
B. "Put their affairs in order, take anti-dementia loved ones, and health care providers
medication and carry on driving" ● Goal: maintain and maximize your health
C. "Talk over the future with loved ones, identify and independence as you face the end of
priorities and plan ahead"
your life
D. Avoid crises by putting assistive technology in
● Should be a compassionate process and
their home
with support and education as your health
changes
● Encourage patients to choose a surrogate
decision-maker
○ Important component of advance
care planning
○ Discuss values and preferences
PREV MED 3-FH SHIFTING EXAM (17 FEBRUARY 2021) Page 7 of 18

[PRV-FH] 03 Care of the Dying (Batch 2022), p. 2


16. Prudent behavior towards LGBT patients prior to any D UNDERSTANDING LGBTQ+ ISSUES
clinic interview includes: ● Creating awareness and understanding on
A. Ask about civil status matters pertaining to the LGBT Community is
B. Ask if he/she has a girlfriend/boyfriend essential to:
C. Calling patient by his/her legal first name ○ promoting cultural competence among
D. Calling patient his/her preferred name healthcare providers
○ ensuring sensitivity towards them
● When addressing LGBT individuals,
healthcare providers should always ask
clients how they identify and/or wish to be
addressed.

Choice A and B: instead of asking “are you


married?” or asking if they have a
boyfriend/girlfriend, ask “do you have a partner?”
or “are you in a relationship?”
Remember: never assume.

[PRV-FH] 04 Optimal Primary Care for LGBTQ+


Patients (Batch 2022), p.1
17. The principles of pain symptom management include the C 5 Principles in Pain Control (WHO)
following EXCEPT ● Define the type and site of pain
A. Establish the etiology of pain ○ Pain is considered to be the "fifth vital
B. Treat each pain specifically sign“
C. Give morphine as needed ○ Then get the PQRST
D. Prescribe pain regimen that give relief 24/7 (Palliative/Provoking, Quality, Radiation
Severity, Timing)
● Anticipate pain breakthrough
● Reviewing regimen frequently and regularly
● Treat “total” pain
● Treat each pain specifically

[PRV-FH] 03 Care of the Dying (Batch 2022), p. 4


18. The physician makes his patient ventilate. What phase of C CEA for Counselling Skills – C: Catharsis
counseling is this?
A. Preparatory Can be used for the following questions:
B. Insight ● What came to your mind when you started
C. Catharsis feeling your symptoms? (Ano ang naisip mo
D. Action noong nakaramdam ka ng sakit?)
● What feelings came out when these thoughts
came to your mind? (Ano ang naging
damdamin mo noong naisip mo ang mga ito?)
● What consequence of your illness makes you
feel this way the most? (Ano ang pinaka
nakakatakot na maaaring mangyari dahil sa
sakit mo?)

[PRV-FH] 03 Care of the Dying (Batch 2022), p. 2


19. Which of the following is a correct wellness screening D Wellness
recommendation for 70 years old? - Advocate wellness for older persons.
A. Pap smear
B. Mammography Laboratory Tests
C. CBC FOBT
D. Fecal occult blood test ● 60-75 y/o - annually
● 76-85 y/o - if with risk
● >85 y/o - do not screen
Bone Densitometry
● >/= 65 y/o - screen not less than every 2 yrs
● 60-64 y/o - if with fracture risk
Pap Smear
● 60-65 y/o - every 5 years
● > 65 y/o with 3 adequate screening - do not
screen
Mammography
● 60-74 y/o - every 2 yrs
● >/= 75 y/o - no recommendation
PREV MED 3-FH SHIFTING EXAM (17 FEBRUARY 2021) Page 8 of 18

If at risk, do FBS, lipid profile, ECG

NOTE: No gender stated in the question, so FOBT


is the safest answer.

[PRV-FH] 01 Geriatrics (Batch 2022), p.7


20. A 68-year-old female is brought to your office by her son D Gestures associated with an emotion or behavior
with a complaint of headaches. She diverts her eyes to Suspicion, secretiveness:
look at her son when answering your questions. Which ● Folded arms, moving away from another
one of the following is the most likely reason for this ● Crossed legs
patient not making eye contact? ● Lack of eye contact (culturally sensitive)
A. She is depressed ● Hand covering mouth
B. She could be a victim of abuse ● Frown
C. She is intimidated by you as a doctor ● Scrunching in with head down
D. She could be hiding the truth from you
● Stolen look, sideways glance
● Sideways positioning
● “Poker face''
● Deception indicated by lack of eye contact
● Anxiety gestures
● Looking at floor
● Frequent swallowing
● Wetting lips
● Throat clearing
21. The risk that well-screened people will become addicted B Pain specialists view addiction as a series of
to opioid drugs when they're taking them for chronic pain specific behavior including:
is very high ● Compulsive use of medication
A. True ● Loss of control over medications
B. False ● Insistence on taking meds despite
deterioration in the quality of life (QOL)
Physical dependence on opioids occurs when
they are taken for more than twenty-four hours.
● Note: it is a normal, expected part of proper
pain rx.
● Meds are slowly tapered over time to ensure
that the patients do not experience severe
withdrawal symptoms due to dependence.

[PRV-FH] 03 Care of the Dying (Batch 2022), p. 5


22. Which of the following has good evidence on insomnia for A Sleep restriction therapy (SRT)
older persons? ● SRT improves time to fall asleep by 12 mins
A. Sleep restriction therapy and time asleep while in bed by 5-10%
B. Melatonin ● SRT improves sleep for 1 in 2-6 patients
C. Aromatherapy
compared to sleep hygiene alone
D. Diphenhydramine
Is melatonin effective for sleeping disorders?
● Quality of melatonin research is poor and at
high risk of bias
● If research is believable, melatonin may
help people fall asleep faster (~10 mins)
and spend more time asleep (~15 mins)
● Both of these amounts may be of limited
clinical value (choice B)

[PRV-FH] 01 Geriatrics (Batch 2022), p. 4


PREV MED 3-FH SHIFTING EXAM (17 FEBRUARY 2021) Page 9 of 18

23. Which of the following tests cannot assess language? D


A. MMSE
B. AD8
C. MOCA
D. Mini cog

[PRV-FH] 01 Geriatrics (Batch 2022), p.2


24. Angelo is a male yet attracted and had relationships with C Sexual Orientation
both genders. This is a reflection of his ● Based on how a person identifies attraction
A. Biologic sex on a physical and emotional basis
B. Gender expression ○ In other words, it refers to a person’s
C. Gender orientation
D. Gender identity
emotional, sexual, or relational
attraction to others.
● May be described by many terms:
○ Heterosexual, bisexual, homosexual
(lesbian or gay), queer, pansexual
and asexual
● Not a choice, but a person’s self-awareness
and acceptance can be influenced by many
factors including cultural environment,
stage of development, and beliefs.

[PRV-FH] 04 Optimal Primary Care for LGBTQ+


Patients (Batch 2022), p.2
25. Who of the following has cognitive impairment? D
A. 68 year old female finished Grade 5 with MMSE
of 22
B. 70 years old male finished Grade 6 with MOCS
score of 25
C. 60 year old female professor with MMSE of 24
D. 65 years old vendor finished second year high
school with MMSE of 22

[PRV-FH] 01 Geriatrics (Batch 2022), p. 2


26. Doctor said "I am sorry, I have some bad news for you C Setting
today. The pathology report revealed that you have ● Should be private and conducive to
adenocarcinoma.” Identify step based on the SPIKES undistracted and focused discussion
A. Setting
B. Perception Perception
C. Information ● Use open ended questions to create a
D. Invitation reasonable accurate picture of how the patient
perceives the medical situation

Invitation
● Obtain the patient’s invitation. Some desire full
information about their condition, but some do
not.

Knowledge (Information)
● Giving medical facts using nontechnical words
PREV MED 3-FH SHIFTING EXAM (17 FEBRUARY 2021) Page 10 of 18

Empathic Responses
● Observe and identify emotions, identify the
reason for the emotion, and let the patient
know that you have connected the emotion
with the reason by making a connecting
statement

Strategy and Summary


● Ask the patients if they are ready for
discussion on treatment options. Check for the
patient’s understanding.

[PRV-FH] 03 Care of the Dying (Batch 2022), pp.


2-3
27. Which inquiry is not necessary when taking sexual C Sexual History
history? ● Encourage patients to choose a surrogate
A. How do you identify your sexual orientation? decision-make
B. "Do you perform vaginal, anal and or oral sex?" ● Should include sexual behavior, orientation,
C. Do you have any relationship with a foreigner? and gender identity
D. Are there sexual concerns you would like to
● Instead of asking “Are you married?”, ask “Do
discuss?
you have a partner?” or “Are you in a
relationship?”
● Next you could ask “have you had female
partners, male partners, or both?”
○ Alternatively, you could start by asking
“are you sexually active?”
○ Then ask, “when you have had sex,
what are the genders of your partners?”
○ If patients are confused by this question,
you could ask “Do you have sex with
men, women, or both?”
● At the end of the sexual history:
○ “Do you have any concerns or questions
about your sexuality, sexual orientation,
or sexual health?”
● You should also ask whether patients identify
as transgender or have a gender-related
concerns:
○ “Since many people are affected by
gender issues, I ask patients if they
have any concerns about this. If this
topic is not relevant to you, tell me and I
will move on.”

[PRV-FH] 04 Optimal Primary Care for LGBTQ+


Patients (Batch 2022), p.3
28. "Patient: I am sick and tired of living with these B
headaches. No one has been able to help me, and none
of the medications are working. For the doctor’s
response, which is the appropriate empathic statement to
make?"
A. “I know how awful this must feel”
B. “I feel how frustrated you must be”
C. “I feel bad for you having to go through this”
D. “I can see that you are frustrated by the lack of
improvement in your symptoms.”

[PRV-FH] 02 Home Care & Difficult Encounters


(Batch 2022) p.5
29. "When taking sexual history, which of the following is D See number 27.
LEAST prudent to ask "
A. Are you sexually active?
B. "Do you have sex with men, women or both? "
PREV MED 3-FH SHIFTING EXAM (17 FEBRUARY 2021) Page 11 of 18

C. How many partners did you have in the past 6


months?
D. Which type of sexual contact brings you most
pleasure?
30. A comprehensive geriatric assessment was done in a 65 C A. Timed Up and Go Test
years old with constipation. Which of the following ● From a patient in sitting position → Stand
findings is a positive screen? without using arms, walk 3 meters, turn
A. 11 seconds to complete the get up and go test around → walk back, and sit down.
B. Unsteady reach but never had a fall ● Doctor will time the process and observe of
C. Muscle mass of 7 kg/m2 gait abnormalities
D. Forgetting which word to use ● Older adult who takes >12 seconds to
complete test is at high risk for falling
B. Functional Reach Test

C. Muscle Mass
● Encourage patients to choose a surrogate
decision-makeI

● If gait speed is >0.8 m/s, measure grip


strength using a Jamar Hand Dynamometer |
○ Grip strength NV
■ Males: >30 kg
■ Females: >20 kg
● If gait speed is ≤0.8m/s, measure muscle
mass using Dual X-ray Absorptiometry or
DEXA scan (appendicular muscle mass)
○ Muscle Mass NV
■ Males: >7.23 kg/m2
■ Females: >5.67 kg/m2
D. A positive test result for dementia will depend on
the scores of the different tests.

[PRV-FH] 01 Geriatrics (Batch 2022)


31. Which of the following is a correct intervention for geriatric A Recommendation for the elderly
syndromes?
A. Daily fiber intake of 25 grams per day to prevent A. Dietary fiber intake recommendation for
constipation constipation prevention: 20-35 g/day
B. 2 glasses of milk daily as part of the Pinggang B. Pinggang Pinoy for senior citizens: 1 glass of
Pinoy to prevent malnutrition milk
C. Tamsulosin helps decrease the size of the C. Tamsulosin only helps in reducing lower urinary
prostate tract symptoms in men with BPH but does not
D. Alendronate and calcium must be given for decrease the prostate’s size
osteopenia D. Alendronate and calcium for osteoporosis

[PRV-FH] 01 Geriatrics (Batch 2022,


32. You can recognize compassion in a professional or family D Empathy refers to feeling what another person is
carer if he/she would feeling. Sympathy means you understand what the
A. Show that they felt sorry for the person with other person is feeling even without feeling it
dementia yourself. Compassion means your feelings have
B. Make sure the person with dementia could prompted you to take action to relieve the suffering
enjoy their usual hobbies and carry out activities of another person.
of daily living in their preferred way
C. Would have excellent communication skills and Choice A shows sympathy
be respectful Choice B shows handling stigma of dementia
D. Be kind and when seeing a patient was in Choice C shows professionalism
distress would react by thinking and doing the
best response Source:
https://positivemindsinternational.com/the-danger-
of-confusing-empathy-or-sympathy-with-
compassion/#:~:text=Empathy%20refers%20to%
20feeling%20what,the%20suffering%20of%20ano
ther%20person.
PREV MED 3-FH SHIFTING EXAM (17 FEBRUARY 2021) Page 12 of 18

33. The following makes Documentation in Home Care C All of the other choices are reasons that make
essential EXCEPT: documentation essential in home care except for
A. Provides evidence of your patient outcomes choice C.
along with the quality of care and improvement
B. Essential to ensure reimbursement from payer Home care is usually less expensive, more
sources convenient and just as effective as the care one
C. Leads to incorrect treatment decisions and gets in a hospital or skilled nursing facility.
expensive and unnecessary diagnostic studies
D. Used to communicate the patients’ progress [PRV-FH] 02 Home Care & Difficult Encounters
with other members of the home health care (Batch 2022), p.1
team
34. The management of symptoms in terminal ill patients A Stages of care of the dying includes
involves a diagnostic evaluation for the cause of each communication, management of symptoms,
symptom when possible and giving the treatment for the hospice care, end of life and bereavement care.
identified cause when reasonable
A. True Management of symptoms includes control of
B. False symptoms such as pain, anorexia, nausea and
vomiting, dyspnea, cough, hiccups, bed sores,
mucositis, urinary incontinence, insomnia and
confusion.

[PRV-FH] 03 Care of the Dying (Batch 2022)


35. One should convene the family when a 9-year old boy A Patients that present with somatization may require
presents with psychiatric care and since the patient is only 9
A. Somatization years old, family support should be stressed.
B. Occasional enuresis
C. Monthly headaches Occasional enuresis, monthly headaches and
D. Failing grades in school failing grades may all be attributed to an underlying
source and may be resolved with appropriate
medication and intervention.

[PRV-FH] 02 Home Care & Difficult Encounters


(Batch 2022), p.4-5
36. Which of the following is TRUE regarding grief, mourning, C Grief usually begins before the anticipated
and bereavement? death|
A. The healthcare team should extend ● Explain the course of their illness and how
bereavement services for no more than one death will come
month after the death of the patient. ● Assure them that their physical symptoms will
B. The physician's attendance at the patient's be controlled
funeral would be inappropriate. ● Develop regular family meetings
C. Grief counseling should begin when the patient
is alive. A. Provides follow-up bereavement care for up to 1
D. Mourning is composed of sequential stages that YEAR AFTER the patient’s death
occur in order. B. NOT inappropriate
D. GRIEF is composed of sequential stages that
occur in order (Denial, Anger, Bargaining,
Depression, Acceptance)

[PRV-FH] 03 Care of the Dying [Batch 2022],


pp.4, 7-8
37. A family meeting should be done in the following situation: B When to consider convening the family?
A. 12 y/o female with acne ● Compliance problem
B. 65 year old female who suffered a stroke ● Treatment failure
C. 4 y/o old boy with acute nasopharyngitis ● Somatization
D. 30 y/o old male for annual employment physical ● Anxiety/Depression
● Substance abuse
● Parent-child problems
● Marital/Sexual difficulties
● Lifestyle problems
● High utilization of medical services by an
individual or family

[PREVMED] Family Medicine FIRST TERM 2018-


2019
38. Which of the following cases carries the greatest risk of B Risk factors for delirium:
delirium? ● Advanced age
A. An 82 year old female with dementia seen in ● Underlying brain diseases such as dementia,
clinic for a UTI stroke, or Parkinson disease, particularly
PREV MED 3-FH SHIFTING EXAM (17 FEBRUARY 2021) Page 13 of 18

B. A 72 year old male admitted to the wards for a when there are current problems with
heart failure exacerbation memory
C. A 78 year old female post-op day #1 for an ● Use of multiple medications (particularly
elective hip repair psychiatric drugs and sedatives), or multiple
D. A 70 year old female admitted to the hospital for medical problems
a hip fracture ● Sudden withdrawal of a regular medication or
cessation of regular alcohol use
● Frailty, malnutrition, immobility
● Advanced cancer
● Undertreated pain (although excessive use of
opioid pain medication for pain control can
also impair brain function
● Immobilization, including physical restraints
● Use of bladder catheters
● Limb fractures
● Interventions, including diagnostic tests
● Poor eyesight or hearing
● Sleep deprivation
● Organ failure (eg, chronic lung disease;
heart, kidney, or liver failure)

https://www.uptodate.com/contents/delirium-
beyond-the-basics#H3
39. "Joanna, 19 year old, wears a crew cut hairstyle, military C Gender expression
uniform and cap; she acts like a real man. This is a ● The spectrum of masculine and feminine
reflection of" characteristic in how one dresses, acts or
A. Biologic sex presents his/herself
B. Gender identity ● “She acts like a real man”
C. Gender expression ● Masculine / Feminine / Gender nonconforming
D. Gender orientation
Biological Sex
● Male / Female

Gender Identity
● The person’s internal sense of their own gender
● Cisgender / Transgender / Genderqueer /
Bigender

Gender Orientation
● Based on how a person identifies attraction on
a physical and emotional basis
● Heterosexual / Bisexual / Homosexual / Queer
/ Pansexual / Asexual

[PREV-FH] 04 Optimal Primary Care for LGBTQ+


Patients [B2022], pp 1-2
40. It is not possible to prevent dementia at present, but a C Risk reduction activities:
variety of actions may reduce risk. Risk reduction ● Be physically active
activities include: ● Eat healthily
A. Avoiding sports with high risk of head injury, ● Don’t smoke
such as boxing ● Drink less alcohol
B. Including more fish and red wine in the diet ● Exercise your mind
C. Exercising more, stopping smoking and ● Take control of your health
maintaining a healthy weight
D. Being very sociable and completing Sudoku *More than 1 answer seems to be right but the
puzzles when not out with friends BEST answer would be C.

https://www.alzheimers.org.uk/about-
dementia/risk-factors-and-prevention/how-reduce-
your-risk-dementia
41. Issues of caregiver neglect or abuse should be explored A Unhygienic/Malodorous patient
in these types of patients: ● May be a sign of caregiver neglect
A. Unhygienic ● Do not show dislike
B. Demanding ● Carry on with the interview and treat the
C. Somatizing problem
D. Frequent flier ● Do your best in conducting interview
● Educate the patient of caregiver on the
importance of hygiene
● Explore possible reasons for it
PREV MED 3-FH SHIFTING EXAM (17 FEBRUARY 2021) Page 14 of 18

Demanding Patient
● Explain the situation
● Address patient’s doubts and explore the
concerns and fears
● Empathize
● Last resort → suggest 2nd opinion

Somatizing Patient
● Collect detailed history of physical symptoms
● Explicitly express empathy towards physical
symptoms throughout the history
● Explore the response to previous questions
● Conduct further physical and cognitive
examination
● Educate and elicit feedback
● Discuss treatment plan and address issues
directly

Frequent flier
● Acknowledge the pattern of frequent visits
● Identify underlying reasons for the frequent
visits
● May be lonely, dependent or too
afraid/embarrassed to ask questions they really
want answered

[PRV-FH] 02 Home Care & Difficult Encounters


(Batch 2022), p. 5
42. Doctor said: “Could you just tell me the general drift of A Summary
what I have just explained to you?” ● Check for the patient’s understanding of the
A. Summary discussion
B. Giving information
C. Setting Giving information
D. Patient's perception ● Warning the patient that bad news is coming
may lessen the shock following the disclosure.
This may also facilitate information processing
● Example: “Unfortunately, I’ve got bad news to
tell you” or “I’m sorry to tell you that…”

Setting
● Arranging privacy, involving significant others,
sitting down and making connection to the
patient, managing time constraints and
interruptions

Patient’s perception
● Before discussing medical information, the
clinician uses open-ended questions to create
a reasonably accurate picture of how the
patient perceives the medical situation.
● Example: “What have you been told about your
medical situation so far?” or “What is your
understanding of the reasons we did the MRI?”

[PRV-FH] 03 Care of the Dying (Batch 2022), p. 2


43. Disclosure allows the patient to make practical and A Benefits of disclosure
emotional adjustments to the terminal illness; thereby ● In a study by Centeno-Cortes et al., there was
providing closure to his or her life. no increase in perceived symptoms of anxiety,
A. True despair, sadness, depression, insomnia or fear
B. False in the group of informed patients compared to
the uninformed patients
● 75% of informed patients were able to share
their concerns about the illness and its
consequences with their relatives.
● Families of informed patients felt that they were
in a better position to care for the patient.
● Allows the patient to make practical (e.g.
making a will, settling bank accounts and
PREV MED 3-FH SHIFTING EXAM (17 FEBRUARY 2021) Page 15 of 18

mortgages) and emotional adjustments to


the illness; thereby providing closure to his
or her life.
● Diagnosis disclosure in cancer is also
correlated with improved symptom control and
patient satisfaction with care.

[PRV-FH] 03 Care of the Dying (Batch 2022), p. 3


44. When someone with a serious, life-limiting illness is no A Artificial nutrition and hydration is a medical
longer able to eat or drink, artificial nutrition will no longer treatment that allows a person to receive nutrition
bring the person back to a healthy state. (food) and hydration (fluids) when they are no
A. True longer able to take them by mouth.
B. False
When someone with a serious, life-limiting illness
is no longer able to eat or drink it usually means
that the body is beginning to stop functioning.
Artificial nutrition and hydration will not bring
the person back to a healthy state.

2006. Artificial Nutrition (Food) and Hydration


(Fluids) at the End of Life. [ebook] Hinds Hospice.
Available at:
<http://hindshospice.org/uploads/1/5/2/5/1525236
8/artificialnutritionandhydration.pdf> [Accessed 17
February 2021].
45. Cancer treatment such as chemotherapy can cause A Gastrointestinal (GI) side-effects of
constipation chemotherapy are a debilitating and often
A. True overlooked clinical hurdle in cancer management.
B. False Chemotherapy-induced constipation (CIC) and
Diarrhea (CID) present a constant challenge in the
efficient and tolerable treatment of cancer and are
amongst the primary contributors to dose
reductions, delays and cessation of treatment.

McQuade, R. M., Stojanovska, V., Abalo, R.,


Bornstein, J. C., & Nurgali, K. (2016).
Chemotherapy-Induced Constipation and
Diarrhea: Pathophysiology, Current and Emerging
Treatments. Frontiers in pharmacology, 7, 414.
https://doi.org/10.3389/fphar.2016.00414

Constipation is a common and distressing


problem for many individuals with cancer
during treatment and palliative care, and perhaps
even during survivorship.

Wickham R. J. (2017). Managing Constipation in


Adults With Cancer. Journal of the advanced
practitioner in oncology, 8(2), 149–161.
46. The following describes the scope of hospice care: C Hospice Care
A. Prolongs the dying process by any artificial ● Highlights quality rather than length of life
means of support ● Consists of palliative and supportive services
B. Offers comfort and focuses on helping the for dying persons and their families
patient become independent ● Provides follow-up bereavement care for up
C. Care and services provided does not end with to 1 year after patient’s death
the death of the patient
D. Benefits patients who are elderly and with [PRV-FH] 03 Care of Dying [B2022] p. 7
chronic illness
47. Which of the following is considered normal in a 73 year A Optimal waist circumference cut-off for ≥70 y/o
old female? ● Males: 109 cm (43 in)
A. Waist circumference of 37 inches ● Females: 98 cm (38 in)
B. Mid upper arm circumference of 20 cm.
C. Grip strength of 18 kg B. Normal mid-arm circumference: 26cm
D. Functional reach test score of 9.5 C. Normal grip strength
● Males: >30 kg
● Females: >20 kg
D. Normal functional reach test cut-off for 70-87
years old
● Males: 13.16 in
PREV MED 3-FH SHIFTING EXAM (17 FEBRUARY 2021) Page 16 of 18

● Females: 10.47 in

[PRV-FH] 01 Geriatrics (Batch 2022), p. 5


48. Dina, 32 year old, believes she is a woman in mind and D Transwoman refers to a woman who was assigned
heart but is still for genital reassignment to complete her as male at birth. She may or may not be identified
womanhood. Dina is a: by others as trans, and may or may not identify
A. Asexual herself as trans, It is grammatically and
B. homosexual definitionally correct to include a space between
C. Transman trans and woman.
D. Transwoman
Genderminorities.com
49. Grace thinks she is neither a man nor woman. Grace D Queer - denoting or relating to a sexual or gender
does not really care if a male or female person will be identity that does not correspond to established
attracted to her. She is described as ideas of sexuality and gender, especially
A. Homosexual heterosexual norms.
B. Heterosexual
C. Bisexual
D. Queer
50. Modified Caregiver Strain Index score of Maria is 27. This B MCSI Scoring
means ● ≤ 23 – Normal
A. Normal ● 24-28 – Predisposition to strain
B. Predisposition to strain ● ≥ 29 – Severe caregiver strain
C. Mild caregiver strain
D. Severe caregiver strain [PRV-FH] 01 Geriatrics (Batch 2022), p. 6

51. In a demanding patient who questions your proposed D Demanding Patient Case Stimulation
diagnostic and therapeutic plan, the most appropriate ● Explain the situation to the patient
thing to do the following except: ● Address the patient’s doubts regarding your
A. Explore the patient's doubts ability
B. Explain the situation to the patient ● Explore the patient’s concerns and fears
C. Encourage patient to get a second opinion ● Empathize
D. Comply with patient so as not to antagonize ● Last resort is to suggest a second opinion
him/her
[PRV-FH] 1.02 Home Care & Difficult Encounters
(Batch 2022) p. 4
52. The goal of hospice care to keep the patient comfortable A Hospice Pain Management
and improve the quality of life through adequate symptom ● Pain management is one of the primary goals
control of hospice care: keeping the patient
A. True comfortable and managing symptoms to
B. False ensure they have the highest quality of life for
as long as they live

All You Need to Know About Pain Management in


Hospice (crossroadshospice.com)
53. Which of the following is a correct recommendation for C In patients >60 years old, what are the benefits of
prevention in older persons? Vit. D on treatment of falls, fractures and mortality?
A. Gingko biloba to prevent dementia ● Vit D use of around 800-1000 IU to reduce
B. Calcium 500 mg once a day fractures and overall mortality in older
C. Vitamin D 800 IU to prevent fracture and fall patients
D. Vitamin A to prevent macular degeneration
[PRV-FH] 01 Geriatrics (Batch 2022), p. 2
54. If the International Prostate Symptom Score (IPSS) score C International Prostate Symptom Score can be
of a patient is 18 and he is very bothered by his utilized to measure the severity of lower urinary
symptoms, this means tract symptoms. A score of:
A. Normal ● 0-7 = mild symptoms
B. Mild incontinence ● 8-19 = moderate symptoms
C. Moderate incontinence ● 20-35 = severe symptoms
D. Severe incontinence
International Prostate Symptom Score (IPSS) |
QxMD
55. Which of the following dietary pointers is correct for C Non-pharmacologic interventions for
elderly? constipation
A. Vitamin supplements should always be given to
older persons.
B. Simple carbohydrates should be increased Intervention Intake Notes
C. Need for water amounting to 1.5 liters/day for a
60 kg elderly male water 1.5 to 2 improves stool
supplementation L frequency if on
PREV MED 3-FH SHIFTING EXAM (17 FEBRUARY 2021) Page 17 of 18

D. Protein intake of 48g/day for a 60 kg elderly


female a high fiber diet

daily fiber intake 20 to 35 intake should


g/day be slowly
increased over
several weeks
to decrease
adverse effects

D. Interventions in elderly with sarcopenia: Protein


requirement of 1 g/kg/day

[PRV-FH] 01 Geriatrics (Batch 2022), p. 5


56. A 65 year old female had Influenza vaccine 6 months ago, C *Erratum*
PPSV 23 last year, Herpes zoster vaccine 5 years ago, PPSV23 was given a year ago based on
Tdap 3 years ago. Which vaccine must be given to her vaccination history. This vaccine is only given
now? every 5 years (if given a second time at all).
A. Flu quadrivalent
B. Herpes zoster PCV13 should be the vaccine that is given 1 year
C. PPSV 23 after the PPSV23 vaccine and is recommended to
D. Varicella zoster protect geriatric patients from severe pneumonia

A. Influenza vaccine annually


B. Herpes zoster vaccine single dose

[PRV-FH] 01 Geriatrics (Batch 2022), p. 7


57. When Joseph was diagnosed with chronic renal disease, D A cornerstone in the management of chronic
family meals were adjusted and tailored to meet Joseph’s illnesses are the lifestyle changes needed to
dietary needs. Which basic premise of family-centered mitigate further complications of the disease and
care was evident? support of the family in this aspect is very
A. The family is the primary source of health important.
beliefs and behaviors
B. The stress of family developmental transitions
may become manifest in physical symptoms
C. Somatic symptoms can serve an adaptive
function within the family and be maintained by
family patterns
D. Families are a valuable resource and source of
support for the management of illness

[PRV-FH] 02 Home care and Difficult Clinical


Encounters (Batch 2022), p.1
58. Doctor said: “I can see how upsetting this is to you. I D Doctor is acknowledging the patient’s feelings and
guess anyone might have that same reaction.” affirming that what they are feeling is justifiable and
A. Invitation normal
B. Knowledge
C. Information
D. Empathic Response

[PRV-FH] 03 Care of the Dying (Batch 2022), p.3


PREV MED 3-FH SHIFTING EXAM (17 FEBRUARY 2021) Page 18 of 18

59. How families adapt to the stresses and changes imposed A


by the experience of living with dying depends on their
previous experiences with death
A. True
B. False

[PRV-FH] 03 Care of the Dying (Batch 2022), p.8


60. Which of the following is considered as the first line option A
in Step 2 of WHO pain management?
A. Oxycodone
B. Methadone
C. Fentanyl
D. Morphine

[PRV-FH] 03 Care of the Dying (Batch 2022), p. 4


PREVENTIVE MEDICINE 3

10 FEBRUARY 2021
Disclaimer: No answer key was given so please use at your own risk. We believe some items have multiple correct answers.
QUESTION ANSWER RATIONALE
1. In communicating with the family of the dementia patient, which of C Handle Stigma
the following health education statements is correct? ● Not all dementia is Alzheimer
A. Pharmacologic regimen can cure the disease ● Orient pedigree
B. Alzheimer’s disease is communicable ● Deterministic vs risk genes
C. Prognosis for patients with Alzheimer’s is 5-8 years ● Not communicable (choice B)
D. Neurobic exercises can revert back memory of patient Inform about prognosis
● there is no cure for AD (choice A).
Current treatment is focused on cognitive,
functional, and behavioral intentions.
(prevent progression of disease but you
cannot revert back the memories of the
patient to their usual memory) (choice D)
● Average lifespan of patient with AD is 4-8
years after diagnosis
Explain Treatment
● No cure for AD and other dementias
(choice D)
● Treatment is both drug and non-drug
therapy

[PREV MED3] 1.01 - Geriatrics page 3


2. Johnny has lung cancer and has been experiencing difficulty C Dyspnea
breathing due to pleural effusion. The most definitive management ● several causes
is: ● Pharmacologic management:
○ Antibiotics: pneumonia
A. low dose morphine
○ Thoracentesis: Pleural Effusion
B. IV antibiotic regimen
C. thoracentesis if effusion is significant
D. Oxygen inhalation by face mask. [PREV MED3] 1.02 - Care of the Dying page 6
3. Which of the following is considered a normal result of the B Mini Nutritional Assessment (MNA) (choice A)
Comprehensive Geriatric Assessment? Screening score for the LONG FORM
A. MNA-Long form score of 14 for a 60-year-old female ● 24-30 - normal nutritional status
● 17-23.5 - at risk of malnutrition
B. Functional reach test of 14 in a 69-year-old female ● <17 - malnourished
C. 28 kg grip strength in a 65-year-old male
D. Get up and go test speed of 13 sec in a 65-year-old Functional Reach Test (choice B)

Age Men Women

20-40 16.73 in 14.64 in


41-69 14.98 in 13.81 in
70-87 13.16 10.47

Grip Strength NV: (choice C)


● Males: >30kg
● Females: >20kg

Timed Up and Go Test (choice D)


● Older adult who takes >12 seconds to
complete test is at high risk for falling

Page 1 of 7
PREVMED 3-FAMILY HEALTH Quiz 1 – GERIATRICS; CARE OF THE DYING; HOME CARE (10 FEBRUARY 2021) Page 2 of 7

[PREV MED3] 1.01 - Geriatrics pages 1,2,5,6

4. Physicians should prescribe opioids in the following manner: A&B WHO 3-step process: (choice A)
A. Oral is the preferred route of administration of opioids ● Step 1: NSAIDs
B. Use short-acting morphine for breakthrough pain control ● Step 2: Weak opioids
● Step 3: Strong opioids
C. Morphine should be given only to patients not prone to
Factors to consider:
addiction ● Absorption and convenience: Oral>Rectal,
D. For pain control, give the medication strictly on as needed IV>IM
basis ● Efficiency and half life

Breakthrough pain (choice B)


treatment: Immediate-release opioids or rescue
medications
● quick acting, potent analgesics (e.g. oral
morphine)
● for times when pain breaks through
normal pain control

Physical dependence on opioids occurs when they


are taken for more than 24 hours. (choice C)

WHO Analgesic Ladder


** Drugs given on an “as required” basis or pro re
nata (PRN) usually results in poor pain control
(choice D)

5. A 76-year-old retired architect has had Alzheimer’s disease for 8 B The first line of treatment for the behavioral
years. He was reviewed by the community Geriatrician last month as symptoms of Alzheimer's is non-drug approaches,
the patient has become agitated. At the time the patient was found but if these strategies fail and symptoms are
severe, medication may be appropriate.
to be in pain and he was given some simple analgesics with good
effect. You assess the patient and establish that he is not in pain
nor has obvious delirium. The patient is wandering on the ward https://www.alz.org/help-support/caregiving/stages-
and saying that he can see rabbits in the corridors. The patient is behaviors/suspicions-delusions
not a danger to self or others. What is the first line management
plan?
A. Quetiapine
B. Ensure well-lit rooms
C. Olanzapine
D. Risperidone
6. The following describes the scope of home care: B HOME CARE
A. Care and services provided end with the death of the patient ● Provision of comprehensive health care,
B. Benefits patients who are elderly and with chronic illness services are provided in places of
C. Aims to prolong the dying process by any artificial means of residence
support ● Goals
D. Services are limited and focused only on the chronically ill ○ To promote, maintain or restore
patient health of the patient
○ To minimize the effect of disability
● Allows a person with special needs to stay
in their home
● Might be good for people who are
getting older, are chronically ill,
recovering from surgery, or disabled

Batch 2022 Trans 1.02 Home Care and Difficult


Encounters, p. 1
7. Which part of the SPIKES is it when you create a comfortable A SETTING up the Interview
environment and encouraging presence of family or friend? Guidelines:
A. Setting ● Arrange for some privacy.
B. Perception ○ Make sure interview room is ideal
C. Invitation
PREVMED 3-FAMILY HEALTH Quiz 1 – GERIATRICS; CARE OF THE DYING; HOME CARE (10 FEBRUARY 2021) Page 3 of 7

D. Knowledge ○ Have tissues ready in case


patients become upset
● Involve significant others.
○ Most patients want to have others
with them, but this should be the
patient’s choice.
○ When there are several family
members, ask the patient to
choose 1 or 2 family
representatives.
● Sit down.
○ Relaxes the patient
○ Try not to have barriers between you
and the patient.
● Make a connection with the patient.
○ Uncomfortable, but it is an important
way of establishing rapport with your
patient
○ Touching the patient on the arm or
holding a hand are other ways to
accomplish this
● Manage time constraints and interruptions
○ Inform the patient of any time
constraints and interruptions that you
may expect.

Batch 2022 Trans 1.02 Home Care and Difficult


Encounters, p. 1
8. Miguel has colon cancer. He has abdominal pain rated as the B
severity of 8/10 on the numerical scale. Based on the WHO step
ladder approach to pain control, the drug of choice is?
A. NSAID
B. Morphine
C. Tramadol
D. Paracetamol

Batch 2022 Trans 1.03 Care of the Dying, p. 4


9. The following makes Documentation in Home Care essential D Documentation in Home Care essential leads to
EXCEPT: correct treatment decisions and avoid expensive and
A. it is used to communicate the patient’s progress with the unnecessary diagnostic studies.
other members of the home health care team
B. it is essential to ensure reimbursement from payer sources
C. it provides evidence of your patient outcomes along with the
quality of care and improvement
D. it leads to incorrect treatment decisions and expensive and
unnecessary diagnostic studies
10. The following is true of fentanyl in cancer pain control, EXCEPT: C The drug of choice medication for WHO STEP 2 pain
A. It is effective and well tolerated for the treatment of chronic management in cancer patients is CODEINE.
pain Fentanyl is under WHO STEP 3 pain management.
B. There is improved convenience and compliance resulting
from administration every 72 hours Batch 2022 Trans 1.03 Care of the Dying, p. 4
C. It is the drug of choice medication for WHO STEP 2 pain
management in cancer patients
D. It is a useful analgesic for cancer patients who are unable
to swallow
PREVMED 3-FAMILY HEALTH Quiz 1 – GERIATRICS; CARE OF THE DYING; HOME CARE (10 FEBRUARY 2021) Page 4 of 7

11. In giving knowledge to the patient in SPIKES: B


A. dump the truth right from the start regardless of the patient’s
preference
B. we should avoid using medical jargons
C. provide them with a false diagnosis so they won't lose hope
D. conspire with the family to withhold information from the
patient

Batch 2022 Trans 1.03 Care of the Dying, p. 4

12. Which of the following is an instrumental activity of daily living? B


A. Using a walker for support
B. Managing day to day purchase
C. Using bathtub with assistance
D. Feeds self without assistance

https://www.nursingcenter.com/ce_articleprint?an=0
0000446-200804000-00023
13. The following are considered adjuvant drugs in cancer pain D Adjunct Analgesics
management, EXCEPT: ● drugs with primary indication other than pain
A. Muscle relaxant that have analgesic properties in some painful
B. Corticosteroid conditions:
C. Antidepressant ○ Corticosteroids (pain d/t inflammation and
D. Multivitamins swelling)
○ Tricyclic antidepressants (amitriptyline,
nortriptyline, doxepine)
○ Anticonvulsants (valproate,
carbamazepine, gabapentine; for
radiculopathies)
○ Benzodiazepines (Diazepam; anxiolytic)
○ Muscle relaxants (tizanidine, lioresal)
○ Anesthetics (for indwelling epidural
catheter)

Batch 2022 Trans 1.03 Care of the Dying, p. 5


14. According to the principle of pain symptom management, the B Disclaimer: we only ruled out the other choices
following is true:
A. all pain are due to cancer itself A - Concept of Total Pain
B. treatment should be specific to the pathophysiology of pain ● Includes not only the physical symptoms
C. you should not combine opioids with non-opioids from the disease or the treatment, but also
D. start the patient on strong opioids even when pain is mild the financial, physical, and spiritual
components
- Causes of pain in people with cancer: etiology
● Pain d/t cancer alone (85-93%) most
common
● pain d/t cancer dx and tx (17-21%)
PREVMED 3-FAMILY HEALTH Quiz 1 – GERIATRICS; CARE OF THE DYING; HOME CARE (10 FEBRUARY 2021) Page 5 of 7

● pain unrelated to cancer nor its tx (2-9%)

C & D - WHO 3- Step Process: The Pain Relief


Ladder

Batch 2022 Trans 1.03 Care of the Dying, p. 4


15. Which is the best example for polypharmacy? Your patient is C Polypharmacy
A. filling her medications at more than 1 drugstore. ● 4 or more prescription medications
B. taking more than 2 medications. ● 3 or more new prescriptions medications in
C. taking more than 9 medications 24 hours
D. taking a potentially inappropriate combination of medicines. ○ 4 meds increase the risk of fall
○ 5 meds increase the risk of
adverse effects
[PREV MED3] 1.01 - Geriatrics p. 4
16. Which of the following is a correct wellness recommendation in a D Salient Features:
68-year-old female who received Influenza vaccine, 3 months ago, - 68-year-old
PCV 13 and PPSV 23, 5 years ago, Tetanus toxoid, 5 years ago. Pap - Female
smear done last year is normal. Mammography done last year was - With influenza and pneumococcal
normal. Has no family history of osteoporosis but claims to have back vaccines, tetanus toxoid
pain. What is the correct intervention now? - Pap smear done last year
A. Give PPSV 23 now - Normal mammography from a year ago
B. Repeat mammography now - No family history of osteoporosis BUT has
C. Do pap smear now back pain
D. Have bone densitometry done
Bone Densitometry
● >65 y/o - screen not less than every 2
years
● 60-64 - if with fracture risk

Pap smear
● 60-65 y/o - every 5 years
● >65 y/o - if with 3 adequate screening, do
not screen

Mammography
● 60-74 y/o - every 2 years
● >75 y/o - no recommendation

[PREV MED3] 1.01 - Geriatrics p. 7


17. Which of the following is a correct provision of the senior citizens A&B RA 9994 Expanded Senior Citizens Act of 2010
act? 1. 20% discount and VAT exemption for:
A. 20% discount on diapers and groceries a. Purchase of medicine including the flu and
B. 20% discount on burial services pneumococcal vaccines and other
C. exemption from income tax payment essential medical supplies, accessories
D. 5% discount on electricity consumption of 150 kwh and equipment [Choice A]
b. Professional fees of MD in all private
hospitals, medical facilities, OPD and home
health care
c. Medical and dental services, diagnostic and
laboratory fees
PREVMED 3-FAMILY HEALTH Quiz 1 – GERIATRICS; CARE OF THE DYING; HOME CARE (10 FEBRUARY 2021) Page 6 of 7

d. Actual transportation fare for land travel and


domestic air transport and sea shipping
vessels
e. Hotel and lodging establishments,
restaurants
f. Funeral and burial services
g. Admissions to cinema
2. Payment of individual income tax below
minimum wage [Choice C is wrong]
3. 5% on water and electricity registered on their
name not exceeding 100 kwh and 30 mm3
[Choice D is wrong]
4. Educational assistance to pursue secondary,
tertiary, vocational and technical courses and
short-term courses in schools
5. Exemption from training fees for
socioeconomic programs
6. Death benefit of Php 2000 to nearest surviving
relative
7. Express lanes in establishments
8. Mandatory PhilHealth coverage for indigenous
senior citizens

Batch 2022 Trans 1.01 Geriatrics, p. 6


18. A 62-year-old man is admitted to the hospital for evaluation of D Angry Patient
abdominal pain. During the interview, he responds to the question ● Allow complete angry outburst.
with a single word and sometimes with sarcastic answers. He does ● Make a conscious effort to slow down
not engage in eye contact, and he frowns as he tells the physician response. Do not reply immediately. Pay
that this is the third time he has been asked these questions. Which special attention to the speed and volume of
of the following is the most appropriate response by the physician? your speech.
A. “Can you tell me more about your abdominal pain?” ● Validate the frustration with empathic
B. “I understand that you do not feel well, but I need to get a comments:
history here.” ○ “I understand that you are upset at having
C. “I’ll come back once you’ve calmed down, and we can talk to wait, and I apologize for the delay.”
more then.” ○ “I can understand why you are upset. I
D. “You sound upset. Tell me more about that.” appreciate your waiting for me.”
● Involve the patient in the outcome. Make a
compromise, find a way to console, such as a
peace offering how one will make it up to him.
○ If one feels angry with what the patient is
telling you, learn to disengage. (No
mention of the physician i.e. you are
getting angry so not choice C)
● Provide the opportunity to discuss his
concerns.
○ “How can I help you today?”

Batch 2022 Trans 1.02 Home Care and Difficult


Encounters, p. 4
19. A 79-year-old woman was admitted for an elective right knee C Delirium
replacement under general anesthesia. She had a past medical ● Complex neuropsychiatric syndrome
history of type 2 diabetes and hypertension and was taking metformin characterized by acute onset of disturbance
and ramipril. After an uneventful surgery she was found to be of consciousness and fluctuating changes
confused and disoriented the next day. in cognition, attention, and perceptual
disturbance
Which of the following predicts delirium over other causes of ● Most common reason for acute cognitive
confusion? dysfunction in hospitalized older people.
● Prevalence of delirium at admission ranges
A. General anesthesia from 10 to 31%, incidence of new delirium per
B. hallucinations admission ranges from 3 to 29% and
C. history of dementia occurrence rate per admission varies
D. inattention between 11 and 42%
● May be prevented in up to a third of older
patients → early recognition is vital

Factors Significantly Associated with Delirium


● Dementia
● Older Age
PREVMED 3-FAMILY HEALTH Quiz 1 – GERIATRICS; CARE OF THE DYING; HOME CARE (10 FEBRUARY 2021) Page 7 of 7

● Comorbid illness
● Severity of medical illness
● Infection
● ‘High-risk’ medication use
● Diminished activities of daily living
● Immobility
● Sensory impairment
● Urinary catheterization
● Urea and electrolyte imbalance
● Malnutrition

“In pooled analyses, dementia (OR 6.62; 95% CI


(confidence interval) 4.30, 10.19), illness severity
(APACHE II) (MD (mean difference) 3.91; 95% CI
2.22, 5.59), visual impairment (OR 1.89; 95% CI
1.03, 3.47), urinary catheterization (OR 3.16; 95%
CI 1.26, 7.92), low albumin level (MD −3.14; 95%
CI −5.99, −0.29) and length of hospital stay
(OR 4.85; 95% CI 2.20, 7.50) were statistically
significantly associated with delirium.”

Ahmed, S., Leurent, B., & Sampson, E. L. (2014). Risk factors


for incident delirium among older people in acute hospital
medical units: a systematic review and meta-analysis. Age and
ageing, 43(3), 326-333.
20. A 75-year-old man with diabetes and osteoarthritis is evaluated in C 75-year old male with DM & OA
your clinic for recurrent falls. Which of the following is most predictive CC: RECURRENT FALLS
of future dependence in his activities of daily living?
A. T score on his bone density test of -2.7 Diagnosis of Falls
B. A greater than 20-point difference between sitting and
standing systolic blood pressure ● Timed Up and Go Test
C. Taking 25 seconds to rise from a chair, walk 10 steps, turn ○ From the patient in sitting position →
around, walk back, and sit down in that same chair Stand without using arms, walk 3 meters,
D. Impaired 2-point discrimination in his lower extremities turn around → walk back, and sit down.
○ Doctor will time the process and observe
of gait abnormalities
○ Older adult who takes >12 seconds to
complete test is at high risk for falling
[Choice C]
● Functional Reach Test
○ Yardstick on wall parallel to floor, at height
of acromion of subject’s dominant arm.
Subject with feet distanced apart, make a
fist, forward flex to 90 degrees.
○ Subject will reach forward without taking
a step or touching the wall
○ Measure distance between start and end
using the head of the metacarpal of the
3rd finger as a reference point.
○ NOTE: Be cautious because the patient
might fall!

Other Choices:
● Osteoporosis (A) and orthostatic hypotension
(B) may also be associated with increased
falls, but C is most diagnostic of fall risk.
● D - Peripheral neuropathy secondary to DM is
not pertinent to pt’s chief complaint.

Batch 2022 Trans 1.01 Geriatrics, pp. 1, 2, 4


PREVENTIVE MEDICINE 3 [APPLIED EPIDEMIOLOGY]

QUIZ 1: GERIATRICS, HOME CARE & DIFFICULT ENCOUNTERS,


CARE OF THE DYING
Disclaimer: No answer key was given, we tried to rationalize everything as much as we could.
QUESTION ANSWER RATIONALE
1. A 78 year old female hypertensive post coronary artery bypass B Vaccines
grafting received influenza vaccine 2 weeks ago, PCV 13 and PPSV ● Pneumococcal vaccine (PCV13 followed by
23 5 years ago, Herpes zoster vaccine 5 years ago, Tetanus toxoid in PPSV23 after 6-12 mos)
2009. What vaccine will you recommend now? à If PPSV23 was administered first, give
A. Tetanus toxoid PCV13 after 1 yr
B. COVID 19 ● Influenza vaccine annually
C. Varicella ● Herpes Zoster vaccine single dose
D. PPSV 23 ● Tdap
à Tdap, TD, Tt – 3 doses (1 Tdap first then 2
Td) at 0, 1, 6-12 mos
à Booster every 10 yrs

By elimination, other vaccines apart from COVID-19


which are not given are still valid.

Geriatrics Trans (2022). p. 7


2. Which of the following is a correct provision of the senior citizen C/D* RA 9994 Expanded Senior Citizens Act of 2010
act?
A. Death benefit of P3000 to nearest surviving relative 1. 20% discount and VAT exemption for:
B. 5% on water and electricity covering 40 mm3 and 200 ● Purchase of medicine invluding the flu and
kilowatt hour respectively pneumococcal vaccines and other
C. 20% discount on dental service excluding VAT essential medical supplies, accessories
D. Mandatory PhilHealth coverage and equipment
● Professional fees of MD in all private
hospitals, medical facilities, OPD and
home health care
● Medical and dental services, diagnostic
and laboratory fees
● Actual transportation fare for land travel
and domestic air transport and sea
shipping vessels
● Hotel and lodging establishments,
restaurants
● Funeral and burial services
● Admissions to cinema
2. Payment of individual income tax below
minimum wage
3. 5% on water and electricity registered on their
name not exceeding 100 kwh and 30 mm3
4. Educational assistance to pursue secondary,
tertiary, vocational and technical courses and
short-term courses in schools
5. Exemption from training fees for socioeconomic
programs
6. Death benefit of Php 2000 to nearest surviving
relative
7. Express lanes in establishments
8. Mandatory Philhealth coverage for indigenous
senior citizens

Geriatrics Trans (2022). p. 6

Page 1 of 5
PREVMED3 – Family Health – Quiz 1 GERIATRICS, HOME CARE & DIFFICULT ENCOUNTERS, CARE OF THE DYING (March 24, 2021) Page 2 of 5

3. All medications used as sleeping aid have adverse effects and C Pharmacologic Management of Insomnia
increase the risk of falls. If medications are needed, the lowest ● Melatonin 1 tab, 2-3h before bedtime
effective dose should be used for the shortest duration, which of the ● Avoid benzodiazepines and hypnotics
following medications is not considered the safest for use in the ● Zolpidem 10mg may be given if no
elderly based on consensus currently? response to nonpharmacologic or
A. Zopiclone melatonin
B. Zolpidem
C. Diazepam Geriatrics Trans (2022). p. 4
D. Doxepin
4. Based on the Asian Working Group for Sarcopenia, which of the A Following extensive deliberations, AWGS 2019
following defines sarcopenia? contends that diagnosing sarcopenia requires
A. Decrease muscle mass, decrease muscle strength, measurements of both muscle quality and quantity
decrease physical performance and defines persons with low muscle mass, low
B. Decrease muscle mass, increase muscle strength, increase muscle strength, and low physical performance
physical performance as having “severe sarcopenia.”
C. Increase muscle mass, decrease muscle strength, increase
physical performance Chen, L.-K., Woo, J., Assantachai, P., Auyeung, T.-
D. Increase muscle mass, increase muscle strength, decrease W., Chou, M.-Y., Iijima, K., … Arai, H. (2020). Asian
physical performance Working Group for Sarcopenia: 2019 Consensus
Update on Sarcopenia Diagnosis and Treatment.
Journal of the American Medical Directors
Association. doi:10.1016/j.jamda.2019.12.012
5. Mrs. Tee consulted after sustaining a fall. She has had three falls B FRAILTY
this year. She also complains of increased lethargy over the last six Clinical Syndrome
months with difficulty going out of her house to even do grocery Including 3 or more of the following:
shopping. She lives on the second floor and used to be able to climb ● Unintentional weight loss of >10lbs in the
the stairs to reach her room. However, in the last six months, she has previous year
been unable to climb up and has to take the lift daily. She does not ● Self-reported exhaustion
want to eat at times and lost 6 kg over the last year from her usual ● Weakness (as measured by grip strength
weight of 46 kg to 40 kg now. What is the possible reason why Mrs. in the lowest 20% by gender and BMI)
Tee’s falling? ● Slow walking speed (lowest 20% by
A. Stroke gender & height)
B. Frailty ● Low physical activity (as measured by kcal;
C. Malignancy lowest 20%)
D. Depression Red Flags
● Slowness in walking
● Sarcopenia
● New onset depression
● > 5 chronic medication
Warning Signs
● Low level of activity
● Exhaustion
● Unintentional weight loss
● Atypical mood swings
Risk Factors
● Chronic diseases
● Physiologic impairment

Geriatrics Trans (2022). p. 6


6. Persons with dementia can be helped to communicate by B/D There is no cure for AD. Current treatment is focused
A. Understanding how different parts of the brain control on cognitive, functional and behavioral intentions.
different abilities (prevent progression of the disease but you cannot
B. Making lists and talking about one thing at a time revert back the memories of the patient to their usual
C. Using “elderspeak” which involves talking to people with memory)
dementia as if they were children
D. Building rapport, listening, and use shorter sentences Some points in communication with people with
dementia should include using short sentences,
asking one question at a time, speaking slowly,
maintaining eye contact, avoiding criticizing or
correcting.

https://www.alz.org/help-support/caregiving/daily-
care/communications
PREVMED3 – Family Health – Quiz 1 GERIATRICS, HOME CARE & DIFFICULT ENCOUNTERS, CARE OF THE DYING (March 24, 2021) Page 3 of 5

For 7-10. Hospice vs. Palliative Care


A. Palliative care
B. Hospice care
C. Both
7. Team is made of doctors and nurses, and includes others such as B Hospice care
social workers, nutritionists, and chaplains ● Consists of palliative and supportive services
8. Can be provided along with curative treatment and may begin at B for dying persons and their families
the time of diagnosis ● Usually those who have less than 6 months to
live.
● Highlights quality rather than length of life
● Provided by interdisciplinary team of
professionals and volunteers (care
coordinator, nurse, physician, counselor,
volunteer coordinator, and spiritual support)
● Provides continuity of care
● Provides family-centered care involving the
patient and family in making decisions
● Available 24/7
● Provides follow-up bereavement care for up to
1 year after the patient’s death

Care of the Dying Trans (2022). p. 7


9. Only symptom relief is provided A Palliative care is any form of medical or treatment
that concentrates on reducing the severity of
disease symptoms rather than striving to halt,
delay, or reverse the progression of the disease.

Care of the Dying Trans (2022). p. 7


10. Care can be rendered at home, in the hospital, or in a facility C Hospice care and palliative care can be both
rendered at home, in hospital, or in a facility.
Care of the Dying Trans (2022). p. 7
11. Which of the following is true of cognitive impairment in B The preferred test for global cognition in PD is the
Parkinson’s disease (PD)? Mini-Mental State Examination, MINI COG, MOCA,
A. The presence of dementia is an exclusion criteria for PD and CDT
B. The preferred test for global cognition in PD is the Mini-
Mental State Examination
C. Cognitive impairment is the last symptom to appear in
Parkinson’s disease dementia
D. Cognitive impairment is associated with the presence of
orthostatic hypotension in PD Geriatrics Trans (2022). p. 2.
12. In the elderly, behavioral interventions are effective and C In cognitive behavioral therapy, stimulus control,
recommended as an initial treatment of insomnia. Which of the sleep hygiene and education, relaxation and
following is not part of Cognitive Behavioral Therapy? imagery, pastoral counselling and support groups
A. Sleep hygiene and education are included.
B. Relaxation and mindfulness
C. Sleep restriction
D. Stimulus control Care of the Dying Trans (2022). p. 4
13. Which of the following is a correct recommendation for the B Pharmacologic Management for Insomnia
corresponding geriatric syndromes? ● Melatonin 1 tab, 2-3 hours before bedtime
A. Terasozin as second line therapy for LUTS-BPH ● Avoid benzodiazepines and hypnotics
B. Zolpidem 10 mg daily for insomnia ● Zolpidem 10 mg may be given if no response
C. Routine use of Risperidone for depression to nonpharmacologic management or
D. Psyllium for constipated elderly to minimize bloating melatonin

Other choices:
● Choice A: Alpha receptor blockers (Tamsulosin,
Terazosin) are effective as first line therapy for
LUTS-BPH
● Choice C: Risperidone is an atypical
antipsychotic used to treat schizophrenia and
bipolar disorders. Pharmacologic management
for depression in the elderly includes Sertraline,
Escitalopram, and Mirtazapine.
● Choice D: Psyllium is included in the
pharmacologic management of constipation but
1bloating and abdominal distention are its
adverse effects.
● Geriatrics Trans (2022). p. 3-5.
PREVMED3 – Family Health – Quiz 1 GERIATRICS, HOME CARE & DIFFICULT ENCOUNTERS, CARE OF THE DYING (March 24, 2021) Page 4 of 5

14. When disclosing the diagnosis to the patient, the physician should B SPIKES Step 1: Setting up the Interview
be: ● Most patients want to have others with them, but
A. Straightforward even if the patient is not ready this should be the patient’s choice (rule out
B. Avoid using medical jargons when explaining the condition Choice C)
C. Tell the family first before the patient
D. Dump all the information on the patient in one sitting. SPIKES Step 4: Giving Knowledge and Information
to the Patient

Warning the patient that bad news is coming may


lessen shock following disclosure and may also
facilitate information processing. (rule out Choice A)

Giving medical facts, the one-way part of the


physician-patient dialogue, may be provided with a
few simple guidelines:
● Start at the level of comprehension and
vocabulary of the patient
● Try to use nontechnical words
● Avoid excessive bluntness (rule out Choice A)
● Give information in small chunks (Rule out
choice D)
● When the prognosis is poor, avoid using
phrases such as “there is nothing more we can
do for you”

Care of the Dying Trans (2022). p. 2.


15. What is the appropriate dose of Vitamin D in the elderly to reduce D Vit D use of around 800-1000 IU reduces fractures
fracture? and overall mortality in older patients.
A. 1500 IU
B. 2000 IU
C. 500 IU
D. 1000 IU Geriatrics Trans (2022). p. 2.
16. A 40 year old patient has brain tumor and has headaches due to C A VAS score of 6/10 indicates moderate pain (VAS
increased ICP, VAS= 6/10. What medications would you prescribe? score of 3-6).
A. Tramadol
B. Tramadol + benzodiazepine According to the WHO Analgesic Ladder, weak
C. Tramadol + corticosteroids opioids such as codeine, dihydrocodeine, and
D. Paracetamol + corticosteroids tramadol can be used for moderate pain.

The patient also has headaches due to increased


ICP, this could be addressed with adjunct use of
corticosteroids.

Other choices:
● Paracetamol is a non-opioid used for mild pain
(VAS score of 1-2)
● Benzodiazepine is an adjunct analgesic
primarily used as an anxiolytic

Care of the Dying Trans (2022). p. 4-5.


For 17-20. SPIKES Matching Type
A. Checking on the patient’s perception
B. Summarizing
C. Getting the patient’s invitation
D. Validating emotions
17. “I understand why you feel this way. Many of my other breast D ● When patients get bad news, their emotional
cancer patients have the same fears with regards to chemotherapy” reaction is often an expression of shock,
isolation, and grief.
● Offer support and solidarity to the patient by
making an empathic response.

Care of the Dying Trans (2022). p. 2


PREVMED3 – Family Health – Quiz 1 GERIATRICS, HOME CARE & DIFFICULT ENCOUNTERS, CARE OF THE DYING (March 24, 2021) Page 5 of 5

18. “If this turns out to be something serious would you like to know C ● Some patients desire full information about their
all the details of what’s going on?” condition, but some do not.
● Discussing information at a time of ordering
tests can cue the physician to plan the next
discussion with the patient.

Care of the Dying Trans (2022). p. 2


19. “What do you understand so far about your medical condition from B ● Ask patients if they are ready for discussion on
our discussions today?” treatment options
● Share responsibility for decision-making with
the patient
● Check for the patient’s understanding of the
discussion

Care of the Dying Trans (2022). p. 3


20. “What things came to your mind when you started losing weight A ● The dictum here is: “before you tell, ask.”
and appetite?” ● Before discussing medical information, the
clinician uses open-ended questions to create a
reasonably accurate picture of how the patient
perceives the medical situation.
● Based on this, you can correct any
misinformation and tailor the bad news to what
the patient understands

Care of the Dying Trans (2022). p. 2


PREVENTIVE MEDICINE - FAMILY HEALTH

SHIFTING EXAM
14 APRIL 2021

***USE AT YOUR OWN RISK!!! Answer key was NOT posted***


QUESTION ANSWER RATIONALE
1. Evidence-based geriatric assessment instruments A/D Evidence-based geriatric assessment instruments available to the
available to the clinician: clinician include screening, evaluation and measurement
A. include screening, evaluation and instruments in multiple domains
measurement instruments in multiple
domains
B. Are largely screening instruments to
detect a condition
C. Have applicability in the outpatient but not Quiz+. (n.d.). Retrieved April 17, 2021, from
the inpatient setting https://quizplus.com/quiz/77495-quiz-4-comprehensive-geriatric-
D. Include screening, evaluation and assessment/question/6248777-evidence-based-geriatric-assessment-
measurement instruments in multiple instruments-available-to
domains

2. Doctor said: “You said three months ago, you B Assessing the Patient’s PERCEPTION
started losing appetite and weight, and felt the ● The dictum here is: “before you tell, ask.”
mass in your abdomen. What came to your mind ● Before discussing medical information, the clinician uses open-ended
when you started experiencing the sign and questions to create a reasonably accurate picture of how the patient
symptoms? perceives the medical situation.
A. Summary and strategy ® e.g. “What have you been told about your medical situation
B. Perception so far?” or “What is your understanding of the reasons we
C. Setting did the MRI?”
● Based on this, you can correct any misinformation and tailor the bad
D. Giving information
news to what the patient understands
● It can also accomplish the important task in determining if the patient
is engaged in any variation of illness denial like:
® Wishful thinking
® Omission of the essential but unfavorable medical details of the
illness
® Unrealistic expectations of treatment

[PRV-FH] 1.03 - Care of the Dying (Batch 2022), p. 2


3. Which of the following best predicts delirium over D Delirium
other causes of confusion? ● Complex neuropsychiatric syndrome characterized by acute onset of
A. Inattention disturbance of consciousness and fluctuating changes in
B. Hallucination cognition, attention, and perceptual disturbance
C. Previous history of delirium ● Most common reason for acute cognitive dysfunction in hospitalized
D. History of dementia older people.
● Prevalence of delirium at admission ranges from 10 to 31%, incidence
of new delirium per admission ranges from 3 to 29% and occurrence
rate per admission varies between 11 and 42%
● May be prevented in up to a third of older patients → early
recognition is vital

Page 1 of 15
PREVENTIVE MEDICINE 3 - FAMILY HEALTH Shifting Exam (14 APRIL 2021) Page 2 of 15

Factors Significantly Associated with Delirium


● Dementia
● Older Age
● Comorbid illness
● Severity of medical illness
● Infection
● ‘High-risk’ medication use
● Diminished activities of daily living
● Immobility
● Sensory impairment
● Urinary catheterization
● Urea and electrolyte imbalance
● Malnutrition

“In pooled analyses, dementia (OR 6.62; 95% CI (confidence interval)


4.30, 10.19), illness severity (APACHE II) (MD (mean difference) 3.91;
95% CI 2.22, 5.59), visual impairment (OR 1.89; 95% CI 1.03, 3.47),
urinary catheterization (OR 3.16; 95% CI 1.26, 7.92), low albumin level
(MD −3.14; 95% CI −5.99, −0.29) and length of hospital stay (OR 4.85;
95% CI 2.20, 7.50) were statistically significantly associated with
delirium.”

Ahmed, S., Leurent, B., & Sampson, E. L. (2014). Risk factors for incident
delirium among older people in acute hospital medical units: a systematic
review and meta-analysis. Age and ageing, 43(3), 326-333.
4. Doctor said “I have the results of the tests, how C Obtaining the Patient’s Invitation
much information do you desire and explained ● Most common reason for acute cognitive dysfunction in hospitalized
to you?” Identify step in the SPIKES model older people.
A. Patient’s perception ● Some patients desire full information about their condition, but some
B. Patient’s emotions do not
C. Invitation ● Discussing information at a time of ordering tests can cue the
D. Knowledge physician to plan the next discussion with the patient.
● “How would you like me to give the information about the test results?
● “Would you like me to give you all the information or sketch out
the results and spend more time discussing the treatment plan?”

[PRV-FH] 1.03 - Care of the Dying (Batch 2022), p. 2


5. Issues of caregiver neglect or abuse should be D Unhygienic/Malodorous patient
explored in these types of patients: ● May be a sign of caregiver neglect
A. Demanding ● Do not show dislike
B. Frequent flier ● Carry on with the interview and treat the problem
C. Somatizing ● Do your best in conducting interview
D. Unhygienic ● Educate the patient of caregiver on the importance of hygiene
● Explore possible reasons for it

PRV-FH] 02 Home Care & Difficult Encounters (Batch 2022), p. 5


6. In this patient type, use open ended questions C Silent/Detached Patient
and explain the importance of sharing ● Speak with the patient in private
information in order for you to help him or her: ● Establish rapport in order to gain confidence
A. Seductive patient ● Explain the importance of sharing information in order to help the
B. Demanding patient patient
C. Silent patient ● Use an open-ended question
D. Somatizing patient ● Encourage a productive clinic visit
● Identify cause of silence

[PRV-FH] 02 Home Care & Difficult Encounters (Batch 2022), p. 4


PREVENTIVE MEDICINE 3 - FAMILY HEALTH Shifting Exam (14 APRIL 2021) Page 3 of 15

7. Which of the following is TRUE of osteoporosis B Studies have shown that vitamin and calcium supplementation does not
interventions in elderly? have a significant effect during early menopause. There is a clearer
A. Calcium has substantial effect on fragility benefit in vitamin D and calcium supplementation in older
fractures in elderly postmenopausal women (mean age of 63 years). There is also insufficient
B. Vitamin D alone has little effect on bone data to support vitamin D supplementation alone, without calcium, to
mass in the early menopausal years prevent osteoporotic fracture in postmenopausal women.
C. Women on hormone replacement therapy
needs 500 mg of calcium
Malabanan AO, Holick MF. Vitamin D and bone health in postmenopausal
D. Aerobic exercises may be recommended
women. J Womens Health (Larchmt). 2003 Mar;12(2):151-6. doi:
for elderly to prevent osteoporosis
10.1089/154099903321576547. PMID: 12737713.
8. The living will is only used at the end of life if a A The Living WIll
person is terminally ill or permanently ● Written or video statement about the kinds of medical care you do or
unconscious to describe under what conditions do not want to receive if you are no longer able to express consent
as an attempt to prolong life should be started ● Includes: resuscitation, desired QOL, end-of-life treatments
or stopped according to patient’s wishes
A. True
B. False

9. Which inquiry is not necessary when taking C Asking whether the patient has a (sexual) relationship with a foreigner is
sexual history? irrelevant in taking a sexual history.
A. How do you identify your sexual
orientation?
B. Do you perform vaginal, anal, and or oral
sex?
C. Do you have any relationship with a
foreigner?
D. Are there sexual concerns you would like
to discuss?

10. Who of the following patients is considered frail? D Frailty


A. 67 years old with unintentional weight loss ● Includes 3 or more of the following:
of 2 lbs in the previous year, limited ® Unintentional weight loss of >10 lbs in the previous year
physical activity, low resistance ® Self-reported exhaustion
B. 75 years old with low physical activity and ® Weakness (as measured by grip strength in the lowest 20% by
weakness gender and BMI)
C. 72 years old with Tinetti balance scale of ® Slow walking speed (lowest 20% by gender and height)
25, weight loss of 10 kg, anorexia ® Low physical activity (as measured by kcal; lowest 20%)
D. 70 years old with low grip strength, slow
walking speed, self-reported exhaustion

11. 76 year old man with a past medical history of B ● Men with PSA level above 7 ng/mL should be referred to a urologist
hypertension, CAD, and obesity presents for a without further testing
routine appointment. He reports that he has ● Men with a PSA level between 4-7 ng/mL, we repeat the PSA testing
been feeling “OK” since his last hospitalization 3 in 6-8 weeks
months previous for pneumonia. When he was
at a community health screening for prostate Screening for Prostate Cancer. UpToDate
cancer awareness 2 weeks earlier, he had his
PSA tested. He reports that his PSA was
4.8ng/mL, and he was told that anything above
4 ng/mL is abnormal. Since that time he has
read about the PSA test and wishes he never
had it checked, as he now worries that he will
have to have a biopsy and operation. Which one
of the following next steps in care is the most
appropriate for the patient?
A. Do transrectal ultrasound
B. Repeat PSA now
C. Repeat the PSA in 6 months and if
doubled, refer him to a urologist for an
aggressive work up.
D. No need for further work up
PREVENTIVE MEDICINE 3 - FAMILY HEALTH Shifting Exam (14 APRIL 2021) Page 4 of 15

12. The following tests were done in a 78 year old B. ● KATZ ADL
female: KATZ ADL score of 4, Lawton IADL ® 6- full function
score of 4/8, and able to get up from a sitting ® 4- moderate impairment
position using the arms of the chair. What is the ® <2- severe functional impairment
functional ability of the patient? ● Lawton IADL
A. Partially dependent ® 0 - low function/dependent
B. Can walk a mile albeit slowly and with a ® 8 - high function/independent
companion
C. Needs a walking aid
D. Independent

13. The caregiver of a family member with dementia B. ● Modified Caregiver Strain Index
was tested Modified Caregiver Strain Index. His ® MCSI Scoring
score is 22. This means ▪ <23 = Normal
A. Severe strain
▪ 24-28 = Predisposition to strain
B. Normal
C. Moderate strain ▪ >29 = Severe strain
D. Predisposition to strain

14. Prudent behavior towards LGBT patients prior A. ● Understanding LGBTQ+ Issues
to any clinic interview includes: ® When addressing LGBTQ+ individuals, healthcare providers
A. Calling patient by his preferred name should always ask clients how they identify or wish to be
B. Asking if he/she has a boyfriend/girlfriend addressed.
C. Ask about civil status ® Choices B or C
D. Calling patient by his or her legal first ▪ Ask instead if they have a PARTNER or are they in a
name relationship.
▪ Remember: Never assume
15. When taking sexual history, which of the B. Sexual History
following is LEAST prudent to ask? ● Encourage patients to choose a surrogate decision-make
A. How many partners did you have in the ● Should include sexual behavior, orientation, and gender identity
past 6 months? ● Instead of asking “Are you married?”, ask “Do you have a partner?” or
B. Which type of sexual contact brings you “Are you in a relationship?”
● Next you could ask “have you had female partners, male partners,
more pleasure?
or both?”
C. Are you sexually active?
● Alternatively, you could start by asking “are you sexually active?”
D. Do you have sex with men, women, or
® Then ask, “when you have had sex, what are the genders of your
both? partners?”
® If patients are confused by this question, you could ask “Do you
have sex with men, women, or both?”
● At the end of the sexual history:
® “Do you have any concerns or questions about your sexuality,
sexual orientation, or sexual health?”
● You should also ask whether patients identify as transgender or have
a gender-related concerns:
® ”Since many people are affected by gender issues, I ask patients if
they have any concerns about this. If this topic is not relevant to
you, tell me and I will move on.”

[PRV-FH] 04 Optimal Primary Care for LGBTQ+ Patients (Batch 2022)


16. Which of the following is a correct D
recommendation for constipation in older
persons?
A. Stimulating agents such as Senna and
Bisacodyl are second line therapy for
constipation in older persons
B. Patients with infrequent bowel movements
respond least to Polyethylene glycol
[PRV-FH] 1.01 - Geriatrics (Batch 2022), p. 5
C. Patients with straining and incomplete
evacuation responds best to bulk forming
“Osmotic laxatives -
agents
Low-dose polyethylene glycol (PEG) (17 g/day) has been demonstrated
D. The initial drug of choice for constipation
to be efficacious and well tolerated in older patients”
should be an osmotic agent
Rao, S. (2020). Constipation in the older adult. In T.W. Post (Ed.)
UpToDate. Available from:
https://www.uptodate.com/contents/constipation-in-the-older-adult
PREVENTIVE MEDICINE 3 - FAMILY HEALTH Shifting Exam (14 APRIL 2021) Page 5 of 15

17. Which of the following is a correct D Vaccines


recommendation for immunization in older ● Pneumococcal vaccine (PCV13 followed by PPSV23 after 6-12 mos)
persons? ® If PPSV23 was administered first, give PCV13 after 1 yr
A. Pneumococcal conjugate vaccine is given ● Influenza vaccine annually
first followed after 3 months by the ● Herpes Zoster vaccine single dose
pneumococcal polysaccharide vaccine ● Tdap
B. Varicella vaccine should be given for 2 ® Tdap, TD, Tt - 3 doses (1 Tdap first then 2 Td) at 0, 1, 6-12 mos
doses 1 month apart ® Booster every 10 years
C. The optimal time to receive the influenza
vaccine is every January of each year
D. Tdap is given initially followed by 2 doses
of Td for those without previous doses [PRV-FH] 1.01 - Geriatrics (Batch 2022), p. 7

18. Predictor of difficult clinical encounters C


A. Physicians working <40 hours/week
B. Severity of symptoms <6/10 point-scale
C. >5 physical symptoms
D. Physician’s level of stress is low to
average

[PRV-FH] 1.02 - Home Care & Difficult Clinical Encounters (Batch 2022),
p. 3-4
19. The following are the treatment options of D
dyspnea in cancer patients depending on
etiology/cause, EXCEPT:
A. Bacterial infection - antibiotics,
antipyretics, supportive care
B. Pleural effusion - thoracentesis
C. Obstruction - corticosteroids
D. Volume overload - anticholinergic agents
and nebulization

Volume overload must be treated with diuretics


[PRV-FH] 1.03 - Care of the Dying (Batch 2022), p. 6
20. For severe pain control, only strong opioids are B
recommended in the WHO step ladder
A. True
B. False

Strong opioids may or may not be accompanied by non-opioids and


adjuvant therapies.
[PRV-FH] 1.03 - Care of the Dying (Batch 2022), p. 4
PREVENTIVE MEDICINE 3 - FAMILY HEALTH Shifting Exam (14 APRIL 2021) Page 6 of 15

21. In a patient with life-limiting terminal phase of “The use of artificial nutrition, defined as a medical treatment that allows
illness and is no longer able to eat or drink, B a non-oral mechanical feeding, for cancer patients with limited life
artificial nutrition should be resorted to bring expectancy is deemed nonbeneficial”
back the person to a healthy state
A. True Baumstarck, K., Boyer, L., Pauly, V., Orleans, V., Marin, A., Fond, G.,
B. False Morin, L., Auquier, P., & Salas, S. (2020). Use of artificial nutrition near
the end of life: Results from a French national population-based study of
hospitalized cancer patients. Cancer medicine, 9(2), 530–540.
https://doi.org/10.1002/cam4.2731
22. Georgia believes in her mind that she is a Gender identity
woman. This is a reflection of her A ● The person’s internal sense of their own gender
A. Gender identity
B. Gender expression
C. Biological sex [PRV-FH] 1.04 - Optimal Primary Care for LGBTQ+ Patients (Batch
D. Gender orientation 2022),
23. Joanna, 19 year old, wears a crew cut hairstyle, Gender expression
military uniform and cap; she acts like a real C ● The spectrum of masculine and feminine characteristics in how one
man. This is a reflection of dresses, acts or presents his/herself
A. Biological sex
B. Gender identity
C. Gender expression [PRV-FH] 1.04 - Optimal Primary Care for LGBTQ+ Patients (Batch
D. Gender orientation 2022), p. 2
24. Which of the following is part of normal aging? A
A. Poor decision making
B. Misplacing things most of the time
C. Difficulty of conversation

https://www.nia.nih.gov/health/memory-forgetfulness-and-aging-whats-
normal-and-whats-not
25. One should convene the family when a 9-year Patients that present with somatization may require psychiatric care and
old boy presents with D since the patient is only 9 years old, family support should be stressed
A. Failing grades in school
B. Monthly headaches Occasional enuresis, monthly headaches and failing grades may all be
C. Occasional enuresis attributed to an underlying source and may be resolved with appropriate
D. Somatization medication or intervention
26. A 68 year old female is brought to your office by B? Signs of Depression
her son with a complaint of headaches. She ● Changing body language
diverts her eye to look at her son when ® Make less eye contact
answering your questions. Which one of the ® Slumped posture
following is the most likely reason for this patient ® Less hand gestures
not making eye contact? ● Not engaging in conversation
A. Intimidated by you as a doctor ® Speak or seem to think more slowly
B. Depressed ® Take them longer to respond
C. Could be a victim of abuse ® Not following what you’re saying or processing it as well as they
D. Could be hiding the truth from you used to
● Neglecting responsibilities
® Lack of personal grooming
® Performance status decline
● Avoiding favorite activities
® Anhedonia is the technical term for this problem
● Sleeping or eating too little or too much
● Declining physical health

https://www.rush.edu/news/recognizing-depression
PREVENTIVE MEDICINE 3 - FAMILY HEALTH Shifting Exam (14 APRIL 2021) Page 7 of 15

27. What is reflective of functional decline in older B? **PROCESS OF ELIMINATION*


adults? A - The IADL differs from ADL as people often begin asking for outside
A. Instrumental activities of daily living are assistance when IADL tasks become difficult to manage independently.
preserved longer than activities of daily C - since there’s the term ALWAYS, the statement itself seem impossible
living to be true
B. Some individuals die of “old age” but have D - Senescence is essentially synonymous with aging. Functional decline
maintained an active and healthy lifestyle
refers more to a loss of independence in self-care capabilities and is
C. it is always possible to prevent functional
associated with deterioration in mobility and in the performance of
deterioration
activities of daily living (ADLs) such as dressing, toileting, and bathing.
D. Functional decline is synonymous with
advance age

28. No one can make decisions on behalf of patients A


who have capacity and that unless a patient
refused information, withholding information at
the request of a relative is not ethically
justifiable.
A. True
B. False

29. Grace thinks she is neither a man nor woman. D Genderqueer


Grace does not really case if a male or female ● A term used by persons who may not entirely identify as either male or
person will be attracted to her. She is described female
as:
A. Heterosexual
B. Bisexual
C. Homosexual [PRV-FH] 1.04 - Optimal Primary Care for LGBTQ+ Patients (Batch
D. Queer
2022), p. 2
30. Which of the following is NOT part of the C Core Components of Comprehensive Geriatric Assessment
Comprehensive Geriatric Assessment? ● Fall risk
A. Lawton’s instrumental activities of daily ● Cognition
living ● Mood
B. Snellen’s Chart ● Polypharmacy
C. Modified Caregiver Strain Index ● Social support
D. Functional Reach Test ● Financial concerns
● Goals of care
● Advance care preferences

Additional components may also include evaluation of the following:


● Nutrition/weight change
● Urinary continence
● Sexual function
● Vision/hearing
● Dentition
● Living situation
● Spirituality

Comprehensive geriatric assessment, UptoDate


https://www.uptodate.com/contents/comprehensive-geriatric-
assessment#H9
31. Which of the following dietary pointers is correct D
for the elderly?
A. Vitamin supplements should always be
given to other persons
B. Simple carbohydrates should be
increased
C. Protein intake of 48g/day for a 60 kg
elderly female
D. Need for water amounting to 1.5 liters/day
for a 60 kg elderly male [PRV-FH] 1.01 - Geriatrics (Batch 2022), p. 5
PREVENTIVE MEDICINE 3 - FAMILY HEALTH Shifting Exam (14 APRIL 2021) Page 8 of 15

32. Which of the following is true for insomnia in A


elderly?
A. Use of melatonin in insomniacs without
melatonin deficiency is controversial
B. Flurazepam may be used for elderly
patients
C. Zolpidem does not have any side effect
among older persons
● B: Benzodiazepines (Diazepam)
D. Sedating antidepressants given anytime
® Anxiolytic; avoided in insomnia
can be used for patients with depression ● C: Zolpidem (Z-drugs)
and insomnia ® Help people fall asleep faster (~13-22 mins) and perhaps ~5%
more sleeping time while in bed
® May increase the risk of mild infections (1 in 43 patients)
® Inconsistent cognitive effects (reduced verbal memory or attention)
● D: Sedating Tricyclic Antidepressants → For patients with insomnia
and anxiety; given at bedtime

[PRV-FH] 1.01 - Geriatrics (Batch 2022), p. 4


33. Angelo is a male yet attracted and had C Sexual Orientation
relationships with both genders. This is a ● Based on how a person identifies attraction on a physical and
reflection of his emotional basis
A. Gender expression
B. Biologic sex [PRV-FH] 1.04 - Optimal Primary Care for LGBTQ+ Patients (Batch
C. Gender orientation 2022), p. 2
D. Gender identity

34. Which is NOT a common health issue among D


the LGBT population?
A. Homelessness, violence
B. Suicide attempts and depression
C. Family planning
D. GUT malignancy

[PRV-FH] 1.04 - Optimal Primary Care for LGBTQ+ Patients (Batch


2022), p. 5
35. Palliative care experts recommend that health A Palliative Care Treatment
care professionals consider the patient’s ● Affirms life and regards death as a normal process
emotional, social, and economic concerns as ● Neither hastens nor postpones death
experience of pain and are often a direct cause ● Provides relief from pain and other distressing symptoms
of suffering. ● Integrates the psychological and spiritual aspects of patient care
A. True ● Offers a support system to help patients live as actively as possible
B. False until death
● Offers a support system to help the family cope during the patient’s
illness and in their bereavement

[PRV-FH] 1.02 - Care of the Dying (Batch 2022), p. 7


36. It is a reaction to loss D ● Bereavement
A. Grief ® The period after a loss which grief is experienced and mourning
B. None of the above occurs
C. Mourning
D. Bereavement [PRV-FH] 1.02 - Care of the Dying (Batch 2022), p. 8
PREVENTIVE MEDICINE 3 - FAMILY HEALTH Shifting Exam (14 APRIL 2021) Page 9 of 15

37. Which of the following is a correct benefit of a 65 A


year old retired office clerk based on RA 9994?
A. 20% discount on pulse oximeter
B. P200,000 cash gift on 100th birthday
C. 15% discount on rabies vaccine
D. Free cataract extraction

[PRV-FH] 1.01 - Geriatrics (Batch 2022), p. 6


38. The goal of hospice care is to provide physical B ● Hospice Care
comfort in order to prolong the dying process ® Consists of palliative and supportive services for dying persons
A. True and their families usually those who have less than 6 months to
B. False live

[PRV-FH] 1.02 - Care of the Dying (Batch 2022), p. 7


39. The following describes the scope of hospice D ● Hospice care
care ® Provides follow up bereavement care up to 1 year after the death
A. Provides any artificial means of life of the patient
support at home
B. Benefits patients who have early stage [PRV-FH] 1.02 - Care of the Dying (Batch 2022), p. 7
cancer and on curative treatment
C. Focuses on the goal of helping the patient
become independent and functional
D. Bereavement care is provided until a year
after the death of the patient

40. Dina, a 32 year old, believes she is a woman in C Transgender


mind and heart but is still for genital ● Natal sex discordant with internal sense of gender
reassignment to complete her womanhood.
Dina is a Even if Dina has not yet transitioned through genital reassignment, she
A. Transman already identifies herself as a woman.
B. Asexual
C. Transwoman [PRV-FH] 1.04 - Optimal Primary Care for LGBTQ+ Patients (Batch
D. Homosexual
2022), p. 2
41. The physician makes his patient ventilate. What A CEA for Counselling Skills - A: Catharsis
phase of counseling is this? Can be used for the following questions:
A. Catharsis ● What came to your mind when you started feeling your symptoms?
B. Preparatory (Ano ang naisip mo noong nakaramdam ka ng sakit?)
C. Action ● What feelings came out when these thoughts came to your mind?
D. Insight (Ano ang naging damdamin mo noong naisip mo ang mga ito?)
● What consequence of your illness makes you feel this way the most?
(Ano ang pinaka nakakatakot na maaaring mangyari dahil sa sakit
mo?)

[PRV-FH] 03 Care of the Dying (Batch 2022), p. 2


42. Which of the following statements expresses C
empathy rather than sympathy?
A. “He is in a better place now”
B. “I feel sorry for you”
C. “I feel your loss”
D. “I know how you feel”

Choice C: “I feel your loss.”


Shows a more personal understanding wherein the physician puts
himself/herself into the patient’s shoes.
PREVENTIVE MEDICINE 3 - FAMILY HEALTH Shifting Exam (14 APRIL 2021) Page 10 of 15

Choice A: “He is in a better place now.”


The physician tries to provide comfort assurance to the patient or family.

Choice B: “I feel sorry for you.”


The physician has a feeling of pity or sympathy for the patient rather than
empathy.

Choice D: “I know how you feel.”


Acknowledges what the patient feels. The physician understands the
patient’s experience. This shows sympathy.

[PRV-FH] 02 Home Care and Difficult Clinical Encounters (Batch 2022), p. 5


43. Adjuvant drugs like muscle relaxants can A Adjunct Analgesics
produce analgesia for muscle spasticity hence ● Drugs with primary indication other than pain that have analgesic
can be added to the standard pain regimen. properties in some painful conditions:
A. True ® Corticosteroids (pain d/t inflammation and swelling)
B. False ® Tricyclic antidepressants (amitriptyline, nortriptyline, doxepine)
® Anticonvulsants (valproate, carbamazepine, gabapentine; for
radiculopathies)
® Benzodiazepines (Diazepam; anxiolytic)
® Muscle relaxants (tizanidine, lioresal)
® Anesthetics (for indwelling epidural catheter)

[PRV-FH] 03 Care of the Dying (Batch 2022), p. 5


44. In a demanding patient who questions your A Demanding Patient Case Stimulation
proposed diagnostic and therapeutic plan, the ● Explain the situation to the patient
most appropriate thing to do are the following ● Address the patient’s doubts regarding your ability
except: ● Explore the patient’s concerns and fears
A. Comply with patient so as not to ● Empathize
antagonize him/her ● Last resort is to suggest a second opinion
B. Encourage patient to get a second opinion
C. Explain the situation to the patient [PRV-FH] 1.02 Home Care & Difficult Encounters (Batch 2022) p. 4
D. Explore the patient’s doubts

45. In early stages of cancer, starting palliative care B Palliative Care Treatment
means stopping all curative treatment ● Affirms life and regards death as a normal process
A. True ● Neither hastens nor postpones death
B. False ● Provides relief from pain and other distressing symptoms
● Integrates the psychological and spiritual aspects of patient care
● Offers a support system to help patients live as actively as possible
until death
● Offers a support system to help the family cope during the patient’s
illness and in their bereavement

According to getpalliativecare.org
● Palliative care is based on the needs of the patient, not on the
patient’s diagnosis
● It is appropriate for any stage and age
● It can be provided along the curative treatment

[PRV-FH] 1.02 - Care of the Dying (Batch 2022), p. 7


https://getpalliativecare.org/whatis/
46. Gerry has cancer with metastasis to the spine B Adjuvant Analgesics
and experiences shooting pain to the left leg and ● Drugs with a primary indication other than pain that have analgesic
foot. Aside from analgesics, what adjuvant properties in some painful conditions
would you consider for the pain regimen? ® Corticosteroids: pain due to inflammation and swelling
A. Tramadol ® Tricyclic Antidepressants: Amitriptyline, Nortriptyline, Doxepin
B. Gabapentin ® Anticonvulsants: Valproate, Carbamazipine, Gabapentine
C. Escitalopram ▪ Used for radiculopathies
D. Zolpidem ® Muscle Relaxants: Tizanidine, Lioresal
® Anesthetics: For indwelling epidural catheter
PREVENTIVE MEDICINE 3 - FAMILY HEALTH Shifting Exam (14 APRIL 2021) Page 11 of 15

● Shooting pain to the left leg and foot = Radiculopathy


● A: Tramadol is an opioid
® It can be used to target severe nociceptive pain from bone or soft
tissue
● C: Escitalopram is an SSRI
● D: Zolpidem is a sedative-hypnotic used to treat insomnia

[PRV-FH] 03 Care of the Dying (Batch 2022), p. 5


47. Patient: I am sick of living with these headaches. C
No one has been able to help me and none of
the medications are working. For the doctor’s
response, which is the appropriate empathetic
statement to make?
A. “I know how awful this must feel”
B. “I feel bad for you having to go through
this”
C. “I feel how frustrated you must be”
D. “I can see that you are frustrated by the
lack of improvement of symptoms”

● Choice C: “ I feel how frustrated you must be”


® Shows empathy
® Shows an understanding of what the patient is feeling
® Can put him/herself in the patient’s shoes
● Choice A: “ I know how awful this must be”
® Shows sympathy
® Acknowledges the patient’s emotional hardships
® Understands the experiences of others
● Choice B: “ I feel bad for you having to go through this”
® Shows sympathy
® Understands the experiences of others
● Choice D: “ I can see that you are frustrated by the lack of
improvement of symptoms”
® Shows sympathy
® Acknowledges the patient’s emotional hardships

[PRV-FH] 02 Home Care and Difficult Clinical Encounters (Batch 2022), p. 5


48. Who of the following patients is at risk of falling? D Diagnosis for Falls
A. 76 years old with Get up and Go test of 10 ● Get up and Go test
seconds ® From patient in sitting position → stand without using arms → walk
B. 73 years old with Chair Stand score of 20
seconds 3 meters → turn around → walk back, sit down
C. 81 years old male with grip strength of 25 ® Doctor will time the process and observe for gait abnormalities
kg ® Older adult who takes >12 seconds to complete the test is at high
D. 68 years old male with functional reach risk of falling
test of 13 inches ● Chair Stand Test
® From patient in sitting position → place hands on opposite
shoulders, crossed at the wrist → on “Go”, rise to a standing
position and sit back down again → repeat for 30 seconds
® Physician should count the number of times the patient is able to
come to a full standing position in 30 seconds
PREVENTIVE MEDICINE 3 - FAMILY HEALTH Shifting Exam (14 APRIL 2021) Page 12 of 15

® A below average score indicates a risk for falls


● Functional Reach Test
® Yard stick on wall parallel to floor, at height of acromion of
subject’s dominant arm. Subject with feet distanced apart, make a
fist, forward flex to 90 degrees.
® Subject will reach forward without taking a step or touching the
wall
® Measure distance between start and end using head of
metacarpal of 3rd finger as reference

® Cut off: 13.16 inches for MEN, 10.47 for WOMEN


▪ Below means that patient is at RISK OF FALLING
● Grip Strength
® Measured using Jamar Hand Dynamometer
® Used to determine the presence of sarcopenia in elderly
® Normal Values
▪ Males: > 30 kg
▪ Females: > 20 kg

● Choice D: Patient’s functional reach test score is only 13 which


is below the cut off of males which is 13.16 inches or the cut off
for his age group which is 14.98 inches.
● Choice A: The patient’s Get up and go test result is 10 seconds which
is within the normal limit of 20 seconds.
● Choice B: The patient is 73 years old giving the cut off score of either
<12 for males or <10 for females. The patient’s score is 20 which is
still within the normal limit.
● Choice C: The patient’s result is below the normal value but this
method is not used to diagnose falls but rather to determine the
presence of sarcopenia in the elderly

[PRV-FH] 1.01 - Geriatrics (Batch 2022) p. 1-2, 6


CDC (2017) Assessment 30-second chair stand
49. How families adapt to stress and changes A
imposed by the experience of living with dying
depends on their previous experiences with
death in the family
A. True
B. False

[PRV-FH] 03 Care of the Dying (Batch 2022), p. 8


PREVENTIVE MEDICINE 3 - FAMILY HEALTH Shifting Exam (14 APRIL 2021) Page 13 of 15

50. Which of the following is a differentiating domain B


between MMSE and MOCA?
A. Delayed recall
B. Semantic memory
C. Attention
D. Visuospatial function

[PRV-FH] 1.01 - Geriatrics (Batch 2022) p. 2


51. If patients take narcotics for any length of time B
for chronic pain, they will become dependent on
them. The withdrawal symptoms mean that they
are addicted
A. True
B. False

● Since the patient only has withdrawal symptoms with an unspecified


time frame, he is not yet addicted

[PRV-FH] 03 Care of the Dying (Batch 2022), p. 5


52. A 70 year old male complains of loss of energy, A*
trouble in concentrating, loss of appetite and
insomnia. He has lost about 10% of his weight
since his last visit a month ago and appears
disheveled. Initially the doctor should
A. Refer him for comprehensive geriatric
assessment
B. Refer him to a psychiatrist
C. Suspect he has depression and ask if he
has crying spells and suicidal ideation
D. Request for fecal occult blood and ● Since the patient appears to have several geriatric syndromes, he
colonoscopy to look for malignancy should undergo a comprehensive geriatric assessment to confirm
these syndromes
® Depression - loss of energy, trouble in concentrating, loss of
appetite and insomnia
® Sarcopenia/malnutrition/frailty - loss of 10% of weight, loss of
appetite
® Caregiver abuse - disheveled

[PRV-FH] 1.01 - Geriatrics (Batch 2022) p. 1


53. A patient has mild pain due to cancer due to an B
inflammatory process. Which of the following is
considered as the first line option in Step 1 of
WHO pain management?
A. Codeine
B. NSAIDs
C. Paracetamol
D. Oxycodone

[PRV-FH] 1.03 - Care of the Dying (Batch 2022) p.4


PREVENTIVE MEDICINE 3 - FAMILY HEALTH Shifting Exam (14 APRIL 2021) Page 14 of 15

54. The following are eligible for Home Care, B? ● Home Care
except: ® Provision of comprehensive health care, services are provided in
A. Frail COPD patient on Home O2 treatment places of residence
B. Post-op patient S/P appendectomy ® Goals
C. Homebound elderly patient with ▪ To promote, maintain or restore health of the patient
debilitating osteoarthritis ▪ To minimize the effect of disability
D. Stroke patient in need of intermittent ® Allows a person with special needs to stay in their home
skilled nursing care, or physical, speech, ® Might be good for people who are getting older, are chronically
or occupational therapy ill, recovering from surgery, or disabled
● Although recovery from surgery is also an indication for home care,
among the choices, post-op patient S/P appendectomy seems to be
the one which requires the least or no amount of home care since all
the other choices show debilitating conditions with special needs.

[PRV-FH] 1.02 - Home Care & Difficult Encounters (Batch 2022) p.1
55. Doctor said: “I am sorry to have to tell you this, D ● Doctor is acknowledging the patient’s feelings and affirming that what
but I have some bad news that can be upset they are feeling is justifiable and normal
you. I understand if this will be difficult for you to
process now.:
A. Invitation
B. Knowledge
C. Information
D. Empathic Response

[PRV-FH] 1.03 - Care of the Dying (Batch 2022) p.3


56. When treating an older adult for anxiety, which B? Antidepressants are the first-line treatment in anxiety disorders. The
type of medication is preferred? selective serotonin reuptake inhibitors (SSRIs) and serotonin
A. Long acting benzodiazepines norepinephrine reuptake inhibitors (SNRIs) are more commonly used in
B. Selective Serotonin Reuptake inhibitor the elderly, due to their tolerability and safety profile in this
C. Tricyclic antidepressant population. Although tricyclic/tetracyclic (TCA) antidepressants and
D. Short acting benzodiazepine monoamine oxidase inhibitors (MAOIs) are indicated for select anxiety
disorders, they are not considered first-line treatment of anxiety disorders
in elderly patients due to side effects and safety considerations.

Crocco, E. A., Jaramillo, S., Cruz-Ortiz, C., & Camfield, K. (2017).


Pharmacological Management of Anxiety Disorders in the Elderly.
Current treatment options in psychiatry, 4(1), 33–46.
https://doi.org/10.1007/s40501-017-0102-4
57. A family meeting should be done in the following D ● When to consider convening the family?
situation: → Compliance problem
A. 30 y/o old male for annual employment → Treatment failure
physical → Somatization
B. 4 y/o old boy with acute nasopharyngitis → Anxiety/Depression
C. 12 y/o female with acne → Substance abuse
D. 65 y/o female who suffered a stroke → Parent-child problems
→ Marital/Sexual difficulties
→ Lifestyle problems
→ High utilization of medical services by an individual or family
[PREVMED] Family Medicine FIRST TERM 2018-2019
58. Which of the following is an abnormal result of a B? ● A: Normal result. A calf circumference of <30.5 cm is considered
comprehensive geriatric assessment? abnormal and suggests protein malnutrition..
A. Calf circumference of 33 in a 68 year old ● B: Values that increase risk of diabetes, hypertension, and
female cardiovascular outcomes include >40 inches in men and 35 inches in
B. 42 inches waist circumference in a 70 year non-pregnant women.
old ● C/D: Normal result if adjusted for education. Abnormal MMSE score
for 8th grade education is 21.
C. Mini mental examination (MMSE) score of
22 in a 70 year old male elementary
[PRV-FH] 1.01 - Geriatrics (Batch 2022) p. 2, 5
graduate
D. Mini mental examination (MMSE) score of
22 in a 70 year old male elementary
graduate
PREVENTIVE MEDICINE 3 - FAMILY HEALTH Shifting Exam (14 APRIL 2021) Page 15 of 15

59. When interviewing the older adult with a C ● Communicating with Dementia Patients
suspected dementia, it is most important that: → Face the patient while talking
A. The clinician get in contact with a family → Talk calmly
member to obtain the history → Speak shortly in simple sentences
B. Mental status be evaluated first in order to → Let the patient do as much as he/she can BUT HELP when she
determine if the patient is a reliable is confused or upset
historian
→ Avoid confrontation – “Don’t say you just asked me that”
C. The examiner use short simple questions
→ Be practical
and recognize non-verbal signs of
→ Clarify feelings
discomfort
D. Postpone the mental status evaluation for
[PREVMED] Family Medicine FIRST TERM 2018-2019
the following visit and establish a rapport
first

60. The use of prophylactic anti-constipatory agents A There does not seem to be a tolerance to the constipatory effects of
is the current standard of care and should be opioids. Therefore, once diagnosed, patients may need to stay on
started in conjunction with the start of the opioid laxatives for as long as the patient is on opioids. The use of prophylactic
like Morphine anti-constipatory agents is the current standard of care. These
A. True should be started in conjunction with the start of the opioid.
B. False
Peppin, J.F. (2012). Opioid-induced constipation: causes and treatment. Painscan,
12(3). https://www.practicalpainmanagement.com/opioid-induced-constipation-
causes-treatments
PREVENTIVE MEDICINE 3 (FAMILY HEALTH)

5 MAY 2021

DISCLAIMER!! No answer key was given after the exam using Respondus. Some of the answers cannot be found in the transes.
Answers in red are not sure.
Use at your own risk.
QUESTION ANSWER RATIONALE
1. The management of symptoms in terminal ill patients involves a A Stages of care of the dying includes communication,
diagnostic evaluation for the cause of each symptom when possible management of symptoms, hospice care and end of
and giving the treatment for the identified cause when reasonable. life and bereavement care.
A. True
B. False Management of symptoms includes control of
symptoms such as pain, anorexia, nausea and
vomiting, dyspnea, cough, hiccups, bed sores,
mucositis, urinary incontinence, insomnia and
confusion.

[PRV-FH] 03 Care of the Dying (Batch 2022)


2. Compared to the general population, lesbian, gay, bisexual, and A ● Young members of the LGBTQ community are at
transgender (LGBT) individuals: a higher risk for cigarette uset
A. are less likely to rate their health as excellent or very good ○ Tend to self-report poor health status in
B. are more likely to rate their health as excellent or very good general
C. rate their health the same as that of the general population
D. No comparative data are available [PRV-FH] 04 Optimal Primary Care for LGBTQ+
Patients [B2022] p. 1
3. Which inquiry is not necessary when taking sexual history? A Sexual History
A. Do you have any relationship with a foreigner? ● Encourage patients to choose a surrogate
B. How do you identify your sexual orientation? ● Should include sexual behavior,
C. Do you perform vaginal, anal or oral sex? orientation, and gender identity
D. Are there sexual concerns you would like to discuss? ● Instead of asking “Are you married?”, ask
Do you have a partner?” or “Are you in a
relationship?”
● Next you could ask “have you had female
partners, male partners, or both?”
○ Alternatively, you could start by
asking “are you sexually active?”
○ Then ask, “when you have had
sex, what are the genders of your
partners?”
○ If patients are confused by this
question, you could ask “Do you
have sex with men, women, or
both?”
● At the end of the sexual history:
○ “Do you have any concerns or
questions about your sexuality,
sexual orientation, or sexual
health?”
● You should also ask whether patients
identify as transgender or have a gender-
related concerns:
○ “Since many people are affected
by gender issues, I ask patients if
they have any concerns about
this. If this topic is not relevant to
you, tell me and | will move on.”

[PRV-FH] 04 Optimal Primary Care for LGBTQ+


Patients (Batch 2022), p.3

TRANS TEAMS 1A AND 2A Page 1 of 14


PREV MED 3-FH SHIFTING EXAM (5 MAY 2021) Page 2 of 14

4. In this patient type, use open ended questions and explain the C Silent/ Detached Patient
importance of sharing information in order for you to help him or her: ● They may appreciate it if they get to tell
A. Somatizing their story in private.
B. Seductive ● Establish rapporto gain confidence.
C. Silent ● Explain the importance of sharing
D. Demanding information in order to help the patient.
● Use an open-ended question.
● Encourage a productive clinic visit
● Identify cause of silence
○ Possible fear of authority figure
○ Barriers
■ Language, personality,
cultural
○ Medical reasons
○ Previous negative experience
with a healthcare provider/
service
[PRV-FH] 02 Home Care & Difficult Clinical
Encounters (Batch 2022), p. 4
5. A sexually active patient was asking about the pros and cons of C ● Pre-contemplation: There is no intention
using a condom. The patient is in what stage of the model of health of taking action.
behavior change? ● Contemplation: There are intentions to
A. Action take action and a plan to do so in the near
B. Precontemplation future.
C. Contemplation ● Preparation: There is intention to take
D. Maintenance action and some steps have been taken.
● Action: Behavior has been changed for a
short period of time.
● Maintenance: Behavior has been
changed and continues to be maintained
for the long-term.
● Termination: There is no desire to return
to prior negative behaviors.
https://www.ruralhealthinfo.org/toolkits/health-
promotion/2/theories-and-models/stages-of-
change#:~:text=Contemplation%3A%20There%20a
re%20intentions%20to,maintained%20for%20the%
20long%2Dterm.
6. A 75 year old patient comes to your office complaining of difficulty A or D A - “Insomnia remains one of the most common
sleeping. She complains of difficulty falling asleep, and finds that she sleep disorders encountered in the geriatric clinic
wakes up twice at night. She tries to stay in bed for a longer amount population…”
of time in order to try to fall back asleep. You advise her that: B - False (see image below)
A. This sleep pattern is a normal condition among older
persons
B. Medication timing, mealtimes and other changes in routine
occurring more proximally in the day rarely affect nighttime
sleep.
C. This sleep pattern may be due to increased synchronization
of the circadian rhythm with aging.
D. This sleep pattern is insomnia and may have one etiology in
C - “Age-related changes in any of the structures
physiological changes that occur in the neuroendocrine
involved in generating or entraining circadian
system.
rhythms, and/or age-related changes in any of the
critical features or processes involved in entrainment
may therefore contribute to altered circadian rhythm
timing with advancing age.”
D - Some studies show the involvement of neurons
and the adrenal system so maybe this is true also

Patel, D., Steinberg, J., & Patel, P. (2018). Insomnia


in the Elderly: A Review. Journal of clinical sleep
medicine : JCSM : official publication of the American
Academy of Sleep Medicine, 14(6), 1017–1024.
https://doi.org/10.5664/jcsm.7172
[PRV-FH] Geriatrics p. 4
Duffy, J. F., Zitting, K. M., & Chinoy, E. D. (2015).
Aging and Circadian Rhythms. Sleep medicine
clinics, 10(4), 423–434.
https://doi.org/10.1016/j.jsmc.2015.08.002
7. Which clause does NOT refer to gender identity? B Gender Identity - A, C, & D
A. A person’s internal sense of being is male or female
PREV MED 3-FH SHIFTING EXAM (5 MAY 2021) Page 3 of 14

B. Pattern of emotional and sexual attraction to people of a ● The person’s internal sense of their own
particular gender gender
C. Some combination of male and female Sexual Orientation - B
D. Neither male nor female ● Based on how a person identifies attraction
on a physical and emotional basis

[PRV-FH] Optimal Primary Care for LGBTQ+


Patients p. 1-2
8. The patient in front of you is manifesting clenched fists, wringing of C Angry, defensive, frightened or resistant
the hands, and restricted breathing patterns. The patient is UNLIKELY patients. Clenched fists, furrowed brows, wringing
to be: of the hands, restricted breathing patterns and
A. Angry and defensive warnings from office staff that something is wrong
B. Resistant can help to identify these patients. When you see
C. Confused these signs, try to uncover the source of difficulty for
D. Frightened and anxious the patient and pay attention to the way his or her
emotions relate to the medical issues at hand. Don't
get drawn into a conflict. Instead, define your
boundaries and recognize when your “triggers” are
invoked, as this will help you to modulate your
response to the situation and allow you to empathize
with the patient. Use reflective statements such as,
“I can understand why you might feel that way,” and
follow with a discussion about what it might take to
resolve the situation

https://www.aafp.org/fpm/2007/0600/p30.html
9. The goal of hospice care to keep the patient comfortable and A Hospice Care and End-of-Life Care
improve the quality of life through adequate symptom control. ● Hospice Care
A. True ○ Consists of palliative and
B. False supportive services on dying
persons and their families
● Palliative Care
○ Any form of medical care or
treatment that concentrates on
reducing the severity of disease
symptoms rather than striving to
halt, delay, or reverse the
progression of the disease itself,
or provide a cure.

[PRV-FH] Care of the Dying p. 7


10. Doctor said “ I am sorry, I have some bad news for you today. The C Giving KNOWLEDGE and Information to the
pathology report revealed that you have adenocarcinoma.” Identify the Patient
step based on the SPIKES: ● Warning the patient that bad news is
A. Setting coming may lessen the shock following the
B. Perception disclosure. This may also facilitate
C. Information information processing.
D. Invitation
[PRV-FH] Care of the Dying p. 2
11. When taking sexual history, which of the following is LEAST D Sexual History
prudent to ask - Encourage patients to choose a surrogate
A. How many partners did you have in the past 6 months? decision-make
B. Do you have sex with men, women or both?
- Should include sexual behavior,
C. Are you sexually active?
D. Which type of sexual contact brings you most pleasure? orientation, and gender identity
- Instead of asking “Are you married?”, ask
“Do you have a partner?” or “Are you in a
relationship?”
- Next you could ask “have you had female
partners, male partners, or both?”
- Alternatively, you could start by
asking “are you sexually active?”
- Then ask, “when you have had
sex, what are the genders of your
partners?”
- If patients are confused by this
question, you could ask “Do you
PREV MED 3-FH SHIFTING EXAM (5 MAY 2021) Page 4 of 14

have sex with men, women, or


both?”
- At the end of the sexual history: ○ “Do you
have any concerns or questions about your
sexuality, sexual orientation, or sexual
health?”
- You should also ask whether patients
identify as transgender or have a gender-
related concerns:
○ “Since many people are affected by
gender issues, I ask patients if they have
any concerns about this. If this topic is not
relevant to you, tell me and I will move on.”

[PRV-FH] 04 Optimal Primary Care for LGBTQ+


Patients (Batch 2022), p.3
12. Anthropometric (physical) measurements are an important part of D Screening tools to assess geriatric nutritional status:
nutrition assessment. Which is the odd one out? - MNA
A. Body Mass Index (BMI) - BMI
B. Weight - Waist circumference
C. Mid upper arm circumference - Mid-arm circumference
D. Ulnar length - Calf circumference

[PREV MED3] 1.01 – Geriatrics (19 AUG 2020) p. 5


13. Prudent behavior towards LGBT patients prior to any clinic C Understanding LGBTQ+ Issues
interview includes: - When addressing LGBTQ+ individuals,
A. Calling patient by his/her legal first name healthcare providers should always ask
B. Ask if he/she has a girlfriend/boyfriend
clients how they identify or wish to be
C. Calling patient his/her preferred name
D. Ask about civil status addressed.
- Choices B or D: Ask instead if they have a
PARTNER or are they in a relationship. ▪
Remember: Never assume

[PRV-FH] 04 Optimal Primary Care for LGBTQ+


Patients (Batch 2022), p.1
14. Which is a preventive care screening for LGBT A Screening and Preventive Measures
A. Pap smear transman with female genitalia ● Mammogram for female patients over
B. Mammogram for a bisexual male 40/50 (lesbians, bisexual transman even
C. Transrectal ultrasound for a transwoman after breast reduction/removal)
D. Pap smear for a bisexual transwoman ● Transwoman using hormones
● Prostate exam for male patients (including
transwoman even after vaginoplasty)
● Pap smear recommended for every
patient at risk of cervical cancer
● Anal pap smear (no universal guidelines)

[PRV-FH] 1.04 – Optimal Primary Care for LGBTQ+


Patients page 5
15. An 81 y/o woman has fallen outside her home on two separate C Falls and fall risk can be reduced with exercise
occasions. She experienced significant bruising from the second interventions in the community-dwelling elderly,
event. The patient is interested in exercises to reduce her fall risk. although the most effective exercise variables are
Which of the following is correct? unknown. Future studies in populations with
A. Ongoing physical therapy supervision is important comorbidities known to increase fall risk will help
B. Exercises should be done with a group of people to be determine optimal, condition-specific fall-prevention
effective programmes.
C. Regular exercise may reduce fall risk for a period of at least
2 years https://www.ncbi.nlm.nih.gov/pmc/articles/PMC2792
D. Strength-training exercises are more important than 788/
balance-training exercises

16. This medication can be useful in specific patients who have D Adjunct Analgesics
dyspnea that is significantly associated with anxiety ● Drugs with a primary indication other than pain
A. Methylprednisolone that have analgesic properties in some painful
B. Celecoxib conditions
PREV MED 3-FH SHIFTING EXAM (5 MAY 2021) Page 5 of 14

C. Amitriptyline ○ Corticosteroids: pain d/t inflammation and


D. Diazepam swelling
○ Tricyclic antidepressants (amitriptyline,
nortriptyline, doxepin)
○ Anticonvulsants (valproate,
carbamazepine,gabapentine):
radiculopathies
○ Benzodiazepines (diazepam): anxiolytic
○ Muscle relaxants (tizanidine, lioresal)
○ Anesthetics: for indwelling epidural catheter

Batch Trans 2022: Care of the Dying, page 5


17. Adjuvant analgesics are drugs that are not primarily used as A See number 16
analgesics but can produce analgesia in certain types of pain hence
can be added to the pain regimen.
A. True
B. False
18. If patients take narcotics for any length of time for chronic pain, B
they will become dependent on them. The withdrawal symptoms mean
that they are addicted.
A. True
B. False

● Since the patient only has withdrawal symptoms


with an unspecified time frame, he is not yet
addicted

[PRV-FH] 03 Care of the Dying [B2022] p. 5


19. Doctor said, “I can see how upsetting this is to you because you D Patient’s emotion
were not expecting this result.” ● When patients get bad news, their emotional
A. Invitation reaction is often an expression of shock,
B. Knowledge isolation, and grief
C. Information ○ Offer support and solidarity to the patient by
D. Empathic Response making an empathic response.
● Four Steps in Making an Empathic Response:
○ Observe for any emotion on the part of the
patient
○ Identify the emotion experienced by the patient
by naming it to oneself
○ Identify the reason for the emotion.
○ Let the patient know that you have connected
the emotion with the reason for the emotion by
making a connecting statement

[PRV-FH] 03 Care of the Dying [B2022] p. 2-3


20. A patient has mild pain due to cancer due to an inflammatory D
process. Which of the following is considered as the first line option in
Step 1 of WHO pain management?
A. Paracetamol
B. Codeine
C. Oxycodone
D. NSAIDs

[PRV-FH] 03 Care of the Dying [B2022] p. 4


21. Pansexual refers to B Pansexuality is sexual, romantic, or emotional
A. Not sexually attracted to other people attraction towards people regardless of their sex or
B. Attraction to people of any gender identity gender identity.
C. Attraction to neither biological sex
PREV MED 3-FH SHIFTING EXAM (5 MAY 2021) Page 6 of 14

D. Attraction to people of one’s own sex


22. Which of the following is a normal finding in a comprehensive B MNA Interpretations
geriatric assessment? • 24-30: Normal Nutritional Status
A. MNA-Long form score of 13 for a 60 y/o female • 17-23.5: At risk of malnutrition
B. MMSE of 23 for a 75 y/o high school graduate
C. Functional reach test of 12 in a 68 y/o male • <17: Malnourshied
D. 6 kg/m2 muscle mass in a 68 y/o male MMSE Interpretations
MMSE Interpretation via EDUCATION
• Abnormal for 8th grade education (21)
• Abnormal for high school education (<23)
• Abnormal for college education (<24)
Functional Reach Test
• 20-40 y/o: 16.73 in men, 14.64 in women
• 41-69 y/o: 14.98 in men, 13.81 in women
• 70-87 y/o: 13.16 in men, 10.47 in women
[PRV-FH] 1.01 Geriatrics [B2022] p. 2
23. When dealing with a demanding patient, the physician should B DEMANDING PATIENT
A. Explain the situation to the staff and the caregiver • Explain the situation to patient
B. Look into the patient’s frustrations/doubts • Address the patient’s doubts regarding
C. Check the patient’s doubts regarding the doctor’s your ability
competency
D. Explore the patient’s doubts regarding the family’s ability to • Explore the patient’s concerns and
care for him fears
→ “You seem doubtful with the plan that I
am suggesting. Is there anything that is
worrying you?”
→ Consider negative past experience
with a doctor or fear of dying after going
through a relative’s death
• Empathize → “I realize this symptom is
alarming because your uncle who recently
passed suffered from aneurysm.”
• Last resort is to suggest a second
opinion.

[PRV-FH] 1.02 Home Care and Difficult Clinical


Encounters [B2022] p. 4
24. Which of the following is the best approach when confronted with D See # 23
an angry and defensive patient:
A. Do not pay attention to the way his or her emotions relate to
the medical issues at hand
B. Do not listen to the warnings from office staff that something
is wrong
C. Tell the patient to do deep breathing exercises for relaxation
D. Try to uncover the source of difficulty for the patient
25. A 79-year-old woman presented with weight loss of 10 kg over the C Red Flags for Old People
preceding 3 months. On questioning, she admitted that she was • >75 years
pessimistic about the future and that she had vague gastrointestinal • Needs help with ADLs/IADLs by CCAC or
symptoms, including loss of appetite. She found it hard to go to sleep caregiver
and tended to wake early in the morning, when she felt particularly • Lives alone
tired, but this improved during the day. The family had noticed her • Falls
complaining of memory problems over the previous 6 months or so. • Delirium/confusion
Her past history included a stay in a psychiatric hospital for 4 months • Incontinence
after the birth of her son, 47 years previously. • >2 admissions to acute care hospital/year
On examination, she was thin. Although examination of her abdomen
• “Failure to thrive”
was normal, she said that she felt there was a blockage there, which
• WHEN SEEN: Investigate and give
she doubted could be helped. There were no other abnormal findings.
necessary intervention
What is the most likely diagnosis?
A. Cancer of the head of the pancreas
*memory problems = most probably dementia
B. Depression
pointing to Alzheimer’s
C. Alzheimer’s disease
D. Hyperthyroidism
[PRV-FH] 1.01 Geriatrics [B2022] p. 1
26. Which is NOT a preventive measure for LGBT B Screening and Preventive Measures
A. Prostate exam for transwoman after vaginoplasty ● Mammogram for female patients over
B. Prostate exam for a transman 40/50 (lesbians, bisexual transman even
after breast reduction/removal)
PREV MED 3-FH SHIFTING EXAM (5 MAY 2021) Page 7 of 14

C. Mammogram for bisexual transman after breast ● Transwoman using hormones


reduction/removal ● Prostate exam for male patients
D. Mammogram for transwomen using hormones (including transwoman even after
vaginoplasty)
● Pap smear – recommended for every
patient at risk of cervical cancer
● Anal pap smear (no universal guidelines)

Batch Trans 2022: Optimal Primary Care for


LGBTQ+ Patients, page 5
27. Disclosure allows the patient to make practical and emotional A Benefits of disclosure
adjustments to the terminal illness, thereby providing closure to his or ● In a study by Centeno-Cortes et al., there
her life was no increase in perceived symptoms of
A. True anxiety, despair, sadness, depression,
B. False insomnia or fear in the group of informed
patients compared to the uninformed
patients
● 75% of informed patients were able to
share their concerns about the illness and
its consequences with their relatives.
● Families of informed patients felt that they
were in a better position to care for the
patient.
● Allows the patient to make practical (e.g.
making a will, settling bank accounts and
mortgages) and emotional adjustments to
the illness; thereby providing closure to his
or her life.
● Diagnosis disclosure in cancer is also
correlated with improved symptom control
and patient satisfaction with care.

Batch Trans 2022: Care of the Dying, page 3


28. Mandating senior citizens to have Philhealth benefits is provision A RA 9994 Expanded Senior Citizens Act of 2010
of the Expanded Senior Citizens law, otherwise known as: 1. 20% discount and VAT exemption for:
A. RA 9994 2. Payment of individual income tax below
B. RA 2010 minimum wage
C. RA 9949 3. 5% on water and electricity registered on
D. RA 9995 their name not exceeding 100 kwh and 30
mm3
4. Educational assistance to pursue
secondary, tertiary, vocational and
technical courses and short-term courses
in schools
5. Exemption from training fees for
socioeconomic programs
6. Death benefit of Php 2000 to nearest
surviving relative
7. Express lanes in establishments
8. Mandatory Philhealth coverage for
indigenous senior citizens

Batch Trans 2022: Geriatrics, page 6


29. Which of the following medications does not have side effect of A
bloating when given to an elderly with constipation?
A. Polyethylene glycol
B. Lactulose
C. Psyllium
D. Senna

30. Which of the following patient care scenarios is most consistent C Delirium
with delirium? ● Complex neuropsychiatric syndrome
An 85 y/o man is admitted to the hospital with abdominal pain: characterized by acute onset of
A. He demonstrates a poor recall of facts disturbance of consciousness and
B. He has a history of an unspecified psychiatric disease. His fluctuating changes in cognition, attention,
speech is rapid and he states that there are voices that and perceptual disturbance
directed him to comes to the hospital ● Most common reason for acute cognitive
dysfunction in hospitalized older people.
PREV MED 3-FH SHIFTING EXAM (5 MAY 2021) Page 8 of 14

C. He is easily distracted during the history-taking session. He ● Prevalence of delirium at admission ranges
has difficulty answering the questions you ask appropriately, from 10 to 31%, incidence of new delirium
and frequently is closing his eyes during your assessment per admission ranges from 3 to 29% and
D. He is not oriented to place or time. He is cooperative, alert, occurrence rate per admission varies
and able to answer your questions between 11 and 42%
● May be prevented in up to a third of older
patients → early recognition is vital

Ahmed, S., Leurent, B., & Sampson, E. L. (2014).


Risk factors for incident delirium among older people
in acute hospital medical units: a systematic review
and meta-analysis. Age and ageing, 43(3), 326-333
31. Dante, a cancer patient is very quiet. You often find him reading C Bargaining
his bible. One day he says “I just want to see my grandson graduate”. ● "Just let me live to see my son graduate.“
What stage is he in? ● "I promise I'll be a better person if...."
A. Denial ● Attempt to negotiate in return for cure |
B. Anger ○ Pledge to fulfill promises or make a deal in
C. Bargaining order to get what is wanted like postponing
D. Acceptance death

[PRV-FH] 1.03 - Care of the Dying (Batch 2022), p.


3
32. An 80 y/o man presents to the ER after a fall from a ladder. His C or D Patient is on morphine so MMSE cannot be done
neighbor found him lying outside. The neighbor thinks that he may
have been there for several hours. The patient is evaluated by the Confusion Assessment Methods (CAM)
emergency medicine resident, who diagnoses a L-sided rib fracture, ● The Confusion Assessment Method (CAM) is a
as well as multiple bruises, including over the patient’s L temporal area standardized evidence-based tool that enables
and L flank. The patient has been given morphine for pain and is non-psychiatrically trained clinicians to identify
unable to respond to your questions. What may be the most accurate and recognize delirium quickly and accurately in
way to assess his baseline cognition? both clinical and research settings. There is also
A. Review the chart for a diagnosis of dementia a CAM-ICU version for use with non-verbal
B. Complete MMSE mechanically ventilated patients.
C. Complete CAM
D. Ask the family members whether the patient had memory https://www.va.gov/covidtraining/docs/The_Confusi
problems on_Assessment_Method.pdf
33. Hailey thinks she is neither a man or woman. Hailey does not really C C. Genderqueer - A term used by persons who
care if a male or female person will be attracted to her. She is may not entirely identify as either male or female
described as: A. Homosexual - A sexual orientation; Attracted to
A. Homosexual the same sex/gender
B. Bisexual B. Bisexual - A sexual orientation; Attracted to the
C. Queer same sex/gender and another different sex/gender
D. Heterosexual D. Heterosexual - A sexual orientation; Attracted to
the opposite sex/gender

[PRV-FH] 1.0 - Optimal Primary Care for LGBTQ+


Patients (Batch 2022), p. 2
34. Amor, 32 y/o, believes she is a woman in mind and heart but is still B Transgender - Natal sex discordant with internal
for genital reassignment to complete her womanhood. Jessica is a: sense of gender
A. Transman B. Transwoman - A woman who was assigned male
B. Transwoman at birth
C. Asexual Transman - A man who was assigned female at birth
D. Homosecual C. Asexual - Lack of sexual attraction
D. Homosexual - A sexual orientation; Attracted to
the same sex/gender

[PRV-FH] 1.04 - Optimal Primary Care for LGBTQ+


Patients (Batch 2022), p. 2
35. In early stages of cancer accepting palliative care means stopping B Palliative Care Treatment
all curative medical treatment ● Affirms life and regards death as a normal
A. True process
B. False ● Neither hastens nor postpones death
● Provides relief from pain and other distressing
symptoms
● Integrates the psychological and spiritual
aspects of patient care
● Offers a support system to help patients live as
actively as possible until death
● Offers a support system to help the family cope
during the patient’s illness and in their
bereavement
PREV MED 3-FH SHIFTING EXAM (5 MAY 2021) Page 9 of 14

According to getpalliativecare.org
● Palliative care is based on the needs of the
patient, not on the patient’s diagnosis
● It is appropriate for any stage and age
● It can be provided along the curative
treatment

[PRV-FH] 1.02 - Care of the Dying (Batch 2022), p.


7 https://getpalliativecare.org/whatis/
36. A 65 y/o female has influenza vaccine 6 months ago, PPSV 23, B Vaccines
Herpes zoster vaccine 5 years ago, Tdap 3 years ago. Which vaccine ● Pneumococcal vaccine (PCV13 followed by
must be given to her now? PPSV23 after 6-12 mos)
A. Varicella ○ If PPSV23 was administered first, give
B. PCV 13 PCV13 after 1 yr
C. Flu quadrivalent ● Influenza vaccine annually
D. Herpes zoster ● Herpes Zoster vaccine single dose
● Tdap
○ Tdap, TD, Tt – 3 doses (1 Tdap first then 2
Td) at 0, 1, 6-12 mos
○ Booster every 10 yrs

[PRV-FH] 01 Geriatrics [Batch 2022] p. 7


37. A physician confronted by an angry and resistance patient can C ANGRY PATIENT
employ which strategy to stay away from potential conflict ● Allow complete angry outburst.
A. Define patient’s boundaries ● Make a conscious effort to slow down response.
B. Be explicitly withdrawn Do not reply immediately. Pay special attention
C. Modulate the response of the people surrounding the to the speed and volume of your speech.
situation ● Validate the frustration with empathic
D. Recognize when your “triggers” are invoked comments:
○ “I understand that you are upset at having to
wait, and I apologize for the delay.”
○ “I can understand why you are upset. I
appreciate your waiting for me.”
● Involve the patient in the outcome. Make a
compromise, find a way to console, such as a
peace offering like how one will make it up to
him.
○ If one feels angry with what the patient is
telling you, learn to disengage. (“time out”)
● Provide the opportunity to discuss his concerns.
○ “How can I help you today?”

[PRV-FH] 02 Home Care & Difficult Clinical


Encounters[B2022] p. 4
38. The living will is only used at the end of life if a person is terminally A The Living Will
ill or permanently unconscious to describe under what conditions an ● Written or video statement about the kinds of
attempt to prolong life should be started or stopped according to the medical care you do/do not want to receive
patient’s wishes. if you are no longer able to express your
A. True consent.
B. False ○ Includes: resuscitation, desired QOL, end-
of-life treatments, etc.
○ When creating this, you should know how
independent want to be and what you want
out of your life |
→ Do you want your life extended for any
reason or only if a cure is possible?
○ Having a treatment or not is your decision.
Nothing is right or wrong. This is your
choice.

[PRV-FH] 03 Care of the Dying [B2022] p. 7


39. Both hospice and palliative care offer a personalized plan of care, B Hospice Care
delivered by an interdisciplinary team, that incorporates what is ● Consists of palliative and supportive services
important to the patient and his or her caregivers in order to achieve for dying persons and their families
the best possible quality of life for patients and families. ○ Usually those who have less than 6 months
A. True to live.
B. False ● It highlights quality rather than length of life
● Provided by interdisciplinary team of
professionals and volunteers (care coordinator,
PREV MED 3-FH SHIFTING EXAM (5 MAY 2021) Page 10 of 14

nurse, physician, counselor,volunteer


coordinator, and spiritual support)
● Provides continuity of care
● It provides family-centered care involving the
patient and family in making decisions
● Available 24/7
● Provides follow-up bereavement care for up to
1 year after the patient’s death
Palliative Care
● Any form of medical care or treatment that
concentrates on reducing the severity of
disease symptoms rather than striving to halt,
delay, or reverse the progression of the disease
itself, or provide a cure
● It should not be confused with hospice care
○ Delivers palliative care to those at the end of
life
[PRV-FH] 03 Care of the Dying [B2022] p. 7
40. The rile for reducing the dangers of polypharmacy usually includes B Management in Polypharmacy
A. Mediation review every 6 months
B. Giving patients simple dosing schedules Pharmacologic Non-pharmacologic
C. Establishing communication with all pharmacies patients use
D. Eliminating all over-the-counter medications patients buy ● Medication review (at ● Write out schedules
every office ● Write out indications
visit and/or after every for each medication
hospitalization) ● Use pill boxes to
● Eliminate track adherence
medications with ● Detailed explanation
duplicate of each medication
effects and the indication to
● Stop ineffective increase adherence
medications or those
with sub-optimal
therapeutic effect
● Add new medications
one at a time
● Advise: “start low
and go slow” when
starting new
medications
● Know all non-
prescription drugs
(herbal medicines,
vitamins)

[PRV-FH] 01 Geriatrics [Batch 2022] p. 4


41. In communicating with the family of the dementia patient, which of D FAMILY MEETING IN DEMENTIA PATIENT
the following health education statement is correct ● Allay fears of the unknown
A. Neurobic exercise can revert back memory of patient ● Handle stigma
B. Pharmacologic regime can cure the disease ○ Not all dementia is Alzheimer
C. Alzhemier’s disease is communicable ○ Orient pedigree
D. Prognosis for patient with Alzheimer’s is 5-8 years ○ Not communicable
● Inform about prognosis
○ There is no cure for AD. Current treatment
is focused on cognitive, functional, and
behavioral intentions.
○ The average lifespan of a patient with AD is
4-8 years after diagnosis.

[PRV-FH] 01 Geriatrics [Batch 2022] p. 3


42. Doctor said: “When we get the results of the CT scan, how would D SIX STEPS OF SPIKESS
you like me to discuss the results with you?” Identify the step in Obtaining the Patient’s INVITATION
SPIKES model ● Some patients desire full information about their
A. Patient’s emotions condition, but some do not.
B. Knowledge ● Discussing information at a time of ordering
C. Patient’s perception tests can cue the physician to plan the next
D. Invitation discussion with the patient.
● “How would you like me to give the information
about the test results?...”
[PRV-FH] 03 Care of the Dying [B2022] p. 2
PREV MED 3-FH SHIFTING EXAM (5 MAY 2021) Page 11 of 14

43. Which of the following is a correct recommendation for the various B A - Quality of melatonin research is poor and at
geriatric syndromes? high risk of bias. If research is believable,
A. Use of melatonin for insomnia in elderly is beneficial based melatonin may help people fall asleep faster (10
on systematic reviews mins) and spend more time asleep (15 mins).
B. Statins do not prevent or cause dementia B - Evidence indicates that statins do not prevent,
C. Docusate is superior to other products for treating treat or cause cognitive impairment or dementia.
medication-induced constipation C - Docusate appears similar to placebo in
D. Testosterone supplementation is safe for androgen decline increasing stool frequency. It is inferior to other
in aging males products for treating functional, medication-induced,
or post-op constipation.
D - Testosterone increases some muscle strength
by 7%. Adverse events include cardiovascular in
those with higher risk. Many results are
inconsistent, high risk of bias, and difficult to
quantify in real world application.

[PRV-FH] 01 Geriatrics [Batch 2022] p. 2, 4, 5, 6


44. For moderate to severe pain control, weak opioids with adjuvants B WHO ANALGESIC LADDER
are recommended in the WHO step ladder. ● Strong opioids with or without non-opioid and
A. True adjuvants are recommended for moderate-
B. False severe pain (score 7-10)

[PRV-FH] 03 Care of the Dying [B2022] p. 4


45. Doctor said: “We can schedule a follow-up next week or whenever C SIX STEPS OF SPIKESS
you are ready to discuss the options for palliative care.” Strategy and Summary
A. Patient’s perception ● Ask patients if they are ready for discussion on
B. Giving information treatment options.
C. Strategy ● Share responsibility for decision-making with
D. Setting the patient.
[PRV-FH] 03 Care of the Dying [B2022] p. 3
46. During admission of a patient diagnosed with metastatic colon CA. C Depression
The following are common indicators of clinical depression related to ● Reaction to the effects of illness
terminal illness, except: ● Anticipation of the approaching death
A. Suicidal ideation ● Inability to sustain hope
B. Difficulty concentrating in everyday task ● Clinical signs include the following:
C. Loss energy ○ Withdrawal
D. Feeling of hopelessness ○ Psychomotor retardation
○ Sleep disturbance
○ Hopelessness
○ Suicidal ideation
[PRV-FH] 03 Care of the Dying [B2022] p. 3
47. The following tests were done on an 78 y/o female: KATZ ADL B Katz Index of Independence In Activities of Daily
score of 4, Lawton IADL score of 4/8 and able to get up from a sitting Living
position using the arms of the chair. WHat is the functional ability of ● 6: Full function
the patient? ● 4: Moderate impairment
A. Needs a walking aid ● < 2: Severe functional impairment
B. Can walk mile albeit slowly and with a companion
C. Partially dependent Lawton - Brody Instrumental Activities of Daily
D. Independent Living Scale
● Score of 0 (Low function, dependent)
● Score of 8 (High function, independent)

***A summary score for the Lawton IADL ranges


from 0 (low function, dependent) to 8 (high function,
independent) for women and 0 through 5 for men to
avoid potential gender bias
How to Try This, AJN, American Journal of Nursing:
April 2008 - Volume 108 - Issue 4 - p 62-63
doi: 10.1097/01.NAJ.0000315264.84446.6b
PREV MED 3-FH SHIFTING EXAM (5 MAY 2021) Page 12 of 14

48. A 77 y/o woman is being seen in your clinic for follow-up after she B Alendronate is FDA-approved for the treatment of
was discharged from a nursing home. Three months ago, she suffered postmenopausal osteoporosis, prevention of
L femoral neck fracture after a fall. Her surgery went well and her postmenopausal osteoporosis, steroid-induced
recovery in the nursing home was unremarkable. SHe has been doing osteoporosis, male osteoporosis, and Paget disease
well at home and is now independent again with all activities of daily of the bone
living. Her medications include cholecalciferol 1000 IU once a day,
calcium 500 mg 3x a day, acetaminophen 500 mg every 4-6 hours as It is available in 5 mg, 10 mg, 35 mg, 40 mg, or 70
needed for pain. Her exam is unremarkable. mg oral tablets; 70 mg tablet for solution; and 70
Vit D 25-OH 40 ng/mL GFR 90mL/min/1.73 m2 mg/75 mL oral solution.

Which of the following is the most appropriate management? Clinical indication directs dosing guidelines:
A. Start teriparatide 20u subcutaneously daily ● Postmenopausal women: 5 mg oral tablet daily
B. Start alendronate 70mg orally once a week or 35 mg oral tablet once weekly
C. Increase calcium intake to 3000 mg once a day
D. Vit D 800 units daily Prinsloo, P. J., & Hosking, D. J. (2006). Alendronate
sodium in the management of osteoporosis.
Therapeutics and clinical risk management, 2(3),
235–249. https://doi.org/10.2147/tcrm.2006.2.3.235
49. Which is a reflective statement that will likely help an emotionally D Approach to an angry patient:
intense patient ● Allow complete angry outburst
A. “Sit down, breathe in, breathe out, things are going to be fine” ● Validate the frustration with empathic
B. “Just relax and we will resolve this” comments
C. “Look, I will listen and not judge you” ○ “I understand that you are upset at having to
D. “I can understand why you might feel that way” wait, and I apologize for the delay”
● Involve the patient in the outcome
● Make a compromise, find a way to console
● Provide the opportunity to discuss his/her
concerns

[PRV-FH] 02 Home Care & Difficult Clinical


Encounters[B2022] p. 4
50. Brio, 21 year old, male , wears a flowing white dress matched with D Gender expression
sophisticated hair and make up. This is demonstration of her ● The spectrum of masculine & feminine
A. Biological sex characteristics in how one dresses, acts or
B. Gender identity presents his/herself
C. Gender orientation ● Can be masculine, femininen or somewhere in
D. Gender expression between

[PRV-FH] 04 Optimal Primary Care for LGBTQ+


Patients [B2022] p. 2
51. Which is not a common health issue among the LGBT population? C
A. Family planning
B. Homelessness
C. Neuromuscular disorders
D. Suicide attempts

[PRV-FH] 1.04 – Optimal Primary Care for LGBTQ+


Patients. (B2022) p.5
52. The caregiver of a faily member with dementia was tested for A Modified Caregiver Strain Index (MCSI) Scoring
Modified caregiver strain index. His score is 30. This means ● ≤ 23 – normal
A. Severe strain ● 24-28 – predisposition to strain
B. Predisposition to strain ● ≥ 29 – severe caregiver strain
C. Moderate strain
D. Normal [PRV-FH] 1.01 – Geriatrics (B2022) p. 6
53. An 80 y/o patient is seen for a routine exam. An annual gait C Timed Up and Go Test
evaluation with a Timed Get Up and Go Test is performed. Her times ● From patient in sitting position → Stand without
is 11s. The most likely prediction from this results is increased: using arms, walk 3 meters, turn around → walk
A. Risk likelihood neurological dependency back, and sit down.
B. Dependency ● Doctor will time the process and observe of gait
C. Fall risk abnormalities.
D. Transferability ● Older adult who takes >12 seconds to complete
test is at high risk for falling.
[PRV-FH] 1.01 – Geriatrics (B2022) p. 1
PREV MED 3-FH SHIFTING EXAM (5 MAY 2021) Page 13 of 14

54. The critical element in the Get Up and Go Test is: B Timed Up and Go Test
A. Agility Description: Measurement of fall which correlates to
B. Balance balance and fall risk.
C. TIme
D. Flexibility https://www.thompsonhealth.com/Portals/0/_Rehabi
litationServices/PT%20Mgmt%20of%20Knee/Functi
onal_Tests.pdf
55. Cancer treatment such as chemotherapy can cause anorexia and A Anorexia and vomiting are prominent side effects of
vomiting. chemo/radiotherapy.
A. True
B. False [PRV-FH] X.02 – Care of the Dying (B2022) p. 5
56. The terms transgender D Transgender
A. Is synonymous with lesbian An umbrella term for people whose gender identity
B. Describes sexual orientation and/or expression is different from cultural
C. Refers to only men expectations based on the sex they were assigned
D. Is independent of sexual orientation at birth. Being transgender does not imply any
specific sexual orientation. Therefore, transgender
people may identify as straight, gay, lesbian,
bisexual, etc.

Sexual orientation and gender identity definitions.


(n.d.). Retrieved May 07, 2021, from
https://www.hrc.org/resources/sexual-orientation-
and-gender-identity-terminology-and-definitions

Many people have assumptions about what it means


to be transgender, but it isn't about surgery, or
sexual orientation, or even how someone dresses.
It's how they feel inside.

Fields, L. (n.d.). What it means to be transgender.


Retrieved May 07, 2021, from
https://www.webmd.com/sex/features/transgender-
what-it-means
57. Which of the following is a routine wellness screening C Wellness
recommendation for 70 years old? ● advocate wellness for older persons
A. Mammography ● Laboratory tests:
B. Pap smear ○ FOBT
C. FOBT → 60-75 y/o – annually
D. CBC → 76-85 y/o – if with risk
→ >85 y/o – do not screen
○ Bone Densitometry
→ ≥ 65 y/o – screen not less than every 2
yrs
→ 60-64 y/o – if with fracture risk
○ Pap Smear
→ 60-65 y/o – every 5 years
→ > 65 y/o with 3 adequate screening –
do not screen
○ Mammography
→ 60-74 y/o – every 2 yrs
→ ≥ 75 y/o – no recommendation
○ If at risk, do FBS, lipid profile, ECG

[PRV-FH] 01 Geriatrics [Batch 2022] p. 7


58. The principles of pain symptoms management include the C 5 Principles in Pain Control (WHO)
following except ● Define the type and site of pain
A. Establish the etiology of pain ○ Pain is considered to be the "fifth vital sign“
B. Treat each pain specifically ○ Then get the PQRST (Palliative/Provoking,
C. Give morphine as needed Quality, Radiation Severity, Timing)
D. Oral route is preferred route of administration of pain ● Anticipate pain breakthrough
medications ● Reviewing regimen frequently and regularly
● Treat “total” pain
● Treat each pain specifically

WHO 3-Step Process: The Pain Relief Ladder


● Factors to consider:
○ Absorption and convenience: Oral >
Rectal, IV > IM
○ Efficiency and Half life
PREV MED 3-FH SHIFTING EXAM (5 MAY 2021) Page 14 of 14

● Oral medication should only be abandoned if


the patient is unable to take or retain them
(therefore, is preferred)

[PRV-FH] 03 Care of the Dying (Batch 2022), p. 4


59. When interviewing the older adult with suspected dementia, it is B Communicating with Dementia Patients
most important that: ● Face the patient while talking
A. The clinician get in contact with a family member to obtain ● Talk calmly
the history ● Speak shortly in simple sentences
B. The examiner use short simple questions and recognize ● Let the patient do as much as he/she can BUT
non-verbal signs of discomfort HELP when she is confused or upset
C. Mental status evaluated first in order to determine if the ● Avoid confrontation – “Don’t say you just asked
patient is a reliable historian me that”
D. Postpone the mental status evaluation for the following visit ● Be practical
and establish a rapport first ● Clarify feelings

[PREVMED] Family Medicine (Batch 2020), p. 5


60. In a patient with life-limiting terminal phase of illness and is no B On Feeding
longer able to eat or drink, artificial nutrition like parenteral nutrition ● Studies demonstrate that parenteral or enteral
should be resorted to bring the person back to a healthy state feeding of patients near death neither improves
A. True symptom control nor lengthens life.
B. False ● Families and caregivers realize that food
pushed upon the unwilling patient may cause
problems such as aspiration.
● Help them to find alternative ways to nurture the
patient.

[PRV-FH] 03 Care of the Dying [B2022] p. 7


PREVENTIVE MEDICINE 3

(17 FEBRUARY 2021)


QUIZ
QUESTION ANSWER RATIONALE
1. "In October 3, 2015, the Department of Health announced the A Secondary Prevention
death of a Saudi national who was based in the Philippines ● Action taken to to identify diseases at their
apparently due to the Middle East Respiratory Syndrome- earliest stages and to apply appropriate
Corona Virus (MERS-CoV). The DOH Task Force MERSCoV treatments to limit their consequences and
did contact tracing of 93 people who might have been exposed severity
to the foreigner during his travel in the country, 81 (55 hospital → Screening and Surveillance
staff, 15 hotel staff, and 3 workers from the funeral parlor) of
whom have been located. All contacts were asymptomatic
except for 12 health workers who were admitted at the Batch 2022 Trans Health Promotion and
Research Institute for Tropical Medicine and the San Lazaro Interventions 1.02
Hospital and monitored for 14 days from the time of exposure
until they were cleared of a possible MERS-CoV infection."
A. Secondary prevention
B. Primary prevention
C. Tertiary prevention
D. Primordial prevention
2. "WHO declared the Philippines polio free in 2000. However, a A Organizational intervention
total of 25 polio infections have been detected in the Philippines ● May be from the following:
since its re-emergence under an outbreak declaration of this → Government agencies
debilitating disease in September 2019. The Department of → Private sector
Health has energized its vaccine campaign in children starting → Non-Governmental
August 2019 in Manila, expanded to cover millions more in Organizations (NGO)
2020-2021."
A. Organizational intervention Batch 2022 Trans Health Promotion and
B. Economic intervention Interventions 1.02
C. Political/legisltative intervention
D. Primary prevention
3. "Individuals establishing homes, residential communities or A Economic intervention
retirement villages solely to suit the needs and requirements of ● Involves giving incentives in cash or in
persons with disability enjoy realty tax holiday for the first five kind
years of operation and priority in the building and/or
maintenance of provincial or municipal roads leading to the Batch 2022 Trans Health Promotion and
previously mentioned home, residential community or Interventions 1.02
retirement village."
A. Economic intervention
B. Political/legislative intervention
C. Organizational intervention
D. Social/Community Intervention
4. "Mayor Enrico Echiverri through an Administrative Order declared A Primary Prevention
Caloocan City a No Smoking City during the nation s 110th ● Action taken to avert the occurrence of
Independence Day in June of 2008. To date, it is still disease
implementing its anti-smoking policies. " → Immunization and health
A. Primary prevention education
B. Secondary prevention
C. Tertiary Prevention Batch 2022 Trans Health Promotion and
D. Primordial prevention Interventions 1.02
5. "Individuals establishing homes, residential communities or A Tertiary Prevention
retirement villages solely to suit the needs and requirements of ● Specific interventions to assist diseased
persons with disability enjoy realty tax holiday for the first five or disabled persons in limiting the
years of operation and priority in the building and/or effects of their diseases or disabilities;
maintenance of provincial or municipal roads leading to the also may include activities to prevent
previously mentioned home, residential community or recurrences of a disease.
retirement village."
A. Tertiary prevention Batch 2022 Trans Health Promotion and
B. Secondary prevention Interventions 1.02
C. Primary prevention
D. Primordial intervention

Page 1 of 18
PRV-CH Quiz 1 and Shifting Exams – Community Health Module (17 FEBRUARY 2021) Page 2 of 18

6. "Mayor Enrico Echiverri through an Administrative Order declared A Political/ Legislative Intervention
Caloocan City a No Smoking City during the nation s 110th ● May be from the following:
Independence Day in June of 2008. To date, it is still → Individuals
implementing its anti-smoking policies. " ■ President
A. Political / Legislative intervention ■ Governor
B. Social/Community intervention ■ Mayor
C. Organizational intervention → Senate/ Congress
D. Economic intervention

Batch 2022 Trans Health Promotion and


Interventions 1.02
7. "Needle sharing among drug users is the main reason why there A Primary Prevention
has been an increase in the number of HIV positive cases in ● Action taken to avert the occurrence of
Cebu City. In this setting, the Cebu City Health Office disease
continuously monitors the HIV situation and issues strong → Immunization and health
reminders for city residents to refrain from engaging in risk education
taking behaviors such as unsafe sexual relations, casual
engagements with multiple sex partners, as well as substance Batch 2022 Trans Health Promotion and
abuse which includes sharing needles in order to deter the Interventions 1.02
spread of HIV. "
A. Primary prevention
B. Secondary prevention
C. Tertiary prevention
D. Primordial intervention
8. "WHO declared the Philippines polio free in 2000. However, a A “energized its vaccine campaign in children”
total of 25 polio infections have been detected in the Philippines
since its re-emergence under an outbreak declaration of this Primary Prevention
debilitating disease in September 2019. The Department of ● Action taken to avert the occurrence of
Health has energized its vaccine campaign in children starting disease
August 2019 in Manila, expanded to cover millions more in → Immunization and health
2020-2021." education
A. Primary prevention
B. Secondary prevention Batch 2022 Trans Health Promotion and
C. Tertiary prevention Interventions 1.02
D. Primordial prevention
9. "Section 22 of Republic Act 9211 'Tobacco Regulation Act of A Political/Legislative
2003' specifically states that beginning July 1, 2008, all forms of Examples:
tobacco advertising in mass media shall be prohibited, except ● Passage of laws requiring use of helmets
those placed inside the premises of point-of-sale while driving motorcycles and bicycles
establishments." ● Regulations requiring agencies and
A. Political / Legislative intervention companies to monitor air pollution and
B. Economical intervention governmental actions to reduce it
C. Tertiary prevention ● Regulations aimed at reducing youth
D. Organizational intervention access to tobacco product and alcohol

Batch 2022 Trans Health Promotion and


Interventions 1.02
10. "In September of 2016, DOH reported that a woman from Iloilo A Organizational
City tested positive for the ZIKA virus. As the first confirmed May be from the following:
local transmission in the Philippines, a team of DOH doctors ● Government agencies
right away went to Iloilo, closely monitored the patient and ● Private sector
looked for other possible ZIKA cases." ● Non-Governmental Organizations
A. Organizational intervention
B. Political/legislative intervention Batch 2022 Trans Health Promotion and
C. Social/ Community intervention Interventions 1.02
D. Economical intervention
11. "The Lung Center of the Philippines undertakes a cancer support A Organizational
program to improve the quality of life of patients diagnosed with Examples:
cancer through services like counseling, education, nursing ● Annual hearing and vision screening in
care, pastoral care, pain management and group therapy." schools
A. Organizational ● Automobile, bicycle, and firearm safety
B. Political/ legislative programs conducted by law enforcement
C. Social/Community agencies
D. Economical ● Development of support groups by
nonprofit organizations and facilities that
provide services to people with special
needs Batch 2022 Trans Health
Promotion and Interventions 1.02
PRV-CH Quiz 1 and Shifting Exams – Community Health Module (17 FEBRUARY 2021) Page 3 of 18

12. "In September of 2016, DOH reported that a woman from Iloilo A “a woman from Iloilo City tested positive for ZIKA
City tested positive for the ZIKA virus. As the first confirmed virus”
local transmission in the Philippines, a team of DOH doctors “closely monitored the patient and looked for other
right away went to Iloilo, closely monitored the patient and possible ZIKA cases.”
looked for other possible ZIKA cases."
A. Secondary prevention Secondary Prevention
B. Tertiary prevention ● Action taken to identify disease at their
C. Primary prevention earliest stages and to apply appropriate
D. Primordial intervention treatments to limit their consequences and
severity
→ Screening and surveillance

Batch 2022 Trans Health Promotion and


Interventions 1.02
13. "Makati City has passed City Ordinance No. 2020-089 requiring A According to the answer key, A is the correct
all its residents, workers, and visitors coming in and out of the answer although we think that since this is an
city to wear masks or any protective equipment amid the ordinance, political/legislative should be the answer.
coronavirus disease 2019 (Covid-19) outbreak. Persons who
violate the ordinance shall be penalized with a fine amounting
to PHP1,000 for the first offense, PHP3,000 for the second
offense, and for third and succeeding offenses, a PHP5,000
fine or an imprisonment of six months, or both fine and
imprisonment, at the discretion of the proper court."
A. Organizational intervention
B. Social/Community intervention
C. Political/legislative intervention
D. Economical intervention
14. "Makati City has passed City Ordinance No. 2020-089 requiring A “wear masks or any protective equipment”
all its residents, workers, and visitors coming in and out of the
city to wear masks or any protective equipment amid the Primary Prevention
coronavirus disease 2019 (Covid-19) outbreak. Persons ● Action taken to avert the occurrence of
whoviolate the ordinance shall be penalized with a fine disease
amounting to PHP1,000 for the first offense, PHP3,000 for the → Immunization and health
second offense, and for third and succeeding offenses, a education
PHP5,000 fine or an imprisonment of six months, or both fine
and imprisonment, at the discretion of the proper court." Batch 2022 Trans Health Promotion and
A. Primary prevention Interventions 1.02
B. Primary prevention
C. Educational prevention (there were items that had repeating choices but for
D. Primordial prevention some reason, only A was accepted as the correct
answer)
15. "Needle sharing among drug users is the main reason why there A
has been an increase in the number of HIV positive cases in Cebu City Health Office: organization
Cebu City. In this setting, the Cebu City Health Office
continuously monitors the HIV situation and issues strong
reminders for city residents to refrain from engaging in risk
taking behaviors such as unsafe sexual relations, casual
engagements with multiple sex partners, as well as substance
abuse which includes sharing needles in order to deter the
spread of HIV. "
A. Organizational intervention
B. Economical intervention
C. Political/legislative intervention
D. Social/ Community
16. The Metropolitan Manila Development Authority (MMDA) A
continuously implements a no nonsense clearing drive in highly “reduce the risk of them incurring physical injuries”
populated areas of the metropolis to remove obstacles and
nuisances that hamper sidewalk use by pedestrians and reduce • Primary
the risk of them incurring physical injuries arising from o Action taken to avert the occurrence of disease
accidents.
A. Primary prevention (there were items that had repeating choices but for
B. Primary prevention some reason, only A was accepted as the correct
C. Secondary prevention answer)
D. Tertiary prevention
PRV-CH Quiz 1 and Shifting Exams – Community Health Module (17 FEBRUARY 2021) Page 4 of 18

17. "Sometime in the past, Cagayan Province Health authorities A Cagayan Province Health authorities: organization
observed a sudden surge in dengue fever cases after recording
a 111 % rise in dengue fever cases as compared with the same
period a year before. To abate a possible dengue epidemic, the
local health department tracked residents suffering from flu-like
symptoms that seek medical attention and ascertain what kind
of illness they are experiencing."
A. Organizational intervention
B. Social/Community intervention
C. Political/legislative
D. Economical
18. The Metropolitan Manila Development Authority (MMDA) A MMDA: organization
continuously implements a no nonsense clearing drive in highly
populated areas of the metropolis to remove obstacles and
nuisances that hamper sidewalk use by pedestrians and reduce
the risk of them incurring physical injuries arising from
accidents.
A. Organizational intervention
B. Social/Community
C. Political/legislative
D. Economical
19. "Food establishment owners and operators in Tacloban City, A “now offer healthy menus and food choices”
after signing the pledge and manifesto of social responsibility
some years back, now offer healthy menus and food choices • Primary prevention
such as less fat, salt and sugar preparations; inclusion of fruits, o Action taken to avert the occurrence of disease
juices, vegetables and legumes; and application of low fat
cooking methods."
A. Primary prevention
B. Secondary prevention
C. Tertiary prevention
D. Primordial prevention
20. "The Lung Center of the Philippines undertakes a cancer support A “cancer support program”
program to improve the quality of life of patients diagnosed with
cancer through services like counseling, education, nursing • Tertiary prevention
care, pastoral care, pain management and group therapy." o Specific interventions to assist diseased or
A. Tertiary prevention disabled persons in limiting the effects of their
B. Tertiary prevention diseases or disabilities; also may include
C. Primary prevention activities to prevent recurrences of a disease.
D. Primordial prevention
(there were items that had repeating choices but for
some reason, only A was accepted as the correct
answer)

21. "Sometime in the past, Cagayan Province Health authorities A “ascertain what kind of illness they are
observed a sudden surge in dengue fever cases after recording experiencing”
a 111 % rise in dengue fever cases as compared with the same
period a year before. To abate a possible dengue epidemic, the • Secondary
local health department tracked residents suffering from flu-like o Action taken to identify diseases at their earliest
symptoms that seek medical attention and ascertain what kind stages and to apply appropriate treatments to
of illness they are experiencing." limit their consequences and severity
A. Secondary prevention
B. Primary prevention
C. Primary prevention
D. Primordial prevention
22. "Sometime in the past, The Food and Nutrition Research A “...to help increase awareness, influence nutrition
Institute (FNRI) website called NutritionSchool.ph was launched practices, modify behaviors and thereafter improve
to help increase awareness, influence nutrition practices, the health conditions of Filipinos.”
modify behaviors and thereafter improve the health conditions
of Filipinos. The project is maintained in part through logistics -Primary prevention
and resources provided by Nestle." o Action taken to avert the occurrence of disease
A. Primary prevention
B. Secondary prevention
C. Tertiary prevention
D. Primordial prevention
PRV-CH Quiz 1 and Shifting Exams – Community Health Module (17 FEBRUARY 2021) Page 5 of 18

23. "Through a joint communique dated August 21 and made A “the World Health Organization (WHO) and
available through the WHO website, the World Health UNICEF”
Organization (WHO) and UNICEF strongly recommend children -Organizational intervention
aged 12 and above to wear masks to help tackle the COVID 19
pandemic under the same conditions as adults, while children
between age six-eleven years should wear them on a risk-
based approach. "
A. Organizational intervention
B. Social/Community intervention
C. Economical
D. Educational
24. "Section 22 of Republic Act 9211 (Tobacco Regulation Act of A RA 9211 - Tobacco Regulation Act of 2003
2003) specifically states that beginning July 1, 2008, all forms ● An act regulating the packaging, use,
of tobacco advertising in mass media shall be prohibited, sale, distribution and advertisements of
except those placed inside the premises of point-of-sale tobacco products and for other purposes
establishments." ● Primary Prevention
A. Primary prevention
B. Secondary prevention (there were items that had repeating choices but for
C. Primary prevention some reason, only A was accepted as the correct
D. Tertiary prevention answer)

25. "In a recent online briefing by DOH Undersecretary Maria A Vaccine = Primary Prevention
Rosario Vergeire, it was revealed that the country's vaccine
expert panel has requested the details of both Phase I and
Phase II clinical trials undertaken by Gamaleya -makers of the
Russian vaccine 'Sputnik' for assessment and thereafter start a
Phase III here in the Philippines upon approval by the
Philippine FDA."
A. Primary prevention
B. Secondary prevention
C. Tertiary prevention
D. Primordial prevention
26. "Food establishment owners and operators in Tacloban City, A "Food establishment owners and operators in
after signing the pledge and manifesto of social responsibility Tacloban City”
some years back, now offer healthy menus and food choices -Social / Community Intervention
such as less fat, salt and sugar preparations; inclusion of fruits,
juices, vegetables and legumes; and application of low fat
cooking methods."
A. Social / Community Intervention
B. Educational
C. Organizational intervention
D. Political/legislative intervention
27. "Through a joint communique dated August 21 and made A “wear masks”
available through the WHO website, the World Health - Primary Prevention
Organization (WHO) and UNICEF strongly recommend children
aged 12 and above to wear masks to help tackle the COVID 19
pandemic under the same conditions as adults, while children
between age six-eleven years should wear them on a risk-
based approach. "
A. Primary prevention
B. Secondary prevention
C. Tertiary prevention
D. Primordial intervention
28. "Sometime in the past, The Food and Nutrition Research A “to help increase awareness”
Institute (FNRI) website called NutritionSchool.ph was launched -Educational intervention
to help increase awareness, influence nutrition practices,
modify behaviors and thereafter improve the health conditions
of Filipinos. The project is maintained in part through logistics
and resources provided by Nestle."
A. Educational intervention
B. Organizational intervention
C. Tertiary prevention
D. Social/community intervention
29. "In October of 2015, the Department of Health announced the A “DOH Task Force”
death of a Saudi national who was based in the Philippines -Organizational Intervention
apparently due to the Middle East Respiratory Syndrome-
Corona Virus (MERS-CoV). The DOH Task Force MERSCoV
did contact tracing of 93 people who might have been exposed
PRV-CH Quiz 1 and Shifting Exams – Community Health Module (17 FEBRUARY 2021) Page 6 of 18

to the foreigner during his travel in the country, 81 (55 hospital


staff, 15 hotel staff, and 3 workers from the funeral parlor) of
whom have been located. All contacts were asymptomatic
except for 12 health workers who were admitted at the
Research Institute for Tropical Medicine and the San Lazaro
Hospital and monitored for 14 days from the time of exposure
until they were cleared of a possible MERS-CoV infection."
A. Organizational intervention
B. Educational intervention
C. Social/Community intervention
D. Economical intervention
30. "In a recent online briefing by DOH Undersecretary Maria A “country’s vaccine expert panel”
Rosario Vergeire, it was revealed that the country's vaccine expert -Organizational Intervention
panel has requested the details of both Phase I and Phase II clinical
trials undertaken by Gamaleya -makers of the Russian vaccine
'Sputnik' for assessment and thereafter start a Phase III here in the
Philippines upon approval by the Philippine FDA."
A. Organizational intervention
B.

~END OF QUIZ~

IKAW NA DI NAKAPANSIN NG
PATTERN AND HIRAP NA HIRAP
MAGRATIO DURING EXAM
PRV-CH Quiz 1 and Shifting Exams – Community Health Module (17 FEBRUARY 2021) Page 7 of 18

SHIFTING EXAMS
QUESTION ANSWER RATIONALE
1. What document specifically consists of a core list of A Philippine National Drug Formulary
essential drugs approved and authorized by the DOH and Essential Drugs List or National Drug Formulary
used by PHILHEALTH for re-imbursement purposes? ● List of drugs
A. Philippine National Drug Formulary ● Prepared and periodically updated by DOH
B. Prohibited Drug List
→ On the basis of health conditions obtaining in
C. Monthly Index of Medical Specialties (MIMS)
D. Philippines Pharmaceuticals & Healthcare the Philippines as well as on an internationally
accepted criteria
● Consists of a core list and a complementary list
BATCH 2022 Trans Philhealth and Related
Legislations, p.8
2. The twin moves of the national government to promote rational A Review and amend promotion and advertising of
drug use include the development of the Philippine National drugs
Drug Formulary and which of the following TASK?
A. Review and amend promotion and advertising of drugs RATIO SY 2019-2020
3. "In computing for the cost of illness, which of the following data are A Average daily income or wage
required?"
A. Average daily income or wage
B. Average days of non-morbid conditions
C. Average cost of savings
D. Number of deaths RATIO SY 2019-2020
4. "Pre-disaster actions and measures to avert or minimize loss of life A Preparedness
and property by community organizing, training, planning, equipping, “to avert or minimize loss of life and property by
stockpiling, hazard mapping, insuring of assets, and public community organizing, training, planning, equipping,
information and education initiatives: pre-disaster actions and stockpiling, hazard mapping, insuring of assets, and
measures to avert or minimize loss of life and property by community public information and education initiatives: pre-
organizing, training, planning, equipping, stockpiling, hazard disaster actions and measures to avert or minimize
mapping, insuring of assets, and public information and education loss of life and property by community organizing,
initiatives" training, planning, equipping, stockpiling, hazard
A. Preparedness mapping, insuring of assets, and public information
B. Risk transfer and education initiatives"
C. Risk assessment
D. Disaster prevention
5. In which of the following situations can PHILHEALTH benefit A Payment of at least (3) monthly contributions
entitlement be availed of? within the immediate six month period prior to
A. Payment of at least three (3) monthly contributions within confinement
the immediate six month period prior to confinement
B. Confinement in an accredited hospital for less than 24 hours Entitlement to Benefits
due to an illness or disease requiring hospitalization → Paid premium contribution for at least three (3)
C. Confinement falls outside the 45 days allowance for room months within the six (6) months prior to the first day
and board but complies with the rule/ policy on single period
of availment
of confinement
D. Confinements in non-accredited hospitals for an elective → Paid in full the required premium for the calendar
surgical procedure year
→ The following need not pay the monthly
contributions to be entitled to the Program’s benefits
■ Retirees and pensioners of the SSS and GSIS
prior to March 4, 1995
■ Members of PhilHealth who have reached the
age of retirement as provided for by law, not gainfully
employed or continuing their practice as professional
and have met the required premium contributions of
at least 120 months

BATCH 2022 Trans Philhealth and Related


Legislations, p.3
6. Preferred methodology for evaluation as it best measures A OUTCOME - the longer-term effects including
behaviour change or modification over time. changes in lifestyle or behavior modification.
A. Outcome evaluation
B. Process evaluation Batch 2022: Health Administration, Managed Care
C. Impact evaluation and Health Economics Trans, p. 4
D. Quality assurance
PRV-CH Quiz 1 and Shifting Exams – Community Health Module (17 FEBRUARY 2021) Page 8 of 18

7. "Which function of management requires SMART objective setting, A PLANNING


strategy (staged intervention) and policy formulation, program design ● Achievement of the objectives in the most
and evaluation schemes for achieving goals, either for the entire efficient and economical manner
enterprise or any organized part it?" ● Use of efficient methods and the
A. Planning development of standards necessary for
B. Directing accurate control
C. Staffing ● Integration of the activities of the different
D. Evaluating units
● Reduction of emergency and unexpected
problems

Rational Planning - an assessment of need; setting


aims and objectives; deciding which methods for
strategies will achieve the objectives; Evaluating
outcome

Project Planning - refers to planning a specific


project which is time-limited and aims to bring about
a define change

Batch 2022: Health Administration, Managed Care


and Health Economics Trans, p. 2
8. "What management principle states that the clearer the line of A Definition of “Scalar Chain”
authority from the highest management position in an enterprise to
every subordinate position, the more effective decision making and B. Division of work - specialization essential to
organizational cohesiveness will be?" efficiency
A. Scalar chain C. Renumration - maximum possible satisfaction
B. Division of work D. Unity of direction - one objective, one plan, one
C. Renumeration head
D. Unity of Direction
Batch 2022: Health Administration, Managed Care
and Health Economics Trans, p. 2
9. "Which function of management involves appraisal of past and A Basic Steps of Control or Evaluation
present experiences by observations, exploration of submitted ● Determining models or basis for control
reports, analysis of statistical data in order to determine deviations which may be based on appraisal of past
and institution of corrective or remedial measures? " experiences
A. Evaluating ● Measurement of performance by
B. Directing observations, reports, statistical data,
C. Staffing determine deviations of differences
D. Planning ● Institution of corrective or remedial
measures
● Gone are the days where we rely on
attendance/attendance. Now, the
attendance is tied up with productivity.

Batch 2022: Health Administration, Managed Care


and Health Economics Trans, p. 1
10. Services offered by the organization adapt to the expressed needs A Core Principles of Quality
of users ● Equity – users have equal access and
A. Responsiveness benefit from services
B. Equity ● Effectiveness – services achieve their
C. Effectiveness intended objectives
D. Appropriateness ● Efficiency – services achieve maximum
benefit for stated costs
● Accessibility – a service is easily available
to users in terms of time, distance and
ethos.
● Appropriateness – service that which users
require
● Acceptability – services satisfy the
reasonable expectations of users.
● Responsiveness – services adapt to the
expressed needs of users.

Batch 2022: Health Administration, Managed Care


and Health Economics Trans, p. 4
PRV-CH Quiz 1 and Shifting Exams – Community Health Module (17 FEBRUARY 2021) Page 9 of 18

11. "Defined as the ability of a system, community or society exposed A Resilience is the ability of a system, community or
to hazards to absorb, accommodate and recover from the effects of a society exposed to hazards to resist, absorb,
hazard." accommodate to and recover from the effects of a
A. Resilience hazard in a timely and efficient manner, including
B. Mitigation through the preservation and restoration of its
C. Capacity building essential basic structures and functions.
D. Preparedness
Definitions are from United Nations Office for
Disaster Risk Reduction, UNISDR Terminology and
Disaster Risk Reduction (Geneva, 2009)
12. Millenium Development Goal Benefit Package under PhilHealth A PhilHealth reiterates that it covers outpatient anti-TB
includes which of the following entitlements? treatment or directly observed treatment short-
A. Outpatient Anti-Tuberculosis Treatment through Directly- course (DOTS) for new cases of pulmonary and
Observed Treatment Short-course (DOTS) extra-pulmonary TB. New case means that the
B. End-stage renal disease eligible for kidney transplantation patient has never had treatment for TB or has taken
(low risk) anti-TB drugs but for less than a month.
C. Coronary Artery Bypass Graft Surgery (standard risk)
D. Chemoradiation with Cobalt and Brachytherapy (low dose) The benefit package is worth P4,000 which covers
for cervical cancer diagnostic work-up, consultation services and anti-
TB drugs. The amount is paid to the accredited TB-
DOTS facility handling the treatment on an outpatient
set-up.

https://www.philhealth.gov.ph/news/2012/outpatient
tb_coverage.html
13. Cremation of persons confirmed to have died of COVID 19 A PRIMARY PREVENTION
A. Primary prevention Action taken to avert the occurrence of disease.
B. Secondary prevention Cremation helps in further stopping the spread of
C. Tertiary prevention COVID-19.
D. Primordial prevention
14. Which subset of Filipinos is required by law to undergo mandatory A The following shall be subjected to undergo drug
drug testing? testing:
A. "Officers and members of the military, police and other law (a) Applicants for driver's license.
enforcement agencies" (b) Applicants for firearm's license and for permit to
B. Applicants for Civil Service Commision accreditation carry firearms outside of residence.
C. Applicants for Driver’s license issuance (c) Students of secondary and tertiary schools.
D. Elite and developmental national athletes (d) Officers and employees of public and private
offices, Officers and employees of public and private
offices, whether domestic or overseas.
(e) Officers and members of the military, police
and other law enforcement agencies.
(f) All persons charged before the prosecutor's office
with a criminal offense having an impossible penalty
of imprisonment of not less than six (6) years and
one (1) day
(g) All candidates for public office whether appointed
or elected both in the national or local government
shall undergo a mandatory drug test.
15. Which of the following best reflects the relationship between the A The need for health is professionally defined as the
need for health care and the demand for health care? health care for which medical intervention is
A. They are different because need is professionally defined indicated in a given population. Demand is the
and demand is patient generated amount of care sought by the public under varying
B. They are always the same assumptions about the cost of care.
C. They are different because need takes cost into
consideration
D. They are the same when barriers to care are minimal
PRV-CH Quiz 1 and Shifting Exams – Community Health Module (17 FEBRUARY 2021) Page 10 of 18

16. "The organic compounds that linger in the environment, A Persistent Organic Pollutants (POPs)
bioaccumulate through the food web, and pose a risk of causing The the organic compounds that persist in the
adverse effects to human health and the environment. " environment bioaccumulate through the food web,
A. Persistent Organic Pollutants (POPs) and pose a risk of causing adverse effects to human
B. Ozone Depleting Substances (ODS) health and the environment. These compounds
C. Greenhouse gases resist photolytic, chemical and biological
D. Poisonous and toxic fumes degradation, which shall include but not be limited to
dioxin, furan, Polychlorinated Biphenyls (PCBs),
organochlorine pesticides, such as aldrin,dieldrin,
DDT, hexachlorobenzene, lindane, toxaphere and
chlordane.
Ozone Depleting Substances (ODS)
substances that significantly deplete or otherwise
modify the ozone layer in a manner that is likely to
result in adverse effects of human health and the
environment such as, but not limited to,
chloroflourocarbons, halons and the like.
Greenhouse gases
Gases that can potentially or can reasonably be
expected to induce global warming, which include
carbon dioxide, oxides of nitrogen,
chloroflourocarbons, and the like.
Poisonous and toxic fumes
Any emissions and fumes which are beyond
internationally - accepted standards, including but
not limited to the WHO guideline values.
17. "To protect drinking water from contamination, which is the A To protect drinking water from contamination, the
minimum distance allowed between site of bathing or washing of following measures shall be observed: prohibition of
clothes and source of drinking water as well as the construction of bathing or washing of clothes within a radius of 25
artesian, deep or shallow wells from any known source of pollution." meters from any well or other source of drinking
A. 25 meters water; and prohibition of the construction of artesian,
B. 50 meters deep or shallow wells within 25 meters from any
C. 75 meters source of pollution.
D. 100 meters
Philippine Clean Water Act Provision
18. "In economics, which of the following indicators best reflects the A GDP- the value of all goods produced and services
gross domestic product adjusted with the net factor income from the rendered in a year
rest of the world?" GNP- the gross domestic product adjusted with the
A. Gross national product (GNP) product net factor income from the rest of the world
B. Gross regional domestic product
C. Consumer price index
D. Gross value added tax
19. "Anything that injures health, endangers life, offends the senses, A Nuisance - Anything that injures health, endangers
or produces discomfort in the community is termed:" life, offends the senses or produces discomforts to
A. Nuisance the community.
B. Litter
C. Rubbish
D. Garbage
20. "Which economic valuation is a primary tool for comparing the A Cost-effectiveness analysis is a way to examine both
cost of a health intervention (any activity, using human, financial, and the costs and health outcomes of one or more
other inputs, that aims to improve health) with the expected health interventions. It compares an intervention to another
gains (reducing the risk of a health problem, reducing the severity or intervention (or the status quo) by estimating how
duration of an illness or disability, or preventing death)." much it costs to gain a unit of a health outcome, like
A. Cost-effective analysis (CEA) a life year gained or a death prevented.
B. Cost-minimization analysis (CMA)
C. Cost-benefit analysis (CBA) Source: CDC.gov
D. Cost-utility analysis (CUA)
21. Permission to disinter remains of persons who died of dangerous A REQUIREMENTS FOR THE DISINTERMENT OR
communicable diseases may be granted after a burial period of how EXHUMATION OF REMAINS OF A PERSON WHO
many years? DIED OF DANGEROUS COMMUNICABLE
A. Five (5) years DISEASE
B. Four (4) years
C. Three (3) years a) Permission to disinter the remains of a person who
D. One (1) year died of a disease other than a dangerous
communicable disease may be granted after such
remains has been buried for a period of five (5) years
PRV-CH Quiz 1 and Shifting Exams – Community Health Module (17 FEBRUARY 2021) Page 11 of 18

22. The National Insurance Act of 2013 covers which sub-population A AN ACT INSTITUTING A NATIONAL HEALTH
of Filipinos? INSURANCE PROGRAM FOR ALL FILIPINOS
A. All Filipinos AND ESTABLISHING THE PHILIPPINE HEALTH
B. Employed Filipinos INSURANCE CORPORATION FOR THE
C. Overseas Filipino workers PURPOSE
D. Marginalized sector
23. The need for effective communication and teamwork are A The principle states that an organization must make
emphasized in which of the following management principles? every effort to maintain group cohesion in the
A. Esprit-de-corp organization. It notes that dividing your competition
B. Subordination is a clever tactic, but dividing your own team is a
C. Order serious error
D. Stability of Tenure

24. "Under the Philippine Disability Act, health professionals play A A = Positive empowerment
important role in promoting the employment of people with disabilities. C-D = Disapproval of people with disabilities
Which statement supports this direction?"
A. Educate employers on work abilities of disabled.
B. Deny employment of a disabled on the basis of absence of
facilities for disabled
C. Situate people with disabilities in the workplace setting on a
very limited capacity
D. Entertain the notion that disabled individuals may not
perform well in a given instance.
25. "Private entities that improve or modify their physical facilities in A Dealing with money most likely relates to economics.
order to provide reasonable accommodation for disabled persons
shall also be entitled to an additional deduction from their net taxable
income, equivalent to fifty percent (50%) of the direct costs of the
improvements or modifications."
A. Economic intervention
B. Educational intervention
C. Social intervention
D. Organizational
26. "When the cause of death is a dangerous communicable disease, A Dangerous Communicable Disease Remains are
the remains shall be buried within how many hours after death?" buried within 12 hours after death and shall not be
A. Twelve (12) hours taken to any place of public assembly. Only the adult
B. members of the deceased's family will be permitted
to attend the funeral
27. "An organized system of health care delivery that offers a A Managed care is defined as “an organized system of
comprehensive package of benefits to voluntarily enrolled members health care delivery that offers a comprehensive
who pay for a fixed, prepaid period of engagement and agreement package of benefits to voluntarily enrolled members
with a Health Maintenance Organization is termed:" who pay for a fixed, prepaid period”
A. Managed care
B. Socialized medicine Health Maintenance Organization in the Philippines:
C. Community based health care PhilCare, Medicard, Maxicare, HMII, Intellicare,
D. Preferred provider care Cocolife, Medocare, Caritas Health Shield, etc.

Batch 2022: Health Administration, Managed Care


and Health Economics Trans, p. 7
28. Consists of solid particles that develop by condensation of gases A Fumes are formed when a metal is heated above its
given off by heated metals or plastics boiling point and its vapours condense into very fine,
A. Fume particles (solid particulates)
B. Gas
C. Vapor https://www.ccohs.ca/oshanswers/safety_haz/weldi
D. Mist ng/fumes.html
29. Which is a description of what the organization is currently doing A Mission Statement:
as it fulfils its purpose of existence? ● a declaration as to why an organization
A. Mission statement exists
B. Vision ● defines the business the organization is
C. Policy currently in
D. Program Design ● concentrates on the present
● reflects an organization’s core
competencies, the basic skills or products
provided

Batch 2022: Health Administration, Managed Care


and Health Economics Trans, p. 3
PRV-CH Quiz 1 and Shifting Exams – Community Health Module (17 FEBRUARY 2021) Page 12 of 18

30. "Which management scheme requires setting up of a large-scale A Strategic Planning refers to planning a large-scale
activity that involves signing a memorandum of understanding or activity involving different partners and staged
agreement amongst multiple partnerships, staged interventions, and interventions. Ideally, this is set to 5 years.
the achievement of goals in a defined future state?"
A. Strategic planning Batch 2022: Health Administration, Managed Care
B. Policy formulation and Health Economics Trans, p. 2
C. Project evaluation
D. Resource allocation
31. Administration is a complex process aimed at which of the Administration refers to “a complex process aimed
following outcomes? A at achieving objectives with the LEAST possible
A. "Achieving objectives with the least possible expenditure of expenditure of time, energy, and money
time, energy and money consistent with the agreed quality consistent with the agreed quality of work.
of work."
B. Supervising and controlling people to work as one Batch 2022: Health Administration, Managed Care
regardless of expenditures and Health Economics Trans, p. 1
C. Coordinating all existing services thru hierarchy regardless
of expenditures
D. “Achieving objectives with the greatest expenditure of time,
energy and money consistent with the agreed quality of
work”
32. "Any solid, semi-solid or liquid waste or residue generated from a A Sludge: any solid, semi-solid or liquid waste or
wastewater treatment plant, water supply treatment plant, or water residue generated from a wastewater treatment
control pollution facility, or any other such waste having similar plant, water supply treatment plant, or water control
characteristics and effects." pollution facility, or any other such waste having
A. Sludge similar characteristics and effects.
B. Sewage
C. Septage Sewage: water-borne human or animal wastes,
D. Effluent excluding oil or oil wastes, removed from residences,
building, institutions, industrial and commercial
establishments together with such groundwater,
surface water and storm water as maybe present
including such waste from vessels, offshore
structures, other receptacles intended to receive or
retain waste or other places or the combination
thereof.

Septage: the sludge produced on individual onsite


wastewater disposal systems, principally septic
tanks and cesspools.

Effluent - discharge from known sources which is


passed into a body of water or land, or wastewater
flowing out of a manufacturing plant, industrial plant
including domestic, commercial and recreational
facilities.

UST FMS Medical Board Review 2019 - Preventive,


Family, & Community Medicine p. 94
33. RA 8749 'Philippine Clean Air Act of 1999' is a comprehensive air A Political/Legislative
quality management policy and program which aims to achieve and ● may be from certain individuals (President,
maintain 'Healthy Air for Filipinos' Governor, Mayor) or the Senate or
A. Political/Legislative intervention Congress
B. Organizational intervention ● passage of laws or regulations
C. Educational intervention
D. Social/Community intervention “RA 8749” - political/legislative
34. "Swab testing (RT-PCR), management intervention including A Secondary Prevention pertains to actions taken to
treatment of symptoms, supportive care, isolation, and experimental identify diseases at their earliest stages and to apply
measures using various COVID 19 protocols." appropriate treatments to limit their consequences
A. Secondary prevention and severity.
B. Primary prevention
C. Tertiary prevention Batch 2022: Health Promotion and Interventions, p.
D. Primordial prevention 1

In the context of the COVID19 pandemic, swab


testing is being conducted in order to effectively
identify individuals with an asymptomatic disease.
Considering this, these patients are already afflicted
with COVID19 therefore eliminating the choices on
PRV-CH Quiz 1 and Shifting Exams – Community Health Module (17 FEBRUARY 2021) Page 13 of 18

primary and primordial preventions. Swab testing is


being done to prevent further spread of the disease
from an already affected individual to other
susceptible contacts, and is not being done simply
as a screening strategy to allow for exemptions to
quarantine protocols.
35. "What function of management involves defining manpower A Staffing or Human Resources involves the
requirements for positions, selecting qualified people from following tasks:
candidates, and professional development for incumbents in order to ● Manning and keeping positions provided
accomplish tasks effectively?" for by the organization structure
A. Staffing ● Defining manpower requirements for jobs
B. Planning ● Selecting candidates for positions
C. Evaluating ● Training or developing candidates and
D. Directing incumbents to accomplish tasks effectively

Batch 2022: Health Administration, Managed Care


and Health Economics Trans, p. 1
36. "Discharge from a known source which is passed into a body of A Effluent
water or land, or wastewater flowing out of a manufacturing plant, ● means discharges from known source
industrial plant including domestic, commercial and recreational which is passed into a body of water or
facilities." land, or wastewater flowing out of a
A. Effluent manufacturing plant, industrial plant
B. Septage including domestic, commercial and
C. Sludge recreational facilities.
D. Sewage
“RA 9275” - Environmental Management Bureau
37. "A water tight receptacle which receives the discharge of a A Septic tank
plumbing system or a part thereof, and designed to accomplish partial ● A water tight receptacle which receives
removal and digestion of suspended solid matter in the sewage the discharge of a plumbing system or
through a period of retention." part thereof, and is designed to
A. Septic tank accomplish the partial removal and
B. Septic privy digestion of the suspended solid matter
C. Chemical privy in the sewage through a period of
D. Concrete vault privy
detention.

From the “Sanitation Code of the Philippines”


38. "In SWOT analysis of the organization's environment, exploration A Those under the external environment or milieu are
and determination of opportunities and threats pertain to which of the issues that cannot be controlled but can be identified.
following considerations?" This includes opportunities and threats.
A. Factors in the external environment or milieu
B. Factors in the internal environment or milieu Those under the internal environment or milieu are
C. Vision and mission statement issues that can be controlled which include the
D. Objectives of the organization strengths and weaknesses.

Batch 2022: Health Administration, Managed Care


and Health Economics Trans, p. 2-3
39. "The Department of Environmental Resources (DENR) is the lead A DENR is considered a government agency. Under
agency tasked to regularly review, revise and publish the national the strategies for health promotion, government
water quality status, integrated water quality management framework agency interventions are classified as being under
and ten (10) - year water quality management area action plan. " Organizational.
A. Organizational intervention
B. Political/legislative intervention Political/legislative intervention pertain to
C. Social/community intervention individuals in higher offices, which include the
D. Educational intervention President, Governors, and Mayors, as well as the
Senate/Congress.

Batch 2022: Health Promotion and Interventions, p.


2
40. "The agency that makes policies, strategies and programs on drug A Dangerous Drugs Board (DDB) is the policy-making
prevention and control." and strategy-formulating body in the planning and
A. Dangerous Drugs Board (DDB) formulation of policies and programs on drug
B. Philippine National Police prevention and control.
C. National Bureau of Investigation
D. DOH Philippine National Police (PNP) is in charge of
implementing their 5-point anti-illegal drug strategy.

National Bureau of Investigation (NBI) is in charge


of, among others, undertaking efficient detection and
PRV-CH Quiz 1 and Shifting Exams – Community Health Module (17 FEBRUARY 2021) Page 14 of 18

investigation of crimes and other offenses against


the laws of the Philippines.

Department of Health (DOH) has three major roles


in the health sector: (1) leadership in health; (2)
enabler and capacity builder; and (3) administrator of
specific services. Its mandate is to develop national
plans, technical standards, and guidelines on health.

41. A-Z benefit package under Philhealth includes entitlement for A Z Benefit Packages include mandatory services for
which of the following diseases and interventions? the totality of care, essential for the treatment of the
A. Prostate Cancer (low to intermediate risk); condition, hospital services such as accommodation,
B. Animal Bite Treatment Package medicines, laboratories and professional fee.
C. Outpatient HIV-AIDS Package Included are: ALL, breast cancer, tetralogy of fallot,
D. Voluntary Surgical Contraception Procedures ventricular septal defect, kidney transplant, coronary
artery bypass graft, prostate cancer, cervical
cancer, Z morph and expanded Z morph, selected
orthopedic implants, peritoneal dialysis, colon and
rectum cancer, premature and small newborns, and
children with: developmental disabilities, disability
mobility impairment, disabilities hearing impairment
and visual disabilities.

Batch 2022: 1.02 – PHILHEALTH AND RELATED


LEGISLATIONS Trans pg. 6

42. "Recognized as the process of using administrative directives, A Disaster Risk Reduction
organizations, and operational skills and capacities to implement The policy objective of anticipating and reducing risk
strategies, policies and improved coping capacities to lessen the is called disaster risk reduction (DRR). Although
adverse impacts of hazards and the possibility of disaster. " often used interchangeably with DRR, disaster risk
A. Disaster Risk Reduction and Management management (DRM) can be thought of as the
B. Disaster Mitigation implementation of DRR, since it describes the
C. Contingency Planning actions that aim to achieve the objective of reducing
D. Disaster response risk. It requires a people-centred and multi-sector
approach, building resilience to multiple, cascading
and interacting hazards and creating a culture of
prevention and resilience

Disaster mitigation measures may be structural (e.g.


flood dikes) or non-structural (e.g. land use zoning).
Mitigation activities should incorporate the
measurement and assessment of the evolving risk
environment.

A contingency plan is a plan devised for an outcome


other than in the usual (expected) plan.

Disaster Response as defined in the RA 10121, is


the provision of emergency services and public
assistance during or immediately after a disaster in
order to save lives, reduces health impacts, ensure
public safety and meet the basic subsistence needs
of the people affected.

https://www.preventionweb.net/disaster-
risk/concepts/drr-drm/
43. "If the cost of preventing a hip fracture is greater than the cost of A The aging of our population heightens the need to
surgical repair, which of the following statements is true?" recognize the interaction of these conditions in order
A. A preventive strategy still may be indicated in such to improve our efforts to prevent hip fractures,
instances. provide acute care that improves outcomes, and
B. “With the presence of hip fracture, conservative provide secondary prevention and rehabilitation that
management still is the best option” returns patients to their previous level of functioning.
C. No attempt should be made to prevent hip fracture until a Identification and treatment of vitamin D deficiency
more cost-effective strategy is devised and osteoporosis and assessment and interventions
D. There is no available regimen to choose from. to reduce falls in patients with dementia can
significantly impact the incidence of first and
subsequent hip fractures.
PRV-CH Quiz 1 and Shifting Exams – Community Health Module (17 FEBRUARY 2021) Page 15 of 18

Menzies, I. B., Mendelson, D. A., Kates, S. L., &


Friedman, S. M. (2010). Prevention and clinical
management of hip fractures in patients with
dementia. Geriatric orthopaedic surgery &
rehabilitation, 1(2), 63–72.
https://doi.org/10.1177/2151458510389465

44. Which of the following statements regarding the relationship A A second study by Ross and Mirowsky (2000) based
between insurance and health care use is correct? on the Survey of Aging, Status and the Sense of
A. "People with less insurance use less health care, which in Control (ASOC) examined the claim that being
effect leads to decreases in both necessary and uninsured contributes to the worse health of persons
unnecessary care." of lower SES.
B. People always use the same amount of health care
independent of their insurance
C. “Compared with other goods and services, people change
the amount of health care they use significantly in response
to changes in insurance”
D. No empiric information is available to determine whether
people adjust their behavior depending on their insurance
coverage
45. Which is a description of what the organization would look like to A A vision statement describes the company's
be in a preferred future when it is fulfilling its purpose? purpose, what the company is striving for, and what
A. Vision it wants to achieve. A vision statement describes
B. Mission statement what a company desires to achieve in the long-run,
C. Program design generally in a time frame of five to ten years, or
D. Policy sometimes even longer.

A mission statement is a short statement of why an


organization exists.

Program design is the process that an organization


uses to develop a program. It is most often an
iterative process involving research, consultation,
initial design, testing and. redesign.

Policy' is a predetermined course of action, which is


established to provide a guide toward accepted
business strategies and objectives.
46. This is a management process which analyzes specific potential A contingency planning is a course of action that an
events or emerging situations which might threaten the society and organization would take if an unexpected event or
the environment and establishes arrangements to enable responses situation occurs.
to such events and situations
A. Contingency planning https://www.preventionweb.net/disaster-
B. Risk Assessment risk/concepts/drr-drm/
C. Mitigation
D. Preparedness
47. Set ceiling for purchase price of selected drugs in the Philippine A The MRP shall be construed as the imposition of
market. maximum prices across all levels of the supply
A. Maximum drug retail price chains,‘including, but not limited to, prices set by
B. Minimum drug retail price manufacturers, traders, distributors, wholesalers,
C. Drug production costs and retailers.
D. Retail profit
https://law.upd.edu.ph/wp-
content/uploads/2020/09/DOH-Administrative-
Order-2020-003

48. "Agency in the Department of Health (DOH) which is headed by a A “The FDA shall be headed by a director-general, with
Director-General (rank of Undersecretary of Health) created in order the rank of undersecretary…”
to protect and promote the right to health of the Filipino people,
establish as well as maintain an effective health products regulatory Rep. Act No. 9711, sec. 7(a)
system."
A. Philippine Food and Drug Administration “To guarantee the safety, quality, purity, efficacy of
B. Philhealth products in order to protect and promote the right to
C. Health Facilities and Service Regulatory Bureau (HFSRB) health of the general public”
D. Insurance Commission
Mission statement of the FDA;
https://www.fda.gov.ph/about/
PRV-CH Quiz 1 and Shifting Exams – Community Health Module (17 FEBRUARY 2021) Page 16 of 18

49. A serious disruption of the functioning of a community or society A “A disaster risk a sudden, calamitous event that
which exceeds the ability of the affected community or society to cope seriously disrupts the functioning of a community or
using its own resources. society and causes human, material, and economic
A. Disaster or environmental losses that exceed the
B. Medical emergency community’s or society’s ability to cope using its own
C. Exposure resources.”
D. Risk
https://www.ifrc.org/en/what-we-do/disaster-
management/about-disasters/what-is-a-disaster/
50. The concept of lifetime health monitoring refers to: A “As a stimulus for further development, we propose
A. Modification of the periodic health examination to focus on a Lifetime Health Monitoring Program that uses
likely conditions in one given individual at a given stage. clinical and epidemiologic criteria to identify specific
B. Routine performance of a calendared comprehensive health goals and professional services appropriate
examination for 10 different age groups.”
C. The aggressive use of diagnostic technology during an
individual’s lifetime Breslow, L., & Somers, A. R. (1977). The lifetime
D. The completion of periodic health surveys compiled on a Health-Monitoring Program. New England Journal of
national database Medicine, 296(11), 601-608.
doi:10.1056/nejm197703172961104
51. "Which of the following duly licensed, DOH and PHILHEALTH A Health Care Provider - Refers to:
accredited health care providers is devoted primarily to the 1. a health care institution, which is duly
maintenance and operation of facilities for health promotion, licensed and accredited devoted primarily
prevention, diagnosis,treatment and care of individuals suffering from to the maintenance and operation of
illness, disease, injury, disability or deformity, drug addiction or in facilities for health promotion, prevention,
need of obstetrical or other medical and nursing care. diagnosis, treatment, and care of
A. Health care institution individuals suffering from illness, disease,
B. Health care professional injury, disability, or deformity, or in need of
C. Health maintenance organization obstetrical or other medical and nursing
D. Preferred provider organization care.
Rep. Act No. 7875, sec. 2(o)
52. "Water containing less than 500 ppm dissolved common salt, A Freshwater - means water containing less than 500
sodium chloride, such as that in groundwater, rivers, ponds and ppm dissolved common salt, sodium chloride, such
lakes." as that in groundwater, rivers, ponds and lakes.
A. Freshwater
B. Drinking water Rep. Act No. 9275, sec. 4(q)
C. Groundwater
D. Wastewater
53. "Which economic valuation determines if health screening, A “In health economics, the purpose of CUA is to
nutrition counseling, lifestyle advise, medications, or cardiac bypass estimate the ratio between the cost of a health-
surgery would provide the most additional years of life expectancy related intervention and the benefit if produces in
(quality adjusted life-years) for an individual diagnosed to have a terms of the number of years lived in full health by
lifestyle disease?" the beneficiaries (Quality Adjusted Life-Years)”
A. Cost-utility analysis (CUA)
B. Cost-benefit analysis (CBA) Batch 2022: Health Administration, Managed Care,
C. Cost-effective analysis (CEA) and Health Economics Trans, p. 9
D. Cost-minimization analysis (CMA)
54. "The Comprehensive Dangerous Drugs Act of 2002 (RA 9165) A Possession of Dangerous Drugs - The penalty of life
penalizes individuals with life imprisonment and a fine of P 500,000.00 imprisonment to death and a fine ranging from Five
to P 10,000,000.00 if caught with possession of which banned hundred thousand pesos (P500,000.00) to Ten
substances?" million pesos (P10,000,000.00) shall be imposed
A. "Ten (10) grams of MDMA or ecstasy , PMA, TMA, LSD, upon any person, who, unless authorized by law,
GHB." shall possess any dangerous drug in the following
B. Ten (10) grams of marijuana quantities, regardless of the degree of purity thereof:
C. Ten (10) grams of methamphetamine hydrochloride or
shabu (8) 10 grams or more of other dangerous drugs such
D. Ten (10) grams of anabolic corticosteroids as but not limited to
methylenedioxymethamphetamine (MDA) or
“ecstasy”, paramethoxyamphetamine (PMA),
trimethoxyamphetamine (TMA), lysergic acid
diethylamine (LSD), gamma hydroxyamphetamine
(GHB)...”

Rep. Act No. 9165, sec. 11


PRV-CH Quiz 1 and Shifting Exams – Community Health Module (17 FEBRUARY 2021) Page 17 of 18

55. "Inter-Agency Task Force on Emerging Diseases recommends A Primary Prevention


frequent hand washing, social distancing (maintaining physical ● Action taken to avert the occurrence of
distance from others), covering coughs and sneezes, keeping disease
unwashed hands away from the face, and wearing of face shields to
deter the transmission of SARsCov2 " Frequent hand washing, social distancing, covering
A. Primary prevention cough and sneezes, keeping unwashed hands
B. Secondary prevention away from face and wearing of face shields are all
C. Tertiary prevention measures done to prevent contracting COVID-19.
D. Primordial prevention
56. The service provided is compatible with the patient's belief A Core Principles of Quality
system. ● Equity – users have equal access and
A. Acceptability benefit from services
B. Accessibility ● Effectiveness – services achieve their
C. Availability intended objectives
D. Adequacy ● Efficiency – services achieve maximum
benefit for stated costs
● Accessibility – a service is easily available
to users in terms of time, distance and
ethos.
● Appropriateness – service that which users
require
● Acceptability – services satisfy the
reasonable expectations of users.
● Responsiveness – services adapt to the
expressed needs of users.

Batch 2022: Health Administration, Managed Care


and Health Economics Trans, p. 4
57. The overall outline of action that includes collection of activities in A Key Elements in Planning
a planned sequence leading to a defined goal or set objectives. Plan
A. Program - How to get from your starting point to your
B. Plan end point and what you want to achieve
C. Strategy Strategy
- Broad framework for action which indicates
goals, methods, and underlying principles
Policy
- Guidelines for practice which set broad
goals and the framework for action
Program
- Overall outline of action; collection of
activities in a planned sequence leading
to a defined goal or goals

Trans: Health Administration, Managed Care, and


Health Economics, p. 1
58. “To date, which tops the list of substances most commonly abused A “Methamphetamine Hydrochloride, commonly
in the Philippines?" known as “Shabu” remains to be the main drug of
A. Methamphetamine Hydrochloride (Shabu) abuse comprising ninety-four percent (93.72%) of
B. Cannabis (Marijuana) the total admission. This is followed by Cannabis
C. Inhalants (Contact Cement) (Marijuana) at twenty-three percent (22.59%), and
D. Opium Contact Cement (Rugby) with less than one percent
(0.73%). Mono drug use is still the nature of drug
taking and the routes of administration are
inhalation/sniffing and oral ingestion.”

Dangerous Drug Board website:


https://www.ddb.gov.ph/component/content/categor
y/45-research-and-statistics

59. Triple burden of disease for the Philippines is best exemplified by A Choice A is the only statement that corresponds to
which of the following scenarios? the triple burden of disease.
A. "Increased health consequences brought about by
exposure of Filipinos to urbanization, industrialization, B- WRONG
climate change and natural disasters." Because cardiovascular disease tops the list
B. Deaths are mainly due to communicable diseases
C. “Ten leading causes of morbidity are predominantly chronic, C-WRONG
degenerative lifestyle diseases.” Because morbidity is predominantly caused by
D. Top ten diseases under morbidity and mortality are non- communicable diseases
preventable causes.
PRV-CH Quiz 1 and Shifting Exams – Community Health Module (17 FEBRUARY 2021) Page 18 of 18

D-WRONG
Because the causes are actually preventable

60. "Activity which incites hatred, serious contempt or severe ridicule A “SEC 41. Vilification. – For purposes of this
of persons with disability" Chapter, vilification shall be defined as:
A. Vilification
B. Discrimination (a) The utterance of slanderous and abusive
C. Indignation statements against a person with disability; and/or
D. Battery
(b) An activity in public which incites hatred
towards, serious contempt for, or severe ridicule
of persons with disability.”

RA 9442- An Act Amending Republic Act No. 7277,


Otherwise Known As The “Magna Carta For
Disabled Persons, And For Other Purposes”

~ END ~

MED STUDENT
B,C, D
A NA DI NAG-ARAL

You might also like